Sie sind auf Seite 1von 441

!

!
!
!
!
!
!

SMLE!Questions!
!
July!2017!5!October!12,!2017!
!
!
!
!
!
!
!
!
!
!
!
!
!
!
!
!
!
!
!
!
!
!
!
!
!
!
!
!
!
Medicine''''''''''''''''''''''''''''''''''''''''''''''''''''''''''''''''''''''''''''''''''''''''''''''''''''''''''''''''''''''''
Infectious'Diseases''''''''''''''''''''''''''''''''''''''''''''''''''''''''''''''''''''''''''''''''''''''''''''''''''''3'
Allergy'and'immunology''''''''''''''''''''''''''''''''''''''''''''''''''''''''''''''''''''''''''''''''''''''''''''26'
Endocrinology'''''''''''''''''''''''''''''''''''''''''''''''''''''''''''''''''''''''''''''''''''''''''''''''''''''''''''28'
Pulmonology'''''''''''''''''''''''''''''''''''''''''''''''''''''''''''''''''''''''''''''''''''''''''''''''''''''''''''''40'
Cardiology'''''''''''''''''''''''''''''''''''''''''''''''''''''''''''''''''''''''''''''''''''''''''''''''''''''''''''''''''47'
Rheumatology'''''''''''''''''''''''''''''''''''''''''''''''''''''''''''''''''''''''''''''''''''''''''''''''''''''''''''58'
Gastroenterology'''''''''''''''''''''''''''''''''''''''''''''''''''''''''''''''''''''''''''''''''''''''''''''''''''''''62'
HematologyEOncology'''''''''''''''''''''''''''''''''''''''''''''''''''''''''''''''''''''''''''''''''''''''''''''''75'
Nephrology''''''''''''''''''''''''''''''''''''''''''''''''''''''''''''''''''''''''''''''''''''''''''''''''''''''''''''''''89'
Neurology''''''''''''''''''''''''''''''''''''''''''''''''''''''''''''''''''''''''''''''''''''''''''''''''''''''''''''''''''94'

Pediatrics''''''''''''''''''''''''''''''''''''''''''''''''''''''''''''''''''''''''''''''''''''''''''''''''''''''''''''''''''''''106'

Family'Medicine''''''''''''''''''''''''''''''''''''''''''''''''''''''''''''''''''''''''''''''''''''''''''''''''''''''''''''168'

Research'''''''''''''''''''''''''''''''''''''''''''''''''''''''''''''''''''''''''''''''''''''''''''''''''''''''''''''''''''''''190'

Emergency'Medicine'''''''''''''''''''''''''''''''''''''''''''''''''''''''''''''''''''''''''''''''''''''''''''''''''''''197'

OB/GYN''''''''''''''''''''''''''''''''''''''''''''''''''''''''''''''''''''''''''''''''''''''''''''''''''''''''''''''''''''''''215'

General'Surgery''''''''''''''''''''''''''''''''''''''''''''''''''''''''''''''''''''''''''''''''''''''''''''''''''''''''''''257'

Orthopedics'''''''''''''''''''''''''''''''''''''''''''''''''''''''''''''''''''''''''''''''''''''''''''''''''''''''''''''''''''287'

ENT'''''''''''''''''''''''''''''''''''''''''''''''''''''''''''''''''''''''''''''''''''''''''''''''''''''''''''''''''''''''''''''''309'

Anesthesia''''''''''''''''''''''''''''''''''''''''''''''''''''''''''''''''''''''''''''''''''''''''''''''''''''''''''''''''''''318'

Dermatology''''''''''''''''''''''''''''''''''''''''''''''''''''''''''''''''''''''''''''''''''''''''''''''''''''''''''''''''''322'

Ophthalmology''''''''''''''''''''''''''''''''''''''''''''''''''''''''''''''''''''''''''''''''''''''''''''''''''''''''''''''336'

Psychiatry''''''''''''''''''''''''''''''''''''''''''''''''''''''''''''''''''''''''''''''''''''''''''''''''''''''''''''''''''''''356'

Basic'Science''''''''''''''''''''''''''''''''''''''''''''''''''''''''''''''''''''''''''''''''''''''''''''''''''''''''''''''''378'
Genetics'''''''''''''''''''''''''''''''''''''''''''''''''''''''''''''''''''''''''''''''''''''''''''''''''''''''''''''''''''435'
!
Medicine!!

Infectious!Diseases!
!!
1.' Trichomaniasis!diseases!what!is!the!treatment?!
Answer:!metronidazole!
!!
!!! 2.!!Shigella,!treatment?!
Answer:!Ampicillin!
MedScape:!Ampicillin!and!TMP9SMZ!are!effective!for!susceptible!strains;!amoxicillin!is!less!effective!
than!this!because!of!its!rapid!absorption!high!in!the!GI!tract.!
First!aid:!TMP9SMX!to!decrease!person9person!spread.!
!!
!!! 3.!!Salmonella,!treatment!?!
Answer:!azithromycin,!cefriaxone,!ciprofloxacin!(Medscape)!
First!aid:!oral!quinolones!or!TMP9SMX!
!!
!!! 4.!!Enterococcus!fragilis,!treatment?!
!!!!!!!! Answer:!clindamycin!
Medscape:!
Penicillin!G!(Pfizerpen),!Cefoxitin!(Mefoxin),!Cefotetan,!Clindamycin!(Cleocin),!Amoxicillin!and!
clavulanate!(Augmentin),!Ticarcillin!and!clavulanate!potassium!(Timentin),!Chloramphenicol,!
Meropenem!(Merrem),!Metronidazole!(Flagyl),!Tigecycline!(Tygacil),!Moxifloxacin!(Avelox),!
Entapenem.!
!!
!!! 5.!!Enterococcus!feacalis,!treatment?!
Answer:!Ampicillin!is!drug!of!choice,!allergic!or!resistant!to!ampicillin:!Vancomycin!
!!
!!! 6.!!Bacteroid!fragilis!Abx?!
Answer:!Ampicillin!"not!sure"!
!!
!!! 7.!!Gun!shoot!with!bowel!perforation?!
Answer:!Metronidazole!+!gentamicin,!or!Metro!+!cefoxitin!
!!
!!!!!!8.!!Rosacea,!treatment!?!
Answe:!erythromycin,!if!Doxycyclin!"tetracycline"!wasn't!in!the!answer!
Medscape:!https://emedicine.medscape.com/article/10714299medication#3!
First!Aid:!treat!with!low9potency!corticosteroids!or!topical!metronidazole.!In!more!severe!disease,!
systemic!antibiotics!may!be!used.!Extremely!severe!cases!can!be!treated!by!short9term!
metronidazole.!
!!
!!! 9.!!Necrotising!faschitis?!
Answer:!pipra/tazo!+!clinda!+!vancomycin,!u!can!use!carbapenem!instead!of!pipra/tazo!
!!
!!
!! 10.!UTI,!with!methicillen!sensitive,!treatment?!
!!!!!!!!! Answer:!cloxacillin!Rx!
!!
!! !11.!Clostridium!difficile!Rx?!
!!!!!!!!! Answer:!metro,!if!no!metronidazole!à!vancomycin!
First!Aid:!Cessation!of!inciting!antibiotic!if!any,!PO!Metronidazole!or!PO!Vancomycin;!IV!
metronidazole!if!the!pt!can’t!tolerate!oral!medications.!
!!
!! !12.!Animal!bite!Rx?!
Answer:!amox/clavulanate!+!tetanus!booster!
!!
!!! 13.!Best!antibiotic!for!gram!5ve!bacilli?!
Answer:!cephalosporin!
!!
!! 14.!Antibiotic!inhibits!Ca!and!aluminium!salt?!
Answer:!tetracycline!
!!
!! 15.!Antibiotic!safe!in!pregnancy?!
Answer:!ampicillin,!penicillin,!ampi/sulbactam,!cefoxitin,!cefotetan,!cefazolin,!clinda,!erythro,!
gentamicin,!
!!
! !16.!Antibiotic!for!UTI,!safe!in!all!trimesters?!
Answer:!ampicillin!
!!
!! 17.!Rheumatic!heart!disease,!allergic!to!penicillin,!prophylactic!before!surgery?!
Answer:!IV!vancomycin!and!gentamycin!
!!
!! 18.!Two!drugs!contraindicated!with!each!others?!
Answer:!tetracycline!and!aluminum!
!!
!!! 19.!Mastoditis,!treatment?!
Answer:!vancomycin!with!ceftriaxone!
!!
!!! 20.!Nisseria!infection!risk?!
Answer:!defect!in!classical!complement!pathway!
!!
!!!!!!21.!Tenia!solium,!found!in!meat!of?!
Answer:!pork!
!!
! !!22.!Tenia!saginata,!found!in!meat!of?!
Answer:!beef!
!!
!!
!!
!! 23.!Chronic!granulomatous!disease?!
Answer:!recurrent!infection!with!staph,!aspergillus,!nocardia,!serratia!and!Burkholderia.!
!!
!! 24.!Patient!comes!from!Africa!and!has!fever,!Dx?!
Answer:!depends!on!incubation!period:!
within!one!week!(7!days):!yellow,!dengue!murburg!fever!
within!3!weeks!to!1!month:!ebola,!lassa!fever,!malaria.!
!!
!!!!25.!Leshmania!treatment?!
Answer:!Miltefosine!
!!
!!!!26.!Vibrio!cholera,!treatment?!
Answer:!Doxycyclin!"30S!protein"!
!!
! 27.!Schistosomia,!treatment?!
Answer:!Praziquantel!
!!
!!!!28.!Inhaled!antiviral!works!by?!
Answer:!neuraminidase!inhibitor!
!!
! 29.!Parasite!infection!in!undercooked!food?!
Answer:!trichinellosis!
!!
!!!!30.!Bacteria!sexual!behavior?!
Answer:!conjugation!
!!
!!!!31.!GAS!throat!infection?!
Answer:!Amoxicillin,!or!erythromycin!if!allergic!to!penicillin!
!!
!!!!32.!Interstitial!lung!disease!with!small!non!necrotizing!granuloma?!
Answer:!hypersensitive!pneumonitis!
!!
!!!!33.!Septic!arthritis!resistant!to!antibiotic?!
Answer:!start!vancomycin!
!!
!!!34.!Convulsion!after!GI!infection?!
Answer:!shigella!
!!
!!!35.!Chemotherapy,!infection!from!venous!line?!
Answer:!pseudomonas!
!!
!!!36.!Streptococcal!throat!infection,!Rx!duration?!
Answer:!!10!days!
!!
!!
!!37.!Ribavirin!Side!effects?!
Answer:!Anemia!
Medscape:!Decreased!Hgb!(25936%)!
!!
!!38.!Anti!TB,!causing!seizure?!
Answer:!isoniazid!
!!
!!39.!Anti!TB,!causing!parasthesia!or!nerve!problem?!
Answer:!!isoniazid!
!!
!!!40.!Anti!TB,!causing!visual!problem?!
Answer:!Ethambutol!
!!
!!!41.!Anti!TB,!causing!vertigo!or!ear!problem?!
Answer:!streptomycin!
!!
!!!42.!Anti!TB,!causing!red!urine?!
Answer:!Rifampicin!
!!
!!!!43.!Farmer!with!sandfly!infection,!what’s!the!treatment?!
Answer:!miltefosine!!(Dx!leishmania)!
Medscape:!https://emedicine.medscape.com/article/2202989medication#2!
!!
!!!!44.!HIV!with!pneumonia,!bubble!soap?!
Answer:!cryptococcus!neoformans!
!!
!!!!45.!RSV!virus!can!be!found!in?!
Answer:!Liver!
!!
! 46.!3!to!4!scenarios!about!TB!(all!were!direct!and!clear)!
!!
!!!!47.!PPD!positive!x!ray!negative,!next?!
Answer:!!isoniazid!for!6!months!
!!
!!!!48.!A!pt!on!anti!TB!drugs!for!4!weeks!developed!needle!sensations?!
Answer:!!isoniazid!
!!
! 49.!Patient!with!recurrent!oral!fungal!infection!and!other!things?!
Answer:!Chronic!mucocutanous!candidiasis!
!!
50.!Patient!was!in!trip!in!Africa!and!had!many!symptoms.!(one!of!it!was!salamon! blenching!
something!in!the!skin).!What!is!the!investigation?!
1.' bone!marrow!
2.' blood!film!
Answer:!
Medscape:!
(Dx):!At!approximately!the!end!of!the!first!week!of!illness,!the!fever!plateaus!at!1039104°F!(39940°C).!
The!patient!develops!rose!spots,!which!are!salmon9colored,!blanching,!truncal,!maculopapules!
usually!194!cm!wide!and!fewer!than!5!in!number;!these!generally!resolve!within!295!days.![2]!These!
are!bacterial!emboli!to!the!dermis!and!occasionally!develop!in!persons!with!shigellosis!or!
nontyphoidal!salmonellosis.![27]!
(Invx):!The!diagnosis!of!typhoid!fever!(enteric!fever)!is!primarily!clinical.!
The!criterion!standard!for!diagnosis!of!typhoid!fever!has!long!been!culture!isolation!of!the!organism.!
Cultures!are!widely!considered!100%!specific.!
Culture!of!bone!marrow!aspirate!is!90%!sensitive!until!at!least!5!days!after!commencement!of!
antibiotics.!However,!this!technique!is!extremely!painful,!which!may!outweigh!its!benefit.![36]!
!!
!!
51.!Student!with!10!of!his!classmate!develop!dry!cough!mild!SOB!and!bilateral!consolidation?!
1.' leogenialla!!my!answer!
2.' Mycoplasma!??!
3.' Staph!
4.' Strept.!
Answer:!
!!
! 52.!Cat!bite,!what!is!the!most!likely!organism!
1.' Staph!aureus!
2.' Pasturella!multicida!
Answer:!B!
!!
!!!53.!associated!with!animal!bites?!
1.' Polybactrial!
2.' Pastrulla!multicodia!
Medscape:!P!multocida!often!exists!as!a!commensal!in!the!upper!respiratory!tracts!of!many!
livestock,!poultry,!and!domestic!pet!species,!especially!cats!and!dogs.!
!!
!!54.!Bitten!by!his!brother!and!there!is!1!cm!injury,!what!you!do?!
1.' Amoxcillin!
2.' Surgical!suture!
Answer:?!!wound!cleaning!and!wound!closure!and!tetanus!prophylaxis!and!antibiotic.!Surgical!
intervention!is!frequently!necessary!and!ranges!from!simple!wound!exploration!and!debridement!to!
repair!of!complex!structures!under!magnification.!(medscape)!
https://emedicine.medscape.com/article/2189019treatment!
!!
!!55.!Dog!bites!are?!
1.' Poly!microbial!
2.' Viral!
3.' Bacterial!
4.' Fungal!
Answer:!A!
!!
!56.!Gun!shot!wound,!B!fragils.!What!to!give!the!patient?!(no!metronidazole!in!the!choices)!
Answer:!Clindamycin!and!others!
!!
!57.!Patient!with!painless!penile!ulcer!with!lymphadenopathy!what!is!the!treatment?!
1.' Benazthine!penicillins!g!(correct)!
2.' penicillin!V!
Answer:!A!
!!
58.!Child!present!with!malaise,!history!of!meningitis!last!week!treated!with!iv!antibiotics.!Lab;!!
HGb:!low,!RBC:!low,!What!antibiotic!he!used?!
1.' Chloramphinicol!
2.' Erythromycin!
Answer:!A!
Medscape:!one!of!the!side!effects!of!chloramphenicol!is!Aplastic!anemia!<1%!
https://reference.medscape.com/drug/chloramphenicol9iv9chloromycetin9342554#4!
!!
59.!The!most!common!organism!associated!with!honeymoon!cystitis?!
A.!S.!Saprophyticus!
B.!E.!Coli!
Answer:!
Medscape:!!coli!causes!70995%!of!both!upper!and!lower!UTIs.!Various!organisms!are!responsible!for!
the!remainder!of!infections,!including!S!saprophyticus,!Proteusspecies,!Klebsiella!species,!
Enterococcus!faecalis,!other!Enterobacteriaceae,!and!yeast.!Some!species!are!more!common!in!
certain!subgroups,!such!as!Staphylococcus!saprophyticus!in!young!women.!However,!S!
saprophyticus!can!produce!acute!cystitis!in!older!women!and!in!young!men!and!should!not!be!
automatically!regarded!as!a!contaminant!in!the!urine!cultures!of!these!individuals.!
!!
60.!The!most!common!organism!of!meningoenchephalitis!is:!
A.!enterovirus!
B.!HSV!
Medscape:!
Primary!meningoencephalitis!
Ubiquitous!in!most!soils!in!most!environments,!N!fowleri!is!also!found!in!warm!freshwater,!
particularly!if!the!water!is!stagnant.![7]!Exposure!to!the!this!amoeba!is!very!common.!Children!
younger!than!2!years!frequently!carry!the!organism!asymptomatically!in!their!nose!and!throat,!
especially!in!warmer!months!and!climates.!!Infection!with!this!pathogen!occurs!in!both!
immunocompromised!and!immunocompetent!individuals.![8]!
PAM!is!an!exceptionally!uncommon!occurrence!resulting!from!CNS!invasion!of!the!typically!healthy!
host!by!N!fowleri.!During!a!period!of!a!few!days!to!2!weeks!after!inoculating!a!patient!who!had!been!
swimming,!diving,!bathing,!or!playing!in!warm,!usually!stagnant,!freshwater,!the!amoebae!migrate!
through!the!cribriform!plate,!along!the!fila!olfactoria!and!blood!vessels,!and!into!the!anterior!
cerebral!fossae,!where!they!cause!extensive!inflammation,!necrosis,!and!hemorrhage!in!the!brain!
parenchyma!and!meninges.![9]!!!
!!
!!
61.!3!yrs!old!baby!with!fever,!neck!rigidity,!culture!revealed!diplocci/Gm5ve!(They!provided!a!
picture!of!diffuse!meningiococcemia!skin!rash!and!asked!about!what!to!give!to!his!Family?!

Answer:!Oral!rifampicin!

62.!Young!man!diagnosed!with!TB!wt!will!u!give!to!his!asymptomatic!family:!
1.' Bcg!
2.' rifampin!
3.' All!other!answers!were!not!related!at!all!
Answer:!B!
!!
63.!A!male!patient!with!headache,!malaise,!lymphadenopathy!(and!other!non5specific!symptoms).!
He!has!absolute!eosinophilia!on!CBC.!What!is!the!cause?!
A9Schistosoma!
B9Amoeba!
C9Giardia!
Answer:!
!!
64.!What!is!the!treatment!of!epidemic!vibrio!cholerae?!
!Medscape:!
●!!!!!!Furazolidone!has!been!the!agent!routinely!used!in!the!treatment!of!cholera!in!children;!however,!
resistance!has!been!reported,!and!ampicillin,!erythromycin,!and!fluoroquinolones!are!potentially!
effective!alternatives.!The!use!of!quinolones!is!contraindicated!in!children!with!cholera.!
●!!!!!!Travelers!to!cholera9affected!regions!should!receive!a!cholera!vaccine.!The!cholera!vaccine!
Vaxchora!is!the!only!one!approved!by!the!FDA!for!cholera!prevention.!It!is!a!live,!weakened!vaccine!
administered!as!a!single,!oral!liquid!dose!of!about!three!fluid!ounces!at!least!10!days!before!travel!to!
a!cholera9affected!region.!The!only!other!existing!cholera9prevention!vaccines!require!2!doses,!
according!to!the!Centers!for!Disease!Control!and!Prevention!(CDC).!A!single9dose!vaccine!is!
especially!beneficial!to!a!person!who!needs!to!travel!to!a!cholera9affected!region!on!short!notice.!
[20]!
●!!!!!!Emerging!drug!resistance!in!certain!parts!of!the!world!is!a!concern,!as!some!V!cholerae!strains!
contain!plasmids!that!confer!resistance!to!many!antibiotics.!In!areas!of!known!tetracycline!
resistance,!therapeutic!options!include!ciprofloxacin!and!erythromycin.!Strains!resistant!to!
ciprofloxacin!have!been!reported!from!Calcutta,!India.!
!!

65.!About!watery!diarrhea!without!blood!and!asked!about!organism?!
9!I!chose!C.!Defficile!(!Not!sure!)!
First!aid:!
C.!difficile!due!to!recent!Abx!use!(penicillin,!cephalosporin,!clindamycin),!fever,!abd!pain,!possible!
systemic!toxicity,!fecal!RBCs!and!WBCs,!most!commonly!in!large!bowel,!but!can!involve!small!bowel.!
Identify!C.!diff!toxins!in!stool.!Sigmoidoscopy!showed!pseudomembranes.!
!!
66.!Scenario!about!female!sexually!active!and!came!with!symptom!I!forgot,!in!lab!results!he!
mentioned!gram!negative!diplococci!and!asked!about!the!diagnosis?!
!ANSWER:!N.!Gonorrhea!
!!
67.!Scenario!typical!of!HIV!with!PCP!infection,!CMV!retinitis,!oral!thrush.!Then!asked!what!will!you!
find?!
CD!4!of!100!
ANSWER:!??!
!!
68.!Hiv!pt!before!ttt!you!must!check?!
ANSWER:!CD4!
!Medscape:!Research!data!led!to!US!guidelines!recommending!that!antiretroviral!therapy!be!initiated!
at!a!CD4!count!threshold!of!350/μL,!although!2013!guidelines!from!the!World!Health!Organization!
(WHO)!now!recommend!a!threshold!of!500/μL.![108,!109]They!also!state!that!in!some!cases,!
antiretroviral!treatment!should!begin!immediately,!regardless!of!the!CD4!count,!including!in!HIV9
positive!serodiscordant!couples,!patients!with!hepatitis9B!coinfection,!pregnant!or!breastfeeding!
women,!and!children!under!age!5!years.![108,!109]!
!!
69.!Vaccine!contraindicated!in!Hiv!pt?!
!ANSWER:!varicella!vaccine!(All!live!vaccines)!
!!
!!
70.!IV!drug!abuser!complain!of!oral!thrush,!signs!and!symptoms!of!pneumonia!,!broncho!alveolar!
lavage!was!done!pneumocysticitis!jiroveci!was!found!then!HIV!test!was!done!and!it!was!+ve!,!what!
is!the!most!likely!predictor!of!her!HIV!infection?!
1.' Pneumocysticitis!jiroveci*!
2.' IV!drug!use!
3.' Candida!
!ANSWER:!A!?!
!!
71.!HIV!patint,!cough!with!bad!odour!sputum,!x!ray!attached!what!is!diagnosis?!
Answer:!PCP!pneumonia!
!!
!!
!!
72.!Latency!period!in!HIV,cell!responsible?!
ANSWER:!CD4+!T!cells!with!a!memory!
!!
73.!patient!with!HIV!and!ask!about!something!I!forget!^_^!but!the!choices!was!:!
1.' Memory!B!cell!
2.' CD!8!T!cell!
3.' Macrophage!
Answer:!
!!
!74.!Co5receptor!of!HIV!?!
ANSWER:!CXCR4!(Not!sure)!
!!
75.!Man!have!one!month!sexual!relation!result!of!HIV!was!negative!when!will!repeat!the!screen!?!
1.' 2!mo!
2.' 3!mo!
3.' 5!mo!
4.' 6!mo!
Answer:!
76.!(652!Qs)!about!alternative!cell!reservoir!for!HIV!(CD!4)!?!
Is!it!CD!8!?!!I!don’t!know!
Monocytes,!other!T!cells,!macrophages,!adipose!tissue,!GALT,!genital!tract,!semen,!bone!marrow,!
and!central!nervous!system!(CNS)!cells!including!both!microglia!and!astrocytes!
!!
77.!16!yrs!old!drug!abuser,!what!invx!should!be!done!to!him?!
1.' Test!for!HIV,!I!chose!this!
2.' Test!of!HBV!
ANSWER:!A!

78.!Case!of!positive!PPD,!no!symptom,!clear!x!ray,!how!to!manage!?!(Nurse!with!10!mm!PPD!test,!
CXR!negative,!what!to!do)!
1.' Isonazide!6!months!
2.' Rifam!6!m!
ANSWER:!A?!
●!!!!!!69month!or!99month!isoniazid!daily,!
●!!!!!!39month!rifapentine!plus!isoniazid!weekly,!
●!!!!!!39!or!49month!isoniazid!plus!rifampicin!daily,!
●!!!!!!3–!or!49month!rifampicin!alone!daily.!
!!
79.!Old!pt!take!antiviral!that!taken!by!inhalation?!
ANSWER:!Zanamivir!
!!
80.!Man!travels!to!Sudan!2!weeks!ago,!now!he!is!presenting!with!fever,!maleas!....!unspecific!
symptoms!(from!3!days).!How!you!will!confirm!diagnosis??!
A)!blood!culture!
B)!Serology!!
C)!Blood!film!
ANSWER:!I!would!choose!C?!
https://emedicine.medscape.com/article/2211349workup#c8!
!!
81.!Pt!with!treated!meningiococal!meningitis!what!to!give!her!close!contacts!
Penicillin!or!similar!abx!to!remove!nasal!carriers**!
1.' Isolate!contacts!
2.' Do!nothing!
3.' Give!vaccine!
!ANSWER:!
First!aid:!close!contact!should!receive!rifampin,!ciprofloxacin!or!ceftriaxone!prophylaxis.!
!!
82.!Pt!with!treated!meningiococal!meningitis!what!to!give!her!close!contacts!
1.' Penicillin!or!similar!
2.' (rifampin!or!ciprofloxacin)!
3.' abx!to!remove!nasal!carriers!Isolate!contacts!
4.' Do!nothing!Give!
ANSWER:!B!
!First!aid:!close!contact!should!receive!rifampin,!ciprofloxacin!or!ceftriaxone!prophylaxis.!
!!
83.!Pictures!of!chancre!painless!ulcer!on!the!penis!how!to!treat?!Dx:!primary!syphilis?!
1.' Penicillin!V!
2.' Pencillin!G!
3.' Steroid!
ANSWER:!B!
!!
!!
84.!Scenario!of!patient!from!India!had!rash!hepatomegaly!Treated!and!after1!year!came!with!rash?!
What’s!the!Dx?!
1.' Dermatological!leishmaniasis!
2.' Leprosy!
ANSWER:!A!
!!
85.!Patient!on!chemotherapy!developed!IV!line!sepsis,!(!bacterial!sepsis)!most!likely!organism?!
ANSWER:!Closest!option!was!pseudomonas!(correct)!
!!
86.!Source!of!infection!in!venipuncture?!
ANSWER:!Site!of!insertion!?!
!!
87.!Patient!on!TB!medication!developed!numbness,!what!would!u!give!him?!
ANSWER:!Vitamin!B6!(pyridoxine)!
!!
!!
!!
88.!safe!vaccine!to!be!given!to!immunocompromised!person?!
1.' MMR!
2.' Pnemococcal!
ANSWER:!B!
!!
89.!Gun!shot!in!the!abdomen!with!bacteroid!fragilis!which!antibiotic??!
ANSWER:!Metrodinazole!
!!
90.!What!would!be!helpful!in!diagnosis!of!N.gonorrhoeae!?!
1.' Gram!stain!
2.' Culture!
3.' PCR!
4.' forgot!it!
ANSWER:!B!
Medscape:!Culture!is!the!most!common!diagnostic!test!for!gonorrhea,!followed!by!the!
deoxyribonucleic!acid!(DNA)!probe,!and!then!the!polymerase!chain!reaction!(PCR)!assay!and!ligand!
chain!reaction!(LCR).!The!DNA!probe!is!an!antigen!detection!test!that!uses!a!probe!to!detect!
gonorrhea!DNA!in!specimens.!
!!
91.!HBV!commonest!virus!which!make!rejection!of!the!exparitis!or!labours!from!working!in!Saudi!
?(Community)!
1.' HBV!
2.' HCV!
3.' HIV!
Answer:!A!same!Q!from!smle!13!
ANSWER:!C!
References:!!http://applications.emro.who.int/emhj/v19/07/EMHJ_2013_19_7_664_670.pdf?ua=1!
!!
92.!How!to!diagnose!factitious!fever?!
1.' Blood!culture.!
2.' Urine!analysis.!
3.' Pulse!
ANSWER:!C!
!!
93.!Case!scenario!about!DM!patient!with!necrotizing!fasciitis,!which!antibiotic?!
1.' penicillin/gentamycin!
2.' piperacillin/!tazobactam*!
ANSWER:!B!
!!
94.!How!to!prevent!MERSA?!
1.' Hand!washing!
2.' Vaccine!
ANSWER:!A!
!!
!!
95.!Hepatitis!diagnosed!by?!
1.' Blood!test!(!I!choose!it!)!
2.' Imaging!
3.' Hx!
ANSWER:!A!
!!
96.!Type!of!Hepatitis!B!vaccine?!
!ANSWER:!recombinant!
!!
97.!20!y.!O!male!!with!small!erythmatous!macules!non!planchable!,!history!of!viral!respiratory!
infection!resolve!spontaneously!last!week,!Lab:!plt:!15!(!very!low!),!What!TREATMENT?!
1.' IVIG!
2.' Splenectomy!
3.' Platelets!transfusion!
ANSWER:!A!
!!
98.!side!effect!of!ribavirin?!
1.' Anemia.!
2.' Renal!damage!
3.' Hepatic!damage.!
ANSWER:!A!
!!
99.!How!to!diagnose!enteric!fever!during!the!first!week?!
1.' Urine!and!stool!culture!
2.' Single!blood!culture!
3.' Multiple!blood!cultures!
4.' Bone!marrow!culture!
ANSWER:!
Medscape:!
●!!!!!!The!diagnosis!of!typhoid!fever!(enteric!fever)!is!primarily!clinical.!
●!!!!!!The!criterion!standard!for!diagnosis!of!typhoid!fever!has!long!been!culture!isolation!of!the!
organism.!Cultures!are!widely!considered!100%!specific.!
●!!!!!!Culture!of!bone!marrow!aspirate!is!90%!sensitive!until!at!least!5!days!after!commencement!of!
antibiotics.!However,!this!technique!is!extremely!painful,!which!may!outweigh!its!benefit.!
●!!!!!!Blood,!intestinal!secretions!(vomitus!or!duodenal!aspirate),!and!stool!culture!results!are!positive!
for!S!typhi!in!approximately!85%990%!of!patients!with!typhoid!fever!who!present!within!the!first!
week!of!onset.!They!decline!to!20%930%!later!in!the!disease!course.!In!particular,!stool!culture!may!
be!positive!for!S!typhi!several!days!after!ingestion!of!the!bacteria!secondary!to!inflammation!of!the!
intraluminal!dendritic!cells.!Later!in!the!illness,!stool!culture!results!are!positive!because!of!bacteria!
shed!through!the!gallbladder.!
!!
100.!Enteric!fever!TTT?!
1.' Ciprofloxacin!
2.' Amoxicillin!
3.' Metronidazole!
ANSWER:!A!
!!
102.!Enteric!fever!micro!description!dx!?!Management!?!
!Answer:!
!!
103.!Best!Diagnosis!of!enteric!fever?!
Medscape:!
●!!!!!!The!criterion!standard!for!diagnosis!of!typhoid!fever!has!long!been!culture!isolation!of!the!
organism.!Cultures!are!widely!considered!100%!specific.!
●!!!!!!Culture!of!bone!marrow!aspirate!is!90%!sensitive!until!at!least!5!days!after!commencement!of!
antibiotics.!However,!this!technique!is!extremely!painful,!which!may!outweigh!its!benefit.!
●!!!!!!Blood,!intestinal!secretions!(vomitus!or!duodenal!aspirate),!and!stool!culture!results!are!positive!
for!S!typhi!in!approximately!85%990%!of!patients!with!typhoid!fever!who!present!within!the!first!
week!of!onset.!They!decline!to!20%930%!later!in!the!disease!course.!In!particular,!stool!culture!may!
be!positive!for!S!typhi!several!days!after!ingestion!of!the!bacteria!secondary!to!inflammation!of!the!
intraluminal!dendritic!cells.!Later!in!the!illness,!stool!culture!results!are!positive!because!of!bacteria!
shed!through!the!gallbladder.!
●!!!!!!Multiple!blood!cultures!(>3)!yield!a!sensitivity!of!73%997%.!Large9volume!(10930!mL)!blood!
culture!and!clot!culture!may!increase!the!likelihood!of!detection.![37]!
●!!!!!!Stool!culture!alone!yields!a!sensitivity!of!less!than!50%,!and!urine!culture!alone!is!even!less!
sensitive.!Cultures!of!punch9biopsy!samples!of!rose!spots!reportedly!yield!a!sensitivity!of!63%!and!
may!show!positive!results!even!after!administration!of!antibiotics.!A!single!rectal!swab!culture!upon!
hospital!admission!can!be!expected!to!detect!S!typhi!in!30%940%!of!patients.!S!typhi!has!also!been!
isolated!from!the!cerebrospinal!fluid,!peritoneal!fluid,!mesenteric!lymph!nodes,!resected!intestine,!
pharynx,!tonsils,!abscess,!and!bone,!among!others.!
104.!Diarrhea!(!wedding!a!couple!of!hours!later!had!diarrhea!with!micr!description)?!
ANSWER:!i!don't’!get!it!(????)!
!!
105.!treatment!of!traveler's!diarrhea?!
ANSWER:!Ciprofloxacin!
!!
106.!Gas!gangrene?!
ANSWER:!cl.perfeenges!
!!
107.Central!line!with!fungal!inf.!What!is!the!TTT?!
ANSWER:!Fluconazole!
!!
109.!which!one!of!the!following!infection!diagnosed!by!stool!analysis!by!finding!an!antigen?!
1.' Ascaris!
2.' Tenia!saginata!
3.' Schistosoma!mansoni!
4.' ………………………!
NOTE:!all!of!the!above!infections!are!diagnosed!by!stool!analysis!by!finding!eggs!or!trophozoites.!The!
infections! which! are! diagnosed! by! finding! antigens! are! H.Pylori! and! giardiasis.! SO,! MAY! BE! THE!
MISSING!ANSWER!HERE!IS!THE!CORRECT!CHOICE.!!!
!!
110.!Treatment!of!shigellosis?!
1.' amoxicillin!
2.' ceftriaxone!!
ANSWER:!B!
!!
111.!INFECTIOUS!case!of!GIT!characterized!by!diarrhea!followed!by!constipation!and!give!you!
bacteriology!finding!which!is!gram!negative!and!other!features!I!cannot!remember,!then!asking!
about!treatment:!
1.' Ciprofloxacin!
2.' Chloramphenicol!
3.' Penicillin!
4.' …………………!
Note:!from!my!reading!salmonella!is!characterized!by!diarrhea!followed!by!constipation!
So!my!answer!was!(A)!but!not!sure!about!it.!
ANSWER:!A!(Dx:!S.!typhi)!
!!
112.!What!would!be!helpful!in!diagnosis!of!gonorrhea?!
1.' gram!stain!
2.' culture!
3.' PCR!
4.' forgot!it!
!ANSWER:!B!
!!
113.!What!is!the!parasite!usually!found!in!beef!?!(No!choices!provided)!
ANSWER:!T.!Saginata!i!think!
!!
114.!Patient!with!lymphadenopathy!&!splenomegaly?!
Answer:!EBV!!!
!!
115.!(3!to!4!scenarios)!about!TB!(all!were!direct!and!clear)!
!!
116.!T.diarrhea!>!shegella!(????)!
!!
117.!The!only!organism!from!human!source?!
UpToDate:!

●!!!!!!Human!bite!wound!pathogens!consist!of!both!aerobic!and!anaerobic!bacteria,!including!streptococci,!
Staphylococcus!aureus,!Eikenella,!Fusobacterium,!Peptostreptococcus,!Prevotella,!and!Porphyromonas!spp![19
3].!In!a!study!of!50!patients!with!infected!human!bites,!the!median!number!of!isolates!per!wound!culture!was!
four![3].!Both!aerobes!and!anaerobes!were!isolated!from!54!percent!of!wounds,!aerobes!alone!were!isolated!
from!44!percent,!and!anaerobes!alone!were!isolated!from!2!percent.!

●!!!!!!Viral!pathogens,!including!hepatitis,!human!immunodeficiency!virus,!and!herpes!simplex!virus,!are!
transmissible!by!human!bites;!clinical!descriptions!are!limited!to!case!reports![499].!Human!bite!transmission!of!
syphilis!has!also!been!described!

!!
118.!Most!common!site!for!central!line!infection?!
1.' Seeding!of!bacteria!
2.' Hospital!workers!
3.' Site!of!insertion!
ANSWER:!C!
(Up!to!date)!
!!
119.!!a!pt!end!stage!liver!disease!on!Central!venous!catheter!developed!sepsis!Culture!showed!
budding!yeast,!Suitable!Rx!is?!
1.' fluconazole!
2.' antiviral!
3.' Abx!
4.' cuspofungin!
ANSWER:!D!
!!
120.!The!transmission!of!maternal!antibodies!to!the!fetus!in!pregnancy!is!a!way!of?!
1.' active!artificial!Immunity!
2.' passive!artificial!immunity!
3.' passive!natural!immunity!
4.' active!natural!immunity!
ANSWER:!C!
!!
121.!Post!cholecystectomy!pt,!developed!unilateral!parotid!swelling,!saliva!was!cloudy,!culture!
was!negative,!What!does!he!have?!
1.' sarcoid!granuloma.!
2.' bacterial!sialadenitis!
3.' sarcoma!
4.' sojgrens!syndrome!!
ANSWER:!B!
!!
122.!female!her!husband!had!gonorrhea!what!best!investigation!for!her?!
1.' Gram!stain!
2.' PCR!
3.' I!forgot!others!
ANSWER:!Could!be!A!if!only!those!choices!(not!sure)!
First!aid!page!220!
!!
123.!13!yrs!old!pt!with!salmonella!infection,!Resistant!to!chloramphenicol,!appropriate!Tx!is?!
1.' continue!chloramphenicol!
2.' add!cipro!
3.' give!cipro!alone!
4.' IM!ceftriaxone!
ANSWER:!C!
!!
124.!I!can’t!remember!the!scenario!but!the!answer!was!Rota!Virus!
!!
125.!food!poisoning!case,!4!family!members!ate!from!a!restaurant,!they!developed!diarrhea!and!
vomiting!and!remit!after!24!hrs.!Culture!showed!gram!positive!bacilli?!
1.' salmonella!
2.' shigella!
3.' SA!
4.' Bacillus!ceres!
ANSWER:!D!
!!
125.!Diagnosis!of!visceral!leishmaniasis?!
1.' blood!film!
2.' bone!marrow!
ANSWER:!A!
https://emedicine.medscape.com/article/2202989workup#c7!
!!
126.!scenario!of!man!came!from!desert!and!got!cutaneous!leishmania!what!is!the!treatment!??!
1.' Oral!miltefosine!
2.' Injectable!paromomycin!
3.' There!was!no!amphotrpcin!B!
Answer:!A!
https://emedicine.medscape.com/article/2202989treatment#d9!
!!
127.!Treatment!of!leishmania!donovani?!
●!!!!!!Liposomal!amphotericin!B!alone,!given!as!a!single!dose!(currently!recommended!as!the!drug!of!
choice!by!the!Kala9Azar!elimination!programme!of!India)!
●!!!!!!Liposomal!amphotericin!B!in!a!single!dose,!in!combination!with!7!days!of!oral!miltefosine!or!10!
days!of!paromomycin!
●!!!!!!Miltefosine!plus!paromomycin!for!10!days!
●!!!!!!Amphotericin!B!deoxycholate:!0.7591!mg/kg/day!via!infusion,!daily!or!on!alternate!days!for!15920!
doses!
●!!!!!!Miltefosine!orally!for!28!days!or!paromomycin!intramuscularly!for!28!days!
●!!!!!!Pentavalent!antimonials:!20!mg!Sb5+/kg/day!intramuscularly!or!intravenously!for!30!days!in!
areas!where!they!remain!effective:!Bangladesh,!Nepal,!and!the!Indian!states!of!Jharkhand,!West!
Bengal,!and!Uttar!Pradesh.!
●!!!!!!https://emedicine.medscape.com/article/2202989treatment#d11!
!
128.!Visceral!leishmaniasis!organism?!
A.!L!donovani!
B.!L.Tropica!
ANSWER:!A!
!!
129.!type!of!leishmaniasis!cause!skin!manifestation?!
1.' L.!Tropica!
ANSWER:!A!(L.!Tropica,!L.!Major,!L.!mexicana)!
https://emedicine.medscape.com/article/2202989clinical!
!!
130.!which!markers!of!HBV!is!present!in!window!phase?!
ANSWER:!Anti!HBc!
WiKi:!IgM!anti9core!(HBc9IgM)!is!the!only!detectable!antibody.!HBV!DNA!may!be!positive!as!well.!
!!
131!pictures!of!chancre!painless!ulcer!on!the!penis!how!to!treat?!(Dx):!primary!syphilis?!
1.' penicillin!V!
2.' pencillin!G!and!
3.' steroid!
ANSWER:!B!
!!
133.!patient!has!been!bitten!by!dog,!and!he!received!rabies!vaccine!18!months!ago!when!he!
travelled!to!some!place,!what!action!should!be!taken!immediately?!
1.' Do!nothing!as!patient!is!immunized.!
2.' Give!rabies!vaccine!and!immunoglobulin.!
3.' Give!immunoglobulin!alone.!
4.' Give!two!doses!of!rabies!vaccine.!
ANSWER:!D!(Correct)!
●!!!!!!Note:!this!q!is!solved!before!in!one!of!the!questions!collection!as!B,!but!what!I!found!that!answer!
D!is!the!correct!choice.!There!are!big!details!about!this!topic.!So!you!do!not!need!to!waste!your!time.!
(previous!note)!
●!!!!!!“If!bitten,!a!vaccinated!person!should!receive!two!more!doses!of!rabies!vaccine;!one!dose!
immediately!and!one!three!days!later.”!http://www.immunize.org/catg.d/p4216.pdf!
●!!!!!!Previously!vaccinated!persons!include!those!who!have!received!the!39dose!preexposure!series!of!
HDCV,!rabies!adsorbed!virus!(RVA),!or!PCECV;!a!full!PEP;!or!a!previous!vaccination!with!any!rabies!
vaccine!with!a!documented!history!of!seroconversion.!HRIG!should!not!be!administered.!
●!!!!!!For!the!vaccine,!administer!2!doses!(1!mL!each)!into!the!deltoid!muscle!on!day!0!and!day!3.!
https://emedicine.medscape.com/article/7855439overview#a8!
134.!Patient!was!bitten!by!a!dog,!he!was!conscious,!alert.!Everything!was!normal.!He!has!a!bite!
wound!in!his!hand.!The!patient!told!you!that!he!took!rabies!vaccine!before!one!year!and!half.!
What!should!you!do?!
A.!!!!!!!!!! give!rabies!vaccine!
B.!!!!!!!!!! Give!two!dose!of!rabies!vaccine!
C.!!!!!!!!!! Observe!for!10!days!
D.!!!!!!!!!! Rabies!immuniglublin!and!vaccine!
ANSWER:!Previous!answer:!D,!but!the!answer!is!B!(according!to!medscape)!as!the!previous!Q.!
explanation.!
!!
135.!Most!common!cutaneous!manifestation!of!antimalarial!medications?!
A9Pruritus!
B9Pigmenation!
C9Photosensitivity!
D9Generalized!yellow!discoloration!of!skin!
ANSWER:!(Chloroquine!A)!+!(tetracycline!D)!Medscape!
!!
136.!Picture!and!they!asked!which!type!of!Malaria?!
ANSWER:!Plasmodium!falciparum!(depend!on!the!picture)!
!!
137.!What!of!the!following!use!scotch!tap!in!diagnosis?!
A.!!!!Giardia!
B.!!!!Malaria!
C.!!!!Schistosomiasis!
ANSWER:!Pinworm!(Enterobiasis)!Medscape!
https://emedicine.medscape.com/article/2256529workup!
!!
!!
138.!Patient!will!go!to!endemic!area!of!malaria!(I!think!Sudan)!and!he!asked!you!about!prophylaxis!
for!malaria?!
A.!!!!Quinine!1week!before!traveling,!until!6!week!after!returning.!
B.!!!!Quinolone!2!days!before!traveling,!until!returned.!
Answer:!
!!
139.!Treatment!of!uncomplicated!Falciparum!malaria?!
!!
140.!Pt!came!from!sudan!I!guess!and!they!suspected!malaria!wt!of!these!inv!most!specific!and!
sensitive?!
A.!!!!Malaria!rapid!test!
B.!!!!Malaria!antibodies!!!!!!!!!!!!!!!!!!!!!!!!!!!!!!!!!!!!!!!!!!!!!!!!!!!! !
C.!!!!Blood!film!
Answer:!
!!
141.!Picture!of!ring!cell!stage!of!malaria.!Asked!the!stage!and!malaria!type.?!

142.!EBV!treated!by?!
1.' Paracetamol!and!rest!
2.' Acyclovir,!
3.' Amoxicillin!
Answer:!
!!
143.!Patient!complain!of!hand!pain!after!bee!sting,!on!examination:!tenderness!+!Axillary!lymph!
node!enlarged,!(!+!picture:!weal!on!forearm)!what!is!dx?!
1.' 9!URTECARIA!
2.' 9!Lypmhangitis!
ANSWER:!B!
!!
144.!Male!want!to!take!his!son!to!south!Asia.!Prophylaxis!for!travelers’!diarrhea?!
1.' Ciprofluxacine!
2.' Ceftrexione!
3.' Doxycycline!
!ANSWER:!A!
!!
145.!how!to!diagnose!factitious!fever?!
A.!!!!!!!!!! Blood!culture!
B.!!!!!!!!!! Urine!analysis!
C.!!!!!!!!!! Heart!rate!
D.!!!!!!!!!! Rheumatoid!factor!
ANSWER:!C!
!!
!146.!patient!who!is!an!IV!drug!abuser!,!on!a!blood!smear!it!was!found!out!that!he!
developed!an!RNA!virus!of!a!Flaviviridae!family!,!the!Virus!is?!
1.' HBV!
2.' HCV!
3.' HDV!
4.' HAV!
!ANSWER:!B!
!!
!!
!!
!!
!!
!147.!Pt!with!4!recurrent!UTI,!what!is!the!dose!and!duration!of!nitrofurantoin!for!this!pt?!
1.' BID!for!3!months!
2.' TID!for!3!months!
3.' Daily!for!6!months!
Answer:!

!!

148.!UTI!treated!with!100mg!nitrofurantoin!what!is!the!duration?!
A)!6h!for!one!month.!
B)once!daily!for!2!month!
C)once!daily!for!six!month!
D)!once!daily!for!one!year.!
ANSWER:!A!
!!
Q.!dose!of!nitrofurantoin?!
!MEDSCAPE:!25mg,!50mg,!100mg!
https://reference.medscape.com/drug/macrobid9macrodantin9nitrofurantoin9342567!!
!!
149.!UTI!case!with!gram!negative!indole!positive?!
A)!E.!coli!
ANSWER:!A!
!!
150.!Patient!is!allergic!to!sulfa!drugs!and!penicillin!and!shellfish.!She!has!UTI!what!antibiotic!you!
will!give?!
A)Nitrofurantoin!!
B)Trimethoprim!Sulfamethoxazole!
C)Amoxicillin!
!ANSWER:!A!(not!sure)!
!!
151.!treatment!of!HSV!2?!
A)!acyclovir!
B)!ribavirin!
!ANSWER:!A!
!!
152.!Syphilis:!Painless!Genital!ulcer!plus!inguinal!lymphadenopathy?!NOT!COMPLETE!
!!
153.!Pt!with!painless!Genital!ulcer,!how!to!diagnose?!
ANSWER:!dark!field!microscopy!
!!
154.!Painful!genital!ulcer,!bleeding,!lab!confirmed!Haemophilus!Ducreyi?!
a)treat!all!close!contacts!
b)treat!all!sexual!partners!
c)treat!symptomatic!sexual!parteners!only!
!ANSWER:!B!
http://emedicine.medscape.com/article/2147379treatment!
!!
!!
!!
!!
156.!Patient!came!with!fatigue,!weight!loss,!and!diarrhea.!He!received!a!blood!transfusion!when!
he!was!in!kenya.!He!has!low!grade!fever,!vitals!are!stable,!skin!ex.!There!is!contagious!molluscum!
in!groin!and!generalized!lymphadenopathy!and!palpable!liver,!diagnosis?!
a)secondry!syphillis!
b)Persistent!chronic!hep!B!
c)HIV!
d)Acute!lymphoma!
ANSWER:!C!
!!
157.!IV!drug!abuser,!what!is!the!most!important!to!test!for!>!HIV!or!HBV?!
ANSWER:!HIV!(?)!
!!
158.!what!is!the!most!common!cause!of!candida!infection?!
ANSWER:!candida!albicans,!there!was!other!choices!of!candida!I!can't!remember!
!!
159.!Human!bite!..6!m(months!?)!prior!tetanus!..what!to!give!..!
a)reassurance!
b)booster!tetanus!
c)suturing!
d)amoxicillin!clavulanic!
MEDSCAPE:!Although!rare,!human!bites!have!been!shown!to!transmit!Clostridium!tetani.Assess!all!
patients!for!tetanus!immune!status!and!update!as!appropriate.!According!to!the!recommendations!
of!the!US!Centers!for!Disease!Control!and!Prevention!(CDC),![13]!tetanus!immune!globulin!and!the!39
dose!vaccine!series!should!be!administered!to!patients!with!an!unknown!tetanus!vaccine!history!or!
those!who!have!received!fewer!than!3!doses.!It!is!also!indicated!for!patients!who!received!the!
complete!tetanus!series,!but!whose!booster!administration!was!more!than!5!years!ago.!For!patients!
with!a!history!of!3!or!more!doses!of!tetanus!and!diphtheria!vaccine!who!received!a!booster!less!than!
5!years!ago,!no!tetanus!booster!is!required.!
!!
160.!Pt!has!fever!and!neck!rigidity,!he!was!found!to!have!a!very!contagious!bacterial!disease.!What!
will!you!give!his!close!contacts?!
a)Prednisolone!
b)Acyclovir!
c)Loratidine!
d)rifampicin!
ANSWER:!D!
!!
161.!prevention!of!brucellosis?!
ANSWER:!Pasteurization!of!milk!
!!
162!Gram!5ve,!lactose!non5ferminting!oxidase!+ve?!
ANSWER:!Pseudomonas!
!!
163.!Pic!of!lobar!pneumonia:!
ANSWER:!You!will!hear!bronchial!breathing!
!!
164.!Question!about!OPV!and!IPV?!
1.' Both!has!serum!antibodies!
2.' Both!prevent!virus!to!enter!the!GI!tract!
3.' Other!options!(not!sure!about)!
ANSWER:!A!
!!
165.!patient!came!complains!of!fever!and!a!sore!throat!on!examination!there!was!exudative!
tonsils!and!posterior!cervical!lymphadenopathy,!the!patient!also!had!mild!splenomegaly.!Tests!
showed!positive!EBV!antibodies!what!does!the!patient!has?!
1.' pharyngitis!
2.' infectious!mononucleosis!
ANSWER:!B!
Source:!medscape!
!!
167.!DM!heavy!smoker!with!whitish!patches!in!the!mouth?!
ANSWER:!Candida!
!!
168.!Bacteria!in!which!human!is!the!reservoir!and!the!infection!is!acquired?!
Answer:!!!
!!
169.!What!is!the!cause!of!pseudomembranous!colitis?!
1.' Bacterial!
2.' Idiopathic!
3.' ……….!
4.' ……...!
Answer:!
Pseudomembranous!colitis!refers!to!swelling!or!inflammation!of!the!large!intestine!(colon)!due!to!an!
overgrowth!of!Clostridium!difficile!(C!difficile)!bacteria.!
Q.!Q!about!lyme!disease.!
!!
170.!Diabetic!old!man,!using!full!dentures!has!white!oral!patches,!tx?!
ANSWER:!Miconazole!(candida)!
!!
171.!Hepatitis!B!chart!with!antigens!and!antibodies.!The!question!is!about!the!window!period?!
ANSWER:!ANTI!HBc!i!think!
!!
172.!Group!of!people!got!diarrhea!after!they!ate!from!outside?!
Two!choices!were!organisms!cause!food!poisoning.!!(They!might!want!the!most!common)!
A.!!!!salmonella!
B.!!!!clostridium!perfringens!
Answer:!
!!
! !
! !
173.!You!started!patient!on!Penicillin.!The!sensitivity!test!came!back!with!cefozlin!resistant.!
What!will!you!do!next?!
A.!!!!!!!!!! Continue!the!same!
B.!!!!!!!!!! Shift!the!patient!to!Vancomycin!
!Answer:!b!
!!
174.!Giardia!micro!description!what!is!the!dx?!!
!Don’t!get!it!?!
!!
175.!CSF!normal!glucose!high!protein!(Not!complete)!
A.!!!!!!!!!! Viral!
B.!!!!!!!!!! TB!
Answer:!
!!
176.!Organisms!of!aseptic!meningitis!(Entroviral,,,,)!
!!
177.!Managment!of!meningitis!(Look!at!the!age)!
!!
178.!HBc!antibody!+!The!remaining!is!negative:!
A.!!!!Chronic!
B.!!!!Acute!
C.!!!!Booster!
D.!!!!Vaccination!
ANSWER:!recovering!from!acute!infection.!
!!
179.!What’s!the!most!common!infection!done!in!pre!employment?!
A.!HAV!
B.!HBV!
C.!HIV!
ANSWER:!C!
!!
180.!case!of!streptococcal!pharyngitis!,treatment!?!
1.' Aspirin!
2.' Penicillin!
ANSWER:!B!
!!
181.!student!e!hx!of!sore!throat,headache,!mild!cough,!low!grade!fever,!chest!pain,tired,general!
malaise?!
A.!!!!Pneumoccoci,!
B.!!!!Mycoplasma,!
C.!!!!aspiration!pneumonia,!
D.!!!!Legionella!
Answer:!
!!
182.!Child!with!abdominal!pain,!diarrhea,!burning!urination.!UA!showed!+ve!nitrates!+ve!
leukoestrase!+ve!protein.!How!to!treat?!
A.!!!!suprax!for!14!days!
B.!!!!TMX9SMP!for!4!days!
C.!!!!cephalosporins!1!high!dose!
D.!!!!Amoxicillin!
!!
183.!Gram!negative,!non!lactose,!sulfa!producing.!Tx?!
A.!!!!Cirprofloxacin!
B.!!!!Ceftriaxon!
C.!!!!Chlom!
ANSWER:!A!(NOT!SURE)!
!!
184.!History!of!travel!to!Africa,!came!back!to!myalgia!arthralgia.!Vitals!were!normal!(no!fever),!
what!is!the!most!likely?!
A.!!!!Ebola!
B.!!!!Yellow!fever!!!!!!!!!!!!!!!!!!!!!!!!!!!!!!!!!!!!!!!!!!!!!!!!! !
C.!!!!Lassa!fever!Chikungunya!!!!!!!!!!!!!!!!!!!!!!!!!!!!!!!!!!!!!!!!!!!!!!!!!!!!
Answer!:most!likely!Yellow!fever,!however!it!is!INCOMPLETE!
!!
185.!pt!has!urine!culture!MRSA!he!is!on!antibiotic!after!few!day!develop!redness!on!face,!neck?!
what!is!antibiotic?!
19pencillan!
29vancomycine!
ANSWER:!B!(REDMAN!SYNDROME)!
!!!!!!!!!!!!!!!!!!!!!!!!!!!!!!!!!!!!!!!!!!!!!!!!!!!!!!!!!!!!!!!!!!!!!!!!!!!!!!!!!!!!!!!!!!!!! !
186.!Which!one!of!these!vaccines!taken!by!intranasal!route?!!!
A.!!!!Zanamavir!
B.!!!!Oseltamivir!
ANSWER:!A!
!!!!!!!!!!!!!!!!!!!!!!!!!!!!!!!!! !
187.!Picture!(!look!like!vesicle!)!start!as!1!only!then!spread!to!arm!legs!and!..!with!lymph5node!
enlargement!?!
a.!!!!!herps!simplex!virus!
b.!!!!!Dermatitis!herpetic!form!!!!!!!!!!!!!!!!!!!!!!!!!!!!!!!!!!!!!!!!!!!!!!!!!!!!!! !
c.!!!!!Varicella!zoster!!!!!!!!!!!!!!!!!!!! !
INCOMPLETE!Q,!BUT!Verecila!zoster!is!the!closest!since!others!are!blistering!dx.!!!!!!!! !
!!!!!!!!!!!!!!!!!!!!!!!!!!!!!!!!!!!!!!!!!!!!!!!!!!!!!!!!!!!!!!!!!!!!!!!!!!!!!!!!!!!!!!!!!!!!!!!!!!!!!!!!! !
188.!50!or!40!years!female!1!day!after!chemotherapy!and!broad!spectrum!antibiotics!developed!
painful!vesicular!rash!in!the!breast?!
A9!rubella!
B9!measles!!!!!!!!!!!!!!!!!!!!!!!!!!!!!!!!!!!!!!!!!!!!!!!!!!! !
C9!varicella!zoster!!! !
Answer:!!!!!!!!!!!!!!!!!!!!!!!!!!!!!!!! !
https://www.ncbi.nlm.nih.gov/pmc/articles/PMC3447017/!
!!!!!!!!!!!!!!!!!!!!!!!!!!!!!!!!!!!!!!!!!!!!! !
!!!!!!!!!!!!!!!!!!!!!!!!!!!!!!!!!!!!!!!!!!!!!!!!!!!!!!!!! !
189.!30!y/o!lady!presents!with!productive!cough,!chest!pain!and!fever!for!the!last!one!week.!O/E!
decreased!air!entry!on!the!right!side,!presence!of!bronchial!breathing.!X5ray!revealed!a!wide!
opacity!on!the!right!side.!Culture!was!methicllin5sensetive,!which!of!the!following!is!the!most!
suitable!drug!for!her?!!!!!!!!!!!!!!!!!!!!!!!!!!!!!!!!!!!!!!!! !
A)!Amoxcicillin.!
!B)!Cloxacillin.!
C)!Pipercillin.!
!D)!Penicillin!G.!
ANSWER:!https://emedicine.medscape.com/article/2342409overview!
!!
190.!Patient!has!lung!infection!taking!antibiotic!then!patient!develop!flushing!in!face!and!hand!
then!came!to!doctor!culture!show!methicillin!resistant!gram!+!in!cluster!
Which!antibiotic!he!took?!
A.!!!!Vanco!
B.!!!!Cephalo!!!!!!!!!
ANSWER:!A!
!!
191.!Human!source!infection?!
A)!salmonella !
B)!E.!Histolytica !
NOT!COMPLETE!
!!
192.!Most!specific!for!AIDS?!
A)night!sweat !
B)!generalized!LAP!
C)!opportunistic!infections !
ANSWER:!B!
!!
193.!Patient!with!hx!of!tick!!!classic"!bull's5eye!rash:!Lyme!yme!disease!also!called!erythema!
migrans!…!NOT!COMPLETE!
Lyme!borreliosis,!is!an!infectious!disease!caused!by!bacteria!of!the!Borrelia!type!
!fever,!headache!and!feeling!tired.[1]!If!untreated,!symptoms!may!include!loss!of!the!ability!to!move!
one!or!both!sides!of!the!face,!joint!pains,!severe!headaches!with!neck!stiffness,!or!heart!palpitation.!
Tx:!early!:!erythromycin,!advanced(!CNS,arthritis)!ceftriaxone!.!
!!!!!!!!!!!!!!!!!!!!!!!!!!!!!!!!! !
!!!!!!!!!!!!!!!!!!!!!!!!!!!!!!!!!!!!!!!!!!!!! !

Allergy!and!immunology!
!!
!
1.!What!drug!causes!angioedema!as!a!side!effect?!
A9!CCB!
B9!ACEIs!
2.!Diarrhea!after!fruit!juice!
A9!fructose!allergy!
Answer:!ACEI!
!!
3.!Bloating!and!gases!after!eating!ice!cream!and!dairy!product.!Which!enzyme!is!deficient?!
A9!Lactase!
B9!Pancreatic!alpha!amylase!
C9!Sacrase!
D9!Isomaltose!
Answer:!Lactase!
!!
4.!What!is!the!first!immune!responsible!agent!in!the!skin?!
A9!Keratinocyte!
!!
5.!Patient!with!tachycardia!and!SOB!after!bee!sting!what!is!the!optimal!treatment?!
A9!Remove!stinger!and!watch!the!patient!
B9!SC!epinephrine!and!IV!antihistamine!
Answer:!SC!epinephrine!and!IV!antihistamine!
!!
6.!Patient!with!multiple!infections;!TB,!staph!...!etc.!He!has!immune!defect!in?!
A9!NADPH!oxidase!
CGD!
!!
7.!The!transmission!of!maternal!antibodies!to!the!fetus!in!pregnancy!is!a!way!of?!
A9!Active!artificial!Immunity !
B9!Passive!artificial!immunity!
C9!Passive!natural!immunity!
D9!Active!natural!immunity!
!Answer:!Passive!natural!immunity!
!!
8.!Which!of!the!following!tends!to!cause!a!severe!disease!in!immunocompromised!individuals!
compared!to!immunocompetent?!
A9!Cryptosporidium!
!!
9.!Patient!travelled!to!Africa!where!TB!bovine!is!endemic!there,!what!prevent!him!from!receiving!
BCG!vaccine,!deficiency!in!what?!
A9!IFN!gamma!
B9!IFN!alpha!
C9!IL!4!
D9!IFN!beta!
!Answer:IFN!gamma!
!!
10.!Female!patient!when!she!ate!fish!she!complained!of!mild!redness!in!limited!area!for!three!
hours!+!diarrhea!which!resolved!after!that,!what!she!has?!
A9!Limited!anaphylaxis!
!!
11.!What!is!the!biochemical!defect!in!x!linked!agammaglobulinemia?!
!Answer:!Mutation!in!Bruton’s!tyrosine!kinase!inhibitor!that!is!essential!for!B!cell!function!
!!
12.!Women!had!insect!bite!then!developed!erythema!and!LN!enlargement!(pic!of!forearm!with!
linear!erythema).!Lab!value!=!increase!WBC.!What!the!diagnosis?!
!Answer:!Lymphangitis!
!!
13.!Primary!immunodeficiency!that!expose!the!patient!to!recurrent!of!viral!and!mold!infections!
A9!T!cell!
B9!B!cell!!!!!!!!!!!!!!!!!!!!!!!!!!!!!!!!!!!!!!!!!!!!!!!!!!!!!!
C9!Complement!deficiency!
Answer:!T!cell!!
!!
14.!Old!patient!had!packed!RBC!transfusion!then!developed!fever,!chills,!irritation!at!transfusion!
site!once!it!started,!what!to!do?!
Answer:!Needs!more!details!but!if!nonhemolytic!febrile!reaction!
!!
Management:!rule!out!hemolytic!reaction!or!infection!
!if!temperature!<38ºC,!continue!with!transfusion!but!decrease!rate!and!give!antipyretics!
!if!temperature!>38ºC,!stop!transfusion,!give!antipyretics!and!anti9histamine!!
!!
15.!Lip!swelling!for!3!years!deficiency!in!which!enzyme?!
A9!C1!esterase!inhibitor!
!!
16.!Rhesus!autoimmunity!in!pregnancy.!What!type!of!hypersensitivity?!
A9!Hypersensitivity!type!2!
!!
17.!CD40L!defect!
A9!Hyper!IgM!syndrome!
!!
17.!A!question!about!x5linked!immunodeficiency!
!!
18.!Question!about!RAST!test!measure!??!!!!!!!!!!!!!!!!!!!!!!!!!!!!!!!!!!!!!!!!!!!!!!!!!!!!!!!!!!!!!!!!!!!!!!!!!!!!!!!!!!!!!! !
A!radioallergosorbent!test!(RAST)!is!a!blood!test!using!radioimmunoassay!test!to!detect!specific!IgE!
antibodies,!to!determine!the!substances!a!subject!is!allergic!to.!This!is!different!from!a!skin!allergy!
test,!which!determines!allergy!by!the!reaction!of!a!person's!skin!to!different!substances.!!!!!!! !
!!
19.!What!is!the!immunoglobulin!responsible!in!HSP!disease?!!!!!!!!!!!!!!!!!!!!!!!!!!!!!! !
A9!IgG!
B9!IgM!
C9!IgA!
D9!IgE!
Answer:!IgA!
!!
20.!!Allergic!to!dust,!what!is!the!test!to!confirm!asthma?!!!!!!!!!!!!!!!!!!!!!!!!!!!!
A9!esinophile!something!
!Answer:!Skin!prick!test!or!Specific!IgE!testing!
!
!
!

Endocrinology!
!
! !
1.Clear!Case!with!lab!results!show:!
Low!K!,!hight!blood!preusser!(Case!of!hyperaldosteronism).!How!to!diagnose?!
A.!!Renin9!aldosterone!ratio!
B.!!17hydroxyprogesterone!
Answer:!A!
!https://emedicine.medscape.com/article/9207139workup!
!!
2.Diabetic!patient,!allergic!to!sulpha!drugs,!on!metformin!and!acarbose.!Not!well!controlled,!
what!to!add!?!
A.!! Novoptions!
B.!! Gliptazine!
Answer:!B!
!!
3.Diabetic!and!pregnant,!hba1c!control!should!be!less!than:!
A.!4!
B.!5!
C.!6!
D.!7!
!Answer:!C!
The!NICE!guidelines!for!Diabetes!in!Pregnancy!(Clinical!Guideline!63)!state!that!women!with!
diabetes!should!aim!to!achieve!an!HbA1c!result!of!43!mmol/mol!(6.1%)!or!lower.!
!!
4.!scenario!of!patient!with!diabetes!and!asked!about!how!does!diabetes!affect!on!wound!
healing?!
Answer:!limit!phagocytosis!
!!
5.5Diabetic!pt!complain!of!(something!in!the!back)!with!multiple!discharging!sinuses!Dx:!
1.!Infected!lymphoma!
2.!Lymphangitis!
3.!Carbuncle!
4.!Furuncle!
! Answer:!C!
!!
6.CASE!scenario!about!diabetic!patient!diagnosed!recently!since!6!months.!Which!one!of!the!
following!renal!investigation!we!should!do!yearly:!
A9serum!creatinine!
B9microalbuminuria!
C9creatinine!albumin!ratio!
D924h!urine!protein!collection!
!Answer:!B!
!!
!!
7.Contraindicated!hypoglycemic!drug!in!diabetic!patient!with!heart!disease!
A.!!pioglitazone!
B.!!sitagliptin!
Answer:!A!
!https://reference.medscape.com/drug/actos9pioglitazone9342726#5!
!!
8.Diabetic!patient!with!lower!limb!paresthesia,!what!to!give!
Answer:!Pregabalin!
!!
9.What!is!the!best!diagnostic!test!for!diabetic!nephropathy!
a)Urine!dipstic!
b)Kidney!function!test!
c)Albumin/creatinine!spot!ratio.!
d)24h!urinary!protiens.!
Answer:!C!
!!
10.!!!A!child!with!growth!retardation,!obesity!and!Stria.!What!investigation!would!u!choose?!
1)!MRI!brain!
2)!adrenal!scintigraphy!
3)!MRI!adrenals!
4)!morning!and!evening!cortisol!level!
Answer:!D!
First!you!do!biochemistry!and!confirm!the!presence!of!Cushing!before!proceeding!to!imaging!
!!
11.!!Pregnant!on!thyroid!medication,!how!much!should!you!increase!the!dose!
A.!! 20%!
B.!! 30%!
C.!! 40%!
D.! 50%!
!Answer:!D!
!!
12.!!5Obese!female,!regular!period!had!acne,!other!examination!normal!what!is!the!best!
investigation?!
Answer:!ACTH!
Cushing!syndrome:!The!right!choice!is!to!check!24h!urinary!free!cortisol!level!
Or!low!dexamethasone!suppression!test.!
ACTH!is!not!used!till!later!on.!
!!
13.!! Pt#with#muscle#weakness,#apart#from#hypotension#and#bradycardia,#his#examination#was#
normal.#His#lab#tests#(high#K+,#low#NA,#low#Chloride,#high#urea)#
What#is#the#etiology#behind#his#presentation?#
A.hyponatremia.
B.hyperkalemia.
C.low.chloride.
D.uremia.
Answer:.B.
..
14.##Target#HbA1c#of#DM2#
Answer:.Less.than.7%.
.http://guidelines.diabetes.ca/executivesummary/ch8.
..
15.##Patient#with#high#HgA1c#what#to#order#also?#
A.Fasting.blood.sugar.
B.CBC.
C.TSH.
Answer:.A.
..
16.##Case#of#Elderly#DM#patient#on#metformin,#did#not#eat#well#for#the#past#5#days.#He#is#
dehydrated,#Investigation#revealed#the#following:#
•#Very#high#Random#blood#glucose#levels#
•#Urine#(–ve#for#ketones,#+ve#for#Glucose).#Normal#renal#function.#Your#Assessment:#
A..Hyperosmolar.Hyperglycemia.
B..lactic.acidosis.
C..Diabetic.Ketoacidosis.(DKA).
Answer:.A.
Hyperosmolar.Hyperglycemic.NonKketotic.Syndrome.(HHNKS).:.a.metabolic.complication.of.
DM,.especially.in.elderly.and.in.type.2.DM,..characterized.by:.
1..Hyperglycemia.
2..Extreme.dehydration.
3..Hyperosmolar.plasma.
4..Altered.consciousness.(late.sign).
Precipitating.factors.for.this.condition.(HHNKS):.
•.Acute.infection.
•.NonKadherence.to.Diabetes.Meds..
https://www.merckmanuals.com/professional/endocrineKandKmetabolicKdisorders/diabetesK
mellitusKandKdisordersKofKcarbohydrateKmetabolism/nonketoticKhyperosmolarKsyndromeKnkhs.
..
17.##Metabolic#syndromes#
Diabetic.drugs.
.??.
..
18.##The#nature#history#of#diabetic#nephropathy#(picture#provided)#when#the#changes#occur#or#
something#like#that#
A...10.years.
B...15.years.
C...20.years.
D..25years.
I.searched.and.asked.some.endocrinologist,.none.gave.me.an.answer..
But.mostly.it's.either.15.or.20.years.
..
19.##[To#differentiate#between#type#1#and#2#DM#
Answer:.Endogenous.insulin.
Or.CKpeptide.
..
20.##Old#patient#came#with#Na#of#123#and#serum#osmolality#of#223#,#K#was#normal.#Urine#
osmolality#was#800.#(No#scenario#or#cause#given)#
The#probable#diagnosis#is:#
1K.conns.disease.
2K.addisons.
3K.SIADH.
4K.Cushing.
Answer:.SIADH.
In.SIADH.the.serum.osmolality.is.less.than.275.mosm.and.urine.osmolality.is.more.than.300.
mosm..
In.conns.there's.hypernatremia.and.hypokalemia,.in.Addison.the.opposite.
https://www.slideshare.net/mobile/mhdsoud/diKsiadhKandKcerebralKsaltKwastingKsyndrome.
..
21.##[Type#of#the#insulin#in#DKA#
Answer:.Regular.
..
22.##Middle#aged#male#with#infertility#and#decrease#libido#and#on#examination#bilateral#
testicular#atrophy,#decrease#hair#and#musculature?#What#would#you#do#next?#(Testicular#
biopsy#or#MRI)#
.Couldn’t.find.an.answer.
..
23.##Male#student#in#high#school,#athletic,#presented#to#the#primary#clinic#for#check#up,#
increase#musculature#with#multiple#acnes#what#you#are#going#to#order#(anabolic#urine#test)#
.Answer:.Anabolic.urine.test.
..
24.##Patient#with#dm#,#which#one#of#the#following#is#contraindicated:#
1.! losartan.
2.! Nifedipine.
3.! Hydrazine.
.Answer:.C.
..
26.##Pt#known#case#of#asthma,#his#wife#has#osteoporosis,#he#has#renal#stone#,#he#eats#
multivitamin,#vit#D#and#calcium#.#
PTH#:#high#
What#is#the#dx?#
Answer:.Primary.Hyperparathyroidism.
https://emedicine.medscape.com/article/127351Koverview#a3.
..
28.##Patient#with#thyroid#storm#symptoms.#What#is#the#first#thing#to#give#the#patient?#
Answer:.Propranolol.
..
29.##Clear#case#of#hyperthyroidism.#What#is#the#most#beneficial#in#the#diagnosis?#
A...Thyroid.scan.
B...FNA.
C...US.
.Once.the.diagnosis.of.hyperthyroidism.has.been.established,.the.cause.of.the.hyperthyroidism.should.be.
determined..Graves'.hyperthyroidism.may.be.clinically.obvious.on.the.basis.of.clinical.findings.such.as.
diffuse.goiter.and.ophthalmopathy..However,.a.24Khour.thyroid.radioiodine.uptake.and.scan.are.
frequently.necessary.to.confirm.the.diagnosis.of.Graves'.hyperthyroidism.and.exclude.other.causes,.
especially.painless.thyroiditis..
.Answer:.A.
..
30.##Smoker,#alcoholic#has#thyroid#nodule.#What#is#the#first#thing#to#do?#
Answer:.Needle.aspiration.
..
31.##Case#scenario#about#patient#with#hypertention,#labs#result#hypernatremia#hypokalemia#
and#asking#about#cell#responsible#for#that?#
K.......fasiculata.
K.......glumerulosa.
K.......reticularis.
Answer:.B.
..
32.##picture#of#vial#of#varl#rix#vaccine#asking#this#vaccine#for#what#
a)varicella.
b)hepatitis.b.
c)influenza.
Answer:.A.
..
33.##Case#about#DKA#,worst#complication#?#
Answer:.cerebral.edema.
..
34.##Water#with#low#iodine,cheer[man#wants#to#treat#children#suspected#to#have#cretinism#?#
A.Add.iodine.
B.Levothyroxine.
Answer:.B.
..
35.## ophthalmology#ex#for#DM2#patient#every#
A..6.months.
B..12.months.
C..24.months.
D..36.Months.
.Answer:.B.
..
36.##A#pt#with#Lab#findings#of#hypothyroidism,#he#has#hoarseness#and#a#large#neck#swelling#
OE#lt#lobe#is#swollen#and#is#larger#than#the#right#one#
Best#Mx#is:#
A..FNA.
B..Thyroid.lobectomy.
C..Radio.isotope.scan.
.Answer:.A.
..
37.##Thyroid#mass#with#intracellular#amyloid#deposit#
1.! Medalley.
2.! Papillary.
3.! Follicular.
4.! NonKhogken.
Answer:.A.
.https://oup.silverchairK
cdn.com/oup/backfile/Content_public/Journal/ajcp/53/5/10.1093/ajcp/53.5.592/2/ajcpath53K
0592.pdf?Expires=1509461530&Signature=Q2aCjU3tMI~IQFuoS~g67hCsnQZwVGnarLrX9s4K
MEg6okBIwgGmFkccs5295SJxodv0csYSYLvJ82kIkkZnHXSMfYPPNq4SiG0GABKQkHBuXgacCsXK
PcMoeOjjysophE~Qdt96UvEQLFeuSCQ4sXUpPbYc2wmWXgWT0rLbx6nyyVitu4Y7g5f3V2Zvn6G0
V1rvJ75T7JKdgsyQfYQLj~wWCTFCeLwHbldDaZhdWQK05L5sK3e~xWOhuevmkpPSDB04PZm0GtdK
1Mqe66QsJVmLyzwVadiRiKzVjafVsGzMbZ8cp61l2DdZyvgdhscjoKjTiVCE0Ee1TFKztT~DuA__&KeyK
PairKId=APKAIUCZBIA4LVPAVW3Q.
..
39.##Question#about#baby#2#weeks#old#who#is#not#active#and#mother#happy#he#is#not#cry#asking#
about#investigations#
A...Kt3.level.
B...Total..T4.
C...free.t.4.
D..mother.t.level.
.Answer:.C.
..
40..#In#a#village#where#the#incidence#of#cretinism#and#iodine#was#less#than#1#microgram#the#
health#promoters#want#to#issue#a#director#for#the#for#the#management#of#those#with#
cretinism..#what#is#the#best#initial#management?#
A...TSH.and.t4.mesruments.
B...Start.thyroxine.medication.
C...Iodine.supplementation.
Answer:.B.
The.mainstay.in.the.treatment.of.congenital.hypothyroidism.is.early.diagnosis.and.thyroid.
hormone.replacement..Optimal.care.may.includes.diagnosis.before.age.10K13.days.and.
normalization.of.thyroid.hormone.blood.levels.by.age.3.weeks..
Only.levothyroxine.is.recommended.for.treatment.
http://emedicine.medscape.com/article/919758Ktreatment#d6.
..
41.##Thyrotoxicosis#crisis,#initial#treatment?#
Answer:.B.blocker.
..
42.##Hypoparathyrodism#,he#will#have#?#
Answer:.Convulsions.
Tetany,.seizures,.QT.prolongation,.twitching.(Chvostek.sign),.spasm.(Trousseau.sign).
..
43.##Propylthiouracil#(#anti#thyroid)#how#it#works?#
https://www.ncbi.nlm.nih.gov/pubmed/23883148.
##
44.##[#Anti#thyroid#causing#pancytopenia:#
Answer:.Methimazole.
This.condition.is.extremely.serious.but.affects.only.one.out.of.every.200.to.500.people.who.
take.an.antithyroid.drug..Older.people.taking.propylthiouracil.and.those.who.take.high.doses.of.
methimazole.may.be.at.higher.risk.of.this.side.effect..
..
45.##[#Antithyroid#used#in#pregnancy:#
Answer:.Propylthiouracil.
.Propylthiouracil.is.the.drug.of.choice.during.the.first.trimester.of.pregnancy.because.it.causes.
less.severe.birth.defects.than.methimazole..Because.there.have.been.rare.cases.of.liver.
damage.in.people.taking.propylthiouracil,.some.clinicians.will.suggest.switching.to.methimazole.
after.the.first.trimester,.while.others.may.continue.propylthiouracil..
..
46.##Cold#intolerance,#wt#gain#>#check#thyroid#Us#
The.right.answer.should.be.to.measure.TSH.and.free.T4.
..
47.##cohns#and#Addison#and#Cushing##basically#secondary#hypertension# Labs#and#
investigations#(3qs)#
##
48.##MOA#of#metformin#in#PCOS:#
Answer:.reduces.insulin.resistance..
..
49.#Diabetic#on#sulfa#and#metformin#came#for#regular#check#up,#his#HBA1C#is#9.2#what#is#your#
management#?#
K.regular.insulin.
K.aspart.
K.pioglitazone\.Glitazone.
Answer:.C.
..
50.#A#45#year[old#man,#who#has#diabetes#and#has#been#on#metformin#and#glimepiride#for#the#
last#four#weeks,#presents#to#the#clinic#with#poor#glycemic#control.#A#history#also#confirms#poor#
dietary#management.#
Which#of#the#following#should#be#prescribed#now?#
A..Acarbose.
B..Repaglinide.
C..Tolbutamide.
D..Pioglitazone.
Answer:.A.
..
51.#Married#man,#type2#dm,#his#wife#is#unable#to#conceive,#o/e#testicular#atrophy,#reduced#
pubic#hair,#testosterone#and#LH#are#low Prolactin#normal Next#step#in#dx:#
A).karyotyping .
B).testicular.us .
C).brain.MRI.
Answer:.C.
..
52.#Type#2#dm#woman,#full#bladder,#incomplete#emptying,#the#etiology#
A).diabetic.neuro....
B).detrousal....
Answer:.A.
..
53.##What#is#the#MOA#of#glipizide#?#
Answer:.Increase.insulin.secretions.from.pancreas.
..
54.##[#What#is#insulin#regimen#that#is#similar#to#normal#physiology?#
Answer:.ASPART.AND.GLARGINE.
..
55.##Many#Q#about#thyroid#in#medicine#I#don’t#remember,#review#what#you#should#do#if#
there’s#mass#and#about#hypo#&#hyperthyroidism,#there’s#investigation#in#the#Questions#
..
56.##Patients#have#D.M.#On#metformin#1#g#and#another#anti#dm,#present#with#increasing#
blood#glucose#at#morning,#what#you#will#give#:#
A..NPH.
B..Lispro.
C..Regular.
.Answer:.A.
..
57.##pt#with#hyperthyroidism,#with#palpitation#,#what#is#the#most#likely#finding#in#his#ECG?#
1.! SVT.
2.! VT.
3.! A.FIB.
Answer:.C.
##
58.##Pt#k/c#of#hypothyroidism#,#use#Levothyroxin#175#mg#,#switch#to#200#mg#.#
Labs#:##Tsh#:#high.####T4#:#normal.##What#is#the#cause#?#
A...ectopic.thyroid.
B...Primary.hypo.
C...Secondary.hypo.
D..Small.dose...
Answer:.D.
..
60.##How#hyperglycemia#affect#wound#healing:#
A...dec.imuune.system.
B...Dec.phagocytosis.
C...Stimulates.bacteria.growth.
Answer:.B.
..
61.#case#of#hyperthyroidism#what#will#be#associated:#
A)AFib.
B).VT.
C).SVT.
D).WPW.
.Answer:.A.
..
62.##Multi[nodular#goiter#treatment:#
A...1Ksurgery.
B...2Kradioidonie.
C...3KbetaKblockers.
D..4Kantithyriod.
.Answer:.A.
..
63.##Case#of#hypothroidism#lipid#screening#after#
a.6.months.
b.1year.
c.3years.
d.5years.
.Answer:.A.
..
64.##GLipizide#mode#of#action?#
1.! increases.insulin.release.from.the.pancreas.
2.! Increase.peripheral.cells.sensitivity.to.insulin.
.Answer:.A.
..
65.##A#known#case#of#Addison#disease#presented#with#weakness,#cold#and#clammy#skin,#
dizziness#and#weak#pulses.#Treatment?#
1.! 0.9%.N/S.over.1hour+hydrocortisone.IV.
2.! 0.9%.N/S.over.8hours+hydrocoriesone.IV.
3.! 0.9%.N/S.over.1.hour.+.fludrocortisone.orally.
4.! 0.9%.N/S.over.8hrs.+fludrocortisone.orally.
.Answer:.A.
https://emedicine.medscape.com/article/765753K
treatment?pa=Rev1p9JVxdupXJ3%2FGoHsV3tvHK9mpdTiE1lxb0CSsSQ8YUkKorinsY0X%2F6L3clY
qX8MwC0EECwzp432Skuf9qw%3D%3D.
..
67.##8#year#with#T1DM#dx#before#1#month#when#to#start#ophtha#screening?#
A)..After.3.years.
B)..After.5.years.
.Answer:.B.
..
68..#Patient#is#diabetic#on#glyburide#she#gained#7kgs#in#a#year.#HBA1C#is#6.6#what#is#your#
management?#
A)..Add.metformin.
b).Stop.glyburide.and.start.her.on.metformin.
.Answer:.B.
..
69..#23#year#old#man#is#complaining#of#polydepsia#and#polyurea#and#he#is#worry#and#afraid#
about#getting#dm#what#is#the#best#value#for#diagnosis.?.
1K.7.7.
2K8.
3K.9.
4K.12.
No.enough.information.
NOTE:.
Table.2.1—Criteria.for.the.diagnosis.of.diabetes FPG.126.mg/dL.(7.0.mmol/L)..Fasting.is.
defined.as.no.caloric.intake.for.at.least.8.h.*.
OR.
2Kh.PG.200.mg/dL.(11.1.mmol/L).during.an.OGTT..The.test.should.be.performed.as.described.by.
the.WHO,.using.a.glucose.load.containing.the.equivalent.of.75.g.anhydrous.glucose.dissolved.in.
water.*.
OR.
A1C.6.5%.(48.mmol/mol)..The.test.should.be.performed.in.a.laboratory.using.a.method.that.is.
NGSP.certified.and.standardized.to.the.DCCT.assay.*.
OR.
In.a.patient.with.classic.symptoms.of.hyperglycemia.or.hyperglycemic.crisis,.a.random.plasma.
glucose.200.mg/dL.(11.1.mmol/L)..
*In.the.absence.of.unequivocal.hyperglycemia,.results.should.be.confirmed.by.repeat.testing..
reference:.American.diabetes.association.2016.
..
Q.##Scenario#about#female#with#hypothyroidism#and#take#levothyroxine#for#something#
duration,#she#is#asymptomatic#now.#
The#lab#result#:#T4#within#range#,#TSH#still#high#1#above#normal#range#,#what#will#you#do:#
K.......continue.with.the.same.dose.
K.......increase.dose.by.…..
K.......Increase.dose.by.….
K.......Increase.dose.by.….
Answer:.Increase.dose.by.25.mcg.
..
Q.#Hypothyroidism#pt#after#2#weeks#of#treatment#lab#show#normal#t4#+#high#tsh:#
Same.dose.&.f\u.1K2.months.
Decrese.dose.&.f\u.1K2.months.
Decrease.dose.&.f\u.1K2.weeks.
Answer:.continue.same.dose.
T4.takes.one.week.only,.while.TSH.needs.2.months.to.stabilize.
..
70.##Lady#who#is#taking#thyroxin#75#mg.#She#stopped#taking#the#drug#3#days#back#and#came#
now#for#refill#of#the#drug.#Labs#show#normal#T4#and#high#TSH#(85).#What#is#the#dose#you’re#
going#to#give#her?#
A.. 50.
B.. 75.
C.. 100.
D..150.
Answer:.C.
..
71.##Side#effect#of#hyperthyroidism#drugs#à#choices#where#medication#
..
72.##most#reliable#marker#for#diagnosing#hypothyroidism;#
1.! K.Serum.T4.
2.! K.Total.T4.
3.! K.TSH.
4.! K.T3.
.Answer:.C.
..
73.##Case#of#hyperthyroidism#what#will#be#associated:#
A)AF.
B).VT.
C).SVT.
D).WPW.
.Answer:.A.
..
74.##Scenario#of#Cushing#syndrome#without#the#diagnosis#asking#which#one#will#be#present?.
Telangiectasia.
In.some.rare.cases.you.find.telangiectasia.
..
75.##Incretin#mode#of#action?#
Answer:.Increase.insulin.secretion.
..
77.##HbA1c#target#for#dm2#àless#than#7#
..
78.##Female#trying#to#lose#weight#but#she#couldn’t,#you#ordered#thyroid#panel#and#it#came#out:#
TSH:#high#
T4#normal.#What’s#the#most#likely#diagnosis?.
1.! hyperthyroidism.
2.! Subclinical.hypothyroidism.
Answer:.B.
..
79.##Patient#with#hyperthyroidism.#Which#drug#more#rapid#action?#
1.! Propylthiouracil.
. B..radioactive.iodine
Answer:.A.
..
80.##What#is#the#mechanism#of#insulin#resistance#in#obesity?#
https://www.ncbi.nlm.nih.gov/pmc/articles/PMC3936017/#
##
81.#Patient#with#hyperthyroidism#on#medication.#Came#with#red#eye.#What#is#the#medication#
responsible#for#that?#
.Iodine.can.cause.exacerbation.of.eye.disease..
That's.the.only.explanation.for.red.eye.
##
82.##Diabetic#on#Metformin.#She#got#pregnant,#what#will#you#do?#
Answer:.Shift.her.on.insulin..
##
83.##Patient#is#having#seizure#episodes#since#3#days.#Blood#osmolality#is#low.#Urine#osmolality#
is#normal.#What#is#the#most#likely#cause?#
A...Cons.syndrome.
B...Addison.syndrome.
C...Cushing.
D..Inappropriate.ADH.secretion.
Answer:.D.
..
Patient#came#to#PHC#complaining#of#fatigue#and#polyuria#,#investigation#:#RBS#450#,#positive#
# ketone#body#,#what#is#the#predominant#type#of#ketone#body#is#present#in#the#urine?
. Answer:.Acetoacetate
..
85.#32#female#c/o#breast#milk#discharge#and#irregular#menses,#ask#about#investigation:#
A..TSH.
B..brain.MRI........................................................................... .
C..biopsy.............................................. .
Answer.:.TSH.(Uptodate.mentions.TSH.as.first.initial.test.before.brain.MRI!).
..
86.#Amenorrhea,#low#prolactin,#what#to#see#in#CT#or#MRI:############################################# #
Answer.:.EMPTY.SELLA.TRUCICA.
..
87.#Mother#developed#gestational#dm#in#pregnancy#then#come#after#giving#birth#at#6#month#
the#HA1C#is#6.9#what#is#the#managment#?#
1.! insulin.
2.! metformin.
3.! sulfenourea.
4.! pigletazone.
Answer:.B.
..
88.#Long#scenario#for#30#years#female#patient#has#manifestation#of#hypothyroidism#
(tirdness.....)#Lab#and#vital#sign#were#provided#and#it#shows,#low#T4,#low#24hours#urine#
cortison,#low#testesteron#,#low#somatomedin#,#low#IGF#.#Beside#thyroid#replacement#what#you#
will#give:#
1.! GH.
2.! Steroid.
Answer:.B.
..
89.#Female#34#years#old#presented#with#breast#milk#discharge#and#9#period#a#year,#which#of#
the#following#most#likely#she#had:#
.Answer:.prolactinoma .
..
90.#Case#of#women#with#hyperprolactenemia#and#they#found#pituitary#adenoma#.7#cm#what#is#
the#management#:#
1.! Cabergoline.
2.! bromocroptine.
3.! surgery.
Answer:.A.
..
91.#Case#of#SIADH#,#they#wrote#lab#results#and#they#want#the#diagnosis:#hyponatremia#and#
hypo[osmolality#,#urine#osmalirty#hhigh#
..
92.#Diabetic#patient#on#NPH#and#regular#insulin#twice#before#breakfast#and#dinner#,#glucose#
was#high#at#4pm#,#how#to#manage#:#
1.! Increase.NPH.before.breakfast.
2.! Increase.regular.before.breakfast.
3.! Increase.NPH.before.dinner.
4.! Increase.regular.before.dinner.
Answer:.A.
..
93.#Patient#with#dm#,#which#one#of#the#following#is#contraindicated:#
.a).losartan.
b).Nifedipine.
c).Hydrazine.sulfate.
Answer:.C.
..
..
.

Pulmonology#
##
1.#Acute#asthma#in#ER#take#SABA?#Next?#
A..Theophylline.
B..Inhaler.ipratropium.
Answer:.B.
.Reference:.http://cursoenarm.net/UPTODATE/contents/mobipreview.htm?19/39/20081.
..
2.#Obstructive#Sleep#Apnea#breathing?.
Answer:.Cheyne–Stokes.respiration..
#Reference:.https://en.wikipedia.org/wiki/Cheyne%E2%80%93Stokes_respiration.
..
3.#Lung#infiltration,#decrease#air#entry,#greenish#sputum#foul#smell.#
A..Bronchiolitis.
B..Bronchiectasis..
Answer:.B.
#Reference:#
https://www.nhs.uk/Conditions/Bronchiectasis/Pages/Symptoms.aspx.
https://www.emedicinehealth.com/bronchiectasis/page6_em.htm.
.
4.#Anti[cancer#drug#causes#reactions,#skin#changes#and#lung#fibrosis:#
A..Bleomycin.
B..Cisplatin.
C..Methotrexate.
Answer:.A..
.
Both.medication.can.cause.
all.of.these.side.effects.but.
bleomycin.more.common.
Reference:#
https://www.drugs.com/sfx/
bleomycinKsideKeffects.html.
https://www.drugs.com/sfx/
methotrexateKsideK
effects.html.
..
..
5.#Patient#complaining#of#
face#flushing#and#head#
heaviness#in#the#morning#
that's#relieved#during#the#
day,#CXR#shows#bilateral#
hilar#mass,#he#is#smoker#for#
30#years?#
A..Hodgkin's.lymphoma.
B..Lung.cancer.
C..COPD.
Answer:.B.
Symptoms.of.Superior.Vena.Cava.Syndrome.
Reference:.https://www.cancer.net/navigatingKcancerKcare/sideKeffects/superiorKvenaKcavaK
syndrome.
..
6.#Old#patient#with#facial#swelling#when#waking#up#&#decrease#during#the#day,#he#is#smoker.#What#
is#the#diagnosis?#
A..COPD.
B..TB.
C..Lung.cancer.
Answer:.C.
Symptoms.of.Superior.Vena.Cava.Syndrome.
Reference:.https://www.cancer.net/navigatingKcancerKcare/sideKeffects/superiorKvenaKcavaK
syndrome.
..
7.#Patient#exposures#to#asbestos#what#will#present#in#chest#x#ray:#
A..Hyperinflation.
B..Bilateral.lungs.fibrosis.
C..Interstitial.lung.disease.
.Answer:.B.
Hyperinflation.may.also.be.present,.so.could.be.A.as.well.
Reference:#
For.A:.http://www.clinicaladvisor.com/pulmonaryKmedicine/asbestosKrelatedKpulmonaryK
disorders/article/625653/.
For.B:.https://www.asbestos.com/asbestosis/diagnosis/.
..
8.#1#year,#fine#crackles,#?#
All.choices.were.not.related.looks.look.bronchopneumonia.
..
9.##Female#with#history#of#pulmonary#embolism,#3#abortion.#Have#positive#lupus:#
I.chose.Antiphospholipid+
. .
10.##Man#works#in#the#new#office#and#he#experienced#cough#and#wheezing#and#respiratory#
symptoms#I#don't#remember#exactly.#Otherwise#he#is#fine#and#the#condition#return#once#he#back#to#
this#office:#
A..Asthma.
B..Allergic.pneumonitis.
.Answer:.B.
.Reference:.https://medlineplus.gov/ency/article/000109.htm.
..
..
11.##Which#of#the#following#is#the#typical#picture#of#primary#TB:#
A..Caseation.of.lymph.node.
B..Cavitation.in.lung.
C..Can’t.remember.other.choices.
.Answer:.A.
.Cavitation.is.uncommon.in.primary.TB,.seen.only.in.10K30%.of.cases.
Reference:.https://www.ncbi.nlm.nih.gov/pmc/articles/PMC3215852/.
..
12.##Lung#abscess,#best#antibiotics?#
A..Cipro.
B..Macrolides.
C..Third.generation.
D..Piperacillin.
.Answer:.A.
.Standard.therapy.for.lung.abscess.with.anaerobic.bacteria.is.clindamycin.but.not.in.the.choices.
Reference:.https://emedicine.medscape.com/article/299425Kmedication#2.
..
13.##Treatment#for#bilateral#pneumonia#
A..Ampicillin.
B..Ciprofloxacin.
C..Vancomycin.
.Answer:.B.
.Depend.on.the.organism,.but.atypical.pneumonia.is.often.diffuse.and.bilateral..
The.first.line.antibiotic.in.atypical.pneumonia.is.macrolide#or#doxycycline.but.both.are.not.in.the.
choices,.and.the.second.line.is.fluoroquinolone.
Reference:#
http://bestpractice.bmj.com/bestKpractice/monograph/18/treatment/details.html.
https://emedicine.medscape.com/article/300157Ktreatment#d6.
.
14.##Most#common#cough#in#adults#
Answer:.post.nasal.drip.
.Reference:.http://www.aafp.org/afp/2003/0101/p169a.html.
..
15.##Middle#age#man#with#acute#onset#of#fever#malaise#anorexia#x[ray#show#bilateral#hilar#and#left#
paratracheal#lymphadenopathy#what#is#the#diagnosis:#
A..HIV.
B..sarcoidosis.
C..TB.
D..rheumatic.fever.
.Answer:.B..
The.most.frequent.radiological.abnormality.involves.enlarged.bilateral.hilar.and.right.paratracheal.
lymph.nodes.in.sarcoidosis.
Reference:.https://www.ncbi.nlm.nih.gov/pmc/articles/PMC3789928/.
..
16.##Smoker#with#CXR#result#of#calcification:#
A..Squamous.cell.carcinoma.
B..Adenocarcinoma.
Answer:.A.
.Most.types.of.lung.cancer.linked.to.smoking.cigarette.but.the.strongest.association.is.with.
squamous.cell.carcinoma.and.small.cell.carcinoma.
Reference:.Rubin's.Pathology:.Clinicopathologic.Foundations.of.Medicine.page.539,.5th.Edition.
..
..
17.##Student#with#history#of#sore#throat,#headache,#mild#cough,#low[grade#fever,#chest#pain,#tired,#
general#malaise?#
A..Pneumococci.
B..Mycoplasma.
C..Aspiration.pneumonia.
D..Legionella.
Answer:.B.
Picture.of.atypical.pneumonia.symptoms..Mycoplasma.and.Legionella.are.organisms.of.atypical.
pneumonia,.but.Legionella.common.has.GI.symptoms.with.diarrhea.
Reference:.http://cursoenarm.net/UPTODATE/contents/mobipreview.htm?39/18/40239.
..
18.##Drug#for#asthma#decreases#secretion#more#than#bronchodilation:#
Answer:#ipratropium.promide.
.
19.##28#years#pregnant#at#20#weeks#of#pregnancy.#Developed#dyspnea#and#resp.#distress#of#sudden#
onset,#what#is#the#diagnostic#test:#
A..2.chest.xKrays.
B..Echocardiogram.
C..CT.scan.
D..V/Q.ventilation..
Answer:.D..
Because.of.the.superior.accuracy.of.V/Q.scanning.and.lower.maternal.breast.irradiation,.there.has.
been.a.movement.away.from.CT.angiography.toward.V/Q.scanning.as.the.diagnostic.test.of.choice.in.
the.workup.of.pulmonary.embolism.in.pregnancy.in.women.with.a.negative.CXR..In.those.patients.
with.an.abnormal.CXR,.CT.angiography.remains.the.test.of.choice..
Reference:#http://cursoenarm.net/UPTODATE/contents/mobipreview.htm?5/7/5233#H13.
..
20.##2#Q#about#squamous#cell#carcinoma#of#lung#
First.was.“xK.ray“,.Second.I.don’t.remember.
..
21.##Exacerbation#of#asthma#>#inhaled#steroids#
..
22.##Old#Pt#have#recurrent#DVT#and#obstruction#SVC,#what#the#cause?#
Lung.carcinoma.
..
23.##Cancer#in#the#middle#of#esophagus,#which#type?#
A..Adenocarcinoma.
B..Scc.
.Answer:.B.
#Reference:.https://www.mayoclinic.org/diseasesKconditions/esophagealKcancer/symptomsK
causes/sycK20356084.
...
24.##Old#male#present#with#mid#clavicle#mass#(smoker#for#20y#and#I#think#he#drinks#Alcohol)#What#is#
your#initial#management?#
A..Core.biopsy.
B..Fine.needle.biopsy.
C..bronchoscopy.
..
Answer:.B.
.If.the.question.mean.thyroid.mass,.so.it.is.B.
Reference:.https://emedicine.medscape.com/article/127491Koverview#a4.
..
25.#40#years#old,#non[smoker,#present#with#weight#loss#and#dyspnea,#he#works#in#electricity#
generation,#what#is#the#cause?#
A..Asbestose.
B..Inhaled.coal.dust.
C..Passive.smoking.
Answer:.A#
Reference:#https://academic.oup.com/annweh/articleK
abstract/37/6/645/130162?redirectedFrom=PDF.
...
26.##Patient#with#obstructive#sleep#apnea#CPAP#not#effective#what#to#do?#
A..Promethazine.
B..Beclomethasone.
C..Ipratropium.
D..Modafinil.
.Answer:.D.
.Reference:.https://emedicine.medscape.com/article/295807Kmedication.
..
27.##34#years#old#man#has#an#intermittent#epigastric#pain#for#3wks.#It#is#worse#by#food#but#helped#
by#some#tablets#he#obtained#from#the#pharmacy.#He#had#a#similar#episode#3yrs#ago#and#his#doctor#
gave#him#a#course#of#3#types#of#tablets#at#the#time.#What#is#the#most#appropriate#next#
investigation?#
A..Abdomen.US.
B..Barium.meal.
C..Urea.breath.test.
.Answer:.C.
#Reference:.https://emedicine.medscape.com/article/176938Kworkup.
..
28.##Lung#cancer#of#stage#1#small#cell#mixed#with#non#small#cell#and#asked#about#management?#
A..Surgery.and.chemotherapy.
B..Chemotherapy.and.radiation.
C..Chemotherapy.plus.something.
.Answer:.A.
Both.of.Small.Cell.Lung.Cancer.&.NonKSmall.Cell.Lung.Cancer.the.standard.management.is.surgical.
resection.
Reference:#https://emedicine.medscape.com/article/279960Ktreatment.
https://emedicine.medscape.com/article/280104Ktreatment.
##
29.##Which#lung#cancer#causes#SIADH#syndrome?#
While.the.association.with.small.cell.lung.cancer.(SCLC).is.well.known,.that.with.non.small.cell.lung.
cancer.(NSCLC).has.been.rarely.reported.
.Reference:.http://www.sciencedirect.com/science/article/pii/S2213007117302381.
..
30.##Long#scenario#about#lung#cancer#on#pathology#(polygonal#cells)#and…?#
Answer:.squamousKcell.lung.cancer.
##
Reference:#https://www.pathologystudent.com/?p=4616.
..
..
31.##Known#asthma#has#whitish#rash#easily#removed#in#mouth.#Which#antiasthmatic#cause#this?#
A..Cromolyn.sodium.
B..Betamethasone.
C..Albutmerol.
.Answer:#B.
Reference:.https://www.nhs.uk/Conditions/OralKthrushKKKadults/Pages/Introduction.aspx.
..
32.##COPD#exacerbation#by#Infection,#patient#has#fever#and#greenish#sputum,#what#is#the#most#
likely#microorganism?.(no.pseudomonas.in.the.answers).
A..staph..Aureus.
B..streptococcus.pneumonie.
C..mycoplasma.pneumonia.
D..homophiles.influenza.
.Answer:.D.
.Haemophilus.influenzae;.Moraxella.catarrhalis.more.common.in.preKexisting.structural.lung.disease.
(CF,.bronchiectasis,.COPD).and.the.elderly.
Pseudomonas,.Haemophilus,.and.pneumococcal.species:.May.produce.green.sputum .
..
Reference:.Kumar.and.Medscape.
https://emedicine.medscape.com/article/300157Koverview.
...
33.##Old#asthmatic#patient#came#with#urinary#retention#diagnosed#with#BPH#and#he#was#found#to#
have#high#blood#pressure#180/110#(exactly#it#was#that#high)#what#to#give#now?#
A..IV.Labetalol.
B..propranolol.
C..prazosin.
D..something.ends.with.lol.,.BBlocker.?.
Answer:.C.
Reference:.http://cursoenarm.net/UPTODATE/contents/mobipreview.htm?19/60/20417#H5.
..
34.###Patient#with#recurrent#hemoptysis#since#3#years.#What’s#the#most#likely#cause?#
Missing#details#
A.! Bronchiactasis.
B.! Tb.
.Answer:.
..
35.#Treatment#of#bronchiolitis#
Answer:#Conservative.
..
Reference:.https://emedicine.medscape.com/article/961963Ktreatment.
##
36.#Old#pt#in#ICU#with#pneumonia,#ask#about#organism:#
Answer:.Pseudomonas.
..
37.#What#is#the#best#investigation#for#determine#severity#of#asthma?#
A..PEFR.
B..ABG.
....Answer:.A.
..
Reference:#http://bestpractice.bmj.com/bestKpractice/monograph/44/diagnosis/tests.html.
..
..
38.#Atypical#pneumonia#management?#
Azithromycin.is.the.drug.of.choice.for.children.with.suspected.or.confirmed.Legionella.disease.
..
39.#About#asthma#management#she#is#on#leukotrienes#since#6#month#then#come#with#every#day#
coughing#relieved#by#sulbutamol#in#ER.#What#you#will#give#her#as#maintenance?#
A..Oral.steroid.and.SABB.on.needed.......... .
B..Inhaled.2.doses.steroid.and.SABB.on.needed..
C..Leukotrienes.and.SABB.on.needed..
D..Long.acting.beta.blocker.and.SABB.on.needed..
.Answer:.B.
Reference:#https://emedicine.medscape.com/article/296301Kguidelines#g3.
............................................. . .
40.#Asthmatic#exacerbation#came#to#the#ER#give#him#sulbutamol#the#relieved#and#the#PEF#70%#
what#you#will#do#next#for#maintenance?#
A..Ipratraupium.
B..Inhaled.steroid.
C..Thyphelen.
..
Answer:.B.
Reference:#https://emedicine.medscape.com/article/296301Kguidelines#g3.
..
41.#Pt#inhaling#3#nanogram#of#Cotton#in#a#factory.#Eventually#the#cotton#will#end#up#
A..Engulfed.by.alveolar.macrophages.
B..Trapped.in.mucocilliary.system.without.reaching.alveoli.
C..Pass.without.any.damage.
D..trapped.in.distal.airway.leading.to.fibrosis.
................................. .
Answer:.D.
Reference:.http://thorax.bmj.com/content/59/12/1095.
...
42.#Bronchospastic#symptoms#(eg,#wheeze#and#cough),#usually#responsive#to#bronchodilator#
therapy#
..
43.#Cavernous#hemangioma#+#pleural#effusion################################################ #
Answer:.Pleural.hemangioma.
.Reference:.https://www.ncbi.nlm.nih.gov/pmc/articles/PMC4821329/.
......... .
44.#What#is#the#most#clinical#sign#associated#with#pulmonary#hypertension#
A..Central.cyanosis.
B..Pulmonary.edema............................... . .
C..Stroke.
D..Lower.limb.edema.and.ascites.
..
Answer:.D.
.Reference:#http://www.aafp.org/afp/2001/0501/p1789.html.
..
#45.#X[ray#of#patient#with#pleural#effusion,#what#U#gonna#hear#during#auscultation:#
A..Decrease.breath.sounds.
B..Decrease.vocal.resonant.
C..Bronchial.breath.sounds.
..
Answer:.A.
Reference:#
https://www.medicinenet.com/pleural_effusion_fluid_in_the_chest_or_on_lung/article.htm.
..
46.#Woman#had#+#pulmonary#TB#since#2#week#her#husband#come#for#tuberculin#test,#minimal#
induration#to#be#positive:#
A..5.
B..9.
C..13.
D..17.
Answer:.A.
Reference:#https://www.cdc.gov/tb/publications/factsheets/testing/skintesting.htm.
..
47.#Definition#of#cheyne#stokes#breathing?#
Abnormal.pattern.of.breathing.characterized.by.progressively.deeper.and.sometimes.faster.
breathing,.followed.by.a.gradual.decrease.that.results.in.a.temporary.stop.in.breathing.called.an.
apnea..The.pattern.repeats,.with.each.cycle.usually.taking.30.seconds.to.2.minute.
..
48.#Patient#complaining#of#dry#cough#and#weight#loss.#Nodular#infiltrate#and#hepatomegaly.#
A..Miliary.TB.
B..Blastomycosis.
.Answer:.A.
.Reference:.http://cursoenarm.net/UPTODATE/contents/mobipreview.htm?33/53/34640.
..
.49.#Loss#of#sensation#over#shoulder#which#part#of#pleura#will#be#affected?#
A..Mediastinal.
B..Pleural.
C..Vesceral.
Answer.:A.
..
Mediastinal.and.central.diaphragmatic.pleural.pain.referred.to.root.of.neck.and.over.shoulder.
(Dermatomes.C3KC5).
.Reference:.Lippincott's.Concise.Illustrated.Anatomy:.Thorax,.Abdomen.&.Pelvis..
“The.reference.about.referred.pain.to.the.shoulder.not.loss.of.sensation”.
..
..
......................................

Cardiology#
.
1..Patient.presented.with.SOB..On.examination.there.was.ejection.systolic.murmur.in.the.aortic.area.
radiating.to.neck..What.is.the.pathophysiology.of.SOB?.
AK.Pumonary.venous.congestion.
BK.systemic.venous.congestion.
Answer:.Pulmonary.venous.congestion.
..
2..Patient.with.mitral.stenosis.and.increased.pressure.in.the.left.atrium,.and.ECG.was.provided..
What.is.the.complication?.
AK.Right.ventricular.hypertrophy.
BK.Pulmonary.HTN.
CK.Dilation.of.the.left.atrium.
Answer:.Dilation.of.the.left.atrium.
..
3..Patient.presented.with.palpitation..Normal.ECG.&.examination..What.is.the.treatment?.
AK.Verapamil.
BK.Propranolol.
CK.Digoxin.
Answer:.Can.not.decide.based.on.messing.information..
..
4..Treatment.of.pericarditis.
AK.Aspirin.
BK.Corticosteroid.
CK.Antiviral.
DK.Antibiotics.
Answer:.Aspirin..
..
5..ECG.pic.showing.anterior.MI..Upon.history,.he.took.sildenafil.(viagra).what.drug.is.contraindicated.
to.be.taken.in.this.case?.
AK.Morphine.
BK.Aspirin.
CK.Nitrate.
.Answer:.Nitrate.
..
6..Patient.complains.of.chest.pain,.describes.it.as.close.fist.crushing.his.sternum..What.is.the.
diagnosis?.
AK.Myocardial.ischemia.
BK.Pulmonary.embolism.
CK.Pericarditis.
Answer:.MI.
..
7..Mid.diastolic.murmur.on.auscultation,.located.at.the.left.sternal.border..What.is.the.diagnosis?.
AK..Mitral.stenosis.
..
8..Long.scenario,.positive.findings.were.radiofemoral.delay.and.on.CXR.there.shows.rib.notching..
What.is.the.diagnosis?.
AK.Coarctation.of.the.aorta.
..
9..Murmur.that.changes.with.changing.the.position?.
AK.Innocent.murmur.
..
10..Patient.with.atrial.fibrillation.on.warfarin.12.mg.had.hematuria,.rectal.bleeding,.and.bruises..His.
INR.is.6.2.what’s.next?.
AK.Reduce.warfarin.
BK.Give.whole.blood,.
CK.Give.vitamin.k.
DK.Replace.warfarin.with.aspirin.
.Answer:.Give.Vitamin.K.
..
11..Patient.with.mitral.stenosis.(0.7.mm),.with.moderate.MR,.RVH.and.Afib,.what’s.the.
management?.
AK.Ballon.
BK.Valve.replacement.
CK.Valve.commissurotomy.
DK.Medical.treatment.
Answer:.Valve.replacement.
Patient.can.not.have.both.MR.and.MS.
However.looking.at.the.size.of.0.7.mm,.it.indicates.severe.stenosis..
..
..
12..Patient.with.aortic.stenosis,.what’s.the.management?.
.Answer:.
Asymptomatic:.serial.echos,.avoid.exertion.
Symptomatic:.avoid.nitrates/arterial.dilators.and.ACEI.in.severe.AS.
Surgery.if:.symptomatic.or.LV.dysfunction.
Surgical.Options.
Valve.replacement:.aortic.rheumatic.valve.disease.and.trileaflet.valve.
–.prior.to.pregnancy.(if.AS.significant).
–.balloon.valvuloplasty.(in.very.young).
Interventional.Options.
Percutaneous.valve.replacement.(transfemoral.or.transapical.approach).
is.an.option.in.selected.patients.who.are.not.considered.good.candidates.for.surgery.
..
13..Congestive.heart.failure.with.systolic.dysfunction,.what’s.the.treatment?.
.Answer:.if.Acute.K>.lasix.ACEI.or.ARBs.and.Correcting.underlying.cause.
Chronic.K>.B.blockers.ACEI.or.ARBs.diurteics.ie.lasix.or.spironolaxtin.
Aspirin.and.statins.if.Previous.MI.
..
..
14..ECG.showing.ST.elevation.MI,.what.is.the.most.appropriate.management?.
AK.LMWH.
BK.Streptokinase.
CK.Thrombolysis.
DK.Stenting.
Answer:.depends.on.duration..If.within.90.minutes.stenting.
..
15..19.years.old.female.with.arthralgia.and.pansystolic.murmur.radiating.to.the.axilla..ESR.is.high..
Which.one.of.the.following.is.the.most.likely.diagnosis?.
AK.SLE.
BK.Miliary.TB.
CK.Hodgkin's.lymphoma.
.Answer:.SLE.
..
16..Severe.aortic.regurgitation..Other.findings?.
AK.Collapsing.pulse.
BK.Low.raising.pulse.
CK.Pulsus.paradoxus.
DK.Pulsus.bisferiens.
Answer:.Pulsus.bisferiens.
..
17..Right.bundle.branch.block?.
..
18..Case.of.cardiac.patient,.he.started.to.develop.sign.and.symptoms.of.heart.failure............ .............
which.one.of.the.following.will.prolong.patient.life?.
AK.ACEIs.
BKdigoxin.
CKdiltiazem.
DKdiuretic.
Answer:.ACEI.
..
19..ECG.showing.inferior.MI,.which.one.of.the.following.arteries.blocked?.
AK.Right.coronary.artery.
BK.Left.coronary.artery.
CK.Circumflex.
.Answer:.RCA.
..
..
20..Patient.with.anterolateral.MI.(ECG.was.provided),.presented.within.45.minutes.and.is.going.for.
cardiac.catheterization..He.was.given.aspirin.and.oxygen.at.the.ER..What.are.you.going.to.give.him.
before.the.procedure?.
AK.Morphine.and.nitroglycine.
BK.Small.dose.of.beta.blocker.in.the.IV.infusion.
CK.Normal.saline.with.pacemaker.
DK.Dopamine.and.nonKepinephrine.
.Answer:.Morphine.and.nitroglycine.
..
21..Systolic.ejection.murmur.in.aortic.area.radiating.to.the.neck,.and.the.patient.has.SOB,.what.is.
the.cause?.
AK.Congestion.in.systemic.venous.
BK.Pulmonary.congestion.
CK.Pulmonary.stenosis.
.Answer:.Pulmonary.congestion.
..
22..Patient.thin.and.tall.and.has.chest.deformity..He.came.to.the.doctor.because.2.of.his.brothers.
died.at.a.young.age,.you.are.afraid.of.which.of.the.following?.
AK.Hypertrophic.cardiomyopathy.
BK.Aortic.root.aneurism.
Answer:.Aortic.root.aneurysm.(Marphan).
..
23..Case.of.chest.pain.since.1.hour..What.enzyme.will.confirm.MI?.
AK.Troponin.
BK.CK.
CK.Lactate.dehydrogenase.
DK.Aspartate.dehydrogenase.
EK.Myoglobin.
Answer:.Troponin.(confirm.MI.but.can.take.up.to.6.Hrs.to.rise).
So.Myoglobin..
..
24..Which.of.the.following.breaks.the.thrombus.in.MI.
AK.Warfarin.
BK.Heparin.
CK.Aspirin.
DK.Streptokinase.
.Answer:.Streptokinase.
..
25..Murmur.heard.best.in.the.2nd.right.left.sternal.border..Which.valve.is.stenosed.?.
AK.Aortic.
BK.Pulmonic.
CK.Tricusped.
DK.Mitral.
Answer:.if.rightK>.aortiv.
leftK>.pulmonary.
..
26..Scenario.with.obese,.heavy.smoker.(20.cigarettes.daily)..Greatest.CVS.risk.factor?.
AK.Age.
BK.Smoking.
CK.Obesity.
Answer:.Obesity.
..
27.Best.treatment.for.symptomatic.acute.rheumatic.fever?.
AK.Penicillin.Q6.hours.
BK.Acetylsalicylic.acid.
CK.Steroids.
DK.Single.dose.of.benzathine.penicillin.
Answer:..Single.dose.of.benzathine.penicillin.
..
28..MI.treatment.
..
29..Case.of.AF.long.term.prophylaxis.
AK.Warfarin.
BK.Beta.blocker.
.Answer:.Warfarin.
..
30..Patient.came.to.ER,.retrosternal.chest.pain,.lasting.for.15.min,.relieved.by.rest,.not.compliant.on.
his.medications..Vitally.stable,.mild.HTN..What.is.best?.
AK.ECG.stress.test.
BK.Cardiac.enzymes.and.follow.up.in.clinic.
Answer:.none.of.the.provided.choices..(ECG.and.Cardiac.enzymes).
..
31..Patient.with.only.palpation,.examination.normal.and.ECG.shows.heart.block.what.is.the.
treatment.
AK.CCB.
BK.Beta.blocker.
CK.Digoxin.
Answer:.depending.on.degree.of.block..if.second.type.II.treatment.is.Pacemaker.
..
32..Common.cardiac.anomaly.associated.with.infective.endocarditis?.
AK.ASD.
BK.VSD.
C.K.TOF.
DK.Truncus.arteriosus.
Answer:.TOF.
..
33..Long.scenario.Of.patient.with.coronary.artery.disease,.what.is.the.appropriate.level.of.LDL.for.
him?.
AK.3.5.K.4.1.
BK.2.4.K.3.4.
Answer:.2.4.K.3.4.
..
34..Which.of.the.following.is.protective.from.heart.diseases?.
AK.HDL.
BK.LDL.
CK.ILD.
DK.HLDL.
Answer:.HDL.
..
35..Which.one.of.the.following.has.more.risk.for.CAD?.
AK.LDL.
BK.HDL.
CK.Triglycerides.
DK.Total.cholesterol.
Answer:.LDL.
..
36..Patient.with.mitral.regurgitation,.what.will.delay.the.need.for.surgery?.
AK.Nifedipine.
BK.ACEIs.
CK.BB.
Answer:.ACEI.
..
37..Man.with.aortic.stenosis.developed.syncope.what.is.the.cause?.
AK.Systemic.hypotension.
..
38..Coarctation.of.aorta.associated.with.which.of.the.following?.
AK.Turner.syndrome.
..
39..Patient.is.hypertensive.in.upper.extremities.and.absent.pulse.in.lower.extremities..What.is.the.
diagnosis?.
AK.Coarctation.of.aorta.
..
40..ECG.showing.second.degree.heart.block,.and.asked.about.the.diagnosis.
..
41..ECG.showing.MI.in.one.of.the.leads,.and.asking.about.the.location.of.the.infarction.
..
42..Patient.with.heart.failure.on.ACEI,.furosemide,.spironolactone,.and.other.drugs..Now.stable.and.
clinically.free,.what.drug.modification.is.appropriate?.
AK.Stop.spironolactone.
..
50..Best.treatment.plan.for.Infective.endocarditis?.
AK.Multiple.Abx.high.dose.for.a.short.time.
BK.Multiple.Abx.extended.over.a.long.period.of.time.
Answer:.Multiple.Abx.extended.over.a.long.period.of.time.
..
..
51..Young.patient.with.BP.160/110,.high.creatinine,.++.proteinuria..What.is.the.drug.of.choice.for.
HTN?.
AK.ACEI.
..
52..VT.on.ECG.+.hypotensive..What.is.the.management?.
AK.Amiodarone.
BK.Adenosine.
CK.DC.shock.
Answer:.DC.shock.
..
53..Man.with.chest.pain.and.transient.LOC.x4.times,.CXR.given..What.is.the.management?.
..
..
54..A.31.yearKold.female.T1DM.presented.to.ED.with.syncope.one.hour.before..She.has.had.
shortness.of.breath.and.chest.pain..
Vital.signs:.hypotension,.tachycardia..
ECG.picture.attached.showing.V3KV4.STE..
What.is.the.diagnosis?.
AK.Pulmonary.embolism.
BK.Lateral.MI.
CK.Wolf.parkinson.white.syndrome.
Answer:.Anterior.MI.
..
..
55..Which.of.the.following.is.wide.complex.tachycardia?.
AK.Afib.with.high.ventricular.rate.
BK.Atrial.flutter.2:1.
CK.Monomorphic.ventricular.tachycardia.
DK.Paroxysmal.supraventricular.tachycardia.
Answer:.Monomorphic.ventricular.tachycardia.
..
56..Obese.patient.with.chest.pain.on.exertion,.what.to.do?.
AK.Treadmill.
BK.Myocardial.perfusion.test.
CK.Coronary.angiography.
.Answer:.Myocardial.perfusion.test.
..
57..ECG.of.2nd.degree.AV.block,.next.step?.
AK.Lidocaine.
BK.Atropine.(if.pacemaker.not.in.the.choices).
CK.Electro.cardioversion.shock.
.Answer:.Atropine.(if.pacemaker.not.in.the.choices).
..
58...HTN.patient.on.ACEIs,.not.controlled,.what.to.add?.(No.CCB.in.choices).
AK.Furosemide.
BK.Beta.blocker.
CK.Thiazide.
Answer:.Thiazide.
..
59..Drug.that.prolongs.survival.in.CAD..
.ACEI.
..
60..What.is.the.drug.that.increases.survival.rate.in.aortic.stenosis?.
AK.Digoxin.
BK.Captopril.
CK.Calcium.channel.blocker.
Answer:.Could.not.find.a.definite.answer.
..
61..Attacks.of.migraine.and.syncope.(not.together)..Family.history.mother.died.due.to.cardiovascular.
cause.at.age.of.35.years..Decreased.sounds.over.carotids.on.auscultation..Vitals.show.hypertension..
AK.Carotid.Doppler.
BK.Blood.pressure.monitoring.
Answer:.Carotid.Doppler.(fibromuscular.dysplasia)..
..
62..Treatment.of.AF.
AK.Beta.blocker.
..
63..Most.likely.scenario.in.rheumatic.fever?.
AK.History.of.URTI.followed.by.knee.joint.swelling.
BK.Painful.finger.joints.and.fever.
Answer:.History.of.URTI.followed.by.knee.joint.swelling.
..
64..Heart.failure.what.is.the.treatment?.
AK.digoxin.
..
65..In.MI.which.lab.test.what.will.be.increased?.
AK.ALP.
BK.CK.
Answer:.CKKMB.
..
66..case.of.old.man.with.chest.pain.and.transient.loss.of.consciousness.(3K4.episodes)..CXR.and.
asked.about.management?.
..
.
67..Cardiac.syncope.what.is.characteristic.of.it?.
AK.Rapid.recovery.
BK.Aura.
CK.Neurological.defect.
Answer:.Rapid.recovery.
..
68..Patient.has.history.of.transit.angina..Now.he.is.asymptomatic.and.doing.well.but.his.ECG.shows.
AF..What.is.your.action?.
AK.Reassure.
BK.Give.digoxin.
CK.Give.antiKcoagulation.
Answer:.Give.anticoagulation.after.calculating.CHADSVASc.score.
..
69..Long.history.of.a.man.with.systolic.murmur.at.....sternal.border..Recently.develop.syncope..What.
is.the.diagnosis?.
AK.AS.
..
70..77.years.old.male.with.early.onset.of.dyspnea.when.moving.50.meters,.has.left.apical.heave.and.
loud.audible.systolic.murmur.most.intense.to.hear.in.left.sternal.border..What.is.the.diagnosis?........................................
. .
AK.Sever.mitral.regurgitate.
BK.Calcified.aortic.stenosis.
CK.Mitral.stenosis.
.Answer:.Calcified.aortic.stenosis.
..
71..Old.patient.present.with.angina.symptoms,.presyncope.and.plural.effusion..Echo.shows.AS,.valve.
is..75.mm..What.is.your.management?.
AK.Medical.management.................................... ............ .
BK.Surgical.repair.
CK.Ballon.valvoplasty.
DK.Observation.
Answer:.Surgical.repair.
..
..
72..Patient.complaining.of.SOB.on.exertion.and.chronic.cough,.on.examination.there.is.
hepatomegaly..Best.test.to.determine.the.cause?.
AK.Alpha.1.antitrypsin.level............................................................... .
BK.ECHO.
Answer:.needs.more.details.it.can.be.heart.failure.or.Alpha.1.antitrypsin.
..
73..History.of.angina.that.become.more.frequent.and.lasted.longer.time.without.elevation.of.cardiac.
enzymes?.
AK.Unstable.angina.
..
..
74..Young.patient.came,.tell.you.2.of.his.brothers.died.when.they.running,.you.are.afraid.of.which.of.
the.following?.
AK.Hypertrophic.cardiomyopathy.
..
..
75..Patient.with.MI.and.in.the.5th.day.he.deteriorated?.
AK.Acute.mitral.regurgitation.
..
76..Patient.with.atrial.fibrillation.and.you.described.for.him.a.medication.what.is.the.mechanism.of.
action.of.that.medication?.
Warfarin:.vitamin.K.carboxylase.inhibitor.
..
77..Which.type.of.congenital.heart.disease.is.least.associated.with.infective.endocarditis?.
AK.ASD.
..
78..Patient.has.chest.pain.on.left.side..Examination.showed.pleuritic.chest.pain.and.friction.rub.on.
left.side..What.is.the.next.step?.
AK.Chest.xKray.
BK.12.lead.ecg.
CK.Echo.
DK.Refer.to.cardio.
Answer:.ECG.
..
79..Patient.with.symptoms.of.endocarditis.but.cultures.are.negative..What.is.the.cause?.
AK.SLE.
..
80..A.33.years.old.man,.smoker,.overweight,.inactive.present.with.heart.problem,.which.of.the.
following.is.most.commonly.associated.with.heart.disease?......................................... ................
AK.Smoking.
BK.Obesity.
CK.Inactivity.
.Answer:.Obesity.
..
81..Diabetic.patient.with.chest.pain.for.3.days,.dyspnea.with.exertion,.history.of.URTI..ECG:.T.
depression..Labs:.high.troponin..What.is.the.diagnosis?.
AK.Myocarditis.
BK.MI.
CK.Constrictive.pericarditis.
.Answer:.Myocarditis.
..
82..Which.drug.if.you.stop.abruptly.will.cause.hypertensive.crisis?.(No.beta.blocker.in.choices).
AK.Clonidine.
..
83..What.type.of.heart.failure.occurs.with.high.cardiac.output?.
..
84..Patient.with.central.chest.pain,.CXR.showed.pleural.effusion.and.ECG.showed.ST.elevation.in.4.
leads..What.is.the.diagnosis?.
AK.MI.
BK.Pericarditis.................. ..
CK.Pneumothorax.
DK.Pneumonia.
Answer:.MI..
..
85..Young.girl.come.with.HTN.crisis,.antihypertensives.are.not.effective,.She.has.history.of.
hypotension.during.appendectomy?.
AK.Pheochromocytoma.
BK.Renal.artery.stenosis.
CK.Renal.vein.thrombosis.
DK.Essential.HTN.
Answer:Pheochromocytoma.
..
86..Post.MI.patient.wants.to.drive..When.will.he.be.able.to.drive?.
AK.4.weeks.
BK.1.week.
Answer:.6.weeks..
..
87..History.of.Palpitation.&.SVT.(ECG)..What.is.the.treatment?.
AK.Deltiazim.
..
88..Picture.of.ECG:.bradycardia,.on.history.he.has.only.palpitation..What.is.the.treatment?.
AK.Digoxin.
BK.Propranolol.
CK.ACEI.
Asnwer:.none.
..
89..clear.scenario.about.essential.hypertension.
..
90.#After#ER#management#of#MI,#what’s#the#next#step?#
A..primary.stenting.
B..give.tPA.
C..give.streptokinase.
Answer:.depends.on.duration.
..
..
91.#A#patient#after#MI#when#can#he#go#back#to#his#regular#routine#or#exercise?#
1).1.week.
2).3.weeks.
3).5.weeks.
Answer:.6.weeks.
##
##
92.#Right#sided#heart#failure#symptoms.#Echo#done#which#was#normal.#
1).coxsackie.B.
2).amyloidosis.
3).Constrictive..pericarditis.
Answer:.Coxackie.B.virus.causing.myocarditis.
..
93.#ECG#shows#inferior#MI,#what#artery#is#occluded#
K.right.coronary.
K.left.coronary.
K.circumflex.
K.conus.
.Answer:.RCA.
..
94..Which.of.those.are.in.diagnosis.of.infective.endocarditis.criteria.considered.major?.
..
95..Best.treatment.for.acute.myocarditis?.
AK.Penicillin.
BK.Corticosteroids.
.Answer:.Supportive.
..
96..Most.common.electrolyte.disturbance.in.a.patient.with.digoxin?.
AK.K.
BK.Na.
Answer:.K.
..
97..Drug.which.decreases.HR.and.Pre.and.after.load?.
AK.Carvidilol.
BK.Nifidepine.or.Amlodepine.
Anwer:.CCB.
..
..
98..Most.common.cause.of.secondary.HTN?.
AK.Renal.disease.
..
99..Diabetic.patient.with.chest.pain.for.3.days,.dyspnea.with.exertion,.and.history.of.URTI..
ECG.shows.T.depression.
Labs:.high.troponin.
What.is.the.diagnosis?.
AK.Myocarditis.
B.K.MI.
C.K.Constrictive.pericarditis.
Answer:.Myocarditis..
..
100..Patient.present.to.the.ER.with.typical.symptoms.of.Ischemic.heart.disease,.he.had.history.of.
erectile.dysfunction..He.is.on.PDEK5.inhibitor.(sildenafil),.which.of.the.following.drug.should.not.be.
given?.
AK.Nitroglycerine.
BK.Metopralol.
CK.Dilitazem.
.Answer:.Nitroglycerine.
..
..
101..Which.one.of.the.following.carries.more.risk.for.CAD?.
AK.55,.male,.DM.
BK.50,.male,.hyperlipidemia.
CK.HTN,.obese.
.Answer:55,.male,.DM.
..
..
102..52.Years.old.female.k/c.of.DM,.recently.diagnosed.with.HTN..Examination.was.normal.except.
for.mild.lower.limb.edema..What.you.will.give?.
AK.Propranolol.
BK.Perindopril.
CK.Atenolol.
DK.Hydrochlorothiazide.
.Answer:.Perindopril..
.
103..Malignant.hypertension.is?..........
..
104..Adult.patient.presented.with.chest.pain.and.palpitation,.ECG.showed.atrial.fibrillation..Vitals:.
HR:.140.BP:.80/50..How.to.treat?.
AK.Digoxin.
BK.Synchronized.Cardioversion...............................................
CK.Beta.blocker.
..
105..Patient.with.hyperthyroidism,.with.palpitation,.what.is.the.most.likely.finding.in.his.ECG?..
a).SVT.................................................
b).VT.
c).A.FIB.
.

Rheumatology#
##
.1.#Symptoms#of#diffuse#systemic#sclerosis.#What#is#the#antibody?#
AK.Anticentromere.antibody.
BK.AntiKtopoisomerase.I.(antiKSclK70).antibodies.
Answer:.B.
..
2.#Tall#&#thin#boy#with#joint#laxity,#also#his#mother#has#the#same#features.#What#is#mode#of#
inheritance?#
Answer:.Autosomal.dominant.(Marfan.syndrome)..
..
3.#Boutonniere#deformity#description?#
Answer:.Flexed.PIP.and.hyperextended.DIP..
..
4.#(REPEATED)#Ehler[Danlos#mode#of#inheritance?#
Answer:.Autosomal.dominant.
..
5.#Case#of#RA#asking#about#genetics?#
Answer:.HLA.B27.
HLA.B27.is.associated.with.seronegative.spondyloarthropathies.(ex:.ankylosing.spondylitis)..
..
6.#Clear#scenario#of#Raynaud's#phenomenon#
..
7.#SLE#prognosis#
Answer:.Degree.of.Renal.involvement.
Medscape:.The.disease.course.is.milder.and.survival.rate.higher.in.persons.with.isolated.skin.and.
musculoskeletal.involvement.than.in.those.with.renal.disease.
##
8.#Polymyalgia#rheumatica#is#associated#with?#
Answer:.Proximal.stiffness.
Toronto.notes:.characterized.by.pain.and.stiffness.of.the.proximal.extremities.
..
9.#Antiphospholipid#syndrome#
..
10.#Which#of#the#following#is#diagnostic#of#lupus?#
AK.AntiKRNP.antibodies.
BK.Cystoid.bodies.on.fundoscopy.
CK.Severe.Raynaud's.phenomena.
Answer:.AntiKRNP.associated.with.both.mixed.connective.tissue.disease.and.SLE..
“Cytoid”.bodies.(AKA.cotton.whool.spots).associated.with.inflammatory.and.ischemic.conditions..
..
11.#Dermatomyositis#associated#with#what?#
Answer:.Malignancy.
Toronto.notes:.malignancy.screening.part.of.management..
..
12.#Polymyalgia#rheumatica,#what#are#the#x[Ray#findings?#
AK.Calcification.in.joints.
BK.Normal................................................................. .
Answer:..B..
Uptodate:.While.subdeltoid/subacromial.bursitis.is.a.characteristic.imaging.feature.of.PMR,.it.is.not.
specific.and.is.seen.in.patients.with.rheumatoid.arthritis.(RA).and.other.shoulder.pathology..
..
13.#Most#specific#test#for#rheumatoid#arthritis?#
AK.CRP.
BK.RF.
CK.HLA.b27.Antidnase.
DK.antiKCCP........ .
Answer:.D.
Toronto.notes:.AntiKCCP.specific.for.RA.(94K98%).
..
14.#Female#with#new#onset#Raynaud's#phenomena#and#upper#GI#endoscope#showed#gastric#
vascular#ectasia#(something#like#that)#what#is#positive#in#blood?#
AK.Anticentromere.
BK.Anti.Scl70.
Answer:.B.
AntiKScl70.(=antiKtopoisomerase).>.systemic.scleroderma..
AntiKcentromete>.CREST..
..
15.#Patient#with#gout,#what#should#be#measured?#
Answer:.Uric.acid.level.
..
16.#Polymalegia#+#polymyositis#
..
17.#Patient#with#recurrent#renal#stones#on#HTN#medication#to#reduced#calcium#excretion#and#cause#
gout?#
AK.Hydrochlorithiazide.
BK.Furosemide.
.Answer:.A.
..
18.#Case#of#gout#and#the#answer#was#indomethacin#
..
19.#Patient#develop#back#pain#in#morning,#pain#improve#with#walking#and#movement.#He#had#the#
same#complain#in#past#1#year.#What#is#the#diagnosis?#
AK.Ankylosing.spondylitis.
BK.Recurrent.gout.
Answer:.A.
..
20.#Patient#with#gout,#which#of#the#following#will#be#inhibited#by#the#drug#you#will#give?#
AK.Xanthine.oxidase.
BK.PRPP.
Answer:.A.
..
21.#Behçet#disease#HLA#association#
Answer:.HLA.B51.
.Medscape:.Carriers.of.HLAKB51/HLAKB5.have.an.increased.risk.of.developing.Behçet.disease.
compared.with.noncarriers..
..
22.#Symmetrical#MCP#&#PIP#pain#increase#at#morning#
Answer:.Rheumatoid.arthritis.
..
23.#Patient#with#oral#and#genital#ulcer#and#arthritis.#What#is#the#diagnosis?#
Answer:.Bechet.disease.
..
24.#Patient#with#bilateral#shoulder#and#hip#stiffness#and#ask#about#the#diagnosis?#
Answer:.Polymyalgia.rheumatica.
.Depends.on.rest.of.scenario..Most.important.characteristic.is.:stiffness.and.pain.without.weakness..
..
25.#Case#about#RA#came#with#bilateral#swelling#in#DIP#due#to?#
Answer:.Synovial.thickening..
..
26.#Scenario#about#a#little#girl#who#had#high#ESR#and#high#ANA#and#no#obvious#symptoms.#What#is#
the#diagnosis?#
Answer:.Latent.lupus.
..
27.#Patient#with#uric#acid#calculi,#with#normal#uric#acid#level,#how#to#prevent#recurrence#of#calculi:#
AK.Allopurinol.
BK.Probenecid.
CK.Disodium.citrate.
Answer:.
Toronto.notes:.Treatment>.increase.fluid.intake,.alkalinization.of.urine.±.allopurinol..Hyperuricosuria.
not.necessarily.present..
..
28.#(REPEATED)#Most#specific#test#for#rheumatoid#arthritis?#
A..CRP.
B..RF.
C..HLA.27.
D..AntiKDNAse.
.Answer:.
..
29.#Patient#with#sudden#severe#shoulder#and#neck#pain#with#stiffness,#also#hip#and#lumbar#spine#
pain,#also#complain#of#fatigue#and#increase#sleepiness,#ESR#was#90.#
AK.Cartilaginous.tissue.degeneration.
BK.Antigen.antibody.mediated.arthritis.
CK.Soft.tissue.with.giant.cell.arteritis.
DK.Connective.tissue.disease.something.
.Answer:C.
..
30.#Case#of#Wegener’s#but#asked#about#the#basic#pathology#of#disease#
..
31.#Marfan#syndrome#features#of#the#mother#and#her#boy:#what#is#the#probability#of#the#children#
to#have#it?#
AK.25%.
BK.50%.
CK.75%.
DK.100%.
Answer:.B.
Autosomal.dominant..
..
32.#Female#presented#with#hand#joints#swelling,#she#was#informed#by#physician#that#she#has#bone#
loss,#she#is#angry#about#her#permanent#bone#loss.#What#is#the#explanation#of#bone#loss?#
AK.Increase.pressure.in.joint.space.
BK.Material.secreted.from.synovial.fluid.
CK.Drug.induced.menopause.
Answer:.B.
.
##
33.#Patient#with#osteoarthritis#and#they#ask#about#the#name#of#nodes#over#the#PIP#joint?#
AK.Heberden.nodes.
BK.Bouchards.nodes.
Answer:.B.
.Heberden’s.>.DIP..
..
34.#Case#about#back#pain#and#its#management#
#
35.#Male#patient#with#oral#ulcers,#abdominal#pain,#nausea,#diarrhea,#severe#headache#and#polyarthritis.#(Attached#
a#pictures#of#penile#ulcer#and#ulcers#over#the#buccal#mucosa).#What#is#the#diagnosis?##
AK.Behcet..
BK.Syphilis.
CK.Ulcerative.colitis.
##
36.#Patient#presented#with#right#knee#pain#and#swelling,#arthrocentesis#done:#
Color:#yellow#
Viscosity:#low#############################################################
WBC:#15.2#
Clarity:#opaque#(Didn't#mention#anything#about#crystals#in#the#aspiration)#
What#is#the#diagnosis?#
AK.Gout.
BK.Septic.arthritis.
CK.Rheumatoid.arthritis.
DK.Pseudo.gout.
Answer:..
.
37.#Somethings#about#dermatomysitis.#Rash#+#
A.! Proximal.muscle.weakness.
B.! Tenderness.of.muscle.
.Answer:.A..
Heliotrope.rash.(periorbital),.Gottron’s.papules.(violaceous.papules.over.knuckles.and.IP.joints).±.poikiloderma..
Shawl.sign:.macular.erythema.over.chest.and.shoulder..Proximal.muscle.weakness.±.pain..Dyspnea.on.exertion..
Toronto.notes..
.
.

Gastroenterology##
..
1.#how#to#differentiate#a#huge#ovarian#mass#from#an#ascites?#
Answer:.Anterior.dullness.and.lateral.Tympani.
..
2.###########Vinson#Plummer#syndrome?#
..
PlummerKVinson.Syndrome.Triad.
•.Iron.deficiency.anemia.
•.Dysphagia.
•.Esophageal.webs.
*.rare.(prevalence.<1.in.1,000,000).but.good.prognosis.when.treated.with.iron.and.esophageal.
dilatation..Toronto.
..
3.###########Patient#complaining#of#abdominal#pain#I#think#with#diarrhea#,#also#his#wife#noticed#he#had#
SOB#and#tightness.#Doctor#order#5[hydroxyindoleacetic#acid#in#urine#Which#cell#responsible?#
A.chromaffin.cell.
B.Enterocell.
C.Lympho.cell.
D.Goblet.cell.
Answer:.A

.
.
4.###########which#markers#of#HBV#is#present#in#window#phase:#
Answer:.HBcAb....
First.aid.step.2.p157.
..
5.###########what's#the#most#common#cause#of#gastroenteritis?#
1).Noro.virus.
2).rotavirus.
Answer??.
For.acute..viral.gastroenteritis:.
In.infants,.most.cases.are.due.to.rotavirus..
In.adults,.the.most.common.cause.is.norovirus..
Medscape.and.ACG.
..
6.###########Causes#of#aphthus#ulcer:#
Celiac.disease.
Ibs.
Bilirubin.retention.
Answer:.A.
Malabsorption.in.gastrointestinal.disorders:.About.3%.of.patients.experience.these.disorders,.
particularly.celiac.disease.(glutenKsensitive.enteropathy).but,.occasionally,.Crohn.disease,.pernicious.
anemia,.and.dermatitis.herpetiformis..HLA.DRW10.and.DQW1.may.predispose.patients.with.celiac.
disease.to.oral.ulceration..
medscape.
..
7.###########Which#drug#will#decrease#baseline#and#acid#secretion#from#stomach:#
K.......Ranitidine.(decreases.secretion).
K.......PPI.
K.......Other.choices.that.I.don’t.remember..
Answer:.B.
..
8.###########Tumor#marker#for#pancreatic#cancer?#
Ca.19.
..
9.###########Patient,#with#diarrhea,#flagellated#protozoa,#how#it#cause#diarrhea?#
A............increase.secretion.of.fluids.
B............kills.normal.flora.
C............coats.the.small.bowl.
D........... absorption.is.prevented.
..
Seems.like.he’s.talking.about.giardia...so:..A.or.D.not.sure.
The.mechanisms.by.which.Giardia.causes.diarrhea.and.intestinal.malabsorption.are.probably.
16].
multifactorial.and.not.yet.fully.elucidated..[ Postulated.mechanisms.include.damage.to.the.
endothelial.brush.border,.enterotoxins,.immunologic.reactions,.and.altered.gut.motility.and.fluid.
hypersecretion.via.increased.adenylate.cyclase.activity..
..
10.##knee#pain#old#age#with#NSAID#3#weeks.#Come#with#bloody#vomitting.#Reason?#
PUD.caused.by.NSAID.
..
11.##Young#Male#have#epigastric#pain,#fullness,#nausea,#postprandial#bloating#for#three#years.#His#
symptoms#appeared#after#he#stayed#in#India#for#one#year#for#studying.#Symptoms#are#worse#with#
food.#What#is#the#diagnosis?#
A..Functional.dyspepsia.
B..H..Pylori.Infection.
C..Esophageal.ulcer..
Answer:.B.most.likely.
..
12.##Pt#treated#for#peptic#ulcer#with#triple#what#u'll#do?#
A............Knothing.
B............KH.pylori.antigen.test.
C............–endoscopy.
(not.sure)..If.in.the.scenario.there’s.symptoms.refractory.to.medication..C.to.rule.out.gastric.cancer.
or.cancer.transformation..All.gastric.ulcers.must.be.biobsied.....First.aid.2.
..
13.##Peptic#ulcer#treated#with#triple#therapy#after#3#weeks.#therapy#now#she#is#free#of#sx.#what#to#
do?#
A............Endoscopy.
B............h.pylori.antigen.
Answer:.B.
American.college.of.gastroenterology.2017:.
Should+we+test+for+teatment+success+after.H..pylori.eradication+therapy?+

Whenever.H.+pylori.infection.is.identified.and.treated,.testing.to.prove.eradication.should.be.
performed.using.a.urea.breath.test,.fecal.antigen.test.or.biopsyKbased.testing.at.least.4.weeks.
after.the.completion.of.antibiotic.therapy.and.after.PPI.therapy.has.been.withheld.for.1–2.
weeks..(Strong.recommendation;.Low.quality.of.evidence.(for.the.choice.of.methods.to.test.for.
eradication:.Moderate.quality.of.evidence))..

..

What+are+the+indications+to+test+for,+and+to+treat,.H..pylori.infection?+

Since.all.patients.with.a.positive.test.of.active.infection.with.H.+pylori.should.be.offered.
treatment,.the.critical.issue.is.which.patients.should.be.tested.for.the.infection.(strong.
recommendation;.quality.of.evidence.not.applicable)..
All.patients.with.active.peptic.ulcer.disease.(PUD),.a.past.history.of.PUD.(unless.previous.cure.
of.H.+pylori.infection.has.been.documented),.lowKgrade.gastric.mucosaKassociated.lymphoid.
tissue.(MALT).lymphoma,.or.a.history.of.endoscopic.resection.of.early.gastric.cancer.(EGC).
should.be.tested.for.H.+pylori.infection..Those.who.test.positive.should.be.offered.treatment.
for.the.infection.(Strong.recommendation;.quality.of.evidence:.high.for.active.or.history.of.
PUD,.low.for.MALT.lymphoma,.low.for.history.of.endoscopic.resection.of.EGC)..

In.patients.with.uninvestigated.dyspepsia.who.are.under.the.age.of.60.years.and.without.alarm.
features,.nonKendoscopic.testing.for.H.+pylori.infection.is.a.consideration..Those.who.test.
positive.should.be.offered.eradication.therapy.(conditional.recommendation;.quality.of.
evidence:.high.for.efficacy,.low.for.the.age.threshold)..

When.upper.endoscopy.is.undertaken.in.patients.with.dyspepsia,.gastric.biopsies.should.be.
taken.to.evaluate.for.H.+pylori.infection..Infected.patients.should.be.offered.eradication.
therapy.(strong.recommendation;.high.quality.of.evidence)..

..

15.##Sudden#severe#generalized#abdominal#pain#in#a#patient#known#to#have#A#Fib:#
Answer:.Mesenteric.ischemia.
..
16.##Treatment#of#Wilson#disease#
penicilamine.or.trientine..and.zinc.but.should.space.the.doses.
..
18.##Healthy#women#asymptomatic#on#regular#check#up#found#to#have#HB#antigen#positive#?#
1Kacute.hepatitis.
2Kchronic.hepatitis.
3Kacute.carrier.
4Kchronic.carrier.
Answer:.A.Not.sure..Depends.what.other.ones.they.did.or.didn’t.do.
##
#
#
#
#
#
#
#
#
#
#
#
#
#
19.##What#is#the#mechanism#of#diarrhea#
A............Decrease.fluid.intake.
B............Decrease.fluid.absorption.
C............Increase.fluid.intake.
D........... Increase.fluid.absorption.
Answer:.B.
..
Q.#What#is#the#mechanism#of#diarrhea#with#patient#has#gardia#infection#?#
1K.reduction.of.absorption.
2K.increase.secretion.of.mucosal...................................................................... .
Answer:.reduction.of.absorption.most.likely.but.2.could.be.right.
..
20.##Case#of#achalasia#clear#sentence#of#bird#like#appearance#on#xray#
..
21.##U#want#to#start#statin#for#a#patient#what#test#u#should#order?#
AKLFT.
BKGGT.
Amswer:.A.
Medscape.
..
22.##Most#common#site#for#crohns?#
Ileocecal.area.
..
23.##Best#test#to#diagnose#hepatitis#B?#
AK.US.
BKblood.test.
CKblood.culture.
DKliver.biopsy.
Answer:.B.
..
24.##Retrosternal#pain#and#barium#swallow#showed#esophageal#corkscrew#appearance?#
AKachalasia.
BK.Esophageal.cancer.
CKdiffuse.esophageal.spasm.
Answer:.C.
..
25.##BRUNNER's#gland#location?#
Answer:.submucosal.duodenum.
..
26.##What#is#the#drug#that#given#with#analgesia#to#decrease#side#effect:"#‫ﻟﻨﺺ‬$ $‫ﺑ&ﺬ‬#"#
A).cimetidine.
B).Metoclopramide..
C).misoprostol.
Answer:.depends.what.they.think.of.as.analgesia….if.it’s.NSAIDS,.choice.C.would.be.good.prophylaxis.
for.ulcer.
..
27.##Which#one#of#the#following#GIT#cell#secret#defensins:#
A:.paratial.cell.
B:.chief.cell.
C:.paneth.cell.
D:.plasma.cell.
Answer:.C.
..
28.##Scenario#of#hepatorenal#syndrome:#
renal.failure.in.cirrhosis.
Classifications#
.preKrenal.(usually.due.to.overKdiuresis).
.acute.tubular.necrosis.
.HRS.
–.Type.I:.sudden.and.acute.renal.failure.(rapid.doubling.of.creatinine.over.2.wk).
–.Type.II:.gradual.increase.in.creatinine.with.worsening.liver.function.(creatinine.doubling.
over.years).
HRS.can.occur.at.any.time.in.severe.liver.disease,.especially.after.
–.overdiuresis.or.dehydration,.such.as.diarrhea,.vomiting,.etc..
–.GI.bleed.
–.sepsis.
Treatment#for.hepatorenal.syndrome.(generally.unsuccessful.at.improving.longKterm.survival).
–.for.type.I.HRS:.octreotide.+.midodrine.+.albumin.(increases.renal.blood.flow.by.increasing.
systemic.vascular.resistance).
–.definitive.treatment.is.liver.transplant.
TORONTO.
..
29.##Scenario#of#zollinger#ellison#syndrome#
GastrinKproducing.neuroendocrine.tumour.that.causes.gastric.acid.hypersecretion.and.peptic.ulcer.
disease..
Å.......Can.be.sporadic.or.associated.with.multiple.endocrine.neoplasia.syndrome.type.1..
Å.......Common.presentation.includes.refractory.peptic.ulcer.disease,.abdominal.pain,.
diarrhoea,.and.gastroKoesophageal.disease..
Å.......The.main.goal.of.treatment.is.control.of.gastric.hypersecretion.with.protonKpump.
inhibitors..
Å.......The.most.common.cause.of.morbidity.and.mortality.is.metastatic.gastrinoma..
..
30.##Orilstat#Mechanism?#
Inhibits.gastric.and.pancreatic.lipases,.prevents.triglyceride.hydrolysis.resulting.in.decreased.
absorption.of.dietary.fats.
..
31.##About#watery#diarrhea#without#blood#and#asked#about#organism?#
Answer:.C..Defficile.(.Not.sure.).
Depends.what.other.options.were.there..Most.cases.of.acute,.watery.diarrhea.are.caused.by.viruses.
(viral.gastroenteritis)..The.most.common.ones.in.children.are.rotavirus.and.in.adults.are.norovirus..
The.ACG.
..
32.##Vit.c#and#iron#absorption?#
Iron.absorption.from.diet.is.enhanced.in.the.presence.of.vitamin.C.and.diminished.by.excess.
calcium,.zinc,.or.manganese.
..
34.##pt#complain#of#constipation#on#abdominal#examination#there#is#a#mass#in#the#lower#abdomen#
you#did#US#which#confirmed#the#presence#of#a#mass#in#which#you#decided#to#do#colonoscopy#which#
find#2#polyps#with#high#grade#hyperplasia#,#when#you#will#follow#up#the#pt#by#colonoscopy?#
AKafter.1.year,.
BK3.yrs.,.
CK8.yrs.
Answer:.A.
Toronto,.after.1.yr.then.every.3K5.yrs.
..
36.##End#stage#liver#disease#with#fungal#infection#ttt#
Answer:.Capsofungin.
..
37.##most#specific#sign#for#perforated#duodenal#ulcer,#
.Answer:.Sudden.epigastric.pain.
..
38.##Patient#diagnosed#with#Barrett's#esophagus#will#have:#
A..Adenocarcioma.
B..Squamous.cell.carcinoma.
Answer:.A.
..
39.##Most#common#cancer#mets#to#stomach:#
A......lung.
B......Prostate.
C......Melanoma.
D......KBrain.
Answer:.A.
If.‘from’.then.liver,.lung,.bone.and.lymph.nodes.
..
40.##Scenario#:#pt#develop#bloody#diarrhea,#abd#pain#and#vomiting,#after#eating#from#restaurant#,#
history#if#hematuria,#
Lab.:.Forgot.!.
HUS:.
..
K.....
If.there’s.conservative,.choose.it..
..
41.##upper#epigastric#pain#+#fever#,#Dx?#(Incomplete.Q).
A............perforated.peptic.ulcer.
B............cholecystitis.
C............appendicitis.
D........... pancreatitis.
..
42.##Side#effect#of#ribavirin:#
A)...........anemia..
B)...........Renal.damage.
C)...........Hepatic.damage..
Answer:.A.
..
43.##Sign#for#low#intestinal#obstruction:#
A)...........Altered.bowel.movement..
B)...........Absolute.constipation.
C)...........Flatulence.
D)..........Diarrhea..
Answer:.B.not.sure.
.
.
.
Also,.revise.functional.obstruction.
..
44.##Patient#with#deoudnal#ulcer#,#what#ttt?#
Answer:..triple.therapy.
..
45.##Clear#case#about#crohn#dis:#child#with#abd#cramps#,#diarhea#....#etc#,#there#is#skip#lesion#,#
transmural#:#
A............K.crohn✅.
B............K.Uc.
C............K.Celiac...
Answer:.A.
GALS.are.crohns:.Granuloma,.All.Layers,.Skip.lesions.
..
46.##Clear#scenario#about#Crohn's#disease#(mentioned#that#she#has#post#meal#periumbilical#pain#
plus#other#symptoms:#
Answer:.Crohn's.disease.
..
47.##Similar#case:#child#with#abd#pain#and#tenderness#,#bloody#diarhea#,#wt#loss.#(#No#lab#results#).#
What’s#the#diagnosis:#
A............crohn.
B............Celiac.
C............UC.
Answer:.C.
..
48.##Celiac#disease#common#site:#
A..Proximal.small.bowl.
B..Distal.small.bowl.
C..Proximal.large.bowl.
D..Distal.large.bowl.
Answer:A.
..
49.##Many#Qs#about#different#type#of#jaundice#easy#just#look#for#indirect#and#direct#bilirubin#levels#
and#liver#enzymes.#
##
50.##Patient#lost#weight#and#has#constipation.#Which#one#has#the#highest#diagnostic#value#?#
Colonoscopy###
##
51.##Patient#is#taking#penicillin#(or#medication#from#penicillin#family).#After#the#second#day#he#
developed#diarrhea.#What#is#the#most#likely#cause?#
A............Psudomembranus.colitis.
B............Something.colitis.
C............Ulcerative.colitis.
Answer:.A.caused.by.C..diff.
..
52.#Within#hours#diarrhea?#
A.Staph.
B.Bacillus.
C.Salmonella.
Answer:.A.
..
53.#Lady#presented#with#jaundice#and#confusion,#her#husband#is#positive#for#hepatitis#B.#Her#labs#
were#attached#and#she#had#high#bilirubin#but#negative#hepatitis#screen,#which#one#of#the#following#
will#you#order?#
A......Serum.copper.level.
B......Serum.ceruloplasmin.level.
..
Copper.would.be.more.specific,.ceruloplasmin.might.be.normal.in.pts.then.it’s.useless.if.it.comes.
back.normal..urinary.copper.24.hrs...BMJ.
54.#male#patient#with#symptoms#of#pancreatitis,#k/c#of#hyperlipidemia#on#treatment#
Labs#
Low#Hgb#
High#MCV#
High#Amylase#
High#AST#
Normal#lipid#profile.#What#is#the#cause?#
A:hyperlipidemia.
B:.Alcohol.consumption.
Answer:.B.i.think..
Alcoholic.Liver.disease.causes.macrocytic.anemia.and.can.cause.pancreatitis..Also,.normal.lipid.
profile.
..
55.#Long#scenario#pt#on#atrovastatin#since#1#yr,increase#LFT:#
Drug.induced.hepatitis.
..
56.#Pt#in#40s#with#obs#jaundice,#labs#showed#direct#bilirubin#40,#ferritin#high??,Alkaline#
phosphatase##high,#AST#elevated,#what#dx?#gallbladder#Stone?#

Could.be.hemochromatosis….high.ferritin,.not.sure.

57.#Which#of#the#following#will#reduce#both#spontaneous#and#contact#induced#acid#secretion#in#the#
stomach?#
PPI.
..
58.Hemochromatosis#has#cirrhosis#before#5#years#came#with#pain#and#jaundice.#What#is#the#next#
step:#
A..US.
B..CEA.
Answer:.A.
..
59.#Most#common#symptom#with#hepatitis#C?######################################################################### #
A:Loss.of.appetite.
B:.Flapping.tremor.
C:.Jaundice.
Answer:.A.
Vague.constitutional.symptoms.are.the.most.common….Toronto.
..
60.#Marker#for#acute#liver#failure?#
This.a.huge.topic.to.summarize.pls.check.the.link:.
http://emedicine.medscape.com/article/177354Kworkup#c8.
..
61:#60#yrs#old#patient#with#missed#teeth#and#white#patches#in#the#marginal#of#tongue#not#relived#by#
??#also#he#had#rash#with#ulcer#(Incomplete.Q).
AK.nurofibramatosis.
BK.dysplasia.
CK.vascular.malformation......................................................................................
........................................................................................
62.#What#is#the#early#sign#of#acute#hepatic#failure?################################################################ #
A:Hepatic.encephalopathy.
B:Palmar.erythema.
C:Spider.naevi..................................... .
Answer:.A.
BMJ.
..
Acute.liver.failure.is.a.broad.term.that.encompasses.both.fulminant.hepatic.failure.and.subfulminant.
hepatic.failure.(or.lateKonset.hepatic.failure)..Fulminant.hepatic.failure.is.generally.used.to.describe.
the.development.of.encephalopathy.within.8.weeks.of.the.onset.of.symptoms.in.a.patient.with.a.
previously.healthy.liver..Subfulminant.hepatic.failure.is.reserved.for.patients.with.liver.disease.for.up.
to.26.weeks.before.the.development.of.hepatic.encephalopathy..
Signs.and.symptoms.of.acute.failure.may.include.the.following:.
Å.......Encephalopathy.
Å.......Cerebral.edema:.May.lead.to.signs.of.increased.intracranial.pressure.(ICP).(eg,.
papilledema,.hypertension,.bradycardia).
Å.......Jaundice:.Often.present.but.not.always.
Å.......Ascites:.Potential.for.hepatic.vein.thrombosis.(budd.chiari).with.rapid.development.
in.the.presence.of.fulminant.hepatic.failure.accompanied.by.abdominal.pain.
Å.......Right.upper.quadrant.tenderness:.Variably.present.
Å.......Change.in.liver.span:.May.be.small.due.to.hepatic.necrosis.or.may.be.enlarged.due.
to.heart.failure,.viral.hepatitis,.or.BuddKChiari.syndrome.
Å.......Hematemesis.or.melena:.Due.to.upper.gastrointestinal.(GI).bleeding.
Å.......Hypotension.and.tachycardia:.Due.to.reduced.systemic.vascular.resistance.
medscape.
..
63.#Patient#has#Wilson#disease#with#signs#of#liver#failure,#tremor,#prominent#upper#vessel,#ascites.#
Treatment#for#this#stage:#
A:Pencllamine.
B:Zinc.
C:Liver.transplant.

D:.Tips.(Transjugular.intrahepatic.portosystemic.shunt).
Answer:.C.
●! Patients.should.be.transplanted.if.they.present.with.severe.hepatic.failure.(Nazer.scores.
≥10).or.if.they.develop.serious.further.deterioration.in.liver.function.while.on.antiKcopper.
medical.therapy..
●! Hepatic.transplant.should.not.be.offered.to.patients.with.neurological.symptoms,.who.
should.be.treated.with.medical.therapy..
..

.............................
.
.

Hematology*Oncology.
##
1.#When#you#give#FFP#with#heparin#
Answer:#DIC#
##
Disseminated#intravascular#coagulation#(DIC)#involves#abnormal,#excessive#generation#of#thrombin#and#fibrin#
in#the#circulating#blood.#During#the#process,#increased#platelet#aggregation#and#coagulation#factor#
consumption#occur.#DIC#that#evolves#slowly#(over#weeks#or#months)#causes#primarily#venous#thrombotic#and#
embolic#manifestations;#DIC#that#evolves#rapidly#(over#hours#or#days)#causes#primarily#bleeding.#Severe,#
rapidly#evolving#DIC#is#diagnosed#by#demonstrating#thrombocytopenia,#an#elevated#PTT#and#PT,#increased#
levels#of#plasma#d[dimer#(or#serum#fibrin#degradation#products),#and#a#decreasing#plasma#fibrinogen#level.#
Treatment#includes#correction#of#the#cause#and#replacement#of#platelets,#coagulation#factors#(in#fresh#frozen#
plasma),#and#fibrinogen#(in#cryoprecipitate)#to#control#severe#bleeding.#Heparin#is#used#as#therapy#(or#
prophylaxis)#in#patients#with#slowly#evolving#DIC#who#have#(or#are#at#risk#of)#venous#thromboembolism .#
(http://www.merckmanuals.com/professional/hematology[and[oncology/coagulation[
disorders/disseminated[intravascular[coagulation[dic)#
Platelet#and#factor#replacement#should#be#directed#not#at#simply#correcting#laboratory#abnormalities#but#at#
addressing#clinically#relevant#bleeding#or#meeting#procedural#needs.#Heparin#should#be#provided#to#those#
patients#who#demonstrate#extensive#fibrin#deposition#without#evidence#of#substantial#hemorrhage;#it#is#
usually#reserved#for#cases#of#chronic#DIC.#Heparin#is#appropriate#to#treat#the#thrombosis#that#occurs#with#
DIC.#It#also#has#a#limited#use#in#acute#hemorrhagic#DIC#in#a#patient#with#a#self[limited#condition#of#acral#
cyanosis#and#digital#ischemia.#
It#is#generally#considered#that#cryoprecipitate#and#coagulation#factor#concentrates#should#not#routinely#be#
used#as#replacement#therapy#in#DIC,#because#they#lack#several#specific#factors#(eg,#factor#V).#Additionally,#
worsening#of#the#coagulopathy#via#the#presence#of#small#amounts#of#activated#factors#is#a#theoretical#risk.#
Specific#deficiencies#in#coagulation#factors,#such#as#fibrinogen,#can#be#corrected#by#administration#of#
cryoprecipitate#or#purified#fibrinogen#concentrate#in#conjuction#with#fresh#frozen#plasma#(FFP)#
administration.#
Data#suggest#that#the#consumption[induced#deficiency#of#coagulation#factors#can#be#partially#rectified#by#
administering#large#quantities#of#FFP,#particularly#in#patients#with#an#international#normalized#ratio#(INR)#
#[ 65]#
higher#than#2.0,#a#2[fold#or#greater#prolongation#of#the#aPTT,#or#a#fibrinogen#level#below#100#mg/dL. The#
suggested#starting#dose#is#15#mg/kg#(https://emedicine.medscape.com/article/199627[treatment#d1)#
##
##
2.#Anemic#patient#on#treatment,#he#came#with#black#stool,#what#is#the#cause#
A[#Ferrous#sulfate#
B[#Iron#dextran#
Answer:#A#
#Ferrous#sulfate#side#effects:#
#(Constipation,#upset#stomach,#black#or#dark[colored#stools,#and#temporary#staining#of#the#teeth.)#
(https://www.drugs.com/ferrous_sulfate.html)#
##
##
3.#Young#patient#with#sickle#cell#anemia,#he#has#penile#pain#and#edematous#shaft#of#penis#(with#
picture),#what#the#diagnosis?#
A[#priapism#
B[#peyronie's#disease#
C[#Peronism#
D[#Paraphimosis#
Answer:#A#
priapism#is#a#well[recognized#complication#of#SCD#(https://emedicine.medscape.com/article/205926[
overview)#Low[flow#or#ischemic#priapism:#This#type#happens#when#blood#gets#trapped#in#the#erection#
chambers.#Most#of#the#time,#there’s#no#clear#cause,#but#it#may#affect#men#with#sickle[cell#disease,#leukemia#
(cancerof#the#blood),#or#malaria.#If#you#don’t#get#treatment#right#away,#it#can#lead#to#scarring#and#permanent#
erectile#dysfunction#(ED).#(https://www.webmd.com/erectile[dysfunction/erectile[dysfunction[priapism#1)#
##
##
4.#Father#with#hemophilia#A#and#mother#carrier.#What#is#the#percentage#that#their#child#will#have#
hemophilia?#
A[#25#%#
B[#50#%#
C[#75#%#
D[#100#%#
Answer:#B#
##
##
50%#will#have#hemophilia#
25%#carrier#
25%#normal#
##
#
#
#
Hemophilia#A#and#hemophilia#B#are#inherited#in#an#X[linked#recessive#pattern.#The#genes#associated#with#
these#conditions#are#located#on#the#X#chromosome,#which#is#one#of#the#two#sex#chromosomes.#In#males#
(who#have#only#one#X#chromosome),#one#altered#copy#of#the#gene#in#each#cell#is#sufficient#to#cause#the#
condition.#In#females#(who#have#two#X#chromosomes),#a#mutation#would#have#to#occur#in#both#copies#of#the#
gene#to#cause#the#disorder.#Because#it#is#unlikely#that#females#will#have#two#altered#copies#of#this#gene,#it#is#
very#rare#for#females#to#have#hemophilia.#A#characteristic#of#X[linked#inheritance#is#that#fathers#cannot#pass#
X[linked#traits#to#their#sons.#In#X[linked#recessive#inheritance,#a#female#with#one#altered#copy#of#the#gene#in#
each#cell#is#called#a#carrier.#(https://ghr.nlm.nih.gov/condition/hemophilia#resources)#

##
5.#Patient#with#multiple#blood#transfusions#and#splenomegaly,#what's#the#cause?#
Answer:#Ankyrin#deficiency#
##
Differential#for#multiple#blood#transfusions#and#splenomegaly:#
Answer:#beta#thalassemia#major#
Patients#with#the#beta#thalassemia#trait#generally#have#no#unusual#physical#findings.#In#patients#with#beta#
thalassemia#major,#the#physical#findings#are#related#to#severe#anemia,#ineffective#erythropoiesis,#
extramedullary#hematopoiesis,#and#iron#overload#resulting#from#transfusion#and#increased#iron#absorption#
Abdominal#examination#may#reveal#changes#in#the#liver,#gallbladder,#and#spleen.#Hepatomegaly#related#to#
significant#extramedullary#hematopoiesis#is#typically#found.#Patients#who#have#received#blood#transfusions#
may#have#hepatomegaly#or#chronic#hepatitis#due#to#iron#overload.#
(https://emedicine.medscape.com/article/206490[clinical?src=refgatesrc1)#
##
6.#Case#about#hematology#all#description#and#investigations#were#consistent#with#hereditary#
spherocytosis,#positive#osmotic#fragility#test#and#jaundice#with#splenomegaly:#
Which#one#of#the#following#is#correct#regarding#this#scenario?#
A[#G6PD#
B[#SPECTRIN#–ANKIRIN#DEFICINCY#
Answer:#B#
most#patients#with#dominant#HS#have#combined#ankyrin#and#spectrin#deficiency#and#that#the#two#proteins#
are#usually#about#equally#deficient.#(https://www.ncbi.nlm.nih.gov/pubmed/8219186)#
(http://www.bloodjournal.org/content/bloodjournal/82/10/2953.full.pdf?sso[checked=true)#
##
##
7.#SCA#mode#of#inheritance?#
Answer:#Autosomal#recessive#
Reference:#STEP#UP#TO#MEDICINE,#pg#333.#
##
8.#Fanconi#anemia#mode#of#inheritance?#
A[#Autosomal#recessive#
B[#Autosomal#dominant#
Answer:#A#
https://ghr.nlm.nih.gov/condition/fanconi[anemia#inheritance#
##
9.#Absorption#of#non[heme#iron#is#enhanced#by#
Answer:#Vitamin#C#
http://ajcn.nutrition.org/content/73/1/93.full#
##
10.#Which#type#of#Hodgkin#lymphoma#has#Reed–Sternberg#cells#and#band#of#collagen#or#fibrosis.#
Which#one#of#the#following#is#the#diagnosis?#
AK.Mixed.cellularity.
BK.Lymphocyte.Kpredominate.
CK.Lymphocyte.–depleted.
DK.Nodular.sclerosis.
Answer:#D#
Reference:#STEP#UP#TO#MEDICINE,#pg#352#
##
11.#You#gave#a#patient#a#drug#and#asked#him#to#come#back#to#check#PT#and#INR#what#was#the#drug?#
AK.Aspirin.
BK.Warfarin.
CK.Enoxaparin.
DK.UF.heparin..
Answer:#B#
Reference:#STEP#UP#TO#MEDICINE,#pg#348#
##
12.#Factor#7#affects#which#lab#result?#
A#normal#aPTT#and#a#prolonged#PT#in#a#patient#with#a#lifelong#history#of#a#tendency#for#mild#or#severe#
bleeding#is#consistent#with#the#diagnosis#of#factor#VII#deficiency#or#the#presence#of#an#inhibitor#to#factor#VII.#
https://emedicine.medscape.com/article/209585[workup#
##
13.#Scenario#with#a#patient#who#came#with#fever#and#altered#level#of#consciousness#for#5#days.#His#
condition#deteriorated#and#developed#body#rash,#and#became#jaundiced.#Labs#show#high#urea#and#
creatinine,#low#platelets#and#anemia#(Scenario#of#TTP).#What's#the#gene#mutation#responsible?#
Answer:#ADAMTS13#
Mutations#in#the#ADAMTS13#gene#cause#the#familial#form#of#thrombotic#thrombocytopenic#purpura.#The#
ADAMTS13#gene#provides#instructions#for#making#an#enzyme#that#is#involved#in#the#normal#process#of#blood#
clotting.#Mutations#in#this#gene#lead#to#a#severe#reduction#in#the#activity#of#this#enzyme.#The#acquired#form#
of#thrombotic#thrombocytopenic#purpura#also#results#from#a#reduction#in#ADAMTS13#enzyme#activity;#
however,#people#with#the#acquired#form#do#not#have#mutations#in#the#ADAMTS13#gene.#Instead,#their#
immune#systems#often#produce#specific#proteins#called#autoantibodies#that#block#the#activity#of#the#enzyme.#
https://ghr.nlm.nih.gov/condition/thrombotic[thrombocytopenic[purpura#genes#
##
14.#Typical#presentation#of#multiple#myeloma,#what#is#the#diagnostic#test?#
A[#Protein#electrophoresis#
B[#CT#head#
Answer:#A#
The#diagnosis#of#multiple#myeloma#is#determined#by#a#number#of#different#diagnostic#tests,#because#
multiple#myeloma#is#difficult#to#diagnose#on#the#basis#of#any#single#laboratory#test#result.##
[#Serum#and#urine#Protein#electrophoresis:#detect#the#presence#of#M#protein.#
[#Skeletal#survey#(x[ray):#detect#lytic#lesions.#
[#MRI#spine:#detect#spinal#cord#compression.#
[#Bone#marrow#aspiration:#presence#of#plasma#cells#in#the#bone#marrow.##Reference:#Davidson’s,#pg#1047#
##
15.#Which#one#of#the#following#follows#a#Mendelian#inheritance?#
A[#Thalassemia#
Answer:#A#
##
Inheritance# Disease#Examples#
Pattern#

Autosomal# Huntington’s#disease,#neurofibromatosis,#achondroplasia,#familial#
Dominant# hypercholesterolemia,#Marfan#syndrome,#von#Willbrand.#

Autosomal# Tay[sachs#disease,#sickle#cell#anemia,#cystic#fibrosis,#phenylketonuria#(PKU),#
Recessive# Thalassemia,#congenital#adrenal#hyperplasia.#

X[linked# Hypophatemic#rickets#(vitamin#D[resistant#rickets),#
Dominant#

X[linked# Hemophilia#A,#Duchenne#muscular#dystrophy,#G6PD#
Recessive#

Reference:#https://www.ncbi.nlm.nih.gov/books/NBK132145/#
https://ghr.nlm.nih.gov/condition/beta[thalassemia#
##
##
16.#Excessive#ice#taking.#What#do#we#call#it?#
Answer:##Pagophagia#
Pagophagia:#A#craving#to#eat#ice,#often#associated#with#anemia#resulting#from#iron#deficiency#
https://medical[dictionary.thefreedictionary.com/pagophagia#
##
18.#75#years#old#male,#asymptomatic,#with#high#WBC,#90%#lymphocytes.#Immunohistochemistry#
(positive#CD19,#CD23,#CD56).#What’s#the#Treatment?#
AK.No.treatment.
BK.Rituximab.+.CVB.
CK.Rituximab.+.Prednisolone.
DK.Cyclophosphamide.
Answer:#A#
Reference:#(Hematology#resident):#the#patient#has#CLL#(#indolent#type),#he’s#asymptomatic#and#he’s#old,#
there#is#no#need#to#subjecting#the#patient#to#chemotherapy#and#it’s#complication.#
If#the#patient#was#symptomatic,#CD20#positive,#and#has#agood#life#expectancy#we#can#give#him#rituximab,#
and#if#the#patient#is#suffering#from#AIH#we#can#add#steroid#to#the#rituximab.#
##
19.#Treatment#of#von[willbrand#disease?#
A[#FFP#
B[#Cryptoprecipitate#
##
Treatment#of#von[Willbrand:#
[DDAVP#(desmopressin).#
[Factor#VIII#concentrates.#
Cryoprecipitate#is#not#recommended#as#treatment#for#vWD.#
Reference:#STEP#UP#TP#MEDICINE,#pg#342#
DAVIDSON’S,#pg#1055#
############################################# #
20.#Pencil#cell#in#blood#smear?#
A[#Sickle#cell#anemia#
B[#Thalassemia#
Answer:#Iron#deficiency#anemia#
http://onlinelibrary.wiley.com/store/10.1002/ajh.21283/asset/21283_ftp.pdf;jsessionid=E4B26FAA449B5D
9EA7C4B538AD19756A.f01t03?v=1&t=j9b7kjl0&s=4f19d363b4f2210f0c451fa46d4af0a066430d51#
Cigar#cells#(also#referred#to#as#pencil#cells)#are#red#blood#cells#that#are#cigar#or#pencil#shaped#on#
Peripheral#blood#smear.#Cigar#cells#are#commonly#associated#with#hereditary#elliptocytosis.#However,#
they#may#also#be#seen#in#iron#deficiency#anemia#and#other#pathological#states#that#decrease#red#blood#
[1]
cell#turnover#and#or#production. #In#the#case#of#iron#deficiency#anemia,#microcytosis#and#
hypochromia#would#also#be#expected.#http://www.wikiwand.com/en/Cigar_cell#
##
21.#Recurrent#hemarthrosis#what#is#the#diagnosis?#
Answer:#Hemophilia#
Hemarthrosis#is#the#most#common#musculoskeletal#manifestation#of#hemophilia.#
http://cursoenarm.net/UPTODATE/contents/mobipreview.htm?14/6/14433#
http://cursoenarm.net/UPTODATE/contents/mobipreview.htm?1/34/1568#
##
22.#Long#scenario,#biopsy#showed#starry#sky#appearance.#What#is#the#gene#affected?#
Answer:#C[MYC#(8;14)#
https://emedicine.medscape.com/article/1447602[overview#a4#
##
23.#Picture#of#sickle#cell#anemia#diagnosis#
##
24.#Two#couple#carrier#of#thalassemia,#probability#of#child#to#have#
thalassemia#
A[#25%#
B[#50%#
C[#100%#
D[#75%#
Answer:#A#
##
##
#
#
#
#
#
25.#Adult#patient#with#thrombocytopenia,#high#creatinine#&#Bun#with#purpura.#What#is#the#
diagnosis?#
Answer:#TTP#(correct)#
Thrombotic#thrombocytopenic#purpura#(TTP)#is#a#rare#blood#disorder#characterized#by#clotting#in#small#blood#
#
vessels#of#the#body#(thromboses),#resulting#in#a#low#platelet#count. In#its#full[blown#form,#the#disease#
consists#of#the#following#pentad:#
x#######Microangiopathic#hemolytic#anemia#
x#######Thrombocytopenic#purpura#
x#######Neurologic#abnormalities#
x#######Fever#
x#######Renal#disease#
Clinical#differentiation#of#hemolytic[uremic#syndrome#(HUS)#and#TTP#can#be#problematic.#Differentiation#is#
often#based#on#the#presence#of#central#nervous#system#involvement#in#TTP#and#the#more#severe#renal#
involvement#in#HUS.#In#HUS,#an#antecedent#history#of#diarrheal#illness#is#more#often#present.#In#fact,#some#
investigators#suggest#a#clinical#classification#of#HUS#based#on#the#presence#or#absence#of#diarrhea.#
##
26.#ITP#case#
[#Immune#thrombocytopenic#purpura#(ITP)#is#a#clinical#syndrome#in#which#a#decreased#number#of#circulating#
platelets#(thrombocytopenia)#manifests#as#a#bleeding#tendency,#easy#bruising#(purpura),#or#extravasation#of#
blood#from#capillaries#into#skin#and#mucous#membranes#(petechiae).#Although#most#cases#of#acute#ITP,#
particularly#in#children,#are#mild#and#self[limited,#intracranial#hemorrhage#may#occur#when#the#platelet#
9 3 ##
count#drops#below#10#×#10 /L#(<10#×#10 /µL); this#occurs#in#0.5[1%#of#children,#and#half#of#these#cases#are#
fatal.#
##
[ITP#is#a#primary#illness#occurring#in#an#otherwise#healthy#person.#Signs#of#chronic#disease,#infection,#
wasting,#or#poor#nutrition#indicate#that#the#patient#has#another#illness.#Splenomegaly#excludes#the#diagnosis#
of#ITP.#
##
[On#complete#blood#cell#count,#isolated#thrombocytopenia#is#the#hallmark#of#ITP.#Anemia#and/or#
neutropenia#may#indicate#other#diseases.#Findings#on#peripheral#blood#smear#are#as#follows:#
x#######The#morphology#of#red#blood#cells#(RBCs)#and#leukocytes#is#normal#
x#######The#morphology#of#platelets#is#typically#normal,#with#varying#numbers#of#large#platelets#
x#######If#most#of#the#platelets#are#large,#approximating#the#diameter#of#red#blood#cells,#or#if#they#lack#
granules#or#have#an#abnormal#color,#consider#an#inherited#platelet#disorder#
Many#children#with#acute#ITP#have#an#increased#number#of#normal#or#atypical#lymphocytes#on#the#
peripheral#smear,#reflecting#a#recent#viral#illness.#
##
27.#Most#common#cause#of#iron#deficiency#anemia?#
Causes#of#iron#deficiency:#
K.Lack.of.iron.in.your.diet..
K.Chronic.blood.loss..
K.Pregnancy.
K.Inability.to.absorb.iron.
https://www.mayoclinic.org/diseases[conditions/iron[deficiency[anemia/symptoms[causes/syc[20355034#
https://emedicine.medscape.com/article/202333[overview#a3#
##
28.#Patient#on#warfarin#came#with#melena#and#PT#high#what#is#the#treatment?#
A[#IV#vitamin#k#
B[#Fresh#frozen#plasma#
C[#Protamine#sulphate#
Answer:#A#
##
29.#Vitamin#k#dependent#factors#
Vitamin#K#serves#as#an#essential#cofactor#for#a#carboxylase#that#catalyzes#carboxylation#of#glutamic#acid#
residues#on#vitamin#K[dependent#proteins.#The#key#vitamin#K[dependent#proteins#include:#
●! Coagulation#proteins:#factors#II#(prothrombin),#VII,#IX#and#X#
●! Anticoagulation#proteins:#proteins#C,#S#and#Z#
##
30.#Patient#with#symptoms#of#gastritis#and#has#anemia.#How#would#you#treat?#
A[#Oral#iron#
B[#IM#iron#
C[#IV#iron#
Answer:#C#
##
31.#What#determines#the#prognosis#in#CLL?#
A[#Stage#of#the#disease#
B[#Age#at#the#time#of#diagnosis#
C[#Bone#marrow#aspiration#
ANSWER:Rai/binet#staging#
The#prognosis#(chance#of#recovery)#depends#on:#
x#### Whether#there#is#a#change#in#the#DNA#and#the#type#of#change,#if#there#is#one.#
x#######Whether#lymphocytes#are#spread#throughout#the#bone#marrow.#
x#######The#stage#of#the#disease.#
x#######Whether#the#CLL#gets#better#with#treatment#or#has#recurred#(come#back).#
x#### Whether#the#CLL#progresses#to#lymphoma#or#prolymphocytic#leukemia.#
x#######The#patient's#general#health.#
##
##
32.#Picture#of#histopathology#of#tumor#with#starry#sky#pattern#
Answer:#EBV#
https://emedicine.medscape.com/article/1447602[overview#a4#
##
33.#K/C#of#Sickle#cell#anemia#presented#with#chest/back/hip#pain.#He#reports#having#previous#
episodes#and#was#hospitalized#for#them.#Now#in#severe#pain.#What#to#do?#
A.#Give#oral#narcotics#and#follow#up.############################################## #
B.#Admission#with#pain#management.####################################################### #
C.#Refer#to#tertiary#hospital#
Did#not#find#a#specific#answer.#
Answer:#B#
Oral#medication#is#not#enough#in#such#case.#Referral#is#right#thing#to#do#but#not#for#acute#management.#
So#you#admit#and#manage#first,#then#refer.#(Opinion)#
For#sickle#cell#crisis,#when#the#severity#of#the#episode#is#assessable,#self[treatment#at#home#with#bed#rest,#
oral#analgesia,#and#hydration#is#possible.#Individuals#with#SCD#often#present#to#the#emergency#department#
(ED)#after#self[treatment#fails.#Do#not#underestimate#the#patient's#pain.#Failure#to#treat#acute#pain#
aggressively#and#promptly#may#lead#to#chronic#pain#syndrome.#According#to#the#2003#BCHS#acute#painful#
crisis#guidelines,#these#patients#should#receive#analgesia#within#30#minutes#of#entering#the#hospital,#with#the#
goal#of#achieving#effective#pain#control#by#60#minutes.#
If#patients#with#SCD#crisis#are#being#transported#by#emergency#medical#services#(EMS),#they#should#receive#
supplemental#oxygen#and#intravenous#hydration#en#route#to#the#hospital.#Some#areas#have#specialized#
facilities#that#offer#emergency#care#of#acute#pain#associated#with#SCD;#many#EDs#have#a#standardized#
treatment#plan#in#place.#https://emedicine.medscape.com/article/205926[treatment#
##
34.#What#is#the#characteristic#for#von[willbrand#disease?#
A[#Prolonged#PT#
B[#Prolonged#PTT#
C[#Factor#12#deficiency#
D[#Factor#VIII#deficiency#
Answer:#D#
https://emedicine.medscape.com/article/206996[overview#
##
35.#Patient#on#warfarin#5#mg,#his#INR#7.#No#signs#of#active#bleeding,#what#to#do?#
AK.Increase.warfarin.dose.to.7.5.mg.
BK.Decrease.warfarin.dose.to.2.5.mg.
CK.Hold.warfarin.for.one.day.and.measure.INR.again.
DK.Continue.the.same.dose..
Answer:#C#
https://emedicine.medscape.com/article/2172018[overview#
##
36.#Sickler#patient#with#splenic#sequestration?#
Splenic#sequestration#occurs#with#highest#frequency#during#the#first#5#years#of#life#in#children#with#sickle#cell#
anemia.#Splenic#sequestration#can#occur#at#any#age#in#individuals#with#other#sickle#syndromes.#This#
complication#is#characterized#by#the#onset#of#life[threatening#anemia#with#rapid#enlargement#of#the#spleen#
and#high#reticulocyte#count.#
Splenic#sequestration#is#a#medical#emergency#that#demands#prompt#and#appropriate#treatment.#Parents#
should#be#familiar#with#the#signs#and#symptoms#of#splenic#sequestration#crises.#Children#should#be#seen#as#
rapidly#as#possible#in#the#emergency#room.#Treatment#of#the#acute#episode#requires#early#recognition,#
careful#monitoring,#and#aggressive#transfusion#support.#Because#these#episodes#tend#to#recur,#many#
advocate#long[term#transfusion#in#young#children#and#splenectomy#in#older#children.#
https://emedicine.medscape.com/article/205926[clinical#
##
37.#How#to#confirm#the#diagnosis#of#patient#with#thalassemia?#
AK.Level.of.hemoglobin.A2.
BK.Genetic.profiling.
Answer:#A#
Elevation#of#the#Hb#A2#level,#demonstrated#by#electrophoresis#or#column#chromatography,#confirms#the#
diagnosis#of#beta#thalassemia#trait.#https://emedicine.medscape.com/article/206490[workup#c7#
##
38.#Patient#with#polycythemia#vera#with#all#values#elevated#Hgb,#WBC#and#platelets.#what's#the#
treatment?#
A[#Phlebotomy#
B[#Meylosuppressive#treatment#
Answer:#A#
All#patients#with#PV#should#undergo#phlebotomy#to#keep#their#hematocrit#below#45%.#Lower#hematocrit#
targets#have#been#proposed#for#women#with#PV,#but#no#empiric#evidence#supports#that#
recommendation.https://emedicine.medscape.com/article/205114[treatment#
##
39.#What#type#of#anemia#that#those#who#are#on#chemotherapy#get?#
Answer:#Aplastic#
x####Radiation#and#chemotherapy#treatments.#While#these#cancer[fighting#therapies#kill#cancer#cells,#
they#can#also#damage#healthy#cells,#including#stem#cells#in#bone#marrow.#Aplastic#anemia#can#be#a#
temporary#side#effect#of#these#treatments.#https://www.mayoclinic.org/diseases[conditions/aplastic[
anemia/symptoms[causes/syc[20355015#

##
40.#Patient#presented#with#fatigue#and#weight#loss.#Lab#show#Hgb#12,#platelets#200,#WBCs#19000#
and#peripheral#blood#film#showed#neutrophilia,#basophilia,#promyelcytes#and#myelocytes.#What's#
the#next#step?#
A[#Flow#cytometry#
B[#Bone#marrow#biopsy#and#aspiration#
C[#Repeat#CBC#
Answer:#A#
CML,#so#you#do#flow#cytometry#to#check#for#Philadilphia#chromosome#(Opinion)#
##
50.#How#to#monitor#the#response#of#iron#treatment?#
A[#Ferritin#
B[#Reticulocyte#count#
C[#Hematocrit#
Answer:#B#
Response#to#iron#therapy#can#be#documented#by#an#increase#in#reticulocytes#5[10#days#after#the#initiation#of#
iron#therapy.#The#hemoglobin#concentration#increases#by#about#1#g/dL#weekly#until#normal#values#are#
restored.#https://emedicine.medscape.com/article/202333[treatment#d13#
In#addition#to#hemolysis,#increased#reticulocytes#may#be#a#response#to#blood#loss#or#the#treatment#of#iron,#
vitamin#B[12,#or#folate#deficiencies.#https://emedicine.medscape.com/article/201066[workup#c8#
##
51.#Patient#with#cut#that#is#continuously#bleeding.#Prolonged#BT#and#thrombocytopenia.#Bleeding#
time#normalizes#with#platelet#transfusion#but#not#FFP.#
A[#Von#Willebrand#
B[#Bernard#Soulier#
C[#Hemophilia#
Answer:#A#
##
52[#Most#important#test#in#Von#Willebrand?#
A[#PT#
B[#PTT#
C[#BT#
Answer:#C#
##
53.#Patient#on#heparin#what#should#be#monitored?########### #
A[#PT#
B[#INR########## #
C[#PTT###
Answer:#C#
##
54.#Patient#has#night#sweating,#weight#loss,#fever,#and#lymph#node#enlargement#in#the#left#
subclavicular#lymph#node.#What#is#the#diagnosis?#
A[#Burkitt#lymphoma####################### #
B[#Hodgkin#lymphoma################################# #
C[#Nonhodgkin#lymphoma#
I#believe#the#question#is#incomplete,#we#can#not#differentiate#between#Hodgkin#and#non#Hodgkin#lymphoma#
without#examining#the#cell#under#the#microscope.#I#would#go#with#Hodgkin#lymphoma#since#they#have#
higher#tendency#to#present#with#B#symptoms.######### #
##
55.#Patient#has#pallor#and#dyspnea#
CBC:#macrocytic#anemia#
#
AST:#High.#Blood#film:#no#megablast.#What#is#the#cause?#
A[#B12#deficiency#
B[#Folate#deficiency#
C[#Drug#induced#
D[#Alcohol#abuse############################## #
Answer:#Alcohol#abuse#
Although#the#mean#corpuscular#volume#is#increased#by#recognized#causes#that#include#vitamin#B12#deficiency#
or#folic#acid#deficiency,#it#is#also#raised#by#high#alcohol#intake#alone.#
https://www.medscape.com/viewarticle/562546_2#
##
56.#Scenario#for#ITP,#what#you#see#in#bone#marrow?#
Answer:#Increase#Megakaryocytes#
The#number#of#megakaryocytes#may#be#increased.#Because#the#peripheral#destruction#of#platelets#is#
increased,#megakaryocytes#may#be#large#and#immature,#although#in#many#cases#the#megakaryocyte#
morphology#is#normal#https://emedicine.medscape.com/article/202158[workup#c7#
##
57.#Patient#with#knee#trauma,#bleeding#in#cut#with#increased#bleeding#time.#He#was#given#FFP#with#
not#improved#but#improved#with#platelet#transfusion?#
A[#VWD#
B[#TTP###################### #
C[#DIC#
Answer:#A#
##
58.#Most#common#type#of#Hodgkin#lymphoma?#
A[#Lymphocytes#predominant#
B[#Lymphocytes#depleted#
C[#Mixed#cellularity#
D[#Nodular#sclerosing#
Answer:#D#
Reference:#DAVIDSON’S,#pg#1042#
#
59.#A#girl#presented#with#fatigue#malaise#after#doing#full#investigation#she#had#+ve#sickling#test#
A[#Aplastic#crisis#
B[#Vasoocclusive#crisis#
Answer:#A#
A#serious#complication#is#the#aplastic#crisis.#This#is#caused#by#infection#with#Parvovirus#B[19#(B19V).#This#
virus#causes#fifth#disease,#a#normally#benign#childhood#disorder#associated#with#fever,#malaise,#and#a#mild#
rash.#https://emedicine.medscape.com/article/205926[clinical.
#
60.#Scenario#of#patient#with#anemia,#blood#film#showed#brown#color.#What#is#the#diagnosis?#
A[#Aplastic#anemia#
B[#Hemolytic#anemia#
Not#enough#information#!#
#
61.#34#years#old#female#presented#with#fever#and#decrease#consciousness#for#6#hours.#
Investigations#shows#low#platelets#only.#What#is#the#diagnosis?#
A[#ITP#
B[#TTP#
C[#Antiphospholipd#syndrome#
Answer:#B#
62.#20#years#old#male,#with#small#erythematous#non[planchable#macules,#history#of#viral#
respiratory#infection#resolve#spontaneously#last#week,#Lab:#platelets#15#(very#low).#What#is#the#
treatment?#################### #
A[#IVIG#
B[#Splenectomy#
C[#Platelets#transfusion#
[the#answer#is#corticosteroids#(ie,#oral#prednisone,#intravenous#[IV]#methylprednisolone,#or#high[dose#
#
dexamethasone) should#remain#the#drugs#of#choice#for#the#initial#management#of#acute#ITP#
[Treatment#with#corticosteroids#may#not#only#reduce#the#rate#of#platelet#destruction#but#may#also#rapidly#
alter#endothelial#cell#integrity#to#facilitate#primary#hemostasis#and#to#reduce#bleeding#and#bruising.#
#
[IV#immunoglobulin#(IVIG)#has#been#the#drug#of#second#choice#(after#corticosteroids)#for#many#years.#
https://emedicine.medscape.com/article/202158[treatment#
63.#Patient#with#features#of#leukemia,#Lab#shows:#blast#cell;#high.#Aure#rodes#in#blood#film.#What#is#
diagnosis?################################# #
A[#AML######### ############################################# #
B[#Aplastic#anemia#
C[#CML#
Answer:#A#
Reference:#STEP#UP#TO#MEDICINE,#pg#356#
64.#Management#of#HSP?#
Answer:#Supportive#
To#date,#no#form#of#therapy#has#been#found#to#shorten#the#duration#of#Henoch[Schönlein#purpura#(HSP)#to#
any#significant#degree.#Therefore,#treatment#remains#primarily#supportive#in#most#cases.#
Management#of#HSP#includes#adequate#hydration;#immediate#discontinuance#of#any#exposure#to#antigenic#
stimulants#(eg,#drugs);#and#follow[up#each#week#for#the#first#month,#every#other#week#for#the#second#month,#
and#monthly#thereafter#until#abnormal#urinary#findings#subside.#
https://emedicine.medscape.com/article/984105[treatment#
##
65.#What#causes#false#positive#sickling#test?#
A[#Anemia#
B[#High#protein#
Answer:#B#
##
66.#What#is#the#cause#of#hyposeplenism?#
A[#Sickle#cell#
B[#Spherocytosis#
Answer:A#
##
67.#A#case#mentioning#presence#of#Reed–Sternberg#cells.#What#is#the#diagnosis?#
#A[#Hodgkin#lymphoma#
B[#Non[Hodgkin#lymphoma#
Answer:#A#
Reference:#DAVIDSON’S,#pg#1042#
##
68.#Patient#known#case#of#sarcoidosis#for#2#years,#labs#showing#anemia.#What#type#of#anemia#he#
most#likely#has?#
Answer:#Anemia#of#chronic#disease#
Anemia#of#chronic#disease:#
This#type#of#anemia#occurs#as#part#of#a#chronic#inflammatory#disorder,#most#often#chronic#infection,#
autoimmune#disease#(especially#RA),#kidney#disease,#or#cancer;#however,#the#same#process#appears#to#begin#
acutely#during#virtually#any#infection#or#inflammation,#including#trauma#or#post[surgery.#(See#also#Anemia#of#
Renal#Disease.)#http://www.msdmanuals.com/professional/hematology[and[oncology/anemias[caused[by[
deficient[erythropoiesis/anemia[of[chronic[disease#
Another#answer#I#found#was#a#rare#association#between#sarcoidosis#and#cold#autoimmune#hemolytic#
anemia.#
https://www.omicsonline.org/open[access/sarcoidosis[and[cold[autoimmune[hemolytic[anemia[a[rare[
association[2329[8790[1000255.php?aid=80787&view=mobile#
##
69.#Patient#started#to#bleed#from#IV#line#and#other#orifices#
Answer:##DIC#
clinical#features#of#DIC:#
[bleeding#tendency:#
[########superficial#hemorrhage#(ecchymosis,#petechiae,#purpura).#
[########bleeding#from#GI#tract,#Urinary#tract,#gingival#or#oral#mucosa.#
[########oozing#from#site#of#procedures#and#incisions.#
[thrombosis:#
1.#####occur#more#often#in#chronic#cases.#
Reference:#STEP#UP#TP#MEDICINE,#pg#344.#
##
70.#Splenomegaly,#high#hgb,#plt?#
Answer:#polycythemia#rubra#vera#
Reference:#STEP#UP#TO#MEDICINE,#pg#358[359#
##
71.#Scenario#about#patient#came#from#Ghana#
Answer:#Burkitt#lymphoma#
##
72.#What#type#of#anemia#is#associated#with#RA?#
Answer:#Normocytic#normochromic#
Patients#with#rheumatoid#arthritis#(RA)#may#exhibit#a#variety#of#hematologic#abnormalities.#Common#
changes#associated#with#active#disease#include#anemia#of#chronic#disease#
http://www.uptodate.com/contents/hematologic[manifestations[of[rheumatoid[arthritis#H3#
anemia#of#chronic#illness#and#chronic#kidney#disease#usually#fall#under#the#classification#of#normochromic,#
normocytic#anemia.#https://emedicine.medscape.com/article/1389854[overview#
##
73.#Patient#on#anticoagulant,#INR:1,#PTT#&#platelet#within#normal,#and#was#given#appointment#
after#2#weeks#to#reevaluate#the#result,#what#is#the#drug?#
A[#Aspirin#
B[#Warfarin#(correct)#
C[#LMWH#
D[#Unfractionated#heparin#
Answer:#B#
##
75.#Case#scenario#of#patient#with#history#of#URTI.#lab#results:#low#Hbg,#high#WBC,#I#think#
reticulocyte#was#within#range#or#decrease,#what#is#the#investigation#you#want#to#do?#
A[#Electrophoresis#
B[#Bone#marrow#biopsy#
I#would#say#it’s#Cold#agglutinin#disease,#so#we#should#do#Coombs#test.#
##
76.#Case#scenario#of#patient#with#history#of#URTI.#lab#results:#low#Hbg,#high#WBC,#I#think#
reticulocyte#was#within#range#or#decrease,#what#is#the#diagnosis?#
A[#Aplastic#anemia#
B[#Hypoplastic#
C[#Immune#hemolytic#
Answer:#C#(Opinion)#
##
78.#asking#about#Ann#Arbor#staging#of#burkitt’s#lymphoma#(multiple#lymph#node#in#same#side#of#
diaphragm)#
A[#Stage#1#
B[#Stage#2#
C[#Stage#3#
D[#Stage#4#
Answer:#B#
#

Stage# Area#of#Involvement#

I# Single#lymph#node#group#
II# Multiple#lymph#node#groups#on#same#side#of#diaphragm#

II# Multiple#lymph#node#groups#on#both#sides#of#diaphragm#

IV# Multiple#extranodal#sites#or#lymph#nodes#and#extranodal#disease#

https://emedicine.medscape.com/article/2007081[overview#
##
##
79.#60#years#old#man#coming#with#tachypnea.#Found#to#have#low#Hg#and#high#LDH#all#other#labs#
were#normal.#What#is#the#diagnosis?#
A[#Aplastic#anemia#
B[#Hemolytic#anemia#
Answer:#B#
Serum#LDH#elevation#is#a#criterion#for#hemolysis.#
LDH#elevation#is#sensitive#for#hemolysis,#but#is#not#specific#since#LDH#is#ubiquitous#and#can#be#released#from#
neoplastic#cells,#the#liver,#or#from#other#damaged#organs.#https://emedicine.medscape.com/article/201066[
workup#c10#
##
80.#Question#about#pernicious#anemia#
Answer:#B12#deficiency#due#to#intrinsic#factor#deficiency#
##
81.#Hook#worm,#how#to#cause#anemia?#
A[#Destruction#of#RBC#
B[#Send#toxins#to#bone#marrow#
C[#Compete#with#host#on#b#12#
Answer:#A#
x#######Classic#hookworm#disease#[#This#is#a#gastrointestinal#(GI)#infection#characterized#by#chronic#blood#loss#
that#leads#to#iron[deficiency#anemia#and#protein#malnutrition;#it#is#caused#primarily#by#N)americanus#and#A)
duodenaleand#less#commonly#by#the#zoonotic#species#A)ceylonicum)
#[ 6]#
x#######Intestinal#blood#loss#secondary#is#the#major#clinical#manifestation#of#hookworm#infection. In#fact,#
hookworm#disease#historically#refers#to#the#childhood#syndrome#of#iron#deficiency#anemia,#protein#
malnutrition,#growth#and#mental#retardation#with#lethargy#resulting#from#chronic#intestinal#blood#loss#
secondary#to#hookworm#infection#in#the#face#of#an#iron#deficient#diet.#
x#######Hookworms#ingest#and#digested#some#of#the#blood#from#the#injured#mucosa#by#means#of#a#multienzyme#
cascade#of#metallohemoglobinases.#Each#Necator#worm#ingests#0.03#mL#of#blood#daily,#whereas#each#
Ancylostoma#worm#ingests#0.15[0.2#mL#of#blood#daily.#Inhibited#host#coagulation#due#to#a#series#of#
anticoagulants#directed#against#factor#Xa#and#the#factor#VIIa–tissue#factor#(TF)#complex,#as#well#as#against#
platelet#aggregation,#further#exacerbates#blood#loss.#
##
##
82.#Known#case#of#sarcoidosis,#presented#with#signs#of#anemia.#From#investigation#he#has#iron#
deficiency#anemia.#What#is#the#pathophysiology?#
A[#Decreased#hepcidin#
B[#Increased#hepcidin#
Answer:#B#
The#hepatic#peptide#hepcidin#was#identified#as#the#systemic#iron[regulatory#hormone.#In#the#efferent#arc,#
hepcidin#regulates#intestinal#iron#absorption,#plasma#iron#concentrations,#and#tissue#iron#distribution#by#
inducing#degradation#of#its#receptor.#Increased#hepcidin#concentrations#in#plasma#are#pathogenic#in#iron[
restrictive#anemias#including#anemias#associated#with#inflammation,#chronic#kidney#disease#and#some#
cancers.#Hepcidin#deficiency#causes#iron#overload#in#hereditary#hemochromatosis#and#ineffective#
erythropoiesis.#
https://www.ncbi.nlm.nih.gov/pmc/articles/PMC4048856/#
#
#83.#Most#common#presentation#in#hemophilia#
AK.Hemarthrosis.
BK.Petechiae.
.Answer:..
.
84.#Case#of#patient#tired,#his#peripheral#blood#show#spherocytes.#
Labs:#low#Hgb#70,#low#platelets#<#15.#What#is#the#treatment?#
AK.Platelet.infusion.
BK.Plasma.exchange....................................................
CK.IV.acyclovir.
..Answer:..
.
85.#Poor#prognostic#factors#of#MM?#
AK.High.calcium.
BK.Low.IgA.
Answer:..
..
86.#Wiskott[Aldrich#Syndrome#typical#symptoms#(diagnosis)?#
AK.Eczema,.thrombocytopenia.and.pyogenic.infections..
.........................................................................
87.#Wiskott[Aldrich#Syndrome#mode#of#inheritance?#
AK.XKlinked.recessive.
..
88.#DIC#
AK.Prolong.APTT.and.PT.,.improve.by.platelet.
..
89.#VWD#
AK.Prolong.bleeding.time.and.improve.by.FFP.and.platelet.but.FFP.should.be.in.larger.amount.
..
90.#Male#patient#complaining#of#back#pain.#Imaging#shows#bony#lytic#lesions#on#multiple#levels#
with#moth#eaten#appearance.#Diagnosis#by?#
AK.Serum.protein.electrophoresis.
BK.Bone.scan.
.Answer:..
.
91.#What#are#you#going#to#find#in#the#blood#film#of#sickle#cell#anemia#patient?##
AK.Spherocytosis.&.shictocytes.
BK.Pencile.cells.
CK.HowellKJolly.body.
DK.Target.cells.
.Answer:..
.
92.#Characteristic#feature#of#NHL?#
AK.Extranodal.involvement.
.Answer:..
.
93.#Sickle#cell#disease#triggering#to#occlusive#crisis?#
AK.Parvo.virus.B19.
BK.EBV.
Answer:.#

Nephrology.
#
1.#Which#one#of#the#following#present#with#positive#anti–glomerular#basement#membrane#
antibody?#
AK.IgA.nephropathy.
BK.Post.streptococcal.glomerulonephritis.
CK.SLE.
DK.Goodpasture.syndrome.
Answer:.D.
Reference:.Toronto.Notes.2017.+.Medscape.
Goodpasture.Syndrome.associated.with.antibodies.against.type.IV.collagen.present.in.lungs.and.
GBM.
..
2.#Case#scenario#of#nephrotic#syndrome#and#asking#about#treatment?#
AK.Prednisone.
BK.Azathioprine.
Answer:.A.
Reference:.Kaplan.Lecture.Notes.
Treatment.of.nephrotic.syndrome.is.to.control.underlying.disease..In.addition,.steroids.are.used.to.
treat.all.forms.of.idiopathic.primary.renal.causes.of.nephrotic.syndrome,.Such.as.membranous,.
minimal.change.disease,.membranoproliferative,.mesangial,.and.focal.segmental.disease..
If.steroids.do.not.work,.the.next.best.step.in.therapy.is.to.add.cyclophosphamide.or.mycophenolate..
Azathioprine.is.sometimes.useful..ACEIs.or.ARBs.are.used.for.all.patients.with.proteinuria,.but.they.
do.not.reserve.the.underlying.disease..
..
3.#Patient#presented#with#hematuria,#on#examination#he#had#bilateral#mass.#What#is#the#most#
likely#diagnosis?#
AK.Polycystic.kidney.disease.
Answer:.A.
Reference:.Toronto.Notes.2017.
Polycystic.changes.are.always.bilateral..It.is.often.asymptomatic,.but.patients.may.present.with.
abdominal.flank.pain/dull.lumbar.back.pain,.hematuria,.nocturia,.HTN.+/K.palpable.kidneys..
..
4.#Athlete#man#with#hematuria#and#calf#muscle#pain?#
AK.Rhabdomyolysis.
Answer:.A.
Reference:.WebMD.
The.“classic.triad”.of.rhabdomyolysis:.muscle.pain.in.the.shoulders,.thighs,.or.lower.back;.muscle.
weakness.or.trouble.moving.arms.and.legs;.and.dark.red.or.brown.urine.or.decreased.urination..
..
5.#Patient#drank#an#ethylene#glycol#containing#product,#what#renal#damage#is#suspected?#
AK.Acute.tubular.necrosis.
Answer:.A.
Reference:.Toronto.Notes.2017.
Etiology.of.ATN.
1..Toxins.
.. a..Exogenous:.antibiotics.(aminoglycosides,.cephalosporins,.amphotericin.B),.antiviral.
(cidofovir),.antineoplastics.(cisplatin,.methotrexate),.contrast.media,.heavy.metals,.other.
(fluorinated.anesthetic,.ethylene.glycol).
.. b..Endogenous:.endotoxins.(bacterial),.myoglobin,.hemoglobin..
2..Ischemia.
.. a..Decreased.circulating.volume.
.. b..Decreased.effective.circulating.volume.
.. c..Vessel.occlusion.
..
6..Patient#with#vomiting#and#hyperventilation,#what#acid#base#disturbance#will#the#patient#have?#
AK.Metabolic.acidosis.and.respiratory.alkalosis.
BK.Metabolic.acidosis.and.respiratory.acidosis.
CK.Metabolic.alkalosis.and.respiratory.acidosis.
DK.Metabolic.alkalosis.and.respiratory.alkalosis.
Answer:#D.
Reference:.Toronto.Notes.2017.
..
7.#Minimal#change#glomerulonephritis#
AK.LM:.normal,.EM:.fusion.of.foot.process.
Answer:#A.
Reference:.Kaplan.Lecture.Notes.
##
8.#Low#bicarb,#low#ph.#What#is#the#diagnosis?#
AK.Metabolic.acidosis.
BK.Metabolic.alkalosis.
CK.Respiratory.acidosis.
DK.Respiratory.alkalosis.
Answer:#A.
##
9.#Patient#with#tender#flank#and#mutation#in#chromosome#16.#What#is#the#mode#of#inheritance?#
AK.Autosomal.dominant.
BK.Autosomal.recessive.
.Answer:#A.
Reference:.Toronto.Notes.2017.
Adult.polycystic.kidney.disease.is.an.autosomal.dominant.disease;.
at.least.2.genes:.PKD1.(chr.16p).and.PKD2.(chr.4q).
..
10.#Young#Patient#with#hemoptysis,#week#later#he#developed#hematuria.#What#is#the#diagnosis?#
AK.Goodpasture.syndrome.
BK.HSP.
CK.PSGN.
Answer:#A.
Reference:.Kaplan.Lecture.Notes.
Goodpasture.syndrome.is.idiopathic.disorder.of.renal.and.lung.disease..Patients.may.present.with.
hematuria,.proteinuria,.hemoptysis,.cough,.and.SOB..
.
11.#Clear#case#of#post[streptococcal#glomerulonephritis.#What#is#the#most#important#step#in#
management?#
AK.High.dose.corticosteroids.
BK.Diuretics.for.high.blood.pressure.
CK.Diuretics.for.edema.
DK.Antibiotics.
Answer:#C.
Reference:.Nephrology.consultant.
If.the.patient.has.active.infection,.the.answer.would.be.antibiotics..If.not,.then.choose.diuretics..
..
12.#Patient#with#recurrent#UTI,#bilateral#hydronephrosis.#Renal#biopsy#showed#multiple#cysts#with#
some#description.#What#is#the#diagnosis?#
AK.Renal.cell.carcinoma.
BK.Polycystic.kidney.disease.
CK.Renal.dysplasia.
Answer:#B.
..
13.#20#years#old#male#with#history#of#hematuria#and#proteinuria#6#gm.#What#you#will#see#in#renal#
biopsy#(history#of#post[streptococcal#glomerulonephritis)?#
AK.Membrane.
BK.Minimal.
CK.Post.infection.
Answer:#Renal.biopsy.will.show.humps.on.EM..IgG.and.C3.will.be.deposited.in.the.mesangium.as.
subepithelial.humps.
Reference:.Kaplan.Lecture.Notes.
For.more.details.about.the.histologic.findings.K.https://emedicine.medscape.com/article/240337K
workup#c7.
..
14.#Patient#woke#up#with#edema#under#eye#+#oral#dehydrated.#Labs:#hypoalbuminemia,#
hyperlipidemia.#What#is#the#treatment?#
AK.Aspirin.
BK.Prednisone.
CK.Cyclophosphamide.
DK.Cyclosporine.
Answer:#B.
Reference:.Kaplan.Lecture.Notes.
##
15.#Picture#of#patient#with#macular#rash#in#buttocks,#associated#with#bilateral#knee#pain#(Henoch[
Schonlein#Purpura).#How#would#you#treat#this#disease?#
AK.Antibiotics.
BK.IVIG.
CK.Corticosteroids.
Answer:#C.
Reference:.Kaplan.Lecture.Notes.
HSP.most.often.resolves.spontaneously..Treatment.is.supportive..Steroids.can.be.used.if.the.disease.
is.progressive.and.severe..
..
16.#Case#of#glomerulonephritis,#which#of#these#considered#a#strong#support#of#your#diagnosis?###
AK.Streptozyme.test.
BK.Low.C3.
Answer:#It.depends.on.the.case..However,.the.single.most.important.test.is.renal.biopsy..
Reference:.Nephrology.consultant.
################################# #
17.#Typical#history#of#SLE#with#urine#casts.#What#is#the#diagnosis?#
AK.SLE.nephropathy.
Answer:#A.
..
18.#Patient#taking#diuretics,#he#has#muscle#weakness,#diarrhea,#irritability.#What#is#the#cause?#
AK.Hypokalemia.
BK.Hyperkalemia.
CK.Hyponatremia.
DK.Hypocalcemia.
Answer:#A.
Reference:.Nephrology.consultant.
##
19.#Patient#have#bilateral#thin#walled#parenchyma#of#kidney?#
AK.PKD.
Answer:#A?.
In.general,.thinKwalled.parenchyma.of.kidney.indicates.the.presence.of.cysts..
Upon.searching.online.I.found.the.following.possible.answers.(if.not.in.choices.I.would.choose.A).
1..Simple.renal.cysts.
2..Lymphangioma.in.adults.is.usually.characterized.by.peripelvic.and.perirenal.thinKwalled.cysts.
##
20.#Patient#with#hypokalemia,#Hypochloremia,#hyponatremia#and#HyperHCO3.#I#think#with#some#
symptoms,#what's#the#1ry#defect?#
AK.NaCl.
BK.H.excretion.......................................................................... .
CK.H.absorption.
DK.K.
Answer:#incomplete.question,.but.the.above.labs.could.be.caused.by.vomiting.
Reference:.Nephrology.consultant.
..
21.#Patient#with#muscle#weakness,#apart#from#hypotension#and#bradycardia,#his#examination#was#
normal.#His#lab#tests#(high#K+,#low#Na,#low#Chloride,#high#urea).#What#is#the#etiology#behind#his#
presentation?#
A..Hyponatremia.
B..Hyperkalemia.
C..Low.chloride.
D..Uremia.
Answer:#the.question.is.incorrect#(nephrology.consultant).
..
22.#Patient#presented#with#hematuria,#weight#loss,#loin#pain#(scenario#of#RCC).#What#is#the#gene#
affected?#
AK.VHL.
Answer:#A#
Reference:.
1K.https://www.ncbi.nlm.nih.gov/pmc/articles/PMC2873025/.
2K.https://www.ncbi.nlm.nih.gov/pubmed/22125026.
Alteration.of.the.VHL.gene.by.mutation,.loss.of.heterozygosity,.and.promoter.methylation.has.been.
found.to.be.important.to.renal.cell.cancer.pathogenesis..
..
23.#Scenario#of#a#patient#whose#thirsty#with#low#urine#output#because#of#ADH#secretion.#(They#
attached#a#picture#of#a#nephrons#with#number#labels#on#each#segment#and#asked#to#choose#which#
area#does#ADH#work#on).#
AK.Collecting.duct.(was.labeled.by.number.5.in.the.picture).
Answer:#A.
Reference:.Toronto.Notes.2017.
..
..
..
..
..
.

.
..
..
..
23.#Patient#with#controlled#DM#and#HTN#he#is#on#ACEI,#furosemide#and#spironolactone.#Electrolyte#
were#normal,#what#should#you#do?#
AK.Stop.spironolactone.
BK.Stop.furosemide.
Answer:#A.
Reference:.https://www.rxlist.com/aldactoneKdrug.htm.
##
24.#Renal#cell#carcinoma#marker?#
AK.M2KPK.
Answer:#?.
Reference:.
1K.https://www.ncbi.nlm.nih.gov/pubmed/3776772.
ESR.and.fibrinogen.were.thought.to.be.useful.for.screening.renal.cell.carcinoma.patients.ESR,.CEA,.
haptoglobin,.fibrinogen.and.C3.for.estimation.of.staging,.and.ESR,.CEA,.haptoglobin.and.C3.for.
estimation.of.prognosis..
.
2K.https://www.ncbi.nlm.nih.gov/pubmed/23113677.
αKenolase.could.be.used.in.the.postoperative.follow.up.of.renal.cell.carcinoma.patients,.whereas.the.
combined.use.of.galectinK1.and.galectinK3.might.represent.a.useful.tool.for.primary.detection..
.
3K.https://www.hindawi.com/journals/dm/2015/251403/.
Unfortunately,.there.is.no.existing.biomarker.for.kidney.cancer.diagnosis..The.currently.available.
biomarkers.appear.to.have.the.most.utility.as.diagnostic.adjuncts,.as.prognostic.indicators,.and.in.
following.up.patients..
..
25.#Case#scenario#regarding#HUS#and#asking#about#treatment?#
Answer:#Plasmapheresis.is.used.to.treat.severe.cases.of.HUS..Mild.disease.resolve.spontaneously..
Reference:.Kaplan.Lecture.Notes.
##
26.#Minimal#change#GN#happens#at#which#age?#
Answer:#The.incidence.of.minimal.change.disease.peaks.in.children.aged.2.years,.with.approximately.
80%.being.younger.than.6.years.at.the.time.of.diagnosis..In.adults,.the.mean.age.of.onset.is.40.years.
Reference:.Medscape.
##
27.#Goodpasture's#syndrome#diagnosis?#
Answer:#the.best.initial.test.to.confirm.the.diagnosis.is.the.level.of.antiKbasement.membrane.
antibodies.to.type.IV.collagen..The.single.most.accurate.test.is.a.lung.or.kidney.biopsy..
Reference:.Kaplan.Lecture.Notes.
##
28.#Calculate#anion#gap#
Answer:#AG.=.Na.–.(HCO3.+.Cl).
Reference:.Toronto.Notes.2017.
Normal.range.10K14.mEq/L.
.

Neurology.
##
1.###########Patient#with#epilepsy#presented#complaining#of#gum#bleeding#and#increase#hair#in#face#
which#of#the#following#drugs#would#be#the#cause:#
A......Phenytoin.
B......Valporic.acid.
C......Carbamazepine.
..Answer:.A.
..
2.###########Treatment#of#trigeminal#neuralgia?#
A......Kcarbamazepine..
B......Knaproxen..
answer:.A.
NOTE:.
Carbamazepine.and.oxcarbazepine.are.considered.first.line.therapy.in.trigeminal.neuralgia.(TN)..
Reference:.Toronto.notes.http://emedicine.medscape.com/article/1145144treatment#d9.
..
3.###########Dental#procedure,#then#pt#developed#numbness#on#lower#or#upper#lip#what#is#the#nerve:#
Inferior#orbital#nerve#
..Answer:.A..Infraorbital.nerve.supplies.upper.lip..
..
4.###########Loss#of#forehead#muscle#action#what#is#the#nerve:#
Answer:.Facial.nerve.
..
5.###########Loss#of#anterior#2/3#tongue#taste:#
Answer:.Fascial.nerve.
Anterior.⅔:.(taste>.facial),.(sensation>.trigeminal)..Posterior.⅔:.glossopharyngeal..
..
6.###########Loss#of#posterior#1/3#tongue#sensation#what#is#the#nerve:#
Answer:.Glossopharyngeal.
..
7.###########Loss#of#gag#reflex#but#normal#uvula:#
1..glossopharyngeal.
2..Vagus.
.Answer:.A.
CN.IX.carries.sensory.information.(afferent.limb.of.reflex)..CN.X.carries.motor.(efferent)...
..
8.###########Pt#had#occipitofrontalis#paralysis#which#branch#of#facial#nerve#is#affected:#
A......Temporal.
B......Buccal.
.Answer:.A.
Frontal.part.from.temporal.branches..Occipital.part.from.posterior.auricular.branch..
..
9.###########Old#man#with#abnormal#hand#and#head#movement#he#took#drug#1#day#before#what#is#the#
drug:#
A............Chloramphenicol.
B............Digoxin.
C............ethambutol.
D........... Metoclopromide.
.Answer:.D.
ChlorPROMAZINE.Antidopaminergic.may.also.cause.dystonia...
..
10.##Child#with#rapid#blinking#of#the#eye#communicate#with#his#parents#during#it#what#is#the#
diagnosis#all#are#normal:#
A)TIC.
B).petit.mal.seizure.
C).tourette.syndrome..
.Answer:.A.most.likely..
Tics:.sudden.rhythmic.movement..
Tourette.syndrome:.2.or.more.tics..
Petit.mal.(absence).seizure.characterized.by.staring.off.into.the.distance,.without.communication..
..
11.##Case#of#myasthenia#gravis.#What#to#give?#
A......Pyridostigimine.
B......Rivastigimine.
C......Other.cholinergic.
.Answer:.A.
Toronto.notes:.symptomatic.treatment.(pyridostygmine)..
..
12.##which#of#the#following#drugs#used#in#Alzheimer's#disease#causes#heptotoxicity?#
1).tacrine.
2).rivastigmine.
3).galantamine.
4).donepezil.
Answer:.A.
.https://www.livertox.nih.gov/AlzheimersDrugs.htm.
..
13.##Which#of#the#following#is#common#cause#of#aseptic#meningitis?#
A)..Enteroviruse.
b).HSV.
Answer:.A.
viral.infection.is.the.most.common.form.of.aseptic.meningitis,.and.enteroviruses.are.the.most.
common.viral.cause..http://emedicine.medscape.com/article/1169489K
overview?pa=cGMbL4akzepNjcC0iphPLqQmWAHFzPxik4mBwCZXH5UtGqvCm5qPNND3MD3gTIsJNFs
YxDuz%2Fz2hge3aAwEFsw%3D%3D.
..
15.##12#yrs#old#wake#up#with#ptosis#and#diplopia#at#morning#then#symptoms#resolves?#
A......Botilism.
B......Myasthenia.gravis.
.Answer:.
Myasthenia.gravis.symptoms.usually.improve.throughout.the.day..
Botulism.starts.6K8.hrs.after.ingestion..Starts.with.paralysis.of.EOM.then.spreads.to.trunk.and.limbs..
Toronto.notes..
..
16.##Treatment#of#acute#migraine?#
Answer:.
1st.line:.acetaminophen,.NSAIDs,.ASA.±.caffeine.2nd.line:.NSAIDs.3rd.line:.5KHT.agonists.±.
antiemetic..
Toronto.notes..
..
17.##25#years#old#male#presented#after#acute#syncopal#attack#when#he#was#standing#in#a#waiting#
line.#He#recalls#feeling#of#dizziness,#blurring#of#vision#and#yawning#four#minutes#before#he#lost#his#
conscious#level.#Diagnosis:#
a)TIA.
b)Ischaemic.attack.
c)Fainting.
b)Out.of.body.experience.
..
18...Old.man.presented.by.his.children,.they.suspect.his.cognitive.function.is.impaired...what.can.you.
tell.them.about.trying.to.improve.his.condition.
A).Daily.exercise.will.help.
B).Brain.exercise.will.help.prevent.dementia.
C).New.measure.of.conducting.his.daily.activities.will.now.be.required.?.
..
19.###Duchenne#muscular#dystrophy#percentage#of#recurrence#
a)50%.
b)25%.
C)2%.
d)7%.
Answer:.A.
For.a.carrier.female,.with.each.pregnancy.there.is.a.one.in.two.(50%).chance.her.sons.will.inherit.the.
disease.allele.and.a.one.in.two.(50%).chance.her.daughters.will.be.carriers..
http://hihg.med.miami.edu/code/http/modules/education/Design/Print.asp?CourseNum=1&Lesson
Num=3.
.
20.##A#middle#age#woman#with#multiple#sclerosis,#complaining#of#urinary#incontinence..she#doesn't#
feel#the#urge#to#empty#her#bladder#but#urine#incontinence#occurs..#
a)Reflex.incontinence.like.urge,.but.no.urgency.
b)Stress.incontinence.
c)Overflow.incontinence.
d)Urge.incontinence.
Answer:C.
..
21.##Prophylaxis#of#migraine#in#asthmatic#pt:#
A).propanol.
B).amitriptyline.
Answer:.B.
..
22.##Adolescent#with#migraines;#what#is#the#long#term#complications:#
A......Hearing.loss.
B......Reflux.esophagitis.
C......Depression.
.Answer:.C.
..
23.##what#is#the#cause#of#ischemic#stroke#in#females:#
A).hypercoagulable.state.
B).previous.DVT.
Answer:.A..Intracranial.atherosclerosis.may.be.the.cause.of.thrombotic.stroke.in.patients.with.
widespread.atherosclerosis..Causes.include.hypercoagulable.state..
https://emedicine.medscape.com/article/1916852Koverview#a5.
..
24.##pt#complain#of#proximal#muscle#weakness#and#ptosis#there#is#improvement#after#
administration#of#anticholinesterase#what#is#the#cause:#
A).autoimmune.
B).viral.induced.
.C).drug.induced.
.D).nutritional..
.Answer:.A..Myasthenia.gravis..
..
25.##What#organism#can#cause#paralysis?#
Answer:.botolinium.
..
26.##Danlos#ehler#what#is#the#type#of#transmission?#
Answer:.Autosomal.dominant.
..Most.common.form.of.Ehlers.Danlos.is.AD..
..
27.##pt#k/c#of#MS#and#he#has#new#MS#attack,#treated#by:#
A......interferon,.
B......IVIG.,.
C......Glucocorticoids.
.Answer:.C.
Acute.treatment:.methylprednisolone.1,000.mg.IV.daily.x.3K7.d.(no.taper.required);.if.poor.response.
to.corticosteroids.may.consider.plasma.exchange.
Toronto.notes..
..
28.##Most#common#cause#cerebral#abscess:#
A......head.trauma.
B......something.in.ear.i.think.OM.,.
C......some.thing.in.nose.
..
29.##Male#present#with#multiple#maculs#5[15#mm#,#and#axilary#frickling#,#his#wife#is#pregnant#,#what#
is#the#mood#of#inheritance?#
Autosomal.
XKlinked.
Mitochondrial.
Answer:.A..Neurofibromatosis.is.autosomal.dominant..
..
30.#Alcoholic#patient#presented#withdrawal#manifestation,#mainly#CNS#(confusion,#wide#gate,#etc.)#
What#would#you#give#him?#
A... thiamine.
B... lorazepam.
Answer:.A..Basic.protocol.includes.thiamine.and.diazepam..
Toronto.notes..
..
31.##Case#of#MG#take#neostigmine#then#present#with#worse#symptoms##to#ER#what#is#the#Rx?#(No#
IVIG#in#answers)#
Answer:.Plasmapheresis.
.For.myasthenic.crisis.IVIG.and.plasmapharesis..
..
32.#Patient#with#facial#Bells#palsy#(UMNL)#which#of#the#following#muscles#will#be#intact?#
AK.orbicularis.oris.
BK.orbicularis.oculi.
CK.masseter.
DK.buccinator.
Answer:.B.if.UMNL..(Bell’s.palsy.is.a.LOWER.motor.neuron.lesion.affecting.whole.side.of.the.face)..
..
33.##Extradural#hematoma#source#of#bleeding:#
AKmiddle.cerebral.artery.
BKmiddle.meningeal.artery.
CKanterior.cerebral.artery.
.Answer:.B..Subdural>.bridging.veins..
..
34.#What's#true#about#dyslexia:#
1.! Prevalence.<1%.
2.! Associated.with.hearing.or.visual.impairment.
3.! Problem.in.brainstem.
Answer:.
Prevalence.10K20%..Poor.vision.is.NOT.a.cause..Neuroimaging.showed.variations.in.the.right.
temporoparietalKoccipital.region..
Medscape.and.Toronto.notes..
..
35.#Male#patient#presented#with#painful#lump#in#his#arm#that#developed#after#some#weight#lifting.#
What#would#be#the#appropriate#next#step?#
AK.Give.diclofenac.
..
36.##Treatment#of#MS#attack#in#ER:#
AK.Oral.antibiotic.
BKIV.antibiotic.
CK.Oral.steroid.
DK.IV.steroid.
.Answer:.D..Toronto.notes..
..
37.##pt#ē#DM#suddenly#develop#weakness#in#half#of#his#body#one#hour#ago,#no#dysarthria#,#no#
headache,#what’s#the#management?#
A......aspirin.
B......anti.plasminogen.
Answer:.B.(Given.that.the.patient.is.a.candidate.+.after.ruling.out.hemorrhage.with.CT).
•.Acute.ischemic.stroke:.thrombolytics.(rtKPA,.e.g..alteplase).if.within.4.5.h.of.symptom.onset.with.
no.evidence.of.hemorrhage.on.CT.scan.
•.Antiplatelet.agents:.prevent.recurrent.stroke.or.stroke.a.er.TIAs,.e.g..Aspirin®.(1st.line);.
clopidogrel,.Aggrenox®.(2nd.line)...Toronto.notes..
..
38.##Epileptic#drug#that#causes#hair#growth?#
.Phenytoin..
..
39.##old#pt#with#neck#stiffness#and#parasthesia#on#morning#exam#there#was#paraspinal#muscle#
spasm#,#ttt?#
Physiotherapy.
..
40.##S&S#of#stroke#,#what#is#the#first#step#in#the#management?#
Answer:.Brain.CT.
.Must.determine.whether.it.is.a.hemorrhagic.or.ischemic.type.of.stroke..
..
41.##pt#with#dm#&#htn#on#medication#had#tremor#with#movement?#(No#cerebellar#lesion#on#choices)#
A............Essential.tremor.
B............Psychological.
C............Parkinsonian.
D........... Physiological.
..
..
42.##Patient#with#meningitis,#culture#showed#gram#positive#bacilli,#what’s#the#organism?#
Listeria.
Answer:.A.
..
43.## #CSF#circulates#in#which#space?#
A... 1/epidural.
B... 2/subdural.
C......3/Subarchnoid.
Answer:.C.
..
..
44.##Drug#of#choice#in#absence#seizures?#
Answer:.Ethosuxamide.
..
45.##Pt#complain#of#proximal#muscle#weakness#and#ptosis#there#is#improvement#after#
administration#of#anticholinesterase#what#is#the#cause:#
A)...........autoimmune.
B)...........viral.induced.
C)...........drug.induced.
D)..........nutritional..
Answer:.A.
..
46.## #Another#scenario#about#Mysthenia#graves.#Part#affected?#
Acetylcholine.receptors.
Answer:.A.
Antibodies..against.Ach.receptors.
..
47.##Women#with#bilateral#breast#nipple#white#discharge#and#vision#disturbance#
Prolactin#level#is#high,#where#is#the#lesion:#
There.was.no.pituitary.lesion.in.choices.BUT.THERE.WAS.Sella.turcica.lesion.
Answer:.sella.turcica.
..
48.## #female,#K/c#of#epilepsy#on#phenytoin,#she#has#been#using#it#for#a#long#time,#she#came#e#a#
complaint#of#hairy#growth#in#her#body,#and#stated#that#she#is#symptom#free#for#about#6#months.#
What#is#the#correct#action?#
1/stop.phenytoin.
2/lower.the.dose.
3/shift.to.another.antiepileptic.(I.chose.this).
4/continue.the.drug..
..
49.#58#years#old#hypertensive#had#hemiplegia#for#10#hours#CT#show#no#hemorrhage#start#
physiotherapy,#examinations#show#hemiplegia,#take#ACIE,#thiazide,#had#acute#peptic#ulcer#before#2#
years#,#patient#need#which#drug:#
A... Noting.
B... Aspirin.
C... Anticoagulant.
D.. Tpa.
Answer:.B.
..
50.##[Parkinson#case#à#low#dopamine#
##
51.##[Another#case#of#parkinson#à#problem#in#substantia#nigra#
..
52.#CSF##readings#were#with#high##both#lymphcytes#&#neutrophils#high#protien#normal#glusose.à.
Cryptococcal.meningitis..my.answer.by.exclusion.
..
53...KToxoplasmosis.encephalitis.in.hiv.scenario.(scenario.of.HIV.pt.with.neurological.symptomes.).
..
54.##[#Stroke#presentation#(unilateral#body#weakness#&#slurred#speach#in#1#hr#)#what's#next#step#?#
CT.of.the.brain.
..
#55.#What#is#the#early#sign#of#increase#intracranial#pressure?#
A......Hypertension.
B......Unconscious.(Change.in.LOC.is.the.first.earliest.sign.in.increased.ICP).
C......Epailateral.papillary.....
D.....Contralateral.papillary....
Answer:.Change.in.LOC.
https://amy47.files.wordpress.com/2010/04/increasedKintracranialKpressureKpp.pptx.
..
56.##6hrs#post#cerebral#infarction#what#will#u#give?#
1KAspirin.
2KTPA.
3KWarfarin.
4KForgot.it.
.Answer:A.
57.##Most#serious#cause#of#stroke#
. K.HTN
. K.Arrhythmias
. K.Smoking
Answer:.A?.
..
58.##Active#seizure#pt#what#to#give#;#
A......Phenotin.
B......Diazepam.
.Answer:.B.
..
59.#Patient#with#guillain#barre,#what#cell#is#affected?#
Answer:.schwan.cell.
..
60.##[Pt#have#medial#squent#the#lesion#is#where#?#
Pone.
Medulla.ablngate.
Midbrain.
.Answer:.A.
CN.VI.nucleus.located.in.pons..Exits.at.pontomedullary.junction..
https://emedicine.medscape.com/article/1198383[overview#
##
61.##Scenario#of#megis#syndrome#(triad),#what#is#the#appropriate#investigation#for#diagnosis#
MRI.pelvis.
Histopatholgy.
Tumor.markers.
.Meig’s.syndrome.(benign.ovarian,.tumour.and.ascites.and.pleural.effusion).
..
62.##Case#scenario#child#with#sign#and#symptoms#of#meningitis#gram#stain#and#culture#result#was#
gram#+ve#cocci#what#is#the#treatment:#
Answer:.ceftriaxone.and.vancomycin.
.Answer:.A.
..
63.##Sign#in#Duchene#muscular#dystrophy?#
Answer:..gowers.sign.
..
64.#Patient#with#left#side#paralysis#(stroke)#what#to#do?#
A......CT.is.first.to.know.type.
B... aspirin.
Answer:.A.
..
65.#CP#patient#all#limbs#affected#but#the#lower#limbs#less#spastic?#
Quadriplegia.
..
66.#Long#scenario#about#patient#come#to#ER#with##chest#pain#radiate#to#left#arm#then#get#coma#and#
death#what#you#will#find#in#the#brain#?#
1K.necrosis.because.of.left.middle.artery.
2Kred.neuron.degeneration.in.hippocampus.
..
67.#Man#lost#smell,#which#lobe#is#affected#
A).parietal.
B).temporal.
C).frontal.
D).occipital.
Answer:.B...
..
68.#Man#with#stroke#and#vision#loss,#affected#lobe:#
A).parietal.
B).temporal.
C).frontal.
D).occipital.
Answer:.D.
..
68...Patient.with.pain.in.lips.and.left.cheek,.with.tenderness,.what.to.give?.
A... Carbamezapine.
B... Amytriptaline.
C... Propanolol.
Answer:.A.
First.line.treatment.of.trigeminal.neuralgia.is.carbamazepine..Toronto.notes..
..
69.#Migraine#+#HTN#?#(#I#got#the#Q#without#B#blocker#in#the#choices)#
B.blocker.
CCB.
Answer:.A.
..
70.##Year#old#male,#newly#married,#axillary#freckles#pigmentations#around#5[15#cm,#his#cousin#has#
the#same#condition..His.wife.is.pregnant.what.is.the.mode.of.inheritance.of.disease.to.child?.
A......autosomal.linked.(.I.think.this.is.the.correct.answer.cause.the.father.might.have.NF).
B......mitrochondrial.
C......xK.linked.
Answer:.C.
..
71.##Asthmatic#lady#with#recurrent#severe#headache#and#photophobia.#She#gets#the#attacks#3[4#
times#per#week.#She#is#unable#to#sleep#and#has#poor#appetite.#What#is#the#prevention?#
AKAmitryptalin.
BKPropranolol.
CK.Verapamil.
Answer:.A.
Prophylaxis:.1..TCA.2..Anticonvulsants.3..Propranolol..Toronto.notes..
..
72.##Teacher#with#band#like#headache?#
Stress.headache.
Answer:.tension.headache..
..
73.##Case#about#cluster#headache.#What#is#the#treatment#?.(‫ﺪ)ﻚ‬+, ‫ﻋﺮﻓ&ﺎ ﻟﻠﻜﻠﺴﺘﺮ‬$ ‫)ﺔ‬67$ ‫ ﻓﻲ‬9‫)ﻣﺎ ﻛﺎ‬.
Ergotamine.
Paracetamol.
Answer:.
Acute.Rx:.•.O2.•.Sumatriptan.(nasal.or.injection)..
Prophylaxis:.•.Verapamil.•.Lithium.•.Methysergide.•.Prednisolone.
Toronto.notes..
Acute:#sumatriptan,#100%#O2#
..
74.##Prevention#of#cluster#headache?#
Verapamil.
..
Q.#Scenario#typical#of#cluster#headache#in#a#male#with#eye#pain,#tearing,#comes#clusters#during#the#
day.#What#prophylactic#treatment?#
A......propanol.
B......valporic.acid.
C......amytriptalin.
D.....verapamil.
Answer:.D..
..
.75.#Pt#with#signs#of#neck#rigidity#and#headache#came#with#seizures,#they#gave#her#lorazepam.#
What’s#the#next#step?#
A..Lumbar.puncture.
B..another.dose.of.lorazepam.
Answer:.A..Do.not.delay.antibiotics.for.lumbar.puncture.or.blood.culture..
Toronto.notes..
..
76.#Pt#with#neck#rigidity#and#abdominal#muscle#rigidity#(clear#extrapyramidal#manifestations),#after#
administering#some#drug.#What#is#the#drug?#
Metochlopramide.
..
77.##[Patient#with#increase#ICP#what#nerve#would#u#examine#before#the#CT#scan#
A......II.
B......III.
C......IV.
D.....X.
Answer:.A.
Increased.ICP.can.cause.CN.VI.palsy..CN.II.will.show.papilledema..
..
78.##To#decrease#stroke#
Answer:.HTN.screening.
..
79.##Elderly#take#haloperidol#and#came#to#ER#afebrile#rolled#eye#
A......Neuroleptic.syndrome.
B......Dyskinesia.
C......Antonia.
.Answer:.B.
..
80.##Amenorrhea,#low#prolactin,#what#to#see#in#CT#or#MRI:#
Answer:.Sella.turcica.
..
81.#60#year#old#male#known#to#have#DM#and#HTN#came#complaining#of#tremors#of#the#upper#limbs#
that#have#been#progressing#during#the#last#6#months#especially#when#the#patient#is#stressed.#It#
increases#with#movement#and#disappears#at#rest.#What#kind#of#tremor#does#he#have?#
1).essential.
2).psychological.
3).Parkinsonian.
4).aggravated.physiological.
.Answer:A.
Parkinsonian>.resting..Essential>.Postural,.worsened.with.sustained.posture..
..
82.##[Pt#came#with#his#daughter#she#noticed#his#father#starting#to#forget#and#recently#started#to#
have#problems#with#his#daily#activity.#What#drug#would#u#start#
A......Pyridostigmine.
B......Rivastigmine.
C......Neostigmine.
.Answer:.B.
Management.of.dementia..Toronto.notes..
..
83.###16#yo#with#GI#symptom#&#bilateral#symmetric#muscle#weakness#
Botulism.
Answer:.
If.preceded.by.exposure,.followed.by.EOM.disturbance.then.spasms.spreading.to.trunk.and.limbs>.
botulism..
If.preceded.by.history.of.URTI.or.GI,.followed.by.ascending.weakness>.Guillian.Barre.syndrome..
..
84.##Female#c/o#breast#milk#discharge#and#irregular#menses,#ask#about#investigation:#
A.tsh.
B.brain.mri.
C.biopsy.
.Answer:.B.
..
85.#Scenario#snake#bite.#What#should#you#do?#
1K.evacuate.venom.
2K.apply.tourniquet.
3K.immobilize.limb.
Answer:C.
.Answer:.Supportive..Wound.irrigation/debridement..Compression.bands.NOT.useful..Antivenom.if.
appropriate..
Toronto.notes..
##
86.##[#Parents#of#child#noticed#that#he's#sitting#alone,#not#developing#well,#playing#with#his#toy,#and#
have#repetitive#behavior#what's#ur#dx:#
AK.Global.developmental.delay.
BK.Autism.
Answer:.B.
Ref:.https://www.cdc.gov/ncbddd/autism/hcpKdsm.html.
..
88.##[What#is#the#pathology#in#Huntington#syndrome?##What#exactly#increase#and#decrease#and#
where#it’s#located?#
1Ksomething.related.to.stratum.whith.nerotransmitter.deffect.have.look.for.it.*.
Huntington's.primarily.affect.the.striatum..
Dopamin.level.is.normal.(.in.contrast.with.Parkinson)..
GABA.is.decrease.(.causing.chorea.because.there.is.no.GABA[inhibitory.neurotransmiter].&.continue.
move)...
The.most.striking.neuropathology.in.HD.occurs.within.the.neostriatum,.in.which.gross.atrophy.of.the.
caudate.nucleus.and.putamen.is.accompanied.by.selective.neuronal.loss.and.astrogliosis..Marked.
neuronal.loss.also.is.seen.in.deep.layers.of.the.cerebral.cortex..Other.regions,.including.the.globus.
pallidus,.thalamus,.subthalamic.nucleus,.substantia.nigra,.and.cerebellum,.show.varying.degrees.of.
atrophy.depending.on.the.pathologic.grade..
..
89.#Patient#with#abnormal#movements.#What#is#the#etiological#drug:#
K.... Digoxin.
K.... Can’t.remember.other.choices.
.(Usually.this.scenario.gives.metoclopramide.as.an.option)..
..
90.#Wernicke's#area#injured,#which#type#of#aphasia?################################################# #
A......can.understand.but.can't.speech..................................................................... .
B......2Kcan't.understand.or.speech.or.write.
Answer:.Can’t.comprehend/understand.but.speaks.fluently.(receptive.aphasia)..
Toronto.notes..
..
91.##Phyenytoin#side#effect?#
Gum.hyperplasia.
##
93.##What#is#a#relative#contraindication#of#Methylergometrine?#
AKAsthma.
BKDM.
CKHTN.
.Answer:.C.
..
95.###############Prevention#of#postherpatic#neuralgia#
Answer:.Antivirals..
•.early.treatment.of.acute.herpes.zoster.with.antivirals.(acyclovir;.longerKacting.famciclovir.
and.valacyclovir.more.effective)..
•.treatment.of.herpes.zoster.with.corticosteroids.DOES.NOT.decrease.PHN..
Toronto.notes..
..
96.#female,#K/c#of#epilepsy#on#phenytoin,#she#has#been#using#it#for#a#long#time,#she#came#e#a#
complaint#of#hairy#growth#in#her#body,#and#stated#that#she#is#symptom#free#for#about#6#
months.#What#is#the#correct#action#?#
A......stop.phenytoin.

B..lower.the.dose .

C.....Shift.to.another.antiepileptic.(.I.chose.this).

D.....Continue.the.drug..

.
.

Pediatrics.
#
Part#1#
●... A#child#presented#with#painless#neck#mass,#cough,#sore#throat#and#generalized#pruritus?#
1.! Streptococcal.pharyngitis.
2.! Hodgkin’s.lymphoma.
3.! Infectious.mononucleosis.
4.! Lyme.disease..
Answer:.C.
https://emedicine.medscape.com/article/963894Kclinical.
..
.
●... 2#month#old#baby#with#creamy#white#plaques#in#the#mouth.#When#he#was#1#week#old#he#
developed#conjunctivitis#treated#with#erythromycin.#What’s#the#best#drug?#
1.! Oral.nystatin.
2.! Oral.tetracycline.
Answer:.A.
http://cursoenarm.net/UPTODATE/contents/mobipreview.htm?4/7/4209.
This.is.a.case.of.oral.thrush.caused.by.candida.albicans.due.to.the.use.of.antibiotics.(erythromycin).
..
●... Mechanism#of#polio#vaccine?#
http://cursoenarm.net/UPTODATE/contents/mobipreview.htm?13/63/14335.
. .

#
●... Child#with#Iron#toxicity#several#hours#ago,#what#will#you#do?#
1.! Gastric.lavage.
2.! Activated.charcoal.
3.! IV.deferoxamine.
Answer:.C.
I.think.its.IV.deferoxamine..
The.Q.lack.a.lot.of.information,.below.is.what.I.could.gather.about.Iron.toxicity..
Acute#Iron#posining:#http://cursoenarm.net/UPTODATE/contents/mobipreview.htm?16/16/16641.
Uptodate:. The. decision. to. perform. GI. decontamination. is. based. upon. the. specific. poison(s).
ingested,.the.time.from.ingestion.to.presentation,.presenting.symptoms,.and.the.predicted.severity.
of.poisoning..GI.decontamination.is.most.likely.to.benefit.patients.who:.Present.for.care.soon.after.
ingestion.(usually.within.one.to.two.hours).

Pediatric#Iron#Toxicity#Treatment#&#Management:#

https://emedicine.medscape.com/article/1011689[treatment#
Indications. for. deferoxamine. include. shock,. altered. mental. status,. persistent. GI. symptoms,.
metabolic.acidosis,.pills.visible.on.radiographs,.serum.iron.level.greater.than.500.µg/dL,.or.estimated.
dose.greater.than.60.mg/kg.of.elemental.iron..
..
●... Scenario#of#a#child#presenting#with#bilious#vomiting.#What’s#the#diagnosis?#
1.! Pyloric.stenosis.
2.! Duodenal.atresia.
3.! Meckel’s.diverticulum.
Answer:.B.
..
https://emedicine.medscape.com/article/1011689Ktreatment.
..
●... 17# year# old# athlete# male# gained# 7# kg# lately# and# he# has# all# the# characteristics# of# normal#
puberty,#he#also#has#a#foul#smelling#breath?#
1.! Anabolic.steroid.
2.! puberty.
..
I.couldn’t.find.a.good.explanation..
But,.Athlete.+.weight.gain.+.bad.breath.all.favors.the.use.of.steroid.
..
●... 3#year#old#girl#ingested#20#pills#of#isoniazid.#What’s#the#treatment?#
##
http://cursoenarm.net/UPTODATE/contents/mobipreview.htm?10/12/10447.
https://emedicine.medscape.com/article/180554Ktreatment#d8.
##
●... A#9#or#12#month#old#asthmatic#child#on#corticosteroids#which#vaccine#to#give?#
##
●... A# child# who’s# always# alone,# doesn’t# own# any# toys# and# doesn’t# play# with# others.# What# is#
the#relation#cause#or#something?#
1.! Intelligence.
2.! Interpersonally.
Answer:.B.
..
●... Scenario#of#a#child#who#swallowed#batteries.#How#would#you#manage#this#case?#
1.! Observation.for.12.hours.
2.! IM.glucagon.
3.! Immediate.endoscopy.
4.! Remove.it.using.a.foley’s.catheter.
Answer:.C.
https://emedicine.medscape.com/article/774838Ktreatment.
..
●... Which#of#the#following#organisms#cause#acute#epiglottitis?#
1.! Influenza.
2.! Hemophilus.influenza.
Answer:.B.
..
●... Neonate# with# umbilical# swelling# and# yellow# discoloration# over# it.# The# patient# is# vitally#
stable.#What’s#the#diagnosis?#There#was#a#picture.##
1.! Urachal.cyst.
2.! Umbilical.granuloma.
3.! Omphalomesenteric.cyst.
https://emedicine.medscape.com/article/935618Koverview#a12.
..
●... An#infant#starts#smiling#at#which#age?#
1.! 4.weeks.
2.! 8.weeks.
3.! 16.weeks.
Answer:.B.
.
Toronto.notes:.

.
Illustrated.textbook.of.pediatric:.

.
●... A#child#who#can#ride#a#tricycle,#what’s#his#age?#
Answer:.3.years.
..
●... Which#one#is#considered#as#delayed#milestone#in#a#13#month#old#child?#
Answer:.Sit.with.support.
..
●... Question#about#Rickets#vs#hypophosphatemia#(know#how#to#differentiate#between#the#lab#
investigations).#
##
●... Diaper#rash#with#satellite#lesion?#
DX:.Candidal.diaper.dermatitis.
Irritant.diaper.dermatitis:.Shiny,.red.macules/patches,.no.skin.fold.involvement..
Candidate. dermatitis:. Erythematous. macerated,. papule/plaques,. Satellite. lesions,.
involvement.of.skin.folds..
http://bestpractice.bmj.com/bestKpractice/monograph/676/diagnosis/historyKandKexamination.html.
##
●... 8# and# a# half# year# old# child# with# breast# bud,# sparse# pubic# hair# and# mild# pigmentation# of#
labia?#
1.! Normal.development.
2.! Precocious.puberty.
Answer:.A.
Precocious. puberty. refers. to. the. appearance. of. physical. and. hormonal. signs. of. pubertal.
development. at. an. earlier. age. than. is. considered. normal.. For. many. years,. puberty. was.
considered.precocious.in.girls.younger.than.8.years;.however,.recent.studies.indicate.that.
signs. of. early. puberty. (breasts. and. pubic. hair). are. often. present. in. girls. (particularly. black.
girls).aged.6K8.years..For.boys,.onset.of.puberty.before.age.9.years.is.considered.precocious..
https://emedicine.medscape.com/article/924002Koverview.
..
●... UTI#treatment#in#a#child?#
Answer:.Augmentin.for.10.days..
https://emedicine.medscape.com/article/969643Ktreatment.
##
●... Child#with#DKA#what’s#the#initial#treatment?#
1.! Bolus.of.normal.saline.
2.! SC.insulin.
3.! IV.insulin.infusion.
Answer:.A.
It. is. important. to. pay. close. attention. to. the. correction. of. fluid. and. electrolyte. loss. during. the. first.
hour. of. treatment.. This. always. should. be. followed. by. gradual. correction. of. hyperglycemia. and.
acidosis.. Correction. of. fluid. loss. makes. the. clinical. picture. clearer. and. may. be. sufficient. to. correct.
acidosis.. The. presence. of. even. mild. signs. of. dehydration. indicates. that. at. least. 3. L. of. fluid. has.
already.been.lost..
https://emedicine.medscape.com/article/907111Ktreatment#d1.
..
●... A#child#with#constipation#since#birth#which#investigation#will#help#establish#diagnosis?#
1.! Plain.x.ray.
2.! Manometry.
3.! Rectal.biopsy.
4.! US.
Answer:.A.
It.is.often.very.difficult.to.differentiate.a.distended.colon.from.small.bowel.on.the.basis.of.a.plain.
abdominal.xKray.of.a.neonate.with.intestinal.obstruction..The.presence.of.airKfluid.levels.is.evidence.
of.obstruction,.but.is.nonKspecific..There.may.be.an.absence.of.air.in.the.lower.pelvis..A.normal.film.
does. not. exclude. the. possibility. of. Hirschsprung's. disease,. but. without. any. evidence. of. distended.
colon.it.is.unlikely.to.be.this.disease,.so.plain.abdominal.xKray.should.always.be.performed.initially..
http://bestpractice.bmj.com/bestKpractice/monograph/750/diagnosis/stepKbyKstep.html.
..
●... 3#day#old#neonate#with#normal#perinatal#history.#Appears#tired,#with#burned#sugar#smell#in#
the#urine?#
1.! Maple.syrup.sugar.disease.
2.! Phenylketonuria.
Answer:.A.
https://emedicine.medscape.com/article/946234Koverview.
..
●... A# child# came# with# symptoms# of# meningitis,# and# culture# showed# gram# [ve# diplococci,# his#
brother#had#contact#with#him.#What#to#do#for#his#brother?#
Answer:.rifampicin..
##
●... Picture#of#CTG,#what’s#the#diagnosis?#
..
●... Newborn# has# tetany# after# delivery.# Calcium:# 1# (normal# 2[3).# One# year# later,# he# has#
recurrent# upper# respiratory# infection# with:# HiB,# HSV,# Pneumocystis# jirovecii,# aspergillus.# What# is#
his#underlying#condition?#(Digeorge#syndrome)#
1.! 22q11.2.deletion.
2.! Depletion.of.cd4.lymphocytes.
3.! Wiskott.Aldrich.syndrome.
4.! NADPH.oxidation.
Answer:.A.
..
●... Child#with#hip#pain,#x[ray#was#normal#but#US#showed#fluid.#Lab#revealed#high#ESR,#and#CRP,#
otherwise#normal.#What#to#do?#
1.! MRI.
2.! CT.Hip.and.pelvis..
3.! Aspiration.
Answer:.A.
The.Q.lacks.some.information..
It.could.be.a.case.of.Transient.synovitis.vs.Septic.arthritis.
Medscape:.
Adding.in.the.CRP.as.a.predictive.factor,.Jung.et.al.found.that.patients.with.4.of.5.predictors.(body.
temperature. >37ºC,. ESR. >20. mm/h,. CRP. >1. mg/dL,. WBC. >11,000/mL,. and. an. increased. hip. joint.
space. of. >2. mm). had. a. high. probability. of. having. septic. arthritis. and. were. candidates. for. further.
study.by.MRI.or.joint.aspiration..
In. settings. in. which. routine. aspirations. of. effusions. is. not. performed,. an. MRI. may. help. physicians.
differentiate.transient.synovitis.from.septic.arthritis..
https://emedicine.medscape.com/article/1007186Kworkup#c5.
..
●... Typical#case#seborrheic#dermatitis:#greasy#lesions#on#a#baby’s#head.#
..
●... What#are#the#findings#upon#auscultating#a#child#diagnosed#with#croup?#
1.! Inspiratory.wheeze.because.of.the.secretions.
2.! Inspiratory.wheeze.because.of.narrowing.of.the.subglottic.area.
Answer:.B.
http://bestpractice.bmj.com/bestKpractice/monograph/681/basics/pathophysiology.html.
https://emedicine.medscape.com/article/962972Koverview.
Croup.manifests.as.hoarseness,.a.sealKlike.barking.cough,.inspiratory.stridor,.and.a.variable.degree.
of. respiratory. distress.. However,. morbidity. is. secondary. to. narrowing. of. the. larynx. and. trachea.
below. the. level. of. the. glottis. (subglottic. region),. causing. the. characteristic. audible. inspiratory.
stridor..
..
..
●... Scenario#of#a#child#with#meningitis.#CSF#findings#are#as#follows:#High#cellular#count,#normal#
glucose#and#high#protein.#What’s#the#diagnosis?#
1.! Tuberculous.meningitis.
2.! Viral.meningitis.
Answer:.A.
It.also.could.be.GBS,.as.they.have.elevated.protein.
..
●... A# child# with# fever# for# 3# days,# then# he# developed# a# rash.# There’s# also# a# picture# showing#
small#spots#in#the#buccal#mucosa#(koplik#spots).#What’s#the#diagnosis?#
1.! Measles.
2.! Kawasaki.
Answer:.A.
..
●... Which#of#the#following#is#part#of#Tetralogy#of#fallot?#
1.! Pulmonary.stenosis.
2.! Left.ventricular.enlargement.
Answer:.A.
..
●... A#baby#with#pansystolic#murmur#in#the#3rd#left#parasternal#area.#He#also#turns#blue#when#
he#cries.#What’s#the#diagnosis?#
Answer:.VSD.
VSD.is.acyanotic.heart.disease,.unless.Eisenmenger’s.syndrome.has.developed.
..
●... 13# month# old# baby# with# a# pan# systolic# murmur.# His# echo# shows# a# 2# ml# opening# in# the#
muscular#interventricular#septum.#How#will#you#manage#this#baby?#
1.! Watchful.waiting.
2.! Surgical.repair.
Answer:.A.
Small.muscular.ventricular.septal.defects.(VSDs).have.a.high.spontaneous.closure.rate.(80K
90%).within.the.first.2.years.of.life.and.often.require.no.medical.or.surgical.management..
Larger.defects.may.not.close.but.may.become.smaller.with.time..
https://emedicine.medscape.com/article/899873Ktreatment.
..
●... A#14#year#old#boy#presented#to#the#clinic#to#take#his#second#dose#of#varicella#vaccine#(His#
1st#one#was#taken#a#year#ago).#Which#of#the#following#is#the#correct#management?#
1.! Give.the.second.dose.
2.! Check.his.varicella.titer.then.give.the.second.dose.
3.! Start.from.the.beginning.and.give.first.and.second.dose.
Answer:.A.
People.13.years.of.age.and.older.(who.have.never.had.chickenpox.or.received.chickenpox.vaccine).
should.get.two.doses.at.least.28.days.apart..
http://www.immunize.org/vis/varicella.pdf.
..
●... A#baby#who#can#sit#in#a#tripod#position,#roll#over#and#reach#out#for#objects.#How#old#is#he?#
Answer:.6.months.
##
●... A#child#who#can#tell#a#story,#draws#his#brother#and#pretends#to#be#a#father.#How#old#is#he?###
1.! 4.years.old.
2.! 5.years.old.
Answer:.B.
..
●... A#mother#presented#to#the#clinic#with#her#3#year#old#child#asking#if#he#has#developmental#
delay?#(Read#about#the#milestones#of#a#3#year[old)#
##
●... A#child#who#can#obey#2#step#commands#and#strangers#can#understand#75%#of#his#words.#
He#asks#questions#frequently.#Which#of#the#following#is#true?#
1.! The.child.has.normal.development.
2.! Delayed.speech.
3.! Delayed.response.
I.couldn’t.find.anything.related.to.(asks.questions.frequently).
Speech.and.language:.
2.years.old:.obey.2.step.commands.
3.years.old:.strangers.can.understand.75%.of.his.words..
##
●... A# child# presented# to# the# ER# after# swallowing# caustic# material# and# he’s# drooling.# What’s#
you#immediate#management?#
1.! Protect.the.airways.
2.! Activated.charcoal.
3.! Emergency.endoscopy.
Answer:.A.
Always.start.with.Airway,.Breathing,.Circulation.
..
●... Scenario#of#a#child#with#recurrent#fractures#and#a#blue#sclera.#What’s#the#diagnosis?#
Answer:.Osteogenesis.imperfecta.
..
●... Child#with#frequent#vomiting#and#GERD.#What’s#the#pathophysiology?#
Answer:.Weak.lower.esophageal.sphincter.pressure.
..
●... In#a#female,#the#spine#stops#growing#after#the#onset#of#menarche#by#how#many#months?#
1.! 6.months.
2.! 12.months.
3.! 24.months.
Answer:.C.
In.93.%.of.girls,.the.first.physical.sign.of.puberty.occurs.about.2.years.before.menarche,.and.
final#height#is#usually#achieved#2.5–3#years#after#menarche#
https://www.ncbi.nlm.nih.gov/pmc/articles/PMC3566248/.
..
●... A# mother# brought# her# 13# month# old# baby# o# the# clinic.# He# was# found# to# have#
developmental#delay.#Which#of#the#following#is#considered#delayed#in#a#baby#his#age?#
1.! Crawling.
2.! Walks.using.furniture.
3.! Can.walk.alone.
Answer:.A.
..
●... Child#presented#with#breath#that#smells#like#garlic.#What’s#the#diagnosis?#
I#couldn’t#find#anything#related#
##
●... Parents# of# a# diabetic# child# called# you# to# tell# you# that# he# lost# consciousness.# What# would#
you#advise#as#part#of#the#initial#management?#
Answer:.IM.glucagon.
..
●... 13#year#old#boy#with#delayed#puberty.#His#growth#parameters#are#normal#and#he#has#fair#
hair#in#the#groin.#How#will#you#manage#him?#
1.! Wait.
2.! Free.testosterone.
3.! Testicular.US.
Answer:.A.
In. most. patients,. however,. the. distinction. between. congenital. GnRH. deficiency. and. constitutional. delay. of.
puberty. remains. uncertain,. and. can. be. resolved. only. with. serial. observations.. In. view. of. these. diagnostic.
difficulties,.the.initial.therapeutic.approach.is.similar.for.both.disorders.[14,15]..The.two.major.options.are:.

●......"Watchful.waiting".with.reassurance.and.psychological.support.for.the.patient.and.family.

●......Administration.of.gonadal.steroids..

ShortKterm.hormonal.therapy.with.testosterone.in.boys.and.with.estrogen.in.girls.may.be.appropriate.when.
the.pubertal.delay.is.severe.or.the.patient's.psychosocial.concerns.about.the.delay.play.a.prominent.role.that.
cannot.be.addressed.by.reassurance.and.education.alone..

Except.under.unusual.circumstances,.therapy.should.be.restricted.to#boys#older#than#14#years.and.girls.older.
than.12.years.who.show.few.or.no.signs.of.puberty.and.are.expressing.considerable.anxiety.about.their.delay..
The.shortKterm.use.of.exogenous.testosterone.in.boys.or.estrogen.in.girls.does.not.appear.to.have.any.longK
term.sequelae.except.for.the.potential.of.skeletal.maturation.that.might.result.in.some.loss.of.adult.height..

http://cursoenarm.net/UPTODATE/contents/mobipreview.htm?40/20/41295.

................
●... Female#with#delayed#puberty.#She#has#a#wide#neck,#no#breast#buds.#What’s#her#diagnosis?#
1.! Turner.syndrome.
2.! Noonan.syndrome.
3.! Fragile.X.syndrome.
Answer:.A.
..
●... 46# year# old# mother# presented# with# her# baby.# He# has# ASD# and# other# features# of# Down#
syndrome?#
Answer:.Trisomy.21.
..
●... A#case#of#precocious#puberty#
http://bestpractice.bmj.com/bestKpractice/monograph/1127/diagnosis/stepKbyKstep.html.
##
●... Antibiotics# used# in# neonatal# sepsis:# either# 3rd# generation# cephalosporins# (cefotaxime)# or#
combination#(ampicillin#+#amoxicillin#+#gentamicin).#
The. current. approach. to. the. treatment. of. earlyKonset. neonatal. sepsis. includes. combined. IV.
aminoglycoside.and.expandedKspectrum.penicillin.antibiotic.therapy..
Cephalosporins.are.attractive.in.the.treatment.of.nosocomial.infection.because.of.their.lack.of.doseK
related. toxicity. and. their. ability. to. reach. adequate. serum. and. cerebrospinal. fluid. (CSF).
concentrations;.however,.their.use.has.led.to.resistance.in.gramKnegative.organisms..
https://emedicine.medscape.com/article/978352Ktreatment#d6.
##
●... Face#cellulitis#in#peds:#group#A#streptococcus#
##
●... Aspirin# in# kids# after# viral# infection:# Reye’s# syndrome# (fulminant# hepatitis# and# cerebral#
edema).#
https://emedicine.medscape.com/article/803683[overview#
##
●... Qs#about#APGAR#score.#
##
●... Milestone#of#a#baby#who#can#hold#his#head#and#when#he#looks#at#his#flying#hand#he#laughs?#
Answer:.4.months.
..
●... Scenario# of# a# child# who# went# to# a# camp# and# developed# fever# and# vomiting# after# coming#
home?#
Answer:.brucellosis.(not.sure).
The.presentation.doesn’t.go.with.brucellosis.or.tickKborne.infections..
It.could.be.simple.gastroenteritis.
..
●... Scenario#of#a#child#who#was#born#pre[term#and#has#shortness#of#breath?#
Answer:.Surfactant.
RDS.caused.by.surfactant.deficiency.
https://emedicine.medscape.com/article/976034Koverview.
..
●... Long# scenario# the# summary# is# a# child# who# was# normal# when# he# was# breast# feeding# and#
now# his# mother# feeds# him# juice# and# he# developed# symptoms# (I# only# remember# jaundice).# What#
should#be#avoided?#
Answer:.Fructose.
..
●... A#child#with#episodes#of#cyanosis#in#lips#and#peripheries#+#precordial#murmur.#What#is#best#
to#reach#a#diagnosis?#
1.! CXR.
2.! ECG.
3.! echocardiography.
Answer:.C.
Echocardiography..(TOF).
..
●... Pediatrics#cases#(rubella#infection#"fever#plus#rash#started#on#the#face#and#spread#into#the#
trunk#and#LL)#
##
●...... Milestone#of#a#child#at#hospital#laughing#and#cooing,#when#the#doctor#approaches#him,#he#felt#
afraid#and#was#looking#for#his#mother?#
Answer:.6.months.
..
●. . . . . . Pediatric#with#purulent#eye#discharge.#Culture#showed#gram#negative#diplococci.#What’s#the#
treatment?#
1.! IV.cephalosporin.
2.! Steroids.
3.! Topical.antibiotics.
Answer:.A.
DX:.Gonococcal.Conjunctivitis.
..
●... A#child#with#cutaneous#hemangioma,#those#hemangiomas#could#be#found#in#which#organ?#
1.! Liver.
2.! Spleen.
3.! Kidneys.
Answer:.A.
https://emedicine.medscape.com/article/1083849Koverview.
Spleen.could.be.involved,.but.most.commonly.the.liver.
..
●. . . . . . 3# year# old# baby# with# fever# and# neck# rigidity.# Culture# revealed# gram# –ve# diplococci# (They#
provided# a# picture# of# diffuse# meningococcemia# skin# rash)# and# asked# about# what# to# give# to# his#
Family?#
Answer:..Oral.rifampicin.
..
●... A#child#brought#to#ER#with#barking#cough,#red#epiglottis,#and#thumb#sign#on#x#ray.#What’s#
the#best#initial#management?#
1.! Examination.of.epiglottis.
2.! Endotracheal.intubation..
3.! Emergent.tracheostomy.
4.! Nasopharyngeal.Tube.
Answer:.B.
DX:.Acute.epiglottitis.
Illustrated:. The. child. should. be. intubated. under. controlled. conditions. with. a. general. anaesthetic..
Rarely,.this.is.impossible.and.urgent.tracheostomy.is.lifeKsaving..
..
●... A# child# with# diarrhea,# abdominal# bloating,# diagnosed# with# giardiasis.# What# is# the# most#
sensitive#test#to#diagnose#his#case?#
1.! 3.consecutive.stool.analysis.
2.! 3.separate.stool.analysis.
3.! Stool.immunoassay.
Answer:.C.
Because. Giardia. cysts. can. be. excreted. intermittently,. multiple. stool. collections. (i.e.,. three. stool.
specimens.collected.on.separate.days).increase.test.sensitivity.1..The.use.of.concentration.methods.
and. trichrome. staining. might. not. be. sufficient. to. identify. Giardia. because. variability. in. the.
concentration.of.organisms.in.the.stool.can.make.this.infection.difficult.to.diagnose..For.this.reason,.
fecal.immunoassays.that.are.more.sensitive.and.specific.should.be.used.
https://www.cdc.gov/parasites/giardia/diagnosis.html.
..
●... Child# from# Ghana# with# a# neck# mass.# Biopsy# showed# starry# sky# appearance,# KI# 67# was#
positive.#Which#genetic#change#is#true?#
Answer:.Over.expression.of.C.myc.gene.
DX:.Burkitt.lymphoma.
..
●... A#child#developed#fever#headache#and#malaise,#after#traveling#to#a#southern#area#on#a#trip#
with# his# friends,# his# mother# thought# that# it# is# normal# after# this# tired# trip,# his# condition# became#
worse# and# he# developed# skin# rash,# sore# throat,# lymphadenopathy,# and# hepatosplenomegaly# on#
examination.#What#test#do#you#want#to#order#to#reach#a#diagnosis?#
Answer:.Monospot.test.
(Infectious.Mononucleosis).
..
●...... A#child#who#has#episodes#of#vomiting#after#feeds,#although#his#appetite#is#good#and#wants#to#
eat#despite#the#vomiting.#His#labs#show#hypokalemia#and#low#chloride.#Most#likely#diagnosis#is?#
Answer:.Hypertrophic.pyloric.stenosis.
The.classic.presentation.of.IHPS.is.the.3K.to.6KweekKold.baby.who.develops.immediate.postprandial,.
nonKbilious,. often. projectile. vomiting. and. demands. to. be. reKfed. soon. afterwards. (a. "hungry.
vomiter")..
http://cursoenarm.net/UPTODATE/contents/mobipreview.htm?21/52/22336.
..
●... A# child# with# different# types# of# infections# including# aspergillosis,# Tb,# and# staph# ...# etc.#
Immune#defect#in?#
Answer:.NADPH.oxidase.
(chronic.granulomatous.disease).
Pathogens. such. as. Aspergillus. spp.,. Nocardia. spp.,. Serratia. marcescens,. and. Burkholderia.
cepacia,.or.recurrent.infections.with.Staphylococcus.aureus,.may.suggest.CGD.
http://bestpractice.bmj.com/bestKpractice/monograph/703/diagnosis/historyKandKexamination.html.
..
●... A# child,# known# case# of# thalassemia,# had# fever# after# blood# transfusion.# What# is# the# best#
measure#to#take#for#prevention#of#fever?#
1.! Antipyretic.
2.! PreKtransfusion.medications.
3.! Leukocyte.depleted.(prior.storage).
4.! Leukocyte.filters.at.transfusion.session..
Answer:.C.
When. red. cells. and. plasma. are. separated. from. whole. blood,. small. amounts. of. residual.
plasma. and/or. leukocyte. debris. may. remain. in. the. red. cell. concentrate.. During. blood.
storage,. these. leukocytes. release. cytokines. that. when. transfused. can. cause. transient.
fevers,.chills,.and.malaise.(without.hemolysis).within.1K6.hours.of.transfusion..This.state.is.
called.febrile.nonKhemolytic.transfusion.reaction.(FNHTR)..
Although.prophylactic.paracetamol.is.often.administered.to.prevent.FNHTRs,.there.is.little.
evidence.in.the.literature.to.support.this.practice..
Cytokine.accumulation.during.storage.of.cellular.components.(especially.in.platelet.units).is.
thought. to. be. the. most. common. event. leading. to. symptoms. of. FNHTRs.. Cytokines. are.
released.by.white.cells.and.preKstorage.leucodepletion.has.reduced.this.risk..
..
●... Cellulitis# in# the# face# of# infants# between# 6# and# 24# months# with# purple# discoloration# is#
caused#by?#
1.! GAS.
2.! Staph.aureus.
3.! Hemophilus.influenzae.
4.! Strep.pneumonia.
Answer:.C.
Haemophilus. influenzae. cellulitis. is. a. cutaneous. condition. characterized. by. a. distinctive. bluish. or.
purplishKred.cellulitis.of.the.face..
Hib.cellulitis.usually.involves.the.face,.head,.or.neck..
Most.cases.occur.in.children.aged.2.years.or.younger..
..
●... Treatment#of#GERD#in#a#child?#
Answer:.Give.antacid.
Pharmacological.therapy.used.in.GERD.include:.
KHistamine.blocker.
KPPI.
KAntacid.
https://www.uptodate.com/contents/acidKrefluxKgastroesophagealKrefluxKdiseaseKinKchildrenKandK
adolescentsKbeyondKtheKbasics.
..
●... Child#with#DM1#got#multiple#hypoglycemic#attacks?#
Answer:.Due.to.honeymoon.period.
http://bestpractice.bmj.com/bestKpractice/monograph/25/treatment/stepKbyKstep.html.
Honeymoon.period.is.one.of.the.causes.of.hypoglycemia.in.DM1.
..
●... Milestone#of#a#child#who#can#draw#a#line?#
Answer:.18.month.(other.options.were.all.younger.than.18.months).
..
●... Intussusception#age#group?#
Intussusception. is. the. most. common. cause. of. intestinal. obstruction. in. infants. between. 6. and. 36.
months.of.age..Approximately.60.percent.of.children.are.younger.than.one.year.old,.and.80.to.90.
percent. are. younger. than. two. years. [3].. Intussusception. is. less. common. before. three. months. and.
after.six.years.of.age..When.it.does.occur.in.these.younger.or.older.age.groups.it.is.more.likely.to.be.
associated.with.a.lead.point..
https://www.uptodate.com/contents/intussusceptionKinKchildren.
##
●... Which#vaccines#are#given#in#a#9#month#old#baby?#
1.! Measles.&.meningococcal.vaccine.
2.! MMR.
Answer:.A.
.

.
..
..
●... Nevus#crepitus#something#like#this#(child#with#scalp#swelling#with#hair#loss#above#it).#
##
●... Vaccine#that#is#contraindicated#in#eczema,#psoriasis#and#contact#dermatitis?#
1.! Rubella.(German.measles).
2.! Measels.
3.! Polio.
4.! Smallpox.
Answer:.D.
Eczema.or.atopic.dermatitis.(in.the.past,.even.if.not.currently.active)..
Patients.with.these.diseases.or.a.history.of.these.diseases.should.not.
be.vaccinated..

Acute,. chronic,. or. exfoliative. skin. conditions,. including. burns,.


impetigo,. chickenpox,. contact. dermatitis,. shingles,. herpes,. severe.
acne,.Darier's.disease.(keratosis.follicularis),.and.psoriasis..Until.these.
conditions.clear,.patients.should.not.be.vaccinated..

http://www.aafp.org/afp/2003/0901/p889.html.
..
●... Pediatric#patient#with#enuresis.##Which#method#is#affective#for#long#time?#
1.! Voiding.before.sleep.
2.! Decrease.fluid.intake.
3.! Alarm.
Answer:.C.
Enuresis.alarms.are.the.most.effective.longKterm.therapy.and.have.few.adverse.effects..
https://www.uptodate.com/contents/nocturnalKenuresisKinKchildrenKmanagement.
..
●... What#a#3#year#old#can#do?#
1.! Climb.stair.
2.! Know.the.names.of.the.days.
3.! Trace.a.triangle.
4.! Catch.a.ball.with.alternative.feet.
Answer:.A.
..
●... 3#year#old#baby#have#a#watery#discharge#from#his#eyes#since#birth#and#redness#what#is#the#
cause?#
1.! Bacterial.conjunctivitis.
2.! Lacrimal.duct.obstruction..
Answer:.B.
http://cursoenarm.net/UPTODATE/contents/mobipreview.htm?35/16/36110.
..
●... Still#birth#what#is#the#right#sentence#(thalassemia#case)?#
1.! Two.normal.beta.and.4.abnormal.alpha.
2.! Two.abnormal.beta.and.4.normal.alpha.
3.! 4.normal.beta.and.2.abnormal.alpha.
4.! 4.abnormal.beta.and.2.normal.alpha.
Answer:.A.
..
●... A#mother#brought#her#7#year#old#girl#to#the#clinic#because#she#has#pubic#hair#growth.#Her#
height# and# weight# are# normal# for# her# age.# On# examination,# no# breast# enlargement# or# other#
secondary#sexual#characteristics.#Clitoris#size#is#normal.#What#do#you#tell#her#mother?#
1.! Early.puberty.
2.! Normal.
3.! Turner.syndrome.
Answer:.A.
https://emedicine.medscape.com/article/924002Kclinical.
..
●... One#year#old#baby#was#breast#feed#till#6#months#of#age#and#was#developing#normally,#then#
the#mother#started#to#give#him#fruit#juices#and#he#started#to#have#developmental#delays.#Which#of#
the#following#substance#he#should#avoid?#
1.! Galactose.
2.! Fructose.
3.! Lactose.
Answer:.B.
Fructose. intolerance;. Affected. individuals. are. completely. asymptomatic. until. they. ingest. fructose..
Thus,.homozygous.neonates.remain.clinically.well.until.confronted.with.dietary.sources.of.fructose..
http://reference.medscape.com/article/944548Koverview#a5.
................................................................... .
●... Doctor#asked#a#child#to#bend#forward#with#the#feet#together,#arms#hanging#and#the#knees#
in#extension#and#the#doctor#is#inspecting#the#patient#from#the#back.#What#is#the#test?#
1.! Adam's.test.
2.! Sexual.abuse.
Answer:.A.
The.patient.takes.off.his/her.tKshirt.so.that.the.spine.is.visible..The.patient.needs.to.bend.forward,.starting.at.
the.waist.until.the.back.comes.in.the.horizontal.plane,.with.the.feet.together,.arms.hanging.and.the.knees.in.
extension..The.palms.are.hold.together..The.examiner.stands.at.the.back.of.the.patient.and.looks.along.the.
horizontal. plane. of. the. spine,. searching. for. abnormalities. of. the. spinal. curve,. like. increased. or. decreased.
lordosis/.kyphosis,.and.an.asymmetry.of.the.trunk..
..
●... Baby#with#only#mild#jaundice#and#everything#else#is#normal.#Indirect#bilirubin#was#elevated#
about#28#maybe#(but#in#the#twenties).#What’s#the#management?#
1.! Observation.
2.! Phototherapy.
Question.lack.some.information,.but.most.probably.it’s.PHOTOTHERAPY.
The.initiation.of.phototherapy.based.on.total.serum.bilirubin.levels,.gestational.age,.age.of.the.infant.in.hours,.
and.individual.risk.factors.A.commonly.used.rule.of.thumb.in.the.NICU.is.to.start.phototherapy.when.the.total.
serum.bilirubin.level.is.greater.than.5.times.the.birth.weight..Thus,.in.a.1Kkg.infant,.phototherapy.is.started.at.
a.bilirubin.level.of.5.mg/dL;.in.a.2Kkg.infant,.phototherapy.is.started.at.a.bilirubin.level.of.10mg/dL.and.so.on..
https://emedicine.medscape.com/article/1894477Koverview#a2.
..
●... Dehydrated#child#(I#think#moderate)#with#severe#vomiting.#What’s#the#management#?#
1.! Pedialyte.10.ml.over.5.min.
2.! Normal.saline.50ml/kg.
3.! ORS.with.ratio.of.1:1.glucose.to.sodium.
Answer:.B.
..
●... 3#years#old#baby#with#dipper#rash#with#no#satellite#lesion.#What’s#the#treatment?#
1.! Barrier.cream.with.frequent.change.of.dipper.
2.! Topical.antibiotics.
3.! Oral.antibiotics.
4.! Topical.steroid.
Answer:.A.
Good.nappyKchanging.hygiene.practices.should.be.ensured..No.matter.which.type.of.nappy.is.used,.it.should.
be.changed.every.2.hours..
If.the.child.is.prone.to.frequent.nappy.rash,.empirically.apply.a.topical.barrier.containing.zinc.oxide,.white.soft.
paraffin,.glycerin,.lanolin,.sucralfate,.or.mineral.oil.at.each.nappy.change..
http://bestpractice.bmj.com/bestKpractice/monograph/676/treatment/details.html.
..
●... Baby#can#sit#without#support#and#turns#his#head#to#the#doctor#and#smiles.#How#old#is#he?#
●... 6.months.
●......9.months.
Answer:.B.
..
●... Tumor# in# the# pediatric# age# group# that# arises# from# kidney# or# adrenals# and# displace# the#
collecting#system?#
●......Wilm’s.
●......Neuroblastoma.
Answer:.B.
Abdominal. neuroblastomas. usually. develop. in. the. retroperitoneum.. Most. arise. from. the. adrenal. gland. and.
displace. the. kidney. inferomedially.. In. rare. cases,. a. neuroblastoma. may. mimic. a. Wilms. tumor,. arising. from.
tissues. in. the. kidney. or. invading. the. kidney.. To. make. diagnosis. complicated,. rare. neuroblastomas. possess.
other.features.more.typical.of.Wilms.tumor.than.of.intrarenal.neuroblastomas..
USMLE[13:.A,.but.I’m.not.sure.of.the.answer.(I.choose.it.because.it.says.displace.displace.the.collecting.system.
and.displacement.goes.with.WilmsK.read.the.explanation.bellow).I.also.asked.a.oncologist.he.said.he.should.go.
with.B.BUT.the.question.is.not.complete.and.he.needs.more.information..
............................................. .
●... A#mother#came#with#her#4#month#old#child#to#vaccinate#him#but#she#reported#that#her#child#
is#having#severe#diarrhea#for#3#days.#What#are#you#going#to#do?#
1.! Refer.all.vaccination.
2.! Give.hepatitis.B.vaccine.
3.! Give.vaccination.without.DTP.
..................... ......... ......... ................................................................................. .
K. . . . . . . . There. is. no. evidence. that. acute. illness. reduces. vaccine. e. cacy. or. increases. vaccine. adverse. events.1,2.
However,.as.a.precaution.with.moderate.or.severe.acute.illness,.all.vaccines.should.be.delayed.until.the.illness.
has. improved.. Mild. illnesses. (such. as. otitis. media,. upper. respiratory. infections,. and. diarrhea). are. NOT.
contraindications.to.vaccination..Do.not.withhold.vaccination.if.a.person.is.taking.antibiotics..
..
K. . . . . . . . A."moderate.or.severe.acute.illness".is.a.precaution.for.administering.any.vaccine..A.mild.acute.illness.
(e.g.,.diarrhea.or.mild.upperKrespiratory.tract.infection).with.or.without.fever.is.not.a.precaution,.and.vaccines.
may.be.given..The.concern.in.vaccinating.someone.with.moderate.or.severe.illness.is.that.a.fever.following.the.
vaccine.could.complicate.management.of.the.concurrent.illness.K.it.could.be.difficult.to.determine.if.the.fever.
was. from. the. vaccine. or. due. to. the. concurrent. illness.. In. deciding. whether. to. vaccinate. a. patient. with.
moderate.or.severe.illness,.the.clinician.needs.to.determine.if.deferring.vaccination.will.increase.the.patient's.
risk.of.vaccineKpreventable.diseases,.as.is.the.case.if.the.patient.is.unlikely.to.return.for.vaccination.or.to.seek.
vaccination.elsewhere..
..
..
●... Case#of#cystic#fibrosis#(+ve#chloride#sweat#test).#They#asked#about#the#gene:#
1.! Chromosome.7.
2.! Chromosome.15.
3.! Chromosome.16.
Answer:.A.
..
●... Child#with#epiglottitis,#x[ray#shows#a#swollen#epiglottis.#What#is#the#first#thing#you’re#going#
to#do?#
1.! Cricothyroidotomy.
2.! Endotracheal.intubation.
3.! Nasopharyngeal.tube.
Answer:.B.
Note:#I#had#the#same#scenario#but#with#different#choices,#they#were:#
1.! Admission.&.refer.to.ENT.
2.! Abx.
3.! Steroids.
Answer:.B.
Appropriate.antibiotics.include.ceftriaxone,.cefotaxime,.and.cefuroxime..
Corticosteroid. administration,. although. advocated. in. the. past. based. on. anecdotal. reports,. remains.
controversial..These.agents.have.no.proven.efficacy.for.treating.epiglottitis..
..
●... 13# year# old# boy# brought# to# the# clinic# by# his# parents# for# delayed# puberty.# He# has# sparse#
pubic#hair,#has#always#been#in#the#25th#percentile.#He’s#healthy.#What#are#you#going#to#do?#
1.! Watchful.waiting.
2.! Give.him.testosterone.
Answer:.A.
In. most. patients,. however,. the. distinction. between. congenital. GnRH. deficiency. and. constitutional. delay. of.
puberty. remains. uncertain,. and. can. be. resolved. only. with. serial. observations.. In. view. of. these. diagnostic.
difficulties,.the.initial.therapeutic.approach.is.similar.for.both.disorders.[14,15]..The.two.major.options.are:.

●......"Watchful.waiting".with.reassurance.and.psychological.support.for.the.patient.and.family.

●......Administration.of.gonadal.steroids..

ShortKterm.hormonal.therapy.with.testosterone.in.boys.and.with.estrogen.in.girls.may.be.appropriate.when.
the.pubertal.delay.is.severe.or.the.patient's.psychosocial.concerns.about.the.delay.play.a.prominent.role.that.
cannot.be.addressed.by.reassurance.and.education.alone..

Except.under.unusual.circumstances,.therapy.should.be.restricted.to#boys#older#than#14#years.and.girls.older.
than.12.years.who.show.few.or.no.signs.of.puberty.and.are.expressing.considerable.anxiety.about.their.delay..
The.shortKterm.use.of.exogenous.testosterone.in.boys.or.estrogen.in.girls.does.not.appear.to.have.any.longK
term.sequelae.except.for.the.potential.of.skeletal.maturation.that.might.result.in.some.loss.of.adult.height..

http://cursoenarm.net/UPTODATE/contents/mobipreview.htm?40/20/41295.
..
..
●......Patient#with#barking#cough.#What#is#the#causative#organism?#
Answer:.Parainfluenza.virus.
..
●... A# hypotensive# baby# with# severe# vomiting# and# watery# diarrhea.# What# is# the# electrolyte#
imbalance?#
1.! Low.na.
2.! High.na.
3.! High.k.
4.! Hyperglycemia.
Answer:.A.
It.could.be.either.hyper.or.hypo.Na.
Since.there.is.hypotension,.I.think.the.answer.is.Low.NA.
●... Blood# in# diaper# with# vomiting# and# constipation.# US# shows# a# donut# sign.# What# is# the#
diagnosis?#
Answer:.Intussusception.
..
●... 3#year#old#child#presented#with#burn#involving#her#whole#right#arm,#with#blisters.#What#is#
the#stage#and#percentage?#
##
##

# #
# #
##
##
●... A#child#diagnosed#with#UTI.#What#to#do#before#the#treatment?#
1.! US.
2.! Cystoscopy..
Answer:.A.
Indications.for.renal.and.bladder.ultrasonography.are.as.follows:.
●... Febrile.UTI.in.infants.aged.2K24.months.
●... Delayed.or.unsatisfactory.response.to.treatment.of.a.first.febrile.UTI.
●... An.abdominal.mass.or.abnormal.voiding.(dribbling.of.urine).
●... Recurrence.of.febrile.UTI.after.a.satisfactory.response.to.treatment.
Cystourethroscopy.is.contraindicated.in.febrile.patients.with.urinary.tract.infections.(UTIs).and.those.
with.severe.coagulopathy..
https://emedicine.medscape.com/article/1829911Koverview#a5.
..
●... Child#with#malignancy#after#bone#marrow#transplant,#by#3#weeks,#his#father#came#because#
his# other# healthy# son# got# chicken# box.# What# would# you# give# the# child# with# the# malignancy# as#
prophylaxis?#
1.! Varicella.vaccine.
2.! Varicella.Ig.
3.! Both.A&B.
4.! Give.acyclovir..
Answer:.B.
The. most. important. use. of. VZIG. is. for. passive. immunization. of. susceptible,. immunocompromised.
children. after. significant. exposure. to. chickenpox. or. zoster.. This. includes. children. with. primary.
immunodeficiency. disorders. and. neoplastic. diseases. and. children. currently. receiving.
immunosuppressive.treatment..
https://www.cdc.gov/Mmwr/Preview/Mmwrhtml/00022690.htm.
..
●... 5#year#old#child#with#hand#and#foot#pain.#What#is#the#diagnosis?#
Answer:.Sickle.cell.anemia.
..
●... What#causes#cyanotic#heart#disease?#
1.! Tetralogy.of.fallot.
2.! Patent.ductus.arteriosus.
3.! VSD.
Answer:.A.
..
●... Missed#vaccination#(study#the#symptom#of#the#disease#Diphtheria,#pertussis,#mumps,#and#
measles).#
##
●... What# is. the# treatment# of# post[streptococcus# glomerulonephritis# in# children# with# edema#
and#HTN?#
1.! High.dose.of.antibiotics.
2.! Diuretic.for.edema.
3.! Diuretic.for.HTN.
4.! High.dose.of.steroids.
Answer:.B.
https://emedicine.medscape.com/article/980685Kmedication#1.
Administer. antibiotics. (penicillin. or. erythromycin). for. 10. days. to. ensure. eradication. of. the.
streptococcus.if.the.disease.is.believed.to.be.acute.poststreptococcal.glomerulonephritis.and.if.risk.
of.contamination.is.present.
Furosemide. is. a. loop. diuretic. that. is. useful. in. patients. with. acute. glomerulonephritis. who. are.
edematous..This.agent.also.has.some.BPKlowering.effect.by.increasing.excretion.of.salt.and.water.via.
interfering. with. the. chlorideKbinding. cotransport. system. in. the. ascending. loop. of. Henle.. In. acute.
hypertensive.states,.administer.furosemide.intravenously.(IV)..
..
http://cursoenarm.net/UPTODATE/contents/mobipreview.htm?17/10/17568.
Loop.diuretics.generally.provide.a.prompt.diuresis.with.reduction.of.blood.pressure.and.edema..In.
our.practice,.intravenous.furosemide.is.given.at.an.initial.dose.of.1.mg/kg.(maximum.40.mg)..
Patients. with. evidence. of. persistent. group. A. streptococcal. infection. should. be. given. a. course. of.
antibiotic.therapy..
..
●... What#is#the#most#common#cause#of#nephrotic#syndrome#in#children?#
A.! ●... Membranous.G.
B.! ●......IgA.nephropathy.
C.! ●......Minimal.change.disease.
D.! ●......Mesangioproliferative.GN.
Answer:.C.
..
●... NICU#child#suddenly#developed#distress#with#absent#breath#sounds.#Where#will#you#place#
the#butterfly#needle?#
●... 2nd.intercostal.space.
●... 3rd.intercostal.space.
●... 4th.intercostal.space.
●... 5th.intercostal.space.

Insert.needle.into.the.pleural.space.(directly.over.the.top.of.the.rib.in.the.second#or#third.intercostal.
space.in.the.midclavicular.line).until.air.is.aspirated.into.the.syringe..Expel.air.through.the.threeKway.
stopcock..
http://www.asph.mobi/Guidelines_Neonatal/Pneumothorax.pdf.
https://www2.health.vic.gov.au/hospitalsKandKhealthKservices/patientKcare/perinatalK
reproductive/neonatalKehandbook/procedures/pneumothoraxKdrainage.
..
●... 6#month#old#infant#brought#by#his#parents#with#history#of#repeated#vomiting;#his#pulse#was#
(190).# He# had# dry# mucous# membranes# and# sunken# anterior# fontanel.# What# is# the# appropriate#
volume#of#fluid#that#should#be#given#initially?#
1.! Bolus.10.ml/kg.of.body.weight.
2.! Bolus.20.ml/kg.of.body.weight.
3.! Slow.infusion.10.ml/kg.of.body.weight.
4.! Slow.infusion.20.ml/kg.of.body.weight.
Answer:.B.
https://emedicine.medscape.com/article/801012Ktreatment#d11.
..
●... Child# unvaccinated# developed# fever,# SOB# and# stridor.# Lateral# X[ray# shows# a# thumb# sign.#
What’s#the#treatment?#
1.! Erythromycin.
2.! Vancomycin.
3.! Ceftriaxone.
4.! Metoconazole.
Answer:.C.
https://emedicine.medscape.com/article/763612Kmedication#2.
..
●... 10# year# old# child# brought# by# his# parents# because# they# were# concerned# about# his# weight.#
He#eats#a#lot#of#fast#food#and#french#fries.#Your#main#concern#in#managing#this#patient#is?#
A... His.parents.concern.about.his.weight.
B... His.BMI.>.33.
C......Family.history.of.heart.disease..
D.....Eating.habits.(fast.food.and.french.fries).
Answer:.D.
..
●... A#4#year#old#child#that#doesn’t#speak#well#except#for#saying#mama#and#papa.#He#responds#
well#to#verbal#commands#and#his#older#brother#had#the#same#problem#when#he#was#in#the#same#
age.#He#is#also#shy.#What#is#the#diagnosis?#
A......Autism.
B......Development.language.disorder..
Answer:.B.
..
●... Parents# brought# their# son# with# cystic# fibrosis,# asking# about# the# chance# of# his# sister# of#
becoming#a#carrier:#
A... 1:2.
B... 2:4.
C... 1:25.
Answer:.B.
https://www.cff.org/WhatKisKCF/Genetics/CFKGeneticsKBasics/.
https://rarediseases.info.nih.gov/diseases/6233/cysticKfibrosis.
.
When.two.carriers.of.an.autosomal.recessive.condition.have.children,.each.child.has.a:.

K........25%.(1.in.4).chance.to.have.CF.
K........50%.(1.in.2).chance.to.be.a.carrier.of.CF.like.each.parent.
K........25%.chance.to.not.have.CF.and.not.be.a.carrier.CF.
When.a.carrier.of.CF.has.a.child.with.a.person.with.CF,.each.child.has.a:.
K........50%.(1.in.2).chance.to.have.CF.
K........50%.(1.in.2).chance.to.be.a.carrier.of.CF.
..
●... What#age#in#months#can#a#child#say#a#few#words?#
A... 6.
B... 9.
C......12.
D.....24.
Answer:.D.
..
●... A#mother#presented#to#the#clinic#with#her#5#month#old#baby#worried#that#he#might#have#
developmental#delay.#Which#of#the#following#milestones#is#appropriate#for#his#age?#
A......Pincer.grip.
B......Reach.objects.
C......Wave.goodbye.
D.....Sit.
Answer:.B.
..
●... A#baby#who#can#run#and#laugh#when#he#sees#the#doctor.#What’s#his#age?#
A... 6.months.
B... 9.months.
C... 12.months.
D.....24.months.
Answer:.D.
..
●... 4#month#old#girl#that#refuses#feeding.#She#becomes#cyanotic#and#cries#whenever#she#feeds.#
On#auscultation:#harsh#ejection#systolic#murmur.#What#is#the#diagnosis?#
A......Aortic.stenosis.
B......Ventricular.defect.
Answer:.A.
Aortic.stenosis:.Usually.asymptomatic;.symptoms.may.include.dyspnea,.easy.fatigue,.chest.pain,.or.
syncope;.newborns.and.infants.may.present.with.CHF.
Systolic. ejection. murmur. (grade. 2. to. 5). best. heard. at. upper. right. sternal. border. with. radiation. to.
carotid.arteries;.left.ventricular.heave;.thrill.at.ULSB.or.suprasternal.notch.
http://www.aafp.org/afp/2011/1001/p793.html.
..
●... A# child# presented# with# a# barking# cough.# What# is# the# most# likely# finding# in# the# physical#
examination?#
A......Stridor.
B......Wheeze.
Answer:.A.
http://www.aafp.org/afp/2011/0501/p1067.html.
..
●......A#scenario#of#a#child#with#frontal#bossing,#bowed#legs#and#a#chest#x[ray#showing#rachitic#rosary.#
Which#of#the#following#is#most#likely#to#be#deficient#in#this#child?#
Answer:.Vitamin.D.
..
●... A#4#year#old#girl#presenting#with#generalized#malaise#and#bony#aches#especially#in#the#hand#
and#feet.#Her#labs#show#high#ANA#and#the#ESR#is#high.#What#is#the#cause?#
A... Latent.lupus.
B... Growing.pain.
No.enough.information..
..
●... Pediatric# case# scenario# about# respiratory# distress# syndrome.# Which# of# the# following# is#
deficient?#
1.! Sphingomyelin.
2.! Dipalmitoylphosphatidylcholine.
Answer:.B.
Surfactant. is. a. complex. lipoprotein. (see. the. image. below). composed. of. 6. phospholipids. and. 4.
apoproteins.. Surfactant. recovered. by. alveolar. wash. from. most. mammals. contains. 70K80%.
phospholipids,. 8K10%. protein,. and. 10%. neutral. lipids,. primarily. cholesterol.. Dipalmitoyl.
phosphatidylcholine. (DPPC),. or. lecithin,. is. functionally. the. principle. phospholipid..
Phosphatidylglycerol.makes.up.4K15%.of.the.phospholipids;.although.it.is.a.marker.for.lung.maturity,.
it.is.not.necessary.for.normal.lung.function..
https://emedicine.medscape.com/article/976034Koverview.
..
..
●... Pediatric#case#of#a#baby#who#vomits#after#feeds#but#has#a#good#appetite#and#wants#to#eat#
despite#vomiting.#During#examination#you#found#an#epigastric#swelling.#What’s#the#next#step?#
1.! Abdominal.US.
2.! XKray.
3.! Barium.enema.
Answer:.A.
..
●... A# case# scenario# about# a# pediatric# patient# with# abdominal# pain# and# vomiting# (case# of#
intussusception).#What#is#the#best#initial#treatment?#
1.! IV.fluid.and.bowel.rest.
2.! Surgery.
3.! Barium.enema.
4.! CT.abdomen.
Answer:.A.
..
●... A# pediatric# case.# Asymptomatic# patient# with# a# picture# of# an# ECG# (I# just# noticed# irregular#
rhythm,# bradycardia# and# PR# interval# was# unequal# some# lead# was# 3# little# squares# and# another# 5#
little# squares).# I# think# it# was# a# case# of# 1st# degree# heart# block.# Then# they# asked# about# the#
consequences?#
1.! Syncope.
2.! Needs.pacemaker.
Answer:.A.
No.enough.information..
FirstKdegree.AV.block.in.and.of.itself.does.not.produce.any.symptoms,.nor.need.pacemaker..
It.could.be.2nd.degree,.Mobitz.I.heart.block.which.is.explained.by.irregular.rhythm.and.unequal.PR.
interval..
I.don’t.know.if.they.mean.temporary.or.permanent.pacemaker..
I.choose.syncope.because.it.can.happen.and.Permanent.pacing.is.rarely.required..
https://emedicine.medscape.com/article/161919Koverview.
..
●... Child#with#rapid#blinking#of#the#eyes.#Can#communicate#with#his#parents#during#it.#What#is#
the#diagnosis?#
A).TIC.
B).Petit.mal.seizure.
C).Tourette.syndrome..
Answer:.A.
http://www.schoolbehavior.com/disorders/tourettesKsyndrome/ticsKandKtourettesKsyndromeK
overview/.
..
.
....#
Part#2#
1). What#can#a#3#year#old#do?.
A)! Climb.stairs.
B)! Know.the.names.of.the.days..
C)! Trace.a.triangle..
D)! Catch.a.ball.with.alternative.feet..
.
Answer.is.A.
.
2). A# 3# year# old# baby# has# watery# discharge# from# his# eyes# since# birth# associated# with#
redness.#What#is#the#cause?.
A)! Bacterial.conjunctivitis..
B)! Lacrimal.duct.obstruction..
.
Answer.is.B.
.
3). A# Researcher# wants# to# measure# obesity# in# children.# He# took# their# BMI.# What# else# he#
should#he#take?.
A)! Girth.measurement...
B)! HDL/LDL.level..
C)! Dietary.habits..
D)! Skin.fold.thickness.
.
Answer.is.D.
.
4). An#18#month#old#child#presented#with#leg#bowing#and#inability#to#walk.#His#labs#show#
high#ca,#low#phosphate,#and#high#ALP.#What’s#the#diagnosis?.
A)! Vitamin.D.deficiency.rickets.
B)! Hypophosphatemia.
C)! Familial.hypophosphatemia..
.
Answer.is.D.
.
Source.is:.
.https://emedicine.medscape.com/article/922305Koverview.
.
familial. hypophosphatemia. also. known. by. XKlinked. hypophosphatemia,. or. vitamin. D.̶. resistant.
rickets.. In. XLH. patients,. the. severe. hypophosphatemia. (<. 2.5. mg/dL). is. associated. with. elevated.
serum. alkaline. phosphatase.. However,. despite. the. severe. hypophosphatemia,. the. serum. calcium,.
PTH,.and.25Khydroxyvitamin.D.concentrations.are.normal..with.high.alkaline.phosphatase..
.
.
#
5). Scenario#of#a#child#with#cafe#au#lait#spots#and#short#stature.#What’s#the#diagnosis?#.
A)! Fanconi.anemia.
.
Answer.is.A..

6). A# 14# month# old# infant# presented# with# n/v.# Upon# examination,# he# was# found# to# have#
depressed# anterior# fontanelle,# decreased# skin# turgor# and# sunken# eyes.# What# is# the#
degree#of#dehydration?.
A)! 5.
B)! 10.
C)! 15.
D)! 20.
.
Answer.is.C.

.
.
7). A#mother#complains#that#her#child#who#is#6#months#old#is#crying#too#much#for#the#past#2#
weeks.# The# mother# said# that# she# was# regularly# feeding# her# child# then# recently#
introduced#cow#milk.#Upon#examination,#there#was#abdominal#distention.#What#is#the#
diagnosis?.
A)! Cow.milk.allergy..
B)! Infantile.colic.
.
Answer.is.A.
.
8). A# child# complains# of# barking# cough# with# inspiratory# stridor.# What’s# the# best# way# to#
diagnose#him?.
A)! Clinically.
B)! XKray.
C)! Laryngeal.swab.
.
Answer.is.A.
.
Source:.emedicine.
https://emedicine.medscape.com/article/962972Kworkup.
Most.importantly,.croup.is.a.clinical.diagnosis..Radiographs.can.be.used.as.a.tool.to.help.confirm.this.
diagnosis,.but.are.not.required.in.uncomplicated.cases.. [20]. .The.anteroposterior.(AP).radiograph.of.
the.soft.tissues.of.the.neck.classically.reveals.a.steeple.sign.(also.known.as.a.pencilKpoint.sign),.which.
signifies. subglottic. narrowing,. whereas. the. lateral. neck. view. may. reveal. a. distended. hypopharynx.
(ballooning).during.inspiration.(see.the.images.below)..[21]. However,.these.xKray.findings.may.not.be.
seen.in.up.to.50%.of.children.with.clinical.symptoms.of.croup..

9). A# scenario# of# a# patient# with# wide# spaced# nipples# and# a# short# neck.# What’s# her#
diagnosis?#.
A)! Turner.syndrome.
B)! Down.syndrome.
.
Answer.is.A.

.
10).Scenario#of#a#female#patient#that#developed#secondary#sexual#characteristics#but#has#no#
vagina.#What#is#the#diagnosis?#.
A)! Mullerian.agenesis.
B)! Ovarian.atrophy.
.
Answer.is.A.

11).A# mother# is# complaining# that# her# baby# has# a# rash# in# the# diaper# area,# that# rash# is#
associated#with#satellite#lesions#around#it.#What#is#the#treatment?.
A)! Metronidazole.
B)! .
Source:.emedicine.
https://emedicine.medscape.com/article/801222Ktreatment.
.
This. is. yeast. diaper. infection,. due. to. the. presence. of. satellite. lesions.. treatment. is. with. topical.
antifungal. medication.. If. candidal. infection. is. suspected,. topical. ointments. or. creams,. such. as.
nystatin,. clotrimazole,. miconazole,. or. ketoconazole. can. be. applied. to. the. rash. with. every. diaper.
change.
.
12).Pediatric#patient#presented#with#a#centripetal#rash#distribution,#more#on#the#trunk#and#
gradually#less#towards#extremities.#What#is#the#incubation#period#of#this#disease?#.
A)! 0K10.
B)! 11K20.
C)! 21K30.
.
If.the.rash.is.vesicle.and.pustules.this.varicella,.incubation.period.is.10K21.days.
Source:.https://www.webmd.boots.com/children/guide/chickenpoxKincubationKperiod.
.
The.incubation.period.is.seven.to.21.(usually.10.to.21).days.after.exposure.to.the.herpes.varicellaK
zoster. virus. to. the. development. of. the. symptoms.. The. disease. is. most. contagious. a. day. or. two.
before.the.rash.appears.and.until.the.rash.is.completely.dry.and.scabbed.over,.about.five.to.six.days.
after.onset.of.the.rash..
.
13).Pediatric#patient#being#treated#for#H#pylori.#He#is#on#metronidazole.#What#to#add?.
A)! Tetracycline.
B)! Clarithromycin.
.
Answer.is.B.
Source:.http://gi.org/guideline/treatmentKofKhelicobacterKpyloriKinfection/.
.
Clarithromycin.triple.therapy.consisting.of.a.PPI,.clarithromycin,.and.amoxicillin.or.metronidazole.for.14.days.
remains.a.recommended.treatment.in.regions.where.H.+pylori.clarithromycin.resistance.is.known.to.be.<15%.
and.in.patients.with.no.previous.history.of.macrolide.exposure.for.any.reason..
.
14).A# Pediatric# patient# is# complaining# of:# Intermittent# dysphagia,# retrosternal# pain# and#
heartburn.#Barium#swallow#showed:#dilated#esophagus.#Synchronous#contractions#and#
gradual#distal#narrowing.#What#is#the#diagnosis?.
A)! Achalasia.
B)! Diffuse.esophageal.spasm.
C)! GERD.
.
Answer.is.A.
Source:.wikipedia.
.
There. is. acute. tapering. at. the. lower. esophageal. sphincter. and. narrowing. at. the. gastroKesophageal.
junction,.producing.a."bird's.beak".or."rat's.tail".appearance..The.esophagus.above.the.narrowing.is.often.
dilated.(enlarged).to.varying.degrees.as.the.esophagus.is.gradually.stretched.over.time..
.
Characteristic.manometric.findings.are:
●! Lower.esophageal.sphincter.(LES).fails.to.relax.upon.wet.swallow.(<75%.relaxation).
●! Pressure. of. LES. <26. mm. Hg. is. normal,>100. is. considered. achalasia,. >. 200. is. nutcracker.
achalasia..
●! Aperistalsis.in.esophageal.body.
●! Relative.increase.in.intraKesophageal.pressure.as.compared.with.intraKgastric.pressure.
.
15).A# pediatric# patient# complaining# of# daily# fever,# fatigue# and# knee# swelling# for# two#
months.# Also,# recently# developed# epistaxis.# Labs:# Low# platelet,# Low# hemoglobin,#
Normal#WBC.#What#is#your#next#step?.
A)! Bone.marrow.aspirate.
B)! ANA.
.
16).A#child#with#swelling.#Which#organ#is#mostly#affected#by#mumps?.
A)! Heart.
B)! Lung.
C)! Testes.
D)! Parotid.
.
Answer.is.D.
.
17).A# pediatric# patient# with# sore# throat,# migrating# arthralgia# and# fever.# What’s# the#
diagnosis?.
A)! Rheumatic.fever.
.
Answer.is.A.

.
18).A# 15[year[old# girl# brought# by# her# mother# because# she# did# not# get# her# period# yet.# On#
examination#she#has#breast#buds,#normal#pubic#hair#and#her#height#has#increased#during#
the#last#year.#Which#one#of#the#following#will#support#your#diagnosis?.
A)! Onset.of.menstruation.
B)! Bone.age.estimation..
.
Answer.A?.
Source:.https://www.girlology.com/pubertyKtimelineKgirls.

Puberty.timeline.in.girls.

●! Breast.buds.are.first.to.develop.for.girls..
●! Shortly. after. breasts. bud. (a. sign. of. circulating. estrogen),. she. will. begin. to. have. vaginal.
discharge.(also.estrogenKdriven)..
●! Pubic.hair.is.second.for.most.girls..
●! The.biggest,.most.rapid.growth.spurt.follows..
●! About. 6. months. after. she. has. grown. the. fastest. (sometimes. 2K3. inches. in. a. matter. of.
months),.she.will.start.her.period..
●! Once.the.period.begins,.most.of.her.growth.is.finished,.but.she.will.continue.to.grow.in.her.
trunk.and.there.may.still.be.some.lengthening.in.her.legs..
.
19).A# school# boy# with# an# itchy# scalp,# 10# other# classmates# are# affected.# What# is# the#
diagnosis?.
A)! Pediculosis.capitis.
.
Answer.is.A.
.
20).An# infant# with# bilateral# absent# red# reflex,# both# pupils# are# reactive.# What# is# the# most#
likely#diagnosis?.
A)! Congenital.cataract.
.
Answer.is.A.
.
21).Scenario#about#pediatric#herpetic#gingivostomatitis.#What#is#the#causative#organism?.
A)! HSV.1.
.
Answer.is.A.
.
22).An#18#month#old#baby#diagnosed#with#meningitis.#What#is#the#management?.
A)! Gentamicin.and.penicillin.
B)! Cefotaxime.and.vancomycin.
.
Answer.is.B.
.

.
.
23).Treatment#of#baby#with#iron#deficiency#anemia#(no#options).#.
A)! Oral.route.
.
Answer.is.A.
.
24).Child# with# repeated# polymicrobial# chest# infections.# Skin# test# was# positive# for# candida#
antigen#and#all#blood#test#are#normal#except:##high#IgG#or#IgM?#And#low#lymphocytes.#
Lymph# node# biopsy# showed:# rudimentary# germinal# centers.# What# is# the#
pathophysiology#of#this#disease?#No#options.#.
#
I.think.it’s.a.case.of.XKlinked.agammaglobulinemia.(XLA),.or.Bruton.agammaglobulinemia.
#
25).What#is#the#treatment#of#typhoid#fever#in#children?.
A)! Chloramphenicol.
.
Source:.emedicine.
https://emedicine.medscape.com/article/968672Ktreatment.
.
Enteric.fever.caused.by.S.+typhi.infection.
●! For.S.+typhi.infection,.initial.empiric.therapy.with.ceftriaxone.is.recommended.due.to.
widespread.resistance..If.susceptible,.chloramphenicol,.ampicillin,.or.TMPKSMZ.may.be.
used..Duration.of.therapy.should.be.14.days..
.
26).A#milestone#question:#A#child#who#knows#the#names#of#colors#and#can#ride#a#tricycle#but#
has#difficulty#in#drawing#a#square?#.
A)! 3.years.old.
.
Answer.is.A.
.
27).A# 4# year# old# child# with# decreased# in# head# growth,# has# weird# hand# movements#
(wringing),#lost#expressive#end#receptive#language#skills#and#lost#his#interest#in#his#social#
environment.#What#is#the#diagnosis?.
A)! Autism.
B)! Mental.retardation.
C)! Rett.syndrome.
.
Answer.is.C.
.
Source:.wikipedia.
https://en.wikipedia.org/wiki/Rett_syndrome.
.
Symptoms. include. small. hands. and. feet. and. a. deceleration. of. the. rate. of. head. growth. (including.
microcephaly).. Repetitive. stereotyped. hand. movements,. such. as. wringing. and/or. repeatedly. putting.
hands.into.the.mouth,.are.also.noted..People.with.Rett.syndrome.are.prone.to.gastrointestinal.disorders.
and.up.to.80%.have.seizures..
.
28).A# child# with# a# red# mass# growing# in# his# shoulder# since# birth.# What’s# the# most# likely#
diagnosis?.
A)! Strawberry.nevus..
.
Answer.is.A.
.
29).A# child# with# developmental# delay,# mild# low# calcium,# pigeon# chest,# bulging# of# frontal#
bone,#mild#high#alkaline#phosphates,#normal#phosphate.#What’s#the#diagnosis?.
A)! Rickets..
B)! Osteoporosis.
C)! Paget.disease.
.
Answer.is.A.
#
30).Anxious#mother#brings#her#2#month#old#baby#having#history#of#diarrhea#and#crying#too#
much#since#2#weeks.#He#was#born#on#the#36th#week,#breast#fed#but#the#mother#started#
bottled#milk#2#weeks#ago.#What#is#the#cause#of#his#abdominal#pain?.
A)! Increased.gases.
B)! Neuro.system.still.not.fully.developed.
C)! Decreased.bowel.peristalsis..
.
31).Child# with# scaly# erythematous# plaques# with# follicular# hyperkeratosis# over# elbows# and#
knees.#What#is#the#other#area#in#the#body#most#likely#to#be#affected?.
A)! Eye.
B)! Adrenal.
C)! Heart..
D)! Kidney.
.
Answer.is.A.
.
Source:..
Blepharitis. involves. chronic. inflammation. of. the. eyelids. and. is. one. of. the. more. common. eye.
problems. associated. with. psoriasis.. Another. common. ocular. finding. is. anterior. uveitis,. which. has.
been.reported.to.occur.in.7%.to.20%.of.patients.with.psoriasis..
..
32).A# 4# week# old# baby# with# his# head# tilted# to# one# side.# Normal# neuro# examination# and#
normal#labs.#What#is#the#diagnosis?#.
A)! Congenital.torticollis.
B)! Cretinism.
C)! Erb’s.palsy..
.
Answer.is.A.
.
33).Boy# (neither# BMI# or# weight# were# mentioned)# eats# fast# food# and# high# fat# diet# (French#
fries)# his# parents# are# concerned# about# his# cholesterol,# both# parents# have# cholesterol#
disease,#you#will#do#cholesterol#test#because:.
A)! BMI.above.30.
B)! Family.Hx.of.cardiovascular.disease.
C)! High.cholesterol.diet.
D)! Parents.concern.
.
34).Child# patient# came# with# scenario# of# chest# infection,# first# day# of# admission# he# was#
treated# with# cefotaxime.# On# the# next# day,# the# patient’s# state# became# bad# with#
decreased#perfusion#and#x[ray#shows#complete#right#sided#opacification#+#hydrothorax.#
What#is#the#causative#organism?.
A)! Strept.Pneumoniae..
B)! Staph..Aureus...
C)! Hemophilus.influenzae.type.b.
D)! Pseudomonas.
.
Answer.is.B.
asked.a.pediatric.consultant.
.
35).Long#scenario#of#a#pediatric#patient#with#the#following#labs:#K:#3,#Na:#124.##How#would#
you#correct#the#electrolyte#imbalance?#.
A)! Normal.saline.and.Kcl.5.over.20.cc/hr.
B)! Normal.saline.and.Kcl.40.over.80.cc/hr.
C)! 1/2.saline.and.Kcl.5.over.20cc/hr.
D)! 1/2.saline.and.Kcl.5.over.20cc/hr.
.
36).Long#scenario#of#a#boy#with#diarrhea#(gastroenteritis#and#dehydration#symptoms)#with#
no# urine# output# (acute# kidney# injury).# And# they# gave# you# labs# (k:# high,# urea:# high,#
sodium#bicarbonate:#low#and#high#creatinine).#What#is#your#immediate#action?.
A)! Dialysis.
B)! Normal.saline..
C)! NaHCO3.
D)! Antibiotics.
.
Answer.is.B.
.
37).A# child# presented# with# fatigue# and# lethargy.# Labs:# Hb# 10.5,# MCV# 40.# What's# your#
management?.
A)! Oral.iron.
B)! IM.iron.
C)! Blood.transfusion.
.
Answer.is.A.
.
38).Calculate#the#fluid#replacement#of#a#baby#who#weighs#15#Kgs.##>#1250#ml.
●! Frist.10.KG.multiply.100.
●! Second.10.KG.multiply.50.
●! So.10*100=1000.
●! 5*50=250.
●! Total:.1000+250=1250ml.
.
39).A# child# with# short# stature# and# no# family# history.# What’s# the# most# likely# underlying#
etiology?.
A)! Constitutional.growth.delay.
B)! Genetic..
C)! Hormonal.
.
Answer.is.A.
.
40).A#clear#scenario#about#croup#(brassy#cough#and#hoarseness).#What’s#the#treatment?.
A)! Racemic.epinephrine.and.oral.steroid.
Source:..http://www.aafp.org/afp/2011/0501/p1067.html.
.
Treatment.of.coup..
A.single.dose.of.dexamethasone.(0.15.to.0.60.mg.per.kg.usually.given.orally).is.recommended.in.all.
patients. with. croup,. including. those. with. mild. disease.. Nebulized. epinephrine. is. an. accepted.
treatment.in.patients.with.moderate.to.severe.croup.
.
41).A#clear#scenario#about#whooping#cough.#What’s#the#causative#organism?.
A)! Pertussis.
B)! Psittacosis.
C)! HiB.
.
Answer.is.A.
.
42).An# 8# year[old# girl# who’s# tired# and# looks# pale.# Her# liver# is# palpable# and# spleen# is# 6# cm#
below#costal#margin.#Her#blood#pressure#is#93/55#and#Hb#is#(low)#and#she#has#positive#
sickle#test#screening.#What#is#the#next#appropriate#investigation#(I#don't#recall#if#she#was#
febrile#or#not)?.
A)! Bone.marrow.aspiration.
B)! Abdominal.US.
C)! Reticulocyte.count.
.
Answer.is.C?.
.
.
.
43).Foul#smelling#urine#in#a#7#year[old#child#with#fever#and#lower#abdominal#pain.#What’s#
the#causative#organism?.
A)! E..Coli.
B)! Proteus.
C)! Klebsiella.
.
Answer.is.A.
.
44).A#child#had#diarrhea#and#vomited#once#with#mild#dehydration:.
A)! Oral.rehydration.solution..
.
45).A#baby#who#can#elevate#his#head#45°#when.#He's#also#cooing#and#smiling.#How#old#is#he?.
A)! 1.month.
B)! 3.months..
C)! 6.months.
.
Answer.is.B.
.
46).Child#who#took#20#pills#of#aspirin.#How#are#you#going#to#manage#him?.
A)! Urine.alkalization..
.
Answer.is.A.
Source:.wikipedia.
Initial.treatment.of.an.acute.overdose.involves.resuscitation.followed.by.gastric.decontamination.by.
administering. activated. charcoal,. which. adsorbs. the. aspirin. in. the. gastrointestinal. tract.. Stomach.
pumping.is.no.longer.routinely.used.in.the.treatment.of.poisonings.but.is.sometimes.considered.if.
the. patient. has. ingested. a. potentially. lethal. amount. less. than. one. hour. before. presentation.[15].
Inducing. vomiting. with. syrup. of. ipecac. is. not. recommended.[10]. Repeated. doses. of. charcoal. have.
been.proposed.to.be.beneficial.in.cases.of.aspirin.overdosing,[16].although.one.study.found.that.they.
might.not.be.of.significant.value.[17].Regardless,.most.clinical.toxicologists.will.administer.additional.
charcoal.if.serum.salicylate.levels.are.increasing..

Intravenous#fluids.
Intravenous. fluids. containing. dextrose. such. as. D5W. are. recommended. to. keep. a. urinary. output.

between.2.and.3.ml/kg/h..

Sodium.bicarbonate.is.given.in.a.significant.aspirin.overdose.(salicylate.level.greater.than.35.mg/dl.6.
hours.after.ingestion).regardless.of.the.serum.pH,.as.it.enhances.elimination.of.aspirin.in.the.urine..It.

is.given.until.a.urine.pH.between.7.5.and.8.0.is.achieved..
.
47).In#a#well[baby#clinic,#the#mother#put#her#child#on#bed#and#he#was#laughing#loudly,#when#
the#doctor#came#to#examine#him,#he#pulled#away#and#wanted#to#reach#his#mother.#How#
old#is#he?#.
A)! 4.months.
B)! 6.months.
C)! 9.months..
.
Answer.is.B.
.
48).A#child#who#is#laughing#and#cooing,#can#support#his#head#but#can’t#move#a#toy#from#one#
hand#to#the#other?.
A)! 4.months.
B)! 6.months.
C)! 8.months.
D)! 2.months..
.
Answer.is.A.
.
49).Complication#of#mumps#in#a#child?.
A)! Meningitis.
B)! Encephalitis.
C)! Orchitis.
.
Answer.is.A.
Source:.http://www.who.int/biologicals/areas/vaccines/mmr/mumps/en/.
The. most. common. complication. of. mumps. in. children. is. meningitis,. sometimes. associated. with.
encephalitis,. and. in. young. adults. orchitis.. Most. complications. due. to. mumps. infection. resolve.
without. permanent. damage.. Death. following. mumps. is. rare. and. is. mostly. due. to. mumps.
encephalitis..
.
50).A#few#hours#after#a#neonate#was#born,#he#started#to#become#jaundiced#and#he#is#getting#
worse#with#time.#What#is#the#diagnosis?.
A)! G6PD..
B)! Pyruvate.kinase.
.
Answer.is.A.
.
51).A# baby# has# constipation# since# birth.# Which# of# the# following# investigations# will# help# in#
diagnosing#his#condition?.
A)! Rectal.biopsy..
B)! Rectal.exam.
C)! Anal.exam.
D)! XKray.
.
Answer.is.A.
Source:.https://emedicine.medscape.com/article/178493Kworkup#c6.
.
The. definitive. diagnosis. of. Hirschsprung. disease. is. confirmed. by. a. fullKthickness. rectal. biopsy.
demonstrating.absence.of.ganglion.cells..The.specimen.must.be.obtained.at.least.1.5.cm.above.the.
dentate.line.because.aganglionosis.may.normally.be.present.below.this.level..Disadvantages.of.fullK
thickness.rectal.biopsy.include.the.necessity.of.general.anesthesia.and.risks.of.bleeding.and.scarring..
.

52).What#is#the#treatment#of#typhoid#fever#in#children?.
Answer.is:.Ceftriaxone.
Source:.https://emedicine.medscape.com/article/968672Ktreatment.
.
Enteric.fever.caused.by.S.+typhi.infection.
●! For.S.+typhi.infection,.initial.empiric.therapy.with.ceftriaxone.is.recommended.due.to.
widespread.resistance..If.susceptible,.chloramphenicol,.ampicillin,.or.TMPKSMZ.may.be.
used..Duration.of.therapy.should.be.14.days..
#
53).A#child#with#a#honey#colored#crust#rash.#What’s#the#diagnosis?#.
Source:#http://www.innerbody.com/diseasesKconditions/impetigo.
#
Impetigo.is.a.skin.infection.caused.by.bacteria,.resulting.in.a.red.rash,.blisters,.and.a.characteristic.
honeyKcolored.crust.on.the.skin..Impetigo.most.commonly.occurs.in.infants.and.children.age.2.to.6..
..
54).A#child#with#coca#cola#colored#urine?.
#
55).A#neonate#with#poor#APGAR#score#and#cyanosis.#How#would#you#manage?#.
#
56).13#year#old#patient#with#salmonella#infection#resistant#to#chloramphenicol.#What#is#the#
appropriate#choice#of#therapy?.
A)! Continue.chloramphenicol.
B)! Add.ciprofloxacin.
C)! Give.cipro.alone.
D)! IM.ceftriaxone.
.
Answer.is.D.
.
57).Child# born# from# a# mother# who# has# gram# negative# streptococcus# positive# at# time# of#
delivery.#After#birth,#child#has#retraction#cyanosis#and#difficulty#breathing.#What’s#your#
next#step#in#management?.
A)! XKray.of.chest!
B)! Blood.culture.and.look.for.infection !
C)! 2.antibiotics.names.were.given.!
.
58).A#child#has#spot#on#teeth.#What#supplement#to#add?#.
A)! Fluoride.
.
Answer.is.A.
.
59).Congenital#adrenal#hyperplasia#test?.
#
Source:.emedicine.
21Khydroxylase. deficiency:. High. serum. concentration. of. 17Khydroxyprogesterone. (usually. >1000.
ng/dL).and.urinary.pregnanetriol.(metabolite.of.17Khydroxyprogesterone).in.the.presence.of.clinical.
features.suggestive.of.the.disease;.24Khour.urinary.17Kketosteroid.levels.are.elevated.
.
60).A#child#swallowed#a#bottle#filled#with#iron#pills,#presented#to#ER#with#abdominal#pain,#
nausea#and#confusion.#What’s#the#best#management?.
A)! .IV.deferoxamine.
.
Answer.is.A.
Source:.https://emedicine.medscape.com/article/815213Ktreatment#d10.
.
In.acute.or.chronic.iron.toxicity,.chelation.therapy.with.deferoxamine.is.indicated.for.patients.with.
serum.iron.levels.>350.mcg/dL.who.have.evidence.of.toxicity,.or.levels.of.>500.mcg/dL.regardless.of.
signs.or.symptoms..In.patients.with.significant.clinical.manifestations.of.toxicity,.chelation.therapy.
should.not.be.delayed.while.one.awaits.serum.iron.levels..

.
61).A#child#with#SCA#presented#with#pain#in#his#penis.#Upon#physical#examination#you#saw#
this#sign.#What#is#the#diagnosis?.

A)! Paraphimosis..

B)! Priapism.

C)! Pyroni's.disease.

Answer.is.B..

62).A#question#about#comparing#breast#milk#with#cow’s#milk?#.

(iron.is.more.in.cow’s.milk).

63).8#month#old#infant#with#meningitis.#What#is#the#causative#organism#in#this#age#group?#.

##################A)#Strep.pneumoniae.

Answer.is.A.
.

64).Neonate#after#delivery#had#bleeding.#What#would#you#give?#.

Source:.emedicine.

Immediately.administer.vitamin.K.subcutaneously.(hold.pressure.on.the.site).for.any.infant.in.whom.
vitamin.K.deficiency.bleeding.is.suspected.or.who.has.serious,.unexplained.neonatal.bleeding...

65).Baby#born#with#1#artery#in#umbilical#cord?.

66).A# 3# year# old# came# to# ER# with# cough# and# SOB,# rhinorrhea# for# three# days.# The# cough# is#
worse#at#night.#What#is#the#most#appropriate#nebulizer#medication?.

A)! Salbutamol.
B)! Ipratropium.bromide..
C)! Chromolyn.sodium..
D)! Racemic.epinephrine..
.
Answer.is.A?.
Since. the. cough. is. worse. at. night. it. goes. more. with. asthma. that. is. triggered. by. respiratory. tract.
infection..
Croup.will.have.inspiratory.stridor.and.barking.cough..
.
.
67).Newborn# with# severe# jaundice# at# birth.# He# was# found# to# be# O+# and# his# mother# is# A[.#
What#is#the#mechanism?.
A)! Fetal.antibody.agglutinating.fetal.antigen.
B)! Fetal.antibody.agglutinating.maternal.antigen..
C)! Maternal.antibody.agglutination.fetal.antigen..
.
Answer.is.C.
Source:.https://en.wikipedia.org/wiki/Rh_disease.
.
Rh.isoimmunization.
The. disease. ranges. from. mild. to. severe,. and. typically. occurs. only. in. some. second. or. subsequent.
pregnancies. of. Rh. negative. women. where. the. fetus's. father. is. Rh. positive,. leading. to. a. Rh+.
pregnancy..If.the.mother.is.Rh.negative.and.the.baby.is.Rh.positive,.the.mother.produces.antibodies.
(including. IgG). against. the. rhesus. D. antigen. on. her. baby's. red. blood. cells.. During. this. and.
subsequent.pregnancies.the.IgG.is.able.to.pass.through.the.placenta.into.the.fetus.and.if.the.level.of.
it. is. sufficient,. it. will. cause. destruction. of. rhesus. D. positive. fetal. red. blood. cells. leading. to. the.
development.of.Rh.disease..
.
68).A#boy#who#took#primaquine#developed#symptoms#of#anemia.#Deficiency#of#which#of#the#
following#caused#his#symptoms?#.
A)! BK.GK6KP.dehydrogenase..
B)! CK.Glucose.6.phosphatase.
C)! DK.Pyruvate.kinase..
.
Answer.is.A.
Source:.https://en.wikipedia.org/wiki/GlucoseK6Kphosphate_dehydrogenase_deficiency#Triggers.
.
Antimalarial. drugs. that. can. cause. acute. hemolysis. in. people. with. G6PD. deficiency. include.
primaquine,.pamaquine,.and.chloroquine...
.
69).A#mother#brought#her#child#to#ED#with#itchy#skin#lesions#that#appeared#after#intake#of#
some# food.# The# lesions# appear# in# a# certain# area# and# stay# for# 2# hours# then# disappear#
then#appear#in#another#area.#What#is#it#called?.
A)! Burrow..
B)! Wheal..
Answer.is.B.
Source:. Wheal:. A. wheal. is. a. rounded. or. flatKtopped,. pale. red. papule. or. plaque. that. is.
characteristically.evanescent,.disappearing.within.24.to.48.hours..
.
70).Long# scenario# of# a# child# who# had# a# high# fever# and# developed# a# tonic[clonic# seizure.#
What#is#management?#.
A)! Fever.control..
B)! Diazepam..
C)! Phenytoin..
Answer.is.A.
Source:.http://www.aafp.org/afp/2006/0515/p1761.html.
.
Current. recommendations. do. not. support. the. use. of. continuing. or. intermittent. neuroleptic. or.
benzodiazepine.suppressive.therapies.after.a.simple.febrile.seizure..
For.patients.who.have.an.ongoing.seizure.at.the.time.of.assessment.(i.e.,.febrile.status.epilepticus),.
intravenous. diazepam. (0.2. to. 0.5. mg. per. kg. of. weight. intravenously. every. 15. minutes. for. a.
cumulative.dosage.of.5.mg.in.children.one.month.to.five.years.of.age).often.is.effective..
.
71).A# baby# who# was# recently# started# on# cow’s# milk,# presenting# with# wheezing# and# SOB.#
Sputum#test#shows#there#is#blood.#Labs#show:#microcytic#hypochromic#anemia.#What’s#
the#treatment?.
A)! Steroids.
B)! Cytotoxic.drugs.
C)! Cromolyn.sodium.
D)! Stop.cow.milk..
.
Answer.is.D.
.
72).A#young#patient#was#playing#presented#with#severe#epigastric#pain,#retching#and#unable#
to#vomit.#Unable#to#pass#NG#tube.#What#is#the#diagnosis?.
A)! Hiatal.hernia.
B)! Volvulus.
..
73).12#year#old#presented#with#growth#retardation#and#abdominal#striae.#What#is#next?.
A)! Morning.and.night.cortisol.level.
B)! MRI.brain.
.
Answer.is.B.
.
74).8# and# half# year# old# brought# by# her# older# sister# for# early# puberty.# Her# sister# menarche#
was#at#14#years#old.#She#presented#with#breast#buds#and#no#pubic#hair#growth.#What#is#
the#cause?.
A)! Precious.puberty.
B)! Normal.development.
C)! Early.thelarche.
.
Answer.is.B.
.
75).Neonate# with# cyanosis# and# shortness# of# breath# while# feeding# with# diaphoresis.#
Presented#with#ejection#systolic#murmur,#3/6,#normal#S1#loud#S2,#best#heard#on#left#side#
of#chest.#What#is#the#cause?.
A)! Large.ASD.
B)! Large.VSD.
C)! PDA.
D)! Mitral.Regurgitation.
.
76).Diarrhea#after#introducing#juices#to#baby#with#positive#reducing#substances#test.#What#
kind#of#intolerance#the#baby#has?#.
A)! Galactose.intolerance.
B)! Fructose.intolerance.
C)! Lactose.intolerance.
.
Answer.is.C.
.
77).One# year# old# baby# with# a# prominent# forehead.# Hemoglobin# electrophoresis# show:# Hb#
A2#2[3%#and#Hb#F#2%.#What’s#the#diagnosis?#.
A)! Alpha.thalassemia.
B)! Beta.thalassemia.major.
C)! Beta.thalassemia.minor.
.
78).A#baby#with#recurrent#jaundice#and#multiple#blood#transfusions,#what#causes#the#RBC#to#
be#destructed,#blood#results#show#low#RBC,#DLT#test#normal#and#some#other#labs#and#
blood#smear#(spirometric??,#nucleated...).
A)! G6PD.
. . . . . #
79).Child# with# repeated# polymicrobial# chest# infections.# Skin# test# positive# for# candida#
antigen.#Blood#test#all#normal#except#high#IgG#or#IgM!?,#Low#lymphocytes.#Lymph#node#
biopsy# showed:# rudimentary# germinal# centers.# What# is# the# pathophysiology# of# this#
disease?#Answer.:.the.diagnosis.is.xKlinked.Bruton’s.Agammaglobulinemia.
#
80).A# child# presented# with# difficulty# in# breathing# and# mild# fever.# The# parent# said# that# he#
had#2#similar#episodes#two#weeks#ago.#What's#your#first#step?.
A)! Observe.
B)! Fluids.and.supportive.care..
C)! Inhaled.steroids..
D)! Ventilatory.support.
.
Answer.is.B?.
.
81).A#scenario#of#a#boy#with#recurrent#infections.#His#brother#died#when#he#was#3#years#old.#
He#has#2#sisters#who#are#alive#and#well.#What#does#he#have?#.
A)! Combined.immunodeficiency.
B)! XKlinked.immunodeficiency.
.
Answer.is.B.
.
82).Child#had#URTI#2#weeks#ago,#presented#with#right#upper#quadrant#pain.#Labs#show#high#
WBC,#low#hg#and#other#labs#not#sure#of#them.#Electrophoresis#shows#inclusion#bodies.#
What's#the#diagnosis?.
A)! Acute.appendicitis.
B)! Sickle.cell.disease.
C)! Lithium.
D)! GI.infarct..
.
83).Child#presented#with#rash#that#started#in#the#back#then#it#was#spread#to#all#his#body.#The#
rash# was# pustular# with# other# areas# vesicular.# The# rash# had# truncal# distribution# more#
than#the#limbs.#What#is#the#incubation#period#for#this#organism?#.
A)! 5K11.days.!
B)! 11K.21.days.!
C)! 23.K.30.days!
.
Answer.is.B.
Source:.https://www.webmd.boots.com/children/guide/chickenpoxKincubationKperiod.
.
The.incubation.period.is.seven.to.21.(usually.10.to.21).days.after.exposure.to.the.herpes.varicellaK
zoster. virus. to. the. development. of. the. symptoms.. The. disease. is. most. contagious. a. day. or. two.
before.the.rash.appears.and.until.the.rash.is.completely.dry.and.scabbed.over,.about.five.to.six.days.
after.onset.of.the.rash..
. . . . . .
84).Child# came# with# knee# swelling# or# bleeding# after# mild# trauma.# Bleeding# time# was#
abnormal#and#it#was#NOT#corrected#after#we#gave#fresh#frozen#plasma.#Then#bleeding#
time# got# back# to# normal# after# giving# platelet# transfusion# (his# platelet# level# was# 50).#
What#does#the#patient#have#(PT#and#PTT#were#not#included#in#the#question)?.
A)! VWF.deficiency.
B)! HSP.
C)! Thrombotic.thrombocytopenic.purpura.(Not.ITP).
D)! Weird.syndrome.name,.can't.remember.
.
85).New# born# totally# healthy# with# left# thigh# bruise.# All# examination# normal.# Prolonged# pt#
and#ptt.#What’s#the#diagnosis?#.
A)! Hemophilia.
B)! Factor.10.deficiency. . . . .
C)! Idiopathic.thrombocytopenic.purpura.
Answer.is.B.
Source:.https://emedicine.medscape.com/article/209867Kworkup.
Coagulation.study.findings.in.patients.with.factor.X.deficiency.include.the.following:.
●! The.prothrombin.time.(PT).is.prolonged.
●! The.activated.partial.thromboplastin.time.(aPTT).is.prolonged.
●! The.Russell.viper.venom.time.(RVVT).is.prolonged;.Russell.viper.venom.cleaves.factor.
X.to.produce.active.factor.Xa.
●! Bleeding.time.is.within.the.reference.range.
.
86).A#child#with#multiple#plaques#on#his#face,#abdomen#and#feet.#What#is#the#Diagnosis?#(In#
the#q#they#mentioned#something#about#histoimmune#and#positive#CD?).
A)! Mycosis.fungoides.
B)! Bowen.disease.
C)! Basel.cell.carcinoma.
If.the.plaques.are.hypopigmented,.with.CD.3.positive,.CD4.positive.and.CD8.negative.this.is.mycosis.
fungoides..
.
.
87).Pediatric#age#group,#mass#on#the#inner#side#of#lower#lip,#non[tender#and#bluish#in#color.#
What#is#the#diagnosis?.
A)! Mucocele.
B)! Gingival.cyst. . . . . .
C)! Ranula.
D)! Epiula.
.
Answer.is.A.
Source:.wikipedia.
The. mucocele. has. a. bluish. translucent. color,. and. is. more. commonly. found. in. children. and. young.
adults..The.most.common.location.to.find.a.mucocele.is.the.inner.surface.of.the.lower.lip..It.can.also.
be. found. on. the. inner. side. of. the. cheek. (known. as. the. buccal. mucosa),. on. the. anterior. ventral.
tongue,.and.the.floor.of.the.mouth....
.
88).Most#common#known#side#effect#of#steroid#use#in#children?.
A)! Excitable.behavior..
B)! Labile.mood.
C)! Growth.retardation..
D)! Intraocular.Hypertension.
.
Answer.is.C?.
.
89).Girl# or# boy# (I# can’t# remember)# hit# puberty.# Till# which# age# the# bone# growth# will#
continue?.
A)! 12.months.
B)! 24.months. . . . .
C)! 36.months.
.
Answer.is.B.
.
90).A# young# girl# came# with# leg# pain# more# in# night.# Examination# is# normal.# Labs:# ESR# and#
ANA#high.#What’s#the#diagnosis?##.
A)! Rheumatoid..
B)! Growing.pain.
C)! Osteoid.osteoma.
D)! Osteosarcoma.
.
.
91).10#year#old#child#with#acute#rheumatic#fever#and#cardiac#involvement.#For#how#long#will#
you#continue#him#on#prophylactic#antibiotics?.
A)! 1.year.
B)! 5.years. . . . . .
C)! 10.years. . . . . .
D)! 15.years.
.
Answer.is.C.
Source:.

.
.
92).A# baby# has# depigmentation# since# birth,# photophobia# and# refractory# error# nystagmus.#
What’s#the#most#likely#complication#associated#with#his#condition?#(Case#of#albinism)#.
A)! Brain.tumor.
B)! Skin.cancer... . . . .
C)! Renal.dysfunction.
.
Answer.is.B.
Source:.https://emedicine.medscape.com/article/1200472Kfollowup.
.
Complications. of. albinism.
See. the. list. below:.
Skin. cancer,. sunburn.
Reduced. visual. acuity.
Social.stigma.
.
.
93).A# child# with. cough,# coryza# and# conjunctivitis.# Presented# with# maculopapular# rash# in#
palm#and#buttocks.#On#examination,#there#are#pustules#in#tonsils.#What#other#physical#
signs#will#find#in#the#patient?.
A)! Spot.on.the.sole.of.the.foot. . . . . . . .
B)! Swollen.tonsils..
C)! Scratch.markers..
D)! Diarrhea.
.
Answer:.It’s.measles,.diarrhea.is.a.common.complication.??.
Ref..http://emedicine.medscape.com/article/966220Kclinical#b3.
.
94).5#month#old#baby#found#to#have#developmental#delay.#Which#one#of#the#following#is#a#
reassuring#sign#for#the#doctor?.
A)! Pincer.grasp..
B)! Reaching.for.objects...
C)! Sits.independently..
.
Answer.is.B.
.
95).Child#with#autoantibodies..#Chances#of#getting#DM#type#1?# # # # .
A)! 40.%..
B)! 60.%..
C)! 80.%..
D)! 100.%. .
.
Source:.http://diabetes.diabetesjournals.org/content/54/suppl_2/S52.
. .
in. the. large. U.S.. study,. the. Diabetes. Prevention. Trial. (DPTK1),. four. autoantibodies. (ICA,. IAA,.
GAD65Ab,.and.IAK2Ab).were.analyzed.to.assess.the.risk.for.developing.diabetes;.98%.of.firstKdegree.
relatives.who.went.on.to.develop.type.1.diabetes.had.one.or.more.autoantibodies,.and.80%.had.two.
or. more. autoantibodies.. Individuals. with. two. or. more. positive. biochemical. autoantibodies. had. a.
68%.5Kyear.risk.for.developing.type.1.diabetes,.and.those.with.all.three.biochemical.antibodies.had.
an.estimated.100%.5Kyear.risk.. . .
.
96).Child#with#fever#and#productive#cough#and#diarrhea#5#times#a#day.#Now,#tolerating#oral#
rehydration?.
A)! Hospitalize.
B)! Augmentin.and.discharge.
C)! Stool.analysis.and.culture.
.
97).Which# of# the# following# vaccines# is# contraindicated# in# a# child# with# progressive#
neurological#problems?#.
A)! Pneumococcal.conjugate.vaccine.
B)! Hib.vaccine.
C)! Dtap.
D)! Varicella.
.
Answer.is.C.
Source:.https://www.cdc.gov/vaccines/hcp/acipKrecs/generalKrecs/contraindications.html.
.
DTaP.
Contraindications:.
1)! Severe.allergic.reaction.(e.g.,.anaphylaxis).after.a.previous.dose.or.to.a.vaccine.component.
2)! Encephalopathy. (e.g.,. coma,. decreased. level. of. consciousness,. . prolonged. seizures),. not.

attributable. to. another. identifiable. cause,. within. 7. days. of. administration. of. previous. dose. of.

DTP.or.DTaP.

Precautions:.
1)! Progressive. neurologic. disorder,. including. infantile. spasms,. uncontrolled. epilepsy,. progressive.
encephalopathy;.defer.DTaP.until.neurologic.status.clarified.and.stabilized.
2)! Temperature.of.≥105°F.(≥40.5°C).within.48.hours.after.vaccination.with.a.previous.dose.of.DTP.

or.DTaP.

3)! Collapse. or. shockKlike. state. (i.e.,. hypotonic. hyporesponsive. episode). within. 48. hours. after.

receiving.a.previous.dose.of.DTP/DTaP.

4)! Seizure.≤3.days.after.receiving.a.previous.dose.of.DTP/DTaP.

5)! Persistent,.inconsolable.crying.lasting.≥3.hours.within.48.hours.after.receiving.a.previous.dose.of.

DTP/DTaP.

6)! GBS.<6.weeks.after.previous.dose.of.tetanusKtoxoid–containing.vaccine.

7)! History. of. ArthusKtype. hypersensitivity. reactions. after. a. previous. dose. of. diphtheriaKtoxoid–

containing. or. tetanusKtoxoid–containing. vaccine;. defer. vaccination. until. at. least. 10. years. have.

elapsed.since.the.last.tetanusKtoxoid–containing.vaccine.

8)! Moderate.or.severe.acute.illness.with.or.without.fever.

.. . . . . .
98).A# 6# year# old# came# to# your# clinic# with# no# vaccine# history# except# bgg# (I# never# read# this#
vaccine#except#in#this#question#in#exam).#What#is#the#appropriate#vaccination#for#him#on#
his#first#clinic#visit?#(time#of#clinic#visit#not#mentioned.#Means#now#or#before#when#he#
was#infant???#To#catch#up??#Or#to#give#6#years#vaccines?). . . . .
A)! MMR.OPV.Varicella.DTap.KKK.6.yrs.vaccines..
B)! MMR.OPV.HiB.
C)! MMR.OPV.Hib.HBV.
.
99).Child#with#multiple#infections:#viral,#fungal#and#bacterial.#He#has#a#defect#in#Cd4?.
A)! Ada.deficiency.
Adenosine# deaminase# deficiency. (also. called. ADA# deficiency. or. ADA[SCID[1]). is. an. autosomal.
recessive[2]metabolic.disorder.that.causes.immunodeficiency..It.occurs.in.fewer.than.one.in.100,000.
live.births.worldwide..
It.accounts.for.about.15%.of.all.cases.of.severe.combined.immunodeficiency.(SCID).[3].ADA.deficiency.
may.be.present.in.infancy,.childhood,.adolescence,.or.adulthood.
……...
##
1). . . . . Child#with#a#history#of#fall#without#losing#consciousness.#He#vomited#twice#and#is#
crying#and#complaining#of#a#headache.#What#should#you#do?#
A).....CT.
B).....Close.observe.
Answer:.A.
Indications.for.CT.scanning.in.a.patient.with.a.head.injury.include.anisocoria,.GCS.score.less.than.12.
(some.studies.suggest.CT.scanning.in.any.pediatric.patient.with.a.GCS.score.of.<.15),.posttraumatic.
seizures,. amnesia,. progressive. headache,. an. unreliable. history. or. examination. because. of. possible.
alcohol.or.drug.ingestion,.loss.of.consciousness.for.longer.than.5.minutes,.physical.signs.of.basilar.
skull.fracture,.repeated.vomiting.or.vomiting.for.more.than.8.hours.after.injury,.and.instability.after.
multiple.trauma..
https://emedicine.medscape.com/article/907273Kworkup#c8.
..
2).....An#18#month#old#female#baby#with#anemia.#What’s#the#diagnosis?#INCOMPLETE#
A).....Homozygous.b.thalassemia.
B).....Homozygous.a.thalassemia.
C).....Carrier.of.a.thalassemia.
Answer:.
K. . . . . . . . Alpha. thalassemia. silent. carrier:. Patients. are. likely. to. be. asymptomatic. and. hematologically.
normal..
K........ AlphaKthalassemia.major.or.homozygous.alpha(0).thalassemia:.Hemoglobin.Bart.hydrops.fetalis.
syndrome.is.typically.caused.when.all.4.alphaKglobin.genes.are.deleted..
K........BetaKthalassemia.minor.(commonly.referred.to.as.betaKthalassemia.trait):.usually.asymptomatic;.
the.mild.microcytic.anemia.is.often.misdiagnosed.as.iron.deficiency.anemia..
K........ BetaKthalassemia.intermedia:.usually.a.similar.presentation.to.betaKthalassemia.major.but.as.a.
toddler. or. older. child;. symptoms. are. usually. less. pronounced. and. the. course. is. usually. more.
insidious..
K........BetaKthalassemia.major.(also.called.Cooley's.anemia):.complete.absence.of.hemoglobin.A;.often.
presents.at.a.few.months.of.age.with.progressive.pallor.and.abdominal.distension;.perinatal.history.
is. most. often. uneventful,. and. the. infant. may. be. pale,. possibly. with. poor. feeding. and. decreased.
activity;.hepatosplenomegaly.and.bony.abnormalities.are.often.present.at.presentation,.most.often.
of.the.skull.(frontal.and.parietal.bossing,.and.chipmunk.facies)..
BUT. (A). is. closer. because. approx.. after. 18. months. HbF. production. reduces. and. HbA. cannot. be.
produced..
Reference:.http://bestpractice.bmj.com/bestKpractice/monograph/251.html.
..
3). . . . . Before# giving# influenza# vaccine# you# have# to# ask# if# the# child# has# an# allergy# from#
which#of#the#following#substances?#
A).....Egg.
Answer:.A.
Reference:.https://www.cdc.gov/flu/professionals/vaccination/vaxKsummary.htm.
..
4)..... A#child#with#barking#cough#only.#No#fever#or#shortness#of#breath.#Vitals#are#normal.#
What’s#your#management?#
A).....Moist.oxygen.
B).....Nebulizer.racemic.acid...................................................................... .
C).....Antibiotics.
Answer:.A..Cool.mist.from.a.humidifier.and/or.sitting.with.the.child.in.a.bathroom.filled.
with.steam.generated.by.running.hot.water.from.the.shower,.help.minimize.symptoms..
Reference:https://emedicine.medscape.com/article/962972Ktreatment.
.... .
5). . . . . A# patient# with# hematuria# and# upon# examination,# he# has# aniridia# (absence# of# the#
iris).#What’s#the#diagnosis?#
A).....Wilms.tumor.
B).....Neuroblastoma.
Answer:.A.
Reference:.https://emedicine.medscape.com/article/989329Koverview........... .
..
..
6). . . . . A#child#came#with#abdominal#mass.#Imaging#showed#renal#tumor.#The#patient#has#
absent#iris.#What#is#the#most#likely#diagnosis?................................... .
A).....Wilms's.tumor.
Answer:.A.
Reference:.https://emedicine.medscape.com/article/989329Koverview........... .
..
7). . . . . Pediatric#patient#known#case#of#acute#lymphoblastic#leukemia#(ALL),#presented#to#
the#ER#with#fever#and#pancytopenia#(lab#results#were#provided).#What#is#your#action?#
A).....Blood.transfusion.
B).....Refer.to.oncology.................................................................... .
C).....Start.Antibiotics.
Answer:.C.
..
8). . . . . Most# common# presentation# of# congenital# heart# disease?.............................
. .
A).....JVD.
B).....Difficulty.feeding.
C).....Ascites.......................................................................... .
Answer:. the. most. common. presentation. of. congenital. heart. disease. is. with. a. heart.
murmur..
Reference:.Illustrated.textbook.of.pediatrics.
..
9).....Which#of#the#following#congenital#heart#defects#is#associated#with#Down#syndrome?#
A).....Endocardial.cushion.defects.such.as.AVSD.
Answer:.A.
Reference:.https://emedicine.medscape.com/article/943216Kclinical#b2.
..
..................................................................... .
10).. A#child#with#foul#smelling#breath#and#seed#like#structures#coming#out#of#the#mouth.#
He#is#also#a#mouth#breather.#No#history#of#fever.#What’s#the#most#likely#diagnosis?#
A).....Pulmonary.disease.
B).....Focal.tonsillitis.
Answer:. B.. Individuals. with. acute. tonsillitis. present. with. fever,. sore. throat,. foul. breath,.
dysphagia,. odynophagia,. and. tender. cervical. lymph. nodes.. Airway. obstruction. may.
manifest. as. mouth. breathing,. snoring,. sleepKdisordered. breathing,. nocturnal. breathing.
pauses,.or.sleep.apnea..
Reference:.https://emedicine.medscape.com/article/871977Kclinical.
..
11). . A#2[year#child#came#to#the#clinic#with#his#mother.#He#scribbles#circles,#runs#around#
and#climbs#onto#the#chair,#plays#with#his#friends#but#does#not#share#his#toys.#He#speaks#
10#words.#He#names#the#picture#you#point#to.#What#is#the#best#thing#to#tell#his#mom?#
A).....He.is.normal.
B).....Delayed.social.development.
C).....Delayed.language.development.
Answer:. A.. He. is. normal. since. sharing. toys. and. cooperative. play. only. starts. at. 4. years. of.
age..
Reference:.Toronto.notes.
..
12)..6#month#old#baby#presented#with#jaundice,#seizure,#irritability#and#vomiting.#He#has#
been# breast[fed# during# his# first# 3# months# of# life,# then# he# has# been# started# on#
commercially#available#milk#formula.#More#recently,#his#mother#introduced#fruit#juices#
into#his#diet.#Investigations#revealed#positive#urine#reducing#substances.#What#element#
should#avoided#in#his#diet?################################################################# #
A).....Galactose..
B).....Fructose..
C).....Phenylalanine..
D)....Irrelevant.choice..
Answer:.B.
..
13). . 4# year# old# child# brought# by# his# parents# to# pediatric# outpatient# clinic# with# them#
complaining# of# his# massive# uncontrolled# appetite# during# the# last# 18# months.# Weight#
was# above# 95th# percentile,# while# height# was# below# 5th# percentile.# Mother# reported#
that#her#son#was#failing#to#thrive#during#his#first#two#years,#beside#the#fact#that#he#was#
developmentally#delayed#compared#to#his#siblings#until#he#caught#up#late.#O/E#he#had#a#
high# forehead,# broad# nose,# small# peripheries# (Hands# and# Feet).# What# is# the# cause#
behind#his#symptoms?####################### #
A).....Genetic..
B).....Metabolic..
C).....Nutritional..
D)....Irrelevant.choice..
Answer:.A.(PraderKWilli.syndrome).
Reference:.Medscape.
..
14).. A#5#year#old#boy#presents#with#tender,#swollen#and#painful#testicle.#O/E#there#was#
absent#cremasteric#reflex.#Which#of#the#following#is#the#correct#diagnosis?#
A).....EpididymoKorchitis..
B).....Testicular.torsion..
Answer:. B.. The. most. sensitive. physical. finding. in. testicular. torsion. is. the. absence. of. the.
cremasteric.reflex..
Reference:.aafp.org.
........................................................................................
15). . Pediatric# case# of# Hemolytic# uremic# syndrome:# he# has# petechia,# hematuria# and#
proteinuria.# 2# weeks# ago,# he# develop# bloody# diarrhea.# The# doctor# prescribed#
symptomatic#treatment#and#probiotics.#Platelets#95,#WBC#48.#What’s#your#next#step?#
A).....Antibiotic.
B).....Platelet.transfusion.
C).....Anticoagulant.
Answer:.Treatment.of.HUS:.
K........Avoid.antibiotics.and.platelet.transfusion..
K........Good.hydration.with.IV.fluids..
K........Blood.transfusion.in.anemic.patients.when.indicated..
K........CCB.for.HUS.with.hypertension..
K........Dialysis.for.renal.failure..
K........Renal.transplant.for.irreversible.failure..
Reference:http://bestpractice.bmj.com/bestpractice/monograph/470/treatment/details.html.
..
16)..Pediatric#patient#has#symptoms#of#epiglottitis#with#stridor#and#distress.#What’s#your#
next#step?#
A).....Antibiotic.
B).....Admit.to.ICU.and.refer.to.ENT.
(No.intubation.in.choices).
Answer:.B.
Reference:.Medscape.
..
17)..Pediatric#patient#comes#with#congested#pharynx,#tonsils#and#plaques#on#the#tongue,#
lips,#and#gingivitis.#No#lesions#on#the#hands#and#feet.#What#is#the#diagnosis?#
A).....Herpes.simplex.virus.
B).....Coxsackie.virus.
Answer:.A..
Reference:.https://emedicine.staging.medscape.com/article/218502Kdifferential.
..
18). . Scenario:#a#patient#developed#bloody#diarrhea,#abdominal#pain#and#vomiting#after#
eating# from# a# restaurant.# History# of# hematuria.# Lab# results# show# anemia.# What# is# the#
diagnosis?########## #
A).....E.coli................................................................................. .
B).....Hemolytic.uremic.syndrome.
Answer:. B.. HUS. is. characterized. by. progressive. renal. failure,. microangiopathic. hemolytic.
anemia.(MAHA),.and.thrombocytopenia..In.typical.HUS,.diarrhea.usually.occurs..
Reference:.Medscape.
................
19). . What# is# the# treatment# for# the# above# question?.............................
. .
A).....Steroids........................................................................... .
B).....Antibiotics.
Answer:.Treatment.of.HUS:.
K........Maintenance.of.good.hydration.is.important.to.minimize.the.likelihood.of.renal.damage..Careful.
attention.needs.to.be.paid.to.avoid.cardiopulmonary.overload,.especially.because.these.patients.are.
at.risk.of.developing.oliguria..
K. . . . . . . . Avoidance. of. antibiotics,. antimotility. (antidiarrheal). agents,. opioids,. or. nonKsteroidal. antiK
inflammatory.drugs.is.advised..
K. . . . . . . . Platelet.transfusions.have.been.associated.with.clinical.deterioration.and.should.be.avoided.if.
possible..
K........For.the.anemia:.blood.transfusion.if.needed..
K........For.the.HTN:.CCB.
K. . . . . . . . If. renal. failure. is. present:. dialysis. is. performed. if. clinically. indicated:. signs. and. symptoms. of.
uremia,. hyperkalemia. (potassium. >6.5. with. ECG. changes),. persistent. severe. acidosis. (bicarbonate.
<10),. hypertension. secondary. to. volume. overload. that. cannot. be. controlled. with. medical. therapy,.
and.necessity.for.transfusion.in.patient.with.volume.overload.and/or.oliguria..
K........In.irreversible.renal.failure:.renal.transplant..
Reference:http://bestpractice.bmj.com/bestpractice/monograph/470/treatment/details.html.
..
20)..What#age#in#months#can#a#baby#say#a#few#words?#
A).....6.months.
B).....9.months.
C).....12.months....................................................................................... .
D)....24.months.
At.12.months:.the.baby.can.say.2.words.
At.24.months:.2K3.word.phrases.and.uses.“I,.me,.you”..
Reference:.Toronto.notes...........
..............
21). . During# delivery# of# a# baby,# there# was# stylomastoid# foramen# trauma.# Which# of# the#
following#features#will#be#evident#when#you#examine#this#baby?#
A).....Loss.of.eye.close...................... .
B).....Loss.of.facial.sensation....... .
C).. Loss.of.mastication.function.
Answer:.A.
Facial.Palsy.(Bell’s.palsy):.
K. . . . . . . . It. is. usually. due. to. pressure. by. the. forceps. blade. on. the. facial. nerve. at. its. exit. from. the.
stylomastoid.foramen.or.in.its.course.over.the.mandibular.ramus..
K. . . . . . . . It.appears.within.1K2.days.after.delivery.due.to.resultant.edema.and.hemorrhage.around.the.
nerve..
K........ Manifestations:.There.is.paresis.of.the.facial.muscles.on.the.affected.side.with.partially.opened.
eye.and.flattening.of.the.nasolabial.fold..The.mouth.angle.is.deviated.towards.the.healthy.side..
...............Reference:.https://www.gfmer.ch/Obstetrics_simplified/foetal_birth_injuries.htm.
................
22). . Clear#case#about#crohn’s#disease:#a#child#with#abdominal#cramping,#diarrhea...etc.#
Endoscopy# shows# skip# lesions# and# transmural# inflammation.# What’s# the# diagnosis?#############################
# #
A).....Crohn’s.disease........................................................... .
B).....Ulcerative.colitis.
C).....Celiac.disease.
Answer:.A.
..
23)..Similar#case:#child#with#abdominal#pain#and#tenderness,#bloody#diarrhea#and#weight#
loss.#(no#labs).#What's#the#diagnosis?........................................ ........ .
A).....Crohn’s.disease............................................................................. .
B).....Celiac.disease.
C).....Ulcerative.colitis.
Answer:.abdominal.pain.and.weight.loss.go.more.with.crohn’s,.while.the.bloody.diarrhea.is.
more. with. UC.. Most. likely,. crohn’s. since. two. of. the. features. in. the. scenario. support. this.
diagnosis..
.
24)..15#month#old#boy#with#meningitis.#What’s#the#best#antibiotic#choice#for#him?#
A).....Vancomycin.+.ceftriaxone .
B).....Ampicillin.(or.penicillin?).+.gentamicin.
Answer:.A..In.infants.and.children:.Initial.antibiotic.selection.should.provide.coverage.for.the.3.most.
common. pathogens:. S+ pneumoniae,. N+ meningitidis,. and. H+ influenzae.. According. to. the. 2004.
Infectious. Diseases. Society. of. America. (IDSA). practice. guidelines. for. bacterial. meningitis,.
vancomycin. plus. either. ceftriaxone. or. cefotaxime. is. recommended. for. those. with. suspected.
bacterial.meningitis.
Reference:.https://emedicine.medscape.com/article/961497Ktreatment#d10.
..
25). . 14# year# old# boy# with# swollen# lips.# Deficiency# of# which# of# the# following# causes# his#
presentation?#
A).....Hereditary.angioedema.
B).....Factor.D.
C).....Anaphylactic.inhibitor.
D)....C1.esterase.inhibitor.
Answer:.D.
Reference:.https://emedicine.medscape.com/article/135604Koverview.
..
26).. A#three#year[old#girl#presented#to#Emergency#Department#with#fever,#vomiting#and#
abdominal# pain# which# began# 10# hours# ago.# Radiological# examination# confirmed# a#
dilated#intestinal#pouch#attached#to#the#anterior#abdominal#wall.#Her#diagnosis#was#the#
persistence#of#a#Meckel’s#diverticulum.#
Which#of#the#following#sites#will#the#surgeon#look#for#this#diverticulum?#
A).....Lower.Duodenum.
B).....Lower.Jejunum.
C).....Lower.Ileum.
D)....Cecum.
Answer:.C.
..
27).. Neonate#presents#with#lethargy,#irritability#and#fever.#Which#of#the#following#is#the#
most#likely#causative#organism?#
A).....Listeria.monocytogens.
B).....Staph.aureus.
C).....N.meningitidis.
Answer:.A.
Reference:.Medscape.
..
28). . A# patient# presents# with# a# cough,# fever,# rhinorrhea,# malaise,# with# conjunctival#
suffusion.# There# are# small,# grayish,# irregular# lesions# surrounded# by# an# erythematous#
base,#on#the#buccal#mucus#membrane#near#the#second#molar#teeth.#
What#is#the#most#likely#diagnosis?#
A).....Measles.
B).....Rubella.
C).....Parainfluenza.
D)....Respiratory.syncytial.infection.
Answer:.A.
29).. A#baby#with#greasy#scaly#rash#at#the#edge#of#the#forehead#and#over#the#cheeks#not#
sparing# the# folds.# Which# of# the# following# is# the# appropriate# treatment?# (Seborrheic#
dermatitis)#
A)..Muropicin.topical.(antibiotic).
Answer:. LowKpotency. topical. corticosteroids,. such. as. hydrocortisone,. desonide,. and.
mometasone. furoate,. have. shown. to. be. efficacious. on. the. face.. Antifungal. therapies. are.
firstKline. therapies. (Ketoconazole,. naftifine,. or. ciclopirox. creams. and. gels). are. effective.
therapies..Systemic.fluconazole.may.help.if.seborrheic.dermatitis.is.severe.or.unresponsive..
Combination.therapy.has.been.recommended...
Reference:.https://emedicine.medscape.com/article/1108312Ktreatment.
..
30)..How#to#manage#croup?#
A).....Nebulized.epinephrine.and.steroid.
B).....Inhaled.salbutamol.and.betamethasone.
Answer:.A.
Reference:.Medscape.
..
31)..Minimum#age#to#give#influenza#vaccine?#
A).....3.months.
B).....6.months.
C).....9.months.
D)....12.months.
Answer:.B..Children.younger.than.6.months.of.age.should.not.be.vaccinated..
Reference:.https://www.cdc.gov/flu/professionals/vaccination/vaxKsummary.htm.
..
32)..Which#of#these#patients#will#most#likely#be#diagnosed#with#rheumatic#fever#from#his#
symptoms?#
A).. Child.with.knee.swelling.and.joint.pain.and.sore.throat..
Answer:.A.
.….
Part#3#
##
1). . . . . A#3#year#old#with#symptoms#of#DM#since#2#weeks#(weight#loss#of#3#kg,#dysphagia,#
and#polyuria.#What#is#the#appropriate#step#to#diagnose#him?######## ###########################################
# #
A).....HLA.DR.3.
B).....Urine.dipstick.
Answer:.B.
Reference:.https://emedicine.medscape.com/article/919999Kworkup.
..
2).....Long#scenario#of#a#boy#k/c#of#G6PD.#Came#with#Hb#of#7.#What#will#you#do?#
A).....Blood.transfusion.
B).....Give.iron....................................................................................... .
C).....Reassurance.
D)....Give.folic.acid.
Answer:.
K........ Affected.patients.should.be.encouraged.to.maintain.a.good.fluid.intake.and.to.eat.a.
light.diet,.as.nausea.is.common..
K........A.haematology.consultation.is.warranted.once.haemolytic.anaemia.is.diagnosed..
K........Folic.acid.is.required.to.supply.increased.RBC.production..
K........ If.severe.anaemia.(haemoglobin.<70.g/L.[<7.g/dL]).with.no.renal.impairment:.blood.
transfusion.
K. . . . . . . . If. severe. anaemia. (haemoglobin. <70. g/L. [<7. g/dL]). with. renal. impairment:. blood.
transfusion.and.renal.dialysis..
Reference:.http://bestpractice.bmj.com/bestKpractice/monograph/704/treatment.html.
..
3).....A#mother#comes#with#her#9#year#old#daughter.#She’s#looking#short#and#her#mother#is#
also#short.#The#bone#age#is#7#years#and#her#labs:#all#normal#except#insulin#like#growth#
hormone#(below#the#normal#range).#What#will#you#do?#
A).....Reevaluation.after.1.year.
B).....Give.growth.hormone.
Answer:.B.
Reference:.https://emedicine.medscape.com/article/920446Ktreatment.
..
4). . . . . Scenario# of# a# child# with# recurrent# chest# infections# and# sinusitis.# What’s# the#
diagnosis?#
A).....Cystic.fibrosis.
B).....Primary.ciliary.dyskinesia.
Answer:.According.to.BMJ.best.practice.Primary.ciliary.dyskinesia.Usually.not.associated.
with. pancreatic. insufficiency;. chronic. purulent. middle. ear. infections,. which. are. less.
common.in.children.with.CF..
Reference:http://bestpractice.bmj.com/bestpractice/monograph/403/diagnosis/differen
tial.html.
..
5). . . . . Month[old#girl#(I#don’t#remember#how#old#exactly),#found#to#have#meningitis#with#
negative#diplococci,#parents#are#also#concerned#about#her#6#year#old#brother.#What#to#
give#her#brother?#
A).....Oral.rifampicin .
B).....Active.immunization .
C).....Ampicillin.&.other.antibiotic .
Answer:.A.
Reference:.https://www.medscape.com/viewarticle/451596_4.
..
6).....A#child#with#history#of#trauma#then#developed#knee#pain#and#swelling#with#
tenderness#on#passive#movement.#What’s#the#next#step?#
A).....Blood.culture.
B).....Empirical.Antibiotics.
C).....Knee.xKray.
D)....Examination.of.synovial.fluid.
Answer:.C.
Reference:.http://bestpractice.bmj.com/bestKpractice/monograph/575/diagnosis/stepK
byKstep.html.
..
7).....Which#of#the#following#deficiencies#is#a#contraindication#for#BCG#vaccine?#
A).....IL.
B).....TNF.
C).....INF.gamma.
Answer:..C..ILK12.deficiency.causes:.low.INF.gamma,.disseminated.mycobacterial.and.viral.
infections..BCG.is.contraindicated..
Reference:.
K.https://www.immunodeficiencysearch.com/interferonKgammaKpathwayKdefects.
K.https://www.ncbi.nlm.nih.gov/pmc/articles/PMC1905729/.
.
8).....Which#of#the#following#is#a#complication#of#mumps?#
A).....Encephalitis.
B).....Hearing.loss.
C).....Facial.palsy.
Answer:.both.A.and.B.are.complications.of.mumps..
Reference:.http://bestpractice.bmj.com/bestKpractice/monograph/1037/followK
up/complications.html.
##
9).....Scenario#of#a#patient#with#sickle#cell#with#a#picture#showing#swollen#foreskin#of#the#
head#of#the#penis?#
A).....Phimosis.
B).....Paraphimosis.
C).....Balanitis.
D)....Priapism.
Answer:.B.
Reference:.https://emedicine.medscape.com/article/777539Kclinical#showall.
..
10)..Tanner#stage#description#in#a#female#(stage#3).#
Answer:#breast.begins.to.become.more.elevated,.and.extends.beyond.the.borders.of.the.
areola,.which.continues.to.widen.but.remains.in.contour.with.surrounding.breast.(11.5–
13).Pubic.hair.becomes.more.coarse.and.curly,.and.begins.to.extend.laterally.(11.5–13).
Reference:.https://www.ncbi.nlm.nih.gov/pmc/articles/PMC2020314/.
..
11)..Child#chewing#his#toys,#associated#with#abdominal#pain,#looks#pale#with#high#
transferrin.#Peripheral#smear:#anisocytosis,#basophilic#stippling#with#target#cells,#
erythroid#hyperplasia#with#maturation#arrest:#
A)Hemolytic.anemia.
B)Sideroblastic.anemia,.
C)IDA.
D)......Thalassemia.trait.
Answer:.B..
Reference:#https://emedicine.medscape.com/article/1389794[workup#c7#
##
12)..Stills#disease#
Answer:#Still's.disease,.a.disorder.featuring.inflammation,.is.characterized.by.high.spiking.
fevers,.salmonKcolored.rash.that.comes.and.goes,.and.arthritis..Still's.disease.is.also.
referred.to.as.systemicKonset.juvenile.idiopathic.arthritis..
Reference:#https://www.medicinenet.com/stills_disease/article.htm#
##
13)..Patient#with#hypotonia,#broad#nose#and#large#forehead,#his#growth#is#...(smaller#
than#peers?).#What’s#the#most#likely#cause?#
A).....Metabolic.
B).....Genetic.
Answer:.B..
K........The.scenario.is.incomplete.however;.the.description.of.dysmorphic.features.goes.
more.with.chromosomal.and.genetic.abnormalities.like.(PallisterKKillian.mosaic.
syndrome:.characterized.by.severe.hypotonia.in.infancy.and.early.childhood,.intellectual.
disability,.distinctive.facial.features,.sparse.hair,.areas.of.unusual.skin.pigmentation,.and.
other.birth.defects..
K........PallisterKKillian.mosaic.syndrome.is.associated.with.a.distinctive.facial.appearance.
that.is.often.described.as."coarse.".Characteristic.facial.features.include.a.high,.rounded.
forehead;.a.broad.nasal.bridge;.a.short.nose;.widely.spaced.eyes;.lowKset.ears;.rounded.
cheeks;.and.a.wide.mouth.with.a.thin.upper.lip.and.a.large.tongue..
Reference:.https://ghr.nlm.nih.gov/condition/pallisterKkillianKmosaicKsyndrome.
..
14)..A#patient#with#seizures#on#anti[epileptic#treatment.#What#will#u#do#regarding#his#
vaccinations?#
A).....Give.all.vaccines.
B).....Don't.give.live.attenuated.vaccine.
C).....Give.all.vaccine.except.DTap.
Answer:.A.
Reference:.https://www.medscape.com/viewarticle/413688.
..
15)..A#32#week#preterm#baby#stayed#in#incubation#for#2#weeks#then#discharged#with#
good#health.#When#to#give#vaccine?#
A).....Correct.for.chronological.age..
B).....Give.vaccine.as.scheduled..
C).....Give.vaccine.half.the.dose..
Answer:.B..Whether.fullKterm.or.preterm,.the.vaccination.schedule.doesn’t.change..
Reference:.https://www.medscape.com/viewarticle/775544_1.
.
16). . Absence# of# moro# reflex# on# the# right# side# of# the# infant# is# due# to?#############################
# #
A).....Intracranial.hemorrhage..
B).....Neonatal.hypoglycemia..
C).....Neonatal.sepsis..
D)....Erb’s.palsy..
Answer:.D.
Reference:.
K. . . . . . . http://bestpractice.bmj.com/bestKpractice/monograph/746/diagnosis/historyKandK
examination.html.
K........https://medlineplus.gov/ency/article/003293.htm.
..
17)..Child#with#DM#type#1.#What's#the#mechanism#of#action#of#the#disease?#
A).....Triglyceride.uptake.
B).....Liver.increase.of.fatty.acid.
.Answer:..
.
18). . A#long#case#with#barking#cough#that#comes#only#at#night#and#resolves.#What’s#the#
diagnosis?#
..
19)..A#baby#has#a#hemangioma.#When’s#the#best#time#to#resect#it?#
A).....1.day.
B).....1.week.
C).....3.months.
D)....6.months.
..
20). . A# baby# fell# from# the# stairs.# He# presented# with# multiple# lacerations# of# different#
degrees#and#a#fracture.#What’s#your#next#step?#
A).....Refer.to.orthopedics.
B).....Call.a.social.worker.
..
21).. Child#woke#up#from#a#nightmare#screaming#and#crying.#Parents#asked#the#child#and#
he#doesn’t#remember#anything#what#sleep#stage?#
A).....1.
B).....2.
C).....3...................................................................................... .
D)....4.
Answer:.C.or.D..These.slow.wave.sleep.parasomnias.include.confusional.arousals,. night.
terrors.(pavornocturnus),.and.sleepwalking.(somnambulism)..
Reference:.https://emedicine.medscape.com/article/1140322Koverview#a4.
..
22). . 3[year[old#child#presenting#with#high#fever#for#the#last#2#days,#vomiting,#refusal#to#
eat#and#red#ears.#Which#of#the#following#will#help#you#find#the#diagnosis?#
A).....CSF.analysis................................................................................. .
B).....Blood.culture...........................................................
C).....Urine.culture.
..
23)..A#mother#brought#her#child#saying#that#he’s#always#sucking#his#thumb#and#when#she#
asks#him#to#stop,#he#doesn’t.#What#are#you#going#to#do?#
A).....Ignore.him.
.
24). . Long#scenario:#18#month#old#baby#presents#with#irritation#and....,#he#was#drinking#
breast#milk#till#the#age#of#9#months.#Then,#he#was#switched#to#cow’s#milk#around#one#
liter#daily.#The#mother#noticed#bad#smelling#stool#for#3#days#with#change#in#consistency.#
The# baby# looks# pale.# Labs# were# provided# and# show# low# Hb.# What# is# the# cause# of# the#
child’s#symptoms?#
A).....Cow.milk.allergy.
B).. Gastrointestinal.infection?............
Answer:..
..
25)..When#to#screen#for#congenital#hypothyroidism?###############################################################################
A).....At.birth.by.cord.blood........................................................................... .
B).....After.1.week.............................................................. .
C).....After.1.month.
D)....After.6.months.
Answer:. updated. AAP. Guidelines. on. Newborn. Screening. and. Therapy. for. Congenital.
Hypothyroidism. recommend. the. screening. of. all. infants. should. be. performed. between.
two. and. four. days. of. birth.. If. this. is. not. possible,. testing. should. be. performed. before.
discharge.or.within.seven.days.of.birth..
..
26). . 24# Year# old# mother# had# a# child# with# suspected# Down# syndrome;# she's# asking# if#
there#is#a#chance#of#another#child#to#have#this#disease.#What#test#will#you#order#to#help?#
A).....Karyotyping.of.child............................................................. .
B).....Karyotype.both.the.mother.and.child.
Answer:. Most. probably. A,. because. you. have. to. karyotype. the. child. first,. then. you.
karyotype.both.parents.depending.on.the.results..
The. extra. chromosome. 21. may. result. from. meiotic. disjunction,. translocation. or.
mosaicism..So,.if.the.karyotype.of.the.child.shows:.
1). . . . . Meiotic.disjunction:.no.need.to.karyotype.parents.and.the.incidence.of.trisomy.
21.is.related.to.maternal.age..
2). . . . . Robertsonian.translocation:.Both.parents.should.be.tested..If.the.mother.is.the.
carrier,.the.risk.is.10K15%,.and.if.the.father.is.the.carrier.the.risk.is.2.5%..If.one.of.the.
parents. carries. the. rare. 21:21. translocation,. then. all. children. will. have. Down.
syndrome..If.none.of.the.parents.are.carry.the.translocation,.the.risk.is.<1%..
Reference:.illustrated.textbook.of.pediatrics.
..
27). . A# child# swallowed# safety# pins.# Abdominal# x[ray# revealed# multiple# pins# in# the#
intestines.#What#are#you#going#to#do?#
A).....Admit.for.observation.&.repeat.xKray.
B).....Discharge.
C).....Exploratory.laparotomy.
Answer:.A..Objects.with.sharp.edges.or.points.present.a.special.problem.because.of.the.
possibility. for. erosion. or. perforation.. These. include. pins,. needles,. tacks,. razor. blades,.
pieces.of.glass,.or.open.safety.pins..Children.who.have.swallowed.such.objects.should.be.
vigilantly.observed..Esophageal.impaction.demands.surgical.removal;.however,.many.of.
these. objects. also. pass. through. the. GI. tract. without. incident. once. they. are. past. the.
gastroesophageal. junction.. Obtain. a. daily. radiograph. (for. radiopaque. objects). and.
monitor.closely.for.signs.of.peritonitis.or.GI.bleeding..In.these.cases,.stools.are.examined.
for.the.foreign.body.in.question..GI.hemorrhage.or.signs.of.peritonitis.mandate.surgical.
exploration.and.removal.of.the.object..
Reference:.https://emedicine.medscape.com/article/933015Ktreatment.
..
28). . A# child# with# drug# overdose# (they# didn't# mention# the# drug).# What's# the#
management?#
A).....Activate.charcoal.
..
29).. A#baby#(I#am#not#sure#about#the#age)#did#not#take#vaccines.#Presented#with#seizure.#
Lab:#high#Ca.#What’s#the#diagnosis?#
A).. Neonate.tetanus.
..
30). . A# mother# came# with# her# boy# for# 6# month# vaccine,# she# mentioned# that# he#
developed#anaphylaxis#after#the#4#month#vaccines#&#needed#hospital#admission.#Which#
of#the#following#is#appropriate?#
A).....Defer.all.vaccines.
B).....Defer.DTP.vaccine.only.
C).....Do.allergy.test.
D)....Give.all.vaccines.&.observe.for.1.hour.
Answer:.C..
Reference:https://www.uptodate.com/contents/image?imageKey=ALLRG%2F81252&topicKey=ALLRG
%2F2074&source=see_link.
..
31). . Child# presented# with# heaves# on# examination.# ECG# shows# RBBB.# On# ECHO:# right#
ventricle# motion# abnormality# and# right# ventricle# hypertrophy.# What# is# the# most# likely#
cause?#
A).....Mitral.prolapse.
B).....ASD.
C).....VSD.
D)....Coarctation.of.aorta.
Answer:.B..
K........Patients.may.have.a.hyperdynamic.right.ventricular.impulse.by.palpation,.especially.
in.older.children.and.adults.with.a.large.leftKtoKright.shunt..
K........ ECG.shows.normal.or.tall.p.waves.indicating.RA.enlargement.or.RKwave.voltages.in.
lead. V1. greater. than. the. upper. limit. of. normal. for. age,. suggesting. right. ventricular.
hypertrophy,.may.be.present.in.larger.defects..
K. . . . . . . . TwoKdimensional. echocardiography. demonstrates. right. atrial. and. ventricular.
enlargement,.as.well.as.the.defect.itself,.especially.for.secundumKtype.defects...
Reference:http://bestpractice.bmj.com/bestKpractice/monograph/1099/diagnosis/stepKbyK
step.html.
##
32). . Child# with# a# history# of# fall# without# losing# consciousness.# He# vomited# twice# and# is#
crying#and#complaining#of#a#headache.#What#should#you#do?#
A).....CT.
B).....Close.observe.
Answer:.A.
Indications.for.CT.scanning.in.a.patient.with.a.head.injury.include.anisocoria,.GCS.score.
less.than.12.(some.studies.suggest.CT.scanning.in.any.pediatric.patient.with.a.GCS.score.
of.<.15),.posttraumatic.seizures,.amnesia,.progressive.headache,.an.unreliable.history.or.
examination. because. of. possible. alcohol. or. drug. ingestion,. loss. of. consciousness. for.
longer. than. 5. minutes,. physical. signs. of. basilar. skull. fracture,. repeated. vomiting. or.
vomiting.for.more.than.8.hours.after.injury,.and.instability.after.multiple.trauma..
Reference:https://emedicine.medscape.com/article/907273Kworkup#c8.
..
33)..An#18#month#old#female#baby#with#anemia.#What’s#the#diagnosis?#INCOMPLETE#
A).....Homozygous.b.thalassemia.
B).....Homozygous.a.thalassemia.
C).....Carrier.of.a.thalassemia.
Answer:.
K. . . . . . . . Alpha. thalassemia. silent. carrier:. Patients. are. likely. to. be. asymptomatic. and.
hematologically.normal..
K. . . . . . . . AlphaKthalassemia. major. or. homozygous. alpha(0). thalassemia:. Hemoglobin. Bart.
hydrops.fetalis.syndrome.is.typically.caused.when.all.4.alphaKglobin.genes.are.deleted..
K........BetaKthalassemia.minor.(commonly.referred.to.as.betaKthalassemia.trait):.usually.
asymptomatic;.the.mild.microcytic.anemia.is.often.misdiagnosed.as.iron.deficiency.
anemia..
K........BetaKthalassemia.intermedia:.usually.a.similar.presentation.to.betaKthalassemia.major.
but.as.a.toddler.or.older.child;.symptoms.are.usually.less.pronounced.and.the.course.is.
usually.more.insidious..
K........BetaKthalassemia.major.(also.called.Cooley's.anemia):.complete.absence.of.
hemoglobin.A;.often.presents.at.a.few.months.of.age.with.progressive.pallor.and.
abdominal.distension;.perinatal.history.is.most.often.uneventful,.and.the.infant.may.be.
pale,.possibly.with.poor.feeding.and.decreased.activity;.hepatosplenomegaly.and.bony.
abnormalities.are.often.present.at.presentation,.most.often.of.the.skull.(frontal.and.
parietal.bossing,.and.chipmunk.facies)..
BUT. (A). is. closer. because. approx.. after. 18. months. HbF. production. reduces. and. HbA.
cannot.be.produced..
Reference:.http://bestpractice.bmj.com/bestKpractice/monograph/251.html.
..
34). . Before# giving# influenza# vaccine# you# have# to# ask# if# the# child# has# an# allergy# from#
which#of#the#following#substances?#
A).....Egg.
Answer:.A.
Reference:.https://www.cdc.gov/flu/professionals/vaccination/vaxKsummary.htm.
..
35).. A#child#with#barking#cough#only.#No#fever#or#shortness#of#breath.#Vitals#are#normal.#
What’s#your#management?#
A).....Moist.oxygen.
B).....Nebulizer.racemic.acid.......................................................................... .
C).....Antibiotics.
Answer:.A..Cool.mist.from.a.humidifier.and/or.sitting.with.the.child.in.a.bathroom.filled.
with.steam.generated.by.running.hot.water.from.the.shower,.help.minimize.symptoms..
Reference:https://emedicine.medscape.com/article/962972Ktreatment.... .
36). . A# patient# with# hematuria# and# upon# examination,# he# has# aniridia# (absence# of# the#
iris).#What’s#the#diagnosis?#
A).....Wilms.tumor.
B).....Neuroblastoma.
Answer:.A.
Reference:.https://emedicine.medscape.com/article/989329Koverview........... .
..
37). . A#child#came#with#abdominal#mass.#Imaging#showed#renal#tumor.#The#patient#has#
absent#iris.#What#is#the#most#likely#diagnosis?................................... .
A).....Wilms's.tumor.
Answer:.A.
Reference:.https://emedicine.medscape.com/article/989329Koverview........... .
..
38). . Pediatric#patient#known#case#of#acute#lymphoblastic#leukemia#(ALL),#presented#to#
the#ER#with#fever#and#pancytopenia#(lab#results#were#provided).#What#is#your#action?#
A).....Blood.transfusion.
B).....Refer.to.oncology.................................................................... .
C).....Start.Antibiotics.
Answer:.C.
Reference:.BMJ.
..
39)..Most#common#presentation#of#congenital#heart#disease?..........................................................
A).....JVD.
B).....Difficulty.feeding.
C).....Ascites.......................................................................... .
Answer:. the. most. common. presentation. of. congenital. heart. disease. is. with. a. heart.
murmur..
Reference:.Illustrated.textbook.of.pediatrics.
..
40)..Which#of#the#following#congenital#heart#defects#is#associated#with#Down#syndrome?#
A).....Endocardial.cushion.defects.such.as.AVSD.
Answer:.A.
Reference:.https://emedicine.medscape.com/article/943216Kclinical#b2.
................................................... .
41).. A#child#with#foul#smelling#breath#and#seed#like#structures#coming#out#of#the#mouth.#
He#is#also#a#mouth#breather.#No#history#of#fever.#What’s#the#most#likely#diagnosis?#
A).....Pulmonary.disease.
B).....Focal.tonsillitis.
Answer:. B.. Individuals. with. acute. tonsillitis. present. with. fever,. sore. throat,. foul. breath,.
dysphagia,. odynophagia,. and. tender. cervical. lymph. nodes.. Airway. obstruction. may.
manifest. as. mouth. breathing,. snoring,. sleepKdisordered. breathing,. nocturnal. breathing.
pauses,.or.sleep.apnea..
Reference:.https://emedicine.medscape.com/article/871977Kclinical.
..
42). . A#2[year#child#came#to#the#clinic#with#his#mother.#He#scribbles#circles,#runs#around#
and#climbs#onto#the#chair,#plays#with#his#friends#but#does#not#share#his#toys.#He#speaks#
10#words.#He#names#the#picture#you#point#to.#What#is#the#best#thing#to#tell#his#mom?#
A).....He.is.normal.
B).....Delayed.social.development.
C).....Delayed.language.development.
Answer:.A..He.is.normal.since.sharing.toys.and.cooperative.play.only.starts.at.4.years.of.
age..
Reference:.Toronto.notes.
..
43)..6#month#old#baby#presented#with#jaundice,#seizure,#irritability#and#vomiting.#He#has#
been# breast[fed# during# his# first# 3# months# of# life,# then# he# has# been# started# on#
commercially#available#milk#formula.#More#recently,#his#mother#introduced#fruit#juices#
into#his#diet.#Investigations#revealed#positive#urine#reducing#substances.#What#element#
should#avoided#in#his#diet?################################################################# #
A).....Galactose..
B).....Fructose..
C).....Phenylalanine..
D)....Irrelevant.choice..
Answer:.B.
..
44). . 4# year# old# child# brought# by# his# parents# to# pediatric# outpatient# clinic# with# them#
complaining# of# his# massive# uncontrolled# appetite# during# the# last# 18# months.# Weight#
was# above# 95th# percentile,# while# height# was# below# 5th# percentile.# Mother# reported#
that#her#son#was#failing#to#thrive#during#his#first#two#years,#beside#the#fact#that#he#was#
developmentally#delayed#compared#to#his#siblings#until#he#caught#up#late.#O/E#he#had#a#
high# forehead,# broad# nose,# small# peripheries# (Hands# and# Feet).# What# is# the# cause#
behind#his#symptoms?####################### #
A).....Genetic..
B).....Metabolic..
C).....Nutritional..
D)....Irrelevant.choice..
Answer:.A.(PraderKWilli.syndrome).
Reference:.Medscape.
..
45).. A#5#year#old#boy#presents#with#tender,#swollen#and#painful#testicle.#O/E#there#was#
absent#cremasteric#reflex.#Which#of#the#following#is#the#correct#diagnosis?#
A).....EpididymoKorchitis..
B).....Testicular.torsion..
Answer:.B..The.most.sensitive.physical.finding.in.testicular.torsion.is.the.absence.of.the....
. cremasteric.reflex..
Reference:.aafp.org.
....................................................................................... .
46). . Pediatric# case# of# Hemolytic# uremic# syndrome:# he# has# petechia,# hematuria# and#
proteinuria.# 2# weeks# ago,# he# develop# bloody# diarrhea.# The# doctor# prescribed#
symptomatic#treatment#and#probiotics.#Platelets#95,#WBC#48.#What’s#your#next#step?#
A).....Antibiotic.
B).....Platelet.transfusion.
C).....Anticoagulant.
Answer:.Treatment.of.HUS:.
K........Avoid.antibiotics.and.platelet.transfusion..
K........Good.hydration.with.IV.fluids..
K........Blood.transfusion.in.anemic.patients.when.indicated..
K........CCB.for.HUS.with.hypertension..
K........Dialysis.for.renal.failure..
K........Renal.transplant.for.irreversible.failure..
Reference:http://bestpractice.bmj.com/bestpractice/monograph/470/treatment/details.
html.
..
47)..Pediatric#patient#has#symptoms#of#epiglottitis#with#stridor#and#distress.#What’s#your#
next#step?#
A).....Antibiotic.
B).....Admit.to.ICU.and.refer.to.ENT.
(No.intubation.in.choices).
Answer:.B.
Reference:.Medscape.
..
48)..Pediatric#patient#comes#with#congested#pharynx,#tonsils#and#plaques#on#the#tongue,#
lips,#and#gingivitis.#No#lesions#on#the#hands#and#feet.#What#is#the#diagnosis?#
A).....Herpes.simplex.virus.
B).....Coxsackie.virus.
Answer:.A..
Reference:.https://emedicine.staging.medscape.com/article/218502Kdifferential.
..
49). . Scenario:#a#patient#developed#bloody#diarrhea,#abdominal#pain#and#vomiting#after#
eating# from# a# restaurant.# History# of# hematuria.# Lab# results# show# anemia.# What# is# the#
diagnosis?########## #
A).....E.coli........................................................... ................
B).....Hemolytic.uremic.syndrome.
Answer:.B..HUS.is.characterized.by.progressive.renal.failure,.microangiopathic.hemolytic.
anemia.(MAHA),.and.thrombocytopenia..In.typical.HUS,.diarrhea.usually.occurs..
Reference:.Medscape.
............... .
50). . What# is# the# treatment# for# the# above# question?.............................
. .
A).....Steroids....................................................................... .
B).....Antibiotics.
Answer:.Treatment.of.HUS:.
K. . . . . . . . Maintenance. of. good. hydration. is. important. to. minimize. the. likelihood. of. renal.
damage.. Careful. attention. needs. to. be. paid. to. avoid. cardiopulmonary. overload,.
especially.because.these.patients.are.at.risk.of.developing.oliguria..
K........ Avoidance.of.antibiotics,.antimotility.(antidiarrheal).agents,.opioids,.or.nonKsteroidal.
antiKinflammatory.drugs.is.advised..
K. . . . . . . . Platelet.transfusions.have.been.associated.with.clinical.deterioration.and.should.be.
avoided.if.possible..
K........For.the.anemia:.blood.transfusion.if.needed..
K........For.the.HTN:.CCB.
K. . . . . . . . If. renal. failure. is. present:. dialysis. is. performed. if. clinically. indicated:. signs. and.
symptoms.of.uremia,.hyperkalemia.(potassium.>6.5.with.ECG.changes),.persistent.severe.
acidosis. (bicarbonate. <10),. hypertension. secondary. to. volume. overload. that. cannot. be.
controlled. with. medical. therapy,. and. necessity. for. transfusion. in. patient. with. volume.
overload.and/or.oliguria..
K........In.irreversible.renal.failure:.renal.transplant..
Reference:http://bestpractice.bmj.com/bestpractice/monograph/470/treatment/details.
html.
..
51)..What#age#in#months#can#a#baby#say#a#few#words?#
A).....6.months.
B).....9.months.
C).....12.months................................................................................... .
D)....24.months.
At.12.months:.the.baby.can.say.2.words.
At.24.months:.2K3.word.phrases.and.uses.“I,.me,.you”..
Reference:.Toronto.notes.......
..............
52). . During# delivery# of# a# baby,# there# was# stylomastoid# foramen# trauma.# Which# of# the#
following#features#will#be#evident#when#you#examine#this#baby?#
A).....Loss.of.eye.close................. .
B).....Loss.of.facial.sensation... .
C).. Loss.of.mastication.function.
Answer:.A.
Facial.Palsy.(Bell’s.palsy):.
K........ It.is.usually.due.to.pressure.by.the.forceps.blade.on.the.facial.nerve.at.its.exit.from.
the.stylomastoid.foramen.or.in.its.course.over.the.mandibular.ramus..
K. . . . . . . . It. appears. within. 1K2. days. after. delivery. due. to. resultant. edema. and. hemorrhage.
around.the.nerve..
K. . . . . . . . Manifestations:. There. is. paresis. of. the. facial. muscles. on. the. affected. side. with.
partially. opened. eye. and. flattening. of. the. nasolabial. fold.. The. mouth. angle. is. deviated.
towards.the.healthy.side..
Reference:.https://www.gfmer.ch/Obstetrics_simplified/foetal_birth_injuries.htm.
............... .
53). . Clear#case#about#crohn’s#disease:#a#child#with#abdominal#cramping,#diarrhea...etc.#
Endoscopy# shows# skip# lesions# and# transmural# inflammation.# What’s# the# diagnosis?#############################
# #
A).....Crohn’s.disease....................................................... .
B).....Ulcerative.colitis.
C).....Celiac.disease.
Answer:.A.
..
54)..Similar#case:#child#with#abdominal#pain#and#tenderness,#bloody#diarrhea#and#weight#
loss.#(no#labs).#What's#the#diagnosis?........................................ ........ .
A).....Crohn’s.disease......................................................................... .
B).....Celiac.disease.
C).....Ulcerative.colitis.
Answer:.abdominal.pain.and.weight.loss.go.more.with.crohn’s,.while.the.bloody.diarrhea.
is.more.with.UC..Most.likely,.crohn’s.since.two.of.the.features.in.the.scenario.support.this.
diagnosis..
.
55)..15#month#old#boy#with#meningitis.#What’s#the#best#antibiotic#choice#for#him?#
A).....Vancomycin.+.ceftriaxone .
B).....Ampicillin.(or.penicillin?).+.gentamicin.
Answer:.A..In.infants.and.children:.Initial.antibiotic.selection.should.provide.coverage.for.
the. 3. most. common. pathogens:. S+ pneumoniae,. N+ meningitidis,. and. H+ influenzae..
According. to. the. 2004. Infectious. Diseases. Society. of. America. (IDSA). practice. guidelines.
for. bacterial. meningitis,. vancomycin. plus. either. ceftriaxone. or. cefotaxime. is.
recommended.for.those.with.suspected.bacterial.meningitis.
Reference:.https://emedicine.medscape.com/article/961497Ktreatment#d10.
..
56)..14#year#old#boy#with#swollen#lips.#Deficiency#of#which#of#the#following#causes#his#
presentation?#
A).....Hereditary.angioedema.
B).....Factor.D.
C).....Anaphylactic.inhibitor.
D)....C1.esterase.inhibitor.
Answer:.D.
Reference:.https://emedicine.medscape.com/article/135604Koverview.
..
57)..A#three#year[old#girl#presented#to#Emergency#Department#with#fever,#vomiting#and#
abdominal#pain#which#began#10#hours#ago.#Radiological#examination#confirmed#a#
dilated#intestinal#pouch#attached#to#the#anterior#abdominal#wall.#Her#diagnosis#was#the#
persistence#of#a#Meckel’s#diverticulum.#
Which#of#the#following#sites#will#the#surgeon#look#for#this#diverticulum?#
A).....Lower.Duodenum.
B).....Lower.Jejunum.
C).....Lower.Ileum.
D)....Cecum.
Answer:.C.
..
58)..Neonate#presents#with#lethargy,#irritability#and#fever.#Which#of#the#following#is#the#
most#likely#causative#organism?#
A).....Listeria.monocytogens.
B).....Staph.aureus.
C).....N.meningitidis.
Answer:.A.
Reference:.Medscape.
..
59)..A#patient#presents#with#a#cough,#fever,#rhinorrhea,#malaise,#with#conjunctival#
suffusion.#There#are#small,#grayish,#irregular#lesions#surrounded#by#an#erythematous#
base,#on#the#buccal#mucus#membrane#near#the#second#molar#teeth.#
What#is#the#most#likely#diagnosis?#
A).....Measles.
B).....Rubella.
C).....Parainfluenza.
D)....Respiratory.syncytial.infection.
Answer:.A.
..
60)..A#baby#with#greasy#scaly#rash#at#the#edge#of#the#forehead#and#over#the#cheeks#not#
sparing#the#folds.#Which#of#the#following#is#the#appropriate#treatment?#(Seborrheic#
dermatitis)#
A).. Muropicin.topical.(antibiotic).
Answer:.LowKpotency.topical.corticosteroids,.such.as.hydrocortisone,.desonide,.and.
mometasone.furoate,.have.shown.to.be.efficacious.on.the.face..Antifungal.therapies.are.
firstKline.therapies.(Ketoconazole,.naftifine,.or.ciclopirox.creams.and.gels).are.effective.
therapies..Systemic.fluconazole.may.help.if.seborrheic.dermatitis.is.severe.or.
unresponsive..Combination.therapy.has.been.recommended...
Reference:.https://emedicine.medscape.com/article/1108312Ktreatment.
..
61)..How#to#manage#croup?#
A).....Nebulized.epinephrine.and.steroid.
B).....Inhaled.salbutamol.and.betamethasone.
Answer:.A.
Reference:.Medscape.
..
62)..Minimum#age#to#give#influenza#vaccine?#
A).....3.months.
B).....6.months.
C).....9.months.
D)....12.months.
Answer:.B..Children.younger.than.6.months.of.age.should.not.be.vaccinated..
Reference:.https://www.cdc.gov/flu/professionals/vaccination/vaxKsummary.htm.
..
63)..Which#of#these#patients#will#most#likely#be#diagnosed#with#rheumatic#fever#from#his#
symptoms?#
A).. Child.with.knee.swelling.and.joint.pain.and.sore.throat..
Answer:.depends.on.the.other.choices,.but.most.likely.A..
Reference:.http://bestpractice.bmj.com/bestK
practice/monograph/404/diagnosis/historyKandKexamination.html.
..
##
64)..Child#with#hip#pain,#x[ray#was#normal#but#US#showed#fluid.#Labs#revealed:#high#ESR#
and#CRP,#otherwise#normal.#What#to#do?#
A).....MRI.
B).....CT.hip.and.pelvis..
C).....Aspiration.
Answer:.most.likely.C.
.
65)..18#month#old#child#took#Hib,#MMRV,#and#...#vaccine#one#week#ago.#He#came#for#HAV#
but#was#not#available.#When#to#give#HAV?#
A).....Immediately....... .
B).....After.1.week.
C).....After.3.weeks.
D)....After.7.weeks.
Answer:.most.likely.A.
..
66)..4#month#old#child#came#for#vaccination#he#has#a#2#day#history#of#watery#diarrhea,#
abdominal#pain#and#vomiting.#What#are#you#going#to#do#regarding#his#vaccination?#
A).....Give.all.vaccines.except.DTap.
B).....Only.give.hep.B.
C).....Give.all.vaccines.except.for.OPV.
D)....Defer.all.vaccines.
Answer:.most.likely.C.
.
.

Family.Medicine.
##
..
Notes:#
K.Drug#for#HTN#reduces#heart#rate#and#peripheral#resistance:.Carvedilol.
K.Drug#for#HTN#reduced#preload#and#cause#vasodilatation:.ACEI.
K.Drug#induced#Hyperpigmentation:.amidarone.
[#Aspirin#SE:.diarrhea.
[#Bisphosphonate#inhibits#osteoclast#activity:.alendronate.
K.Bisphosphonate.works.by.ADP.bla.bla:.coldronate,.etodronate.and.toldronate.
[#Med#for#osteoprosis#causes#heartburn:#risedronate.
[#analgesia#for#cholecystitis:.Meperidine.
K.HgF#inducer:.hydroxyurea,.Na.butyrate,.decitibine.and.5Kazacytidine.
K.Monitor#high#cholesterol:.every.6.months.
[#Borderline#HgA1c;.repeat.after.1.year.
K.Angular#cheilosis:.Vit.b6.
[#Measles:.10K14.D.occupation.
[#Varicella:.10K21.D.
[#Smoking#withdrawal#peak:.3K5.days.
[#Optha#screening#for#DM:#
K.type.1.DM:.annually.beginning.5.years.after.onset.of.DM.
K.Type.2.DM:.annually.beginning.at.time.of.diagnosis.
[#Breast#self#exam:.monthly.
[#Mammogram#normal:.repeat.every.2.years.
[#Most#common#infection#in#saudi#arabia#and#disable#patient#from#work:.HBV.
[#High#cholesterol#pt,#next#follow:.6.months.
[#DM#screening:.if.normal.repeat.after.3.years,.if.preKdiabetic:.yearly..
[#Safe#in#prgenancy:.DTAP,.HBV.and.influenza.
[#HBV:.HBsAg,.then.HBc.Igm,.then.HBs.antibody.
[#Treat#water#from#entameba:.boiling.
[#Cow#milk:.more.in.protein.
[#Parasite#with#meat:.trichinosis.
[#CRC#screening#in#UC.patients.starts.8.years.after.the.Diagnosis..
[#Burkitt#lymphoma:#BCL2.overexpression,.ki67.positive.in.hitsopathology.
K.HIV.CD.&gt;200:.can.be.given.MMR.and.varicell.
1... Researcher#want#to#measure#obesity#in#children#he#included#BMI#and#gender.#what#to#add#
to#asses#risk#of#obesity?#
A... Girth.measurement..
B... HDL/LDL.ratio.
C... Dietary.habits.
D.....Skin.fold.Thickness.
Answer:#D#
BMI.correlates.with.fat.content.measured.by.skin.fold.thickness..
..
2.## Pt#every#menstrual#period#has#depression#even#post#menstrual,#these#problem#are#
continuous#for#6#month,#on#examination#pt#depressed#>>#
Answer:.major.depression.disorders”.and.need.referral.to.psychiatry..
..
3.## About#secondry#prevention#
a).....Kpersonal.education.
b).. Ksecreening.
Answer:.B..
..
5.## nulliparous#came#to#you#age#53#he#mother#got#breast#cancer#when#she#was#38#what#is#the#
most#important#thing#to#check#for#her#abnormal#vaginal#bleeding#?#
Answer:.AUB.is.considered.a.very.alarming.in.postmenopausal.and.highly.suggests.endometrial.
cancer.which.mandates.full.work.up..[.Endometrial.cancer.wkx:..sampling.by.biopsy.or.D&C.+.
hysteroscopy,.pelvic.ultrasound..].All.DDx.in.the.figure.are.important.causes.that.we.need.to.rule.
out..
..
.
.
..
..
.
..
6.## A#man#tested#HIV#+ve.#You#told#him#that#he#should#inform#his#wife#but#he#refused.#What#
should#you#do?#
AKInform.the.ministry.of.health.
BKInform.his.wife.regardless.
CKTry.to.convince.him.to.tell.her..
Answer:#B.#
Reference:.https://www.webmd.com/hivKaids/features/talkKaboutKhivKpositive#1.
..
7.## Best#diagram#used#for#screening#test#?#
Answer:.Nomogram..A.Thorough.search.was.done.without.knowing.the.answer..
..
8.## Best#way#to#treat#water#against#entamebeoa#histolytica#?#
a).. Cholorization.
b).. Boiling.
.Answer:.B..
..
9.## 1st#responsible#for#Bp#regulation#:########################################################################################### #
A............heart.
B............aorta.
C............arterioles............................................ . .
D........... capillaries.
Answer.:... B...
..
High#BP#in#aorta#and#carotid#sinuses#increase#=>##+#Baroreceptor#=>#CVS#centre#in#the#medulla.#
“sympathetic#to#parasympathetic#ratio]######################################################################################### #
Baroreceptor.reflex:.Baroreceptors.in.the.high.pressure.receptor.zones.detect.changes.in.arterial.

pressure..These.baroreceptors.send.signals.ultimately.to.the.medulla.of.the.brain.stem,.specifically.

to.the.rostral.ventrolateral.medulla.(RVLM)..The.medulla,.by.way.of.the.autonomic.nervous.system,.

adjusts.the.mean.arterial.pressure.by.altering.both.the.force.and.speed.of.the.heart's.contractions,.
as.well.as.the.systemic.vascular.resistance..The.most.important.arterial.baroreceptors.are.located.in.

the.left.and.right.carotid.sinuses.and.in.the.aortic.arch.

10.##Pic#of#diagram#showing#progress#of#diabetic#nephropathy#(GFR#and#albuminuria)#cases#in#
years,#then#ask#about#the#specific#point#of#change#
a).....10.year......................................................................................... .
b).....15.year.
c).....20.year.
d).....25.year........................................... ....... .
Answer:.C..
..
..
..
..
..
..
.

.
..
..
.#
#
11.##Which#of#these#vitamins#r#involved#in#degeneration#of#spinal#cord#lead#to#sensory#and#motor#
manifestation#?#
Answer:.Vit.b6,.Vit.b12,.Vit.b1.Vit.E...................................................................................
http://emedicine.medscape.com/article/1152670Koverview#a5.
..
12.##which#one#of#the#following#cancers#directly#related#to#smoking:#
AK.Colon.
BK.Bladder.
CK.Testicular.
DK.Small.cancer..
Answer:#B....
Compared.to.never.smokers,.TCC.risk.was.threefold.higher.in.former.smokers.(95%.CI.2.07K4.18).and.more.
than.sixfold.higher.in.current.smokers.(95%.CI.4.54K9.85)..TCC.risk.steadily.increased.with.increasing.intensity.
(OR.for.≥25.cigarettes/day.8.75;.95%.CI.3.40K22.55).and.duration.of.smoking.(OR.for.≥50.years.5.46;.95%.CI.
2.60K11.49).... .
References:.https://www.ncbi.nlm.nih.gov/pubmed/24964779..................................................................................................
. ................................................................................................ .
13.##Treatment#of#HBV#(started#since#one#month)#
a).....lamivudine.
b).....Interferon.
Answer:.B..
..
..
14.##Dose#of#glucagon#in#IV?#
a).....0.1.
b).....0.2.
c).....1.
d).....2.
Answer:.C...
In.severe.hypoglycaemia.give.1mg.‘1.unit’.and.repeat.q15min.once.or.twice.and.give.dextrose.as.
soon.as.its.available.if.nor.response..
References:.https://reference.medscape.com/drug/glucagenKglucagonK342712.
##
15.##Reason#to#start#screening#for#cancer?#
a).. If.it.can.change.the.natural.history.of.the.disease..
Answer:.A..no.other.options.are.available..
..
16.##Evidence#of#screening#for#hyperlipidemia#in#adults#with#no#cardiac#risk#factors#
Answer:#.not.recommended.if.CHD.do.not.exist..
. .
.

..
.
.
..
17.##how#to#monitor#unfractionated#heparin?#
a).....aptt.
b).....pt.INR.
Answer:.A....
The.most.common.methods.for.monitoring.UFH.are.the.activated.partial.thromboplastin.time.(aPTT).
and.antifactor.Xa.heparin.assay.(antiKXa.HA)..
References:.https://www.medscape.com/viewarticle/746710.
..
18.##Patient#with#small#cell#cancer#grade#III,#developed#back#pain#in#the#last#24#hours,#something#
like#that.#What#to#do?#
a).....MRI.
b).....steroids.and.MRI.
c).....do.nothing.
d).. radiation.
..
Answer:#B.#
In.SCLC.the.most.common.initial.presentation.is.backpain.and.in.patient.with.stage.III.the.
management.is.palliative.or.by.chemotherapy.and.even.if.it’s.an.indication.for.Mets.and.no.
recommendation.for.Steroids.or.MRI.unless.there’s.focality,.and.radiation.is.not.used.in.SCLC.III..
..
.
.
.
..
19.##Colonoscopy#shows#~2#hyperplastic#polyps.#When#to#repeat?#
A............1.
B............3.
C............5.
Answer:.10#years...
hyperplastic.polyps.are.benign...Ref:.http://www.aafp.org/afp/2015/0115/p93.html.
..
20.##23#year#old#man#is#complaining#of#polydpisa#and#polyurea#and#he#is#worry#and#afraid#about#
getting#dm#what#is#the#best#value#for#diagnosis?#Typical#senario#and#answer#
1K.7.7.
2K8.
3K.9.
4K.12.......................................................... .
(UpTodate)............................................................................. .
The.diagnosis.of.diabetes.in.an.asymptomatic.individual.can.be.established.with.any.of.the.following.
criteria:.fasting.plasma.glucose.(FPG).values........................... ....................................................................
. .
≥126.mg/dL.(7.0.mmol/L),.twoKhour.plasma.glucose.values.of.................. . .
≥200.mg/dL.(11.1.mmol/L).during.an.oral.glucose.tolerance.test.(OGTT),.and.A1C.values.≥6.5.percent.
(48.mmol/mol)... . .
..
21.#young#female#came#for#checkup#her#labs#show#
Low#Hbg#
High#MCV#
High#AST#
What#is#the#cause?#
A... folate.deficiency.
B... vitamin.B12.deficiency.
C... alcohol.abuse.
Answer:.C.
not.sure.
..
22.#Colon#cancer#screen#recommended#grad#A#which#age#group:#
AK45K65.
BK50K65.
CK50K75.
Answer:.C.
Not.sure!.
..
23.##70#years#old#professor#presented#with#episodes#of#headache,#all#his#labs#is#normal,#BP#>#140/80#
What#is#the#diagnosis:#
A......Essential.
B......Secondary.
.Answer:A..
24.##Community#medicine#employee#want#to#implement#a#campaign#for#a#leading#cause#of#death#in#
Saudi#Arabia.#What#is#the#appropriate#answer:#
A......Breast.cancer.
B......DM.
C......Coronary.artery.disease.
Answer:.DM.
Ref:.https://www.cdc.gov/globalhealth/countries/saudi_arabia/default.htm.
..
25.##17#yo#athlete#male#gain#7kg#lately#and#he#has#all#characteristic#of#normal#puberty,#he#also#have#
a#foul#smell#breathing?#
AK.anabolic.steroid.
BK.puberty.
CK...
.Answer:.most.likely.A..
..
26.##Scenario#of#a#couple#pre[marital#screening#and#they#poor#blood#labs#for#both,#
Male:#
Hb#low#
Mcv#low#
Hb#A2#more#than#3.5#
Similar#lab#results#for#the#female#
What#is#the#risk#their#child#will#develop#thalassemia?#
1K.25%.
2K.50%.
3K.75.
4K.100.
Answer:A.
..
27.##Middle#aged#man#came#to#PCC#for#regular#check#up.#
He#has#been#smoking#1#pack#a#day#for#40#years.#He#also#consumes#chewed#tobacco.#
Vital#signs:#
BP#=#120/80#
Labs:#
Fasting#blood#glucose#normal#
Lipid#profile#normal#
Renal#function#normal#
What#should#you#screen#for#next#year?#
AK.lung.cancer.
BKHypertension.(correct).
CKDM.
DK.dyslipidemia.
Answer:.A.
Ref:.https://www.cdc.gov/cancer/lung/basic_info/screening.htm.
..
28.##Best#way#for#pain#measurement#in#pt#with#different#language:#
Answer:.Faces.(Drawings).
..
29.##which#of#the#following#supplements#can#decrease#risk#of#some#cancer?#
A............Kfiber.
B............K.vitamin.D.
C............***folic.acid.
.Answer:.
..
30.##The#maximum#accepted#level#of#LDL#in#mmol/l??#
8.8.
..
http://www.cholesterolmenu.com/cholesterolKlevelsKchart/.
..
31.##Pre[diabetic#what#will#be#the#next#visit?#
A... 3.months.
B... 6.months.
C... 12.months.
Answer:.C.
Ref:.http://www.dartmouthKhitchcock.org/endo/preKdiabetes.html.
..
32.##Patient#came#to#the#clinic#and#everything#was#normal#except#HBA1C#which#was#5.9.#When#are#
you#going#to#repeat#and#see#her#again?#
1......3.months.
2......6.months.
3......12.months.
4......36.months.
..
Answer:.6.months.
Ref:.https://www.diabetes.org.uk/professionals/positionKstatementsKreports/diagnosisKongoingK
managementKmonitoring/new_diagnostic_criteria_for_diabetes.
..
33.##Patient#with#high#BP,#otherwise#normal,#when#to#check#BP#again?#
A......6.month,.(3.MONTHS.if.has.a.disease.with.it,.like.HF).
B......1.year.
C......3.years.
.Answer:B.
..
34.##Osteoporosis#most#commonly#due#to?#
Answer:.AgingK.menopause.
.Ref:.https://www.emedicinehealth.com/osteoporosis/page2_em.htm.
..
35.##patient#with#hypertrigelicridemia,#came#with#abdominal#and#back#pain#,his#symptomes#due#to#
?#
Answer:.HypertrigylesrideK.pancreatitis.
..
36.##Pt#on#statin#for#high#cholesterol#,now#hight#trigylesride#,what#to#add#?#
Answer:.Fibrate.
..
37...patient.diagnosed.with.scoliosis.according.to.the.cobb.angle,.when.the.orthopedic.refer.should.
be.done?.
A..5.
B..10.
C..15.
D..20.
.Answer:D.
..
38.##[Man#smoker#,#55#y#,#no#family#history#of#DM#,#his#hemoglobin#A1c#and#blood#sugar#in#the#
border#line#of#being#diabetics#according#to#lap#reading#,#when#to#follow#up#again#?#
3.m.
6.m.
12.m.✅.borderline.
24.m.
36.m.
Answer:.every.12m..
Ref:.http://www.dartmouthKhitchcock.org/endo/preKdiabetes.html.
..
40.##Exercise#to#prevent#osteoporosis#
A... Weight.bearing.exercises.
B......Low.resistance.high.repetitive.
Answer:...
..
41.##The#daughter#of#an#old#age#patient#complains#of#progressively#decreasing#in#memory#and#
change#in#personality#of#her#father,##ttt:#
Answer:.Refer.to.geriatric.clinic.
..
42.##[Treatment#of#refractory#hiccup?#
A... Gabapentin.
B... Chlorapromazine.
Answer:.B.
https://emedicine.medscape.com/article/775746Kmedication.
..
43.##[When#to#screen#for#congenital#thyroid#disease#?#
A... Umbalical.blood.sample.
B... After.3.month.
C... After.year.
.Answer:.A.
..
44.##Want#to#stop#smoking,#but#now#he#wants#to#stop,#which#phase#he#is#in#?#
A............Precontplation.
B............Contplation.
C............action.
D........... Preparation.
Answer:.D.
1..Precontemplation—not.yet.acknowledging.that.there.is.a.problem.
2..Contemplation—acknowledging.that.there.is.a.problem,.but.not.yet.ready.or.willing.to.make.a.
change.
3..Preparation/determination—getting.ready.to.change.behaviors.
4..Action/willpower—changing.behaviors.
5..Maintenance—maintaining.the.behavior.changes.
6..Relapse—returning.to.old.behaviors.and.abandoning.new.changes.
..
45.##Patient#can't#sustain#erection,#he#had#similar#problem#2#years#ago#(or#2[3#months#ago).#he#has#
daily#morning#erection.#
To#whom#you#should#refer#him?#
A..Urology.
B..Psychiatry.
C..Endocrinology.
D..Neurology.
Answer:.B.
..
46.##Which#one#of#the#following#is#the#best#for#screening#male#above#59#years#old?#
1......Highly.sensitive.occult.blood.test.annualy.
2......Sigmoidoscopy.every.3.years.with.Highly.sensitive.occult.blood.
3......Sigmoidoscopy.every.5.years.
4......Colonscopy.every.5.years.
Answer:.
..
47.##Saudi#man#came#for#routine#checkup#what#you#will#screen#him#
A......Hypertension,obesity,DM.
B......Hyperlipidemia,hypertension,obesity.
C......Hypertension,DM,hyperlipidemia.
Answer:.D.?.
..
48.##adolescent#female,#found#to#have#BMI#greate#than#95th#percentile#on#her#routince#visit.#What#
is#the#most#appropriate#action?#
A......avoid.discussion.about.weight..
B......decrease.calories.intake.
C......Interdisciplinary.plan.something.
D.....Tell.her.she.is.larger.than.most.people.her.age..
.Answer:C.
..
49.##If#you#treat#diabetic#with#Vit#D.#What#type#of#prevention#is#this?#
A... Primary.
B... Secondary.
C... Tertiary.
Answer:.B.
..
Q.#Newly#diagnosed#osteoprosis,#you#gave#vitamin#D,#what#is#the#type#of#prevention#?#
.
50.##Baby#with#yellow#teeth#and#dental#carries#in#both#side#
A......nursing.bottle.caries.
B......other.options.
.Answer:..A.
..
51.##Chronic#pain#syndrome#management#?#
NSAID.
..
52.##Term#used#to#call#ppl#who#eat#ice#
K.amylophagia.(starch).
K.Geophagia(dirt/soil).
K.Pagophagia(ice).
Answer:.C.
..
53.##What’s#minimum#volume#of#blood#to#do#a#culture:#
10_20.ml.
..
54.##Patients#have#D.M.#On#metformin#1#g#and#another#anti#dm#present#with#increasing#
blood#glucose#at#morning,#what#you#will#give#:#
A..NPH.
B..Lispro.
C..Regular.
Answer:.A.
.It.depends,..before.breakfast.it.means.fasting,.so.the.answer.would.be.long.acting.insulin,.most.
likely.NPH.or.LANTUS.
..
55.##Female#with#CAD#,#prescribe#for#her#drug#lower#lipid#.#Present#with#facial#flush#,#what#is#the#
drug#?#
K.statin.
K.Nicain.my.answer.
K.Fibrate.
Answer:.statin.
..
56.##what#is#the#MOA#of#glipizide#?#
A..Increase.insulin.secretions.from.pancreas.
Answer:.A.
..
57.##Degree#for#screening#of#abdominal#aortic#aneurysm#for#pt#age#70#y/o#who#never#smoked#?#
..
..
58.##A#16#YO#boy#with#unilateral#gynecomastia,#what#to#advice#him?#
A..reassure.that.most.males.have.this.problem.and.will.disappear.in.a.few.years..
B..Compress.with.bandage.overnight..
.Answer:.A.
Ref:.https://emedicine.medscape.com/article/120858Ktreatment.
..
59.##A#Pt#with#CHF#and#HTN#on#meds,#he#developed#dizziness#every#morning#due#to#a#certain#
medication#and#we#stopped#it,#what#Type#of#prevention#is#this:#
A..primary.
B..secondary.
C..tertiary.
D..quaternary.
My.answer:.B.not.sure.
..
60.##Pt#with#untreated#lower#UTI#for#14#day,#what#is#the#percentage#that#this#infection#will#ascend#
to#the#kidney?#
A... .05.
B... .5.
C... 5.
D.. 50.
.Answer:..
..
61.##Elderly#patient#with#dm,#HTN#,#dyslipdemia#,#what#is#the#target#LDL#level#for#him?#
K.2.3K4.6.
‫ﺻﻞ ﺟﺖ‬$‫ ﺑﺎﻟﻔﻮ‬C‫ ﻏﺮ)ﺒ‬F‫ﻗﺎ‬H$.K...
..
A.target.LDL.cholesterol.level.below.70.to.80.mg/dL.(1.81.to.2.07.mmol/L).is.recommended.for.
people.who.have.CVD.and.have.multiple.major.risk.factors.(eg,.people.with.diabetes.or.who.smoke)..
A.target.LDL.cholesterol.level.less.than.100.mg/dL.(2.59.mmol/L).is.recommended.for.people.who.
have.CVD.but.do.not.have.many.additional.risk.factors..Lifestyle.changes.as.well.as.nonstatin.
medications.may.be.recommended.when.LDL.cholesterol.levels.are.higher.than.100.mg/dL.(2.59.
mmol/L)..
refL.https://www.uptodate.com/contents/highKcholesterolKtreatmentKoptionsKbeyondKtheKbasics.
..
62.##[old#patient#known#to#have#acquired#immunodeficiency#presented#with#cough#and#night#sweat#
,#he#did#mentoux#test#which#was#negative#but#culture#was#positive#for#tuberculosis#what#is#most#
likely:#
A......culture.is.false.positive.
B......mentoux.is.false.negative.
C......mentoux.is.not.a.screening.test.for.TB.
D.....the.patient.should.be.screened.with.heaf.test..
Answer:.B.
https://en.m.wikipedia.org/wiki/Mantoux_test.
.
.
Ethics#and#community#Medicine#
..
1......Patient#trying#to#quit#smoking,#but#he#couldn’t,#he#came#to#you#for#help:#what#behavior#change#
at#this#stage?#
AKprecontemplation.
bKcontemplation.
CKAction.
DKpreparation.
..
Answer:#Preparation.#
. . . . .

. . . . .
. . .

. . .

..
..
..
..
..
..
..
..
..
..
..
..
..
..
..
.
2......A#doctor#is#giving#a#lecture#about#hypertension#and#its#medication,#A#drug#company#
representative#came#to#you:#what#will#you#do?#
##
ANSWER:#Physicians.ought.to.refuse.to.visit.with.representatives.as.a.matter.of.both.professional.
integrity.and.sensible.time.management..
REFERENCES:#https://www.ncbi.nlm.nih.gov/pmc/articles/PMC1466797/#
..
3......What’s#minimum#volume#of#blood#to#do#a#culture:#
..

.
REFERENCES:#
https://www.amc.edu/pathology_labservices/specimen_collection/specimen_col
lection_docs/Blood_Cultures022713.pdf.
##
4......Female#150k.g.#Weight#and#height#160#according#to#BMI?#
AKI.Obesity.
BKII.obesity.
CKIII.obesity.
..
ANSWER:#Class#III.#.Formula.:.kg/.height.squared.meters..[..150/.[.1.6*1.6].=.58].
..
.

.
.
.
..
.
5......Example#of#opening#question?#
AKTell.me.about.the.pain.
..
6......A#type#2#DM#pt#has#an#appointment#in#the#clinic#at#11#o'clock,#the#doctor#came#at#12:30#
because#there#was#a#serious#case#in#the#hospital,#the#pt#was#angry,#what#will#you#do#as#a#doctor#?#
AKApologize.and.expect.that..
..
ANSWER:#.the.answer.depends.on.the.options.provided.in.the.Q.but.the.following.points.should.be.
considered;..Do’s.[.keep.a.safe.distance,.stay.calm.and.cool,..acknowledge.the.emotional.status.“.
angry,anxious.etc”,.acknowledge.legitimacy.“.without.blaming.anyone”,.listen.actively.with.eyeKtoK
eye.contact,.ask.open.Q.“.what.makes.you.feel.so”,.explore.the.reasons.and.possible.ways.of.
treatments.when.suitable,.apologies..when.appropriate].Don’t#[.interrupt,.rude.language,.deny.
reality,.get.angry,.challenge.the.pt.,.defensive.responses.“We.have.been.doing.our.best,.don't.you.
realize.how.hard.we.have.been.trying?.].
REFERENCES:.http://careers.bmj.com/careers/advice/viewKarticle.html?id=1854.
##
7......best#one#considered#as#open[ended#question#in#pt#with#chest#pain?#
AKTell.me.about.the.pain.
BKwhen.did.the.pain.start.
CKwhere.is.the.pain..
..
ANSWER:#Tell#me#about#your#pain.#
##
..
..
..
..
..
..
..
8......definition#of#epidemiology?#
AKDistribution.and.determinant.of.the.disease..
..
ANSWER:..Epidemiology.is.the.study.of.the.distribution.and.determinants.of.healthKrelated.states.or.
events.(including.disease),.and.the.application.of.this.study.to.the.control.of.diseases.and.other.
health.problems..Various.methods.can.be.used.to.carry.out.epidemiological.investigations:.
surveillance.and.descriptive.studies.can.be.used.to.study.distribution;.analytical.studies.are.used.to.
study.determinants..
REFERENCES:..http://www.who.int/topics/epidemiology/en/.
##
..
9......You#got#invited#in#diabetes#campaign,#and#they#offered#you#to#advertise#about#a#drug,#what#to#
do:#
AKAvoid.any.inform.about.drug....
ANSWER:#Physicians.ought.to.refuse.to.visit.with.representatives.as.a.matter.of.both.professional.
integrity.and.sensible.time.management..
..
..
10...Type#of#hepatitis#B#vaccine?#
..
ANSWER:#.two.types.exist;.plasma#derived.[purified.HBsAg.obtained.from.the.plasma.of.persons.
with.chronic.HBV.infection.].,.and.recombinant#vaccines.[use.HBsAg.synthesized.in.yeast.or.
mammalian.cells.into.which.the.HBsAg.gene.(or.HBsAg/preKHBsAg.genes).has.been.inserted.by.
plasmids]..
REFERENCES:.http://www.who.int/immunization/topics/WHO_position_paper_HepB.pdf.
##
11...Which#of#the#following#reduce#cancers?#
AKVit.D.
BKFibers...... .
CKSalt.
ANSWER:#Vitamin#D.#
Å......Molecular,.genetic.and.clinical.data.in.humans.are.scarce.but.they.suggest.that.vitamin.D.is.
protective.against.colon.cancer..
Å......Intake.of.dietary.fiber.is.inversely.associated.with.colorectal.cancer.risk...
REFERENCES:#
Å........https://www.ncbi.nlm.nih.gov/pubmed/22383428.
Å.......https://academic.oup.com/jnci/article/102/9/614/893779/DietaryKFiberKandKColorectalKCancerKRiskKAKNested..
..
12...If#patient#that#you#treated#gave#an#expensive#watch#as#a#gift,#what#will#you#do?#
AKAccept.it.and.tell.to.not.do.it.again.
BKRefuse.it.
CKAccept.it.
ANSWER:#Refuse#it.##.It’s.quite.controversial.and.the.actions.depends.on.the.timing.of.the.gift,.
relationship.with.the.patient,.type.of.gifts.but.usually.for.expensive.gifts.its.best.to.refuse...
REFERENCES:..https://www.ncbi.nlm.nih.gov/pmc/articles/PMC3860914/.
..
13...Most#common#infection#in#saudi#arabia#and#disable#patient#from#work:.
..
ANSWER:#.Hepatitis.B.infection.was.the.most.common.cause.(57.5%),.followed.by.noncommunicable.
diseases.(21.2%).and.hepatitis.C.infection.(17.4%).
REFERENCES:#https://www.ncbi.nlm.nih.gov/pubmed/24975313.
..
14...Treat#water#from#entameba:#
..
ANSWER:#Amebiasis.can.be.prevented.by.increased.sanitation.and.effective.and.safe.disposal.of.
human.excreta..Travelers.should.avoid.unpeeled.fresh.vegetables.and.fruits.and.drink.only.boiled.or.
bottled.water..Avoiding.sexual.practices.that.involve.fecalKoral.contact.can.reduce.infection.in.
homosexuals..In.mental.institutions.recurrent.outbreaks.of.amebiasis.can.be.prevented.by.routine.
screening.of.stool.and.treating.infected.patients..
REFERENCES:.http://www.antimicrobe.org/new/b137.asp.
..
15...Researcher#want#to#measure#obesity#in#children#he#took#BMI#what#else#he#should#take?#
1Kgirth.measurement..
2KHDL/LDL.level.
3Kdietary.habits.
4Kforgot.
ANSWER:#.girth/.waist.measuruments...
References:##https://www.hsph.harvard.edu/obesityKpreventionKsource/obesityKdefinition/howKtoK
measureKbodyKfatness/.
##
..
16...How#to#prevent#plague#
AKKilled.rodent.
ANSWER:#kill.rodents.and.their.habitnant,.wear.gloves.while.touching.infected.animals,..DEET.
repellent.sprays,.flea.control.products.on.animals..
REFERENCES:#https://www.cdc.gov/plague/prevention/index.html.
..
17...Ministry#of#health,#prevent#some#group#of#ppl#to#do#Hajj#&#Umrah#in#2015,#due#to#MERSA[#Co;#
whose#those#population?#
AK.Lactating.women.
BK.Young..kids..>.12.years.
CK.elderly.with.DM.
DK.HBV.
..
ANSWER:#Elderly#+#DM.#Using.a.case–control.design,.we.assessed.differences.in.underlying.medical.
conditions.and.environmental.exposures.among.primary.caseKpatients.and.2–4.controls.matched.by.
age,.sex,.and.neighborhood..Using.multivariable.analysis,.we.found.that.direct.exposure.to.
dromedary.camels.during.the.2.weeks.before.illness.onset,.as.well.as.diabetes.mellitus,.heart.
disease,.and.smoking,.were.each.independently.associated.with.MERSKCoV.illness...
REFERENCES:#https://wwwnc.cdc.gov/eid/article/22/1/15K1340_article.
##
18...A#doctor#is#going#to#present#in#hypertension#conference.#Company#that#produced#new#drug#
suggested#to#sponsor#his#trip.#What#he#is#supposed#to#do:#
AKDisclose.that.he’s.sponsored.
BKAvoid.mentioning.the.new.drug.at.all.
CKReject.the.offer..
..
ANSWER:#Reject#the#offer.#
REFERENCES:#
.
19...Inactive#person,#overweight,#smoker,#44#years#old#man#with#heart#problems,#What#is#the#most#
common#associated#with#heart#disease:#
AKSmoking.
BKObesity.
CKInactivity.
ANSWER:#Smoking..Smoking.is.a.major.cause.of.heart.disease..It.is.estimated.that.smoking.increases.
the.risk.of.stroke,.coronary.heart.disease.and.impotence.by.100%..Smoking.increases.the.risk.of.
death.from.undiagnosed.coronary.heart.disease.by.300%..
REFERENCES:.https://www.worldKheartKfederation.org/resources/riskKfactors/.
..
20...how#to#prevent#MERSA#?#
AKby.hand.washing.
BKvaccine..
ANSWER:#hand#washing.#As.a.general.precaution,.anyone.visiting.farms,.markets,.barns,.or.other.places.
where.dromedary.camels.and.other.animals.are.present.should.practice.general.hygiene.measures,.including.
regular.hand.washing.before.and.after.touching.animals,.and.should.avoid.contact.with.sick.animals.
REFERENCES:.http://www.who.int/mediacentre/factsheets/mersKcov/en/.
..
21...The#daughter#of#an#old#age#patient#complains#of#progressively#decreasing#in#memory#and#
change#in#personality#of#her#father,#ttt:#
AKRefer.to.geriatric.clinic..
ANSWER:#refer#to#geriatric.#
References:#.previous.MCQ’s.
..
22...Important#in#Quit#Smoking?#
AKPatient.desiree..
..
23...what#is#more#risk#for#CAD#
AK55..male.Dm.
BK50.male.hyperlipidemia.
CKHTN+.obese.
AMSWER:##C.#
#REFERENCES:#Toronto.Notes.2016..
.

.
24...What#is#have#more#risk#for#CAD#
AKLDL..
BKHDL.
CKTriglycerides.
DKTotal.cholesterol.
#ANSWER:#LDL.#
#
25...Calculate#disabilities#days#of#URTI#?#
AK292.5.(.correct.)..
..
ANSWER:#.usually.the.number.of.disability.days.is.calculated.by.subtracting.the.number.of.absent.
days.from.the.total.number.of.days.per.year..The.reported.Duration.of.an.URI.episode.is.7.4.days.
with.25%.of.cases.lasting.up.to.two.weeks...
..
26...mammogram#can#detect#breast#cancer#before#clinical#examination#by#how#many#years#?#
AK1.
BK2.
CK3.
DK4.
ANSWER:##2#years..
..
28...Which#of#one#of#the#following#is#considered#as#a##secondary#prevention#method?#
AKpersonal.education.
BKscreening.
ANSWER:#screening.#
..
.
.
..
..
29...commonest#virus#which#make#rejection#of#the#expatriates#or#labors#from#working#in#Saudi?(#
community#)#
a).hbv.
b).hcv.
c).hiv.
Answer:#A#
same.Q.from.smle.13.
References:..http://applications.emro.who.int/emhj/v19/07/EMHJ_2013_19_7_664_670.pdf?ua=1.
..
30...In#a#village#where#the#incidence#of#cretinism#and#iodine#was#less#than#1#microgram#the#health#
promoters#want#to#issue#a#director#for#the#for#the#management#of#those#with#cretinism..#what#is#
the#best#initial#management#
a).. TSH.and.t4.measurements.
b).. Start.thyroxine.medication..
c).. Iodine.supplementation..
..
ANSWER:#B.#
..
..
..
31...varicella#vaccine:#
AK2.doses.6.weeks.apart...
BK2.doses.4.weeks.apart.
..
ANSWER:##.2#doses#4#weeks#apart.#For.children.aged.7.through.12.years,.the.recommended.minimum.
interval.between.doses.is.3.months.(if.the.second.dose.was.administered.at.least.4.weeks.after.the.first.
dose,.it.can.be.accepted.as.valid);.for.persons.aged.13.years.and.older,.the.minimum.interval.between.
doses.is.4.weeks..
REFRENCES:.https://www.cdc.gov/vaccines/schedules/hcp/imz/catchupKshell.html.
..
32...What’s#the#most#effective#way#to#disseminate#health#education?#
AKmass.media.
..
ANSWER:#.Mass.media..
..
33...Best#diagram#used#for#screening#test#?#
Nomogram,.and.other.s.??.
..
ANSWER:.I.couldn’t.find.a.reference.of.any.information.relevant.to.this..
..
..
34...Want#to#stop#smoking,#but#now#he#wants#to#stop,#which#phase#he#is#in?#
AK.Precontemplation.
BK.Contemplation.
CKaction.
DKPreparation.
..
1..Precontemplation—not.yet.acknowledging.that.there.is.a.problem.
2..Contemplation—acknowledging.that.there.is.a.problem,.but.not.yet.ready.or.willing.to.make.a.
change.
3..Preparation/determination—getting.ready.to.change.behaviors.
4..Action/willpower—changing.behaviors.
5..Maintenance—maintaining.the.behavior.changes.
6..Relapse—returning.to.old.behaviors.and.abandoning.new.changes.
..
35...best#parameter#for#screening#?#
AKspecify,.
BKsensitivity.
..
ANSWER:##Sensitivity.#
REFERENCES:.http://sphweb.bumc.bu.edu/otlt/mphK
modules/bs/bs704_probability/bs704_probability4.html.
..
36...Community#medicine#employee#want#to#implement#a#campaign#for#a#leading#cause#of#death#in#
Saudi#Arabia.#What#is#the#appropriate#answer:#
AKBreast.cancer.
BKDM.
CKCoronary.artery.disease.
ANSWER:#.DM..top.ten.leading.causes.of.death.in.KSA.in.order.[.RTA.>.DM.>.back/neck.injuries.>.IHD.

>.depressive.disorders>congenital.anomalies.of.the.heart.>..Drug.use.>.Skin.disorders.>.CKD.>.

cancer..

REFERENCES:.https://www.cdc.gov/globalhealth/countries/saudi_arabia/default.htm.

37...Want#to#stop#smoking,#went#to#hospital#and#asked#for#treatment:#
ANSWER:.action.
..
38...Colon#cancer#screen#recommended#grad#A#which#age#group:#
AK45K65..
BK50K65..
CK50K75..
ANSWER:#C#
REFERENCES:#https://www.cancer.org/cancer/colonKrectalKcancer/detectionKdiagnosisKstaging/acsK
recommendations.html.
..
39...A#man#newly#diagnosed#with#type#2#diabetes#2#weeks#ago,#he#came#for#follow#up#with#you#at#
the#clinic.#You#were#busy#in#the#ward.#His#appointment#is#supposed#to#be#at#10:00#and#now#it#is#
11:15,#the#patient#is#very#angry.#What#should#you#do?#
A).explore.the.reason.of.his.anger.
B).explain.why.you.are.late.
C).show.empathy.to.his.newly.diagnosed.condition..
ANSWER:#A#
..
40...Smoking#withdrawal#peak:#
ANSWER:.3K5.days..
REFERENCES:.https://vapingdaily.com/quittingKeffects/nicotineKwithdrawalKtimeline/.
..
41...Breast#self#exam:#
A... monthly.
ANSWER:#Breast.exams,.either.from.a.medical.provider.or.selfKexams,.are.no.longer.recommended..

REFERENCES:#https://www.cancer.org/latestKnews/americanKcancerKsocietyKreleasesKnewKbreastK
cancerKguidelines.html.
..
42...Blood#preserved#at#22#C#degree?#
ANSWER:#.PRBC;s.is.stored.between.1K6.degrees,.FFP.stored.K18.to.K30,..platelet.stored.at.22.
degrees..
REFERENCES:#https://library.med.utah.edu/WebPath/EXAM/LabMedCurric/LabMed06_02.html.
..
43...Newly#diagnosed#osteo,#you#gave#vitamin#D,#what#is#the#type#of#prevention?#
ANSWER:#tertiary.prevention..
..
44...prevention#of#brucellosis?#
AKPasteurization.of.milk..
ANSWER:#The.most.rational.approach.for.preventing.human.brucellosis.is.the.control.and.elimination.of.the.
infection.in.animals..Pasteurization.of.milk.is.another.protective.mechanism..Vaccination.of.cattle.is.
recommended.for.control.of.bovine.brucellosis.in.enzootic.areas.with.high.prevalence.rates..
REFERENCES:.http://www.who.int/zoonoses/diseases/brucellosis/en/.
.

Research!!
..
..
1... What#is#the#type#of#bias#in#meta[analysis?#
##
A... Tendancy.to.answer.questions.untruthfully.or.misleadingly.on.a.survey.
B.. Recall.retrospective.studies..
C....Loss.of.follow.up.
D.....Publication...
##
Answer:.D.
Reference:.https://www.ncbi.nlm.nih.gov/pmc/articles/PMC3868184/.
..
2... What#is#the#most#common#type#of#study#that#risks#a#recall)bias?#
##
Answer:.Case.control.
Reference:.http://jech.bmj.com/content/58/8/635.
..
3.#What#is#the#most#common#type#of#study#that#risks#non9
response)bias?#
##
Answer:.Cross.sectional.
Reference:.http://www.bmj.com/content/348/bmj.g2573.full.
.
4..What#does#a#confidence#interval#of#95%#mean?#
#
Answer:#A.95%.confidence.interval.reflects.a.significance.level.of.
0.05..If.it.is.hypothesized.that.a.true.parameter.value.is.0.but.the.
95%.confidence.interval.does.not.contain.0,.then.the.estimate.is.
significantly.different.from.zero.at.the.5%.significance.level..
Reference:#https://en.wikipedia.org/wiki/Confidence_interval.
..
.
5..If#a#raesearcher#wants#to#start#a#study#and#wants#only#the#subjects#that#do#not#have#diabetes#to#
participate.#What#will#be#high#in#the#test#?#
Answer:.Specifity.
Reference:#https://www.ncbi.nlm.nih.gov/pmc/articles/PMC2636062/.
##
6..What#is#the#definition#of#epidemiology?#
.Answer:.Epidemiology.is.the.study.of.the.distribution.and.determinants.of.healthKrelated.states.or.
events.in.specified.populations,.and.the.application.of.this.study.to.the.control.of.health.problems.
Reference:#https://www.cdc.gov/ophss/csels/dsepd/ss1978/lesson1/section1.html..
7............ What#does#“p[value”#mean?#
#Answer:.PKvalue.or.probability.value.is.the.probability.for.a.given.statistical.model.that,.when.the.
null.hypothesis.is.true,.the.statistical.summary.would.be.the.same.as.or.of.greater.magnitude.than.
the.actual.observed.results..
##
Reference:#https://en.wikipedia.org/wiki/PKvalue.
#.
8............ What#is#a#Standard#deviation?#
#Answer:#The.standard.deviation.is.a.measure.that.is.used.to.quantify.the.amount.of.variation.or.
dispersion.of.a.set.of.data.values..A.low.standard.deviation.indicates.that.the.data.points.tend.to.be.
close.to.the.mean.(also.called.the.expected.value).of.the.set,.while.a.high.standard.deviation.
indicates.that.the.data.points.are.spread.out.over.a.wider.range.of.values..
Reference:#https://en.wikipedia.org/wiki/Standard_deviation.
##
##
9..Best#test#for#screening?#
#Answer:#sensitivity.#
#.
10...A#study#aims#at#exploring#the#association#of#cigarette#smoking#and#the#risk#of#IHD.#Results#were#
as#follows#(Numbers#are#not#exact,#but#are#used#for#clarification#purposes):#
K.NonKsmokers.OR:.0.1....................................... . .
K.Mild.smokers..OR:.1..
K.Heavy.smokers.OR:.2..
K.Extensive.heavy.chain.smokers.OR:.5..
Which#of#these#is#true#about#this#study#findings?#
A).RiskKAssociation.relationship..
B).DoseKdependant.relationship..
Answer:.B.
Explanation:.The.odd.ratio.is.increasing.as.the.patient.is.smoking.heavier..
................. . .
11...Post#test#probability#of#a#diagnostic#test:#
A..likelihood.ratio.
B..predictive.value.
Answer:.A.
Reference:.https://www.ncbi.nlm.nih.gov/pmc/articles/PMC4025141/.
Explanation:.The.liklihood.ratio.is.a.probability.of.an.individual.without.the.condition.having.the.test.
result..
..
..
..
..
12...At#daycare#center#10#out#of#50#had#red#eye#in#the#1st#week,#another#30#developed#same#
condition#in#the#next#week#.What#is#the#attack#rate?#
20%.
40%.
60%.
80%.
Answer:.80%.
Referance:.https://www.cdc.gov/ophss/csels/dsepd/ss1978/lesson3/section2.html.
Explanation:.Attack.rate.=.Number.of.new.cases.in.the.population.at.risk./.Number.of.persons.at.risk.

in.the.population.

13...long#scenario,#he#wants#the#equation#of#relative#risk:#
Answer:.RR.=.
Reference:.https://en.wikipedia.org/wiki/Relative_risk.
Explanation:#
▪....A.=.The.number.of.people.who.both.had.the.exposure.and.developed.the.disease.
▪....B.=.The.number.of.people.who.had.the.exposure.but.did.not.develop.the.disease.
▪....C.=.The.number.of.people.who.did.not.have.the.exposure.but.did.develop.the.disease.
▪....D.=.The.number.of.people.who.neither.had.the.exposure.nor.developed.the.disease.
..
14...A#study#aims#at#exploring#the#association#of#high#fat#intake#and#prostatic#cancer.,#Group#1#has#
prostatic#cancer#(1000)#patients#with#50#high#fat#intake,#group#2#doesn't#have#cancer#(1000)#pt#with#
10#high#fat#intake#
a)OR.0.52.
b)OR.5.2.
c)RR.0.52.
d)RR.5.2.
Answer:.b.
Reference:.First.aid.
Explanation:.OR.is.typically.used.in.case.control.studies.

.
..
15...In#a#study#they#are#selecting#every#10th#family#in#the#city,#what#is#the#type#of#study?#
A... Systematic.random.study.
B... Stratified.random.study.
C... Non.randomized.study.
Answer:.A.
Reference:#https://en.wikipedia.org/wiki/Systematic_sampling.
##
##
16...In#a#cohort#study#on#lubricant#oil#use#and#urinary#bladder#CA#done#over#20#years#10,000#
exposed#10,000#non#exposed#750#exposed#got#CA#150#non#exposed#got#CA#Then#they#asked#about#
the#incidence#in#1000#in#one#year?#
A... 2.25.
B... 45.
C... .45.
D.. .225.
Answer:.A.
Explanation:.In.20.years:.20,000.patients.were.studied.,.900.cases.of.CA.in.total..

So.in.one.year.900/20=45.case.for.20,000.patients.

For.1000:.45.x1000/20,000=2.25.
17...Lung#cancer#affected#80#of#100#smokers,6#of#500#nonsmokers.#What#is#Relative#risk#reduction?#
A... 33:1.
B... 55:1.
C... 66:1.
Answer:.C.
Reference:#:.https://en.wikipedia.org/wiki/Relative_risk.
##

Explanation:# .
..
18.#Repeated#question#(Question#5)#
..
19...In#a#systematic#review,#which#one#of#the#following#can#be#done#to#decrease#selection#bias?#
A... English.literature.
B... Setting.inclusion/exclusion.criteria.
C... Including.papers.with.positive.outcomes.only.
Answer:.B.
Reference:.https://www.ncbi.nlm.nih.gov/books/NBK126701/.
..
20...Research#question#about#cumulative#incidence:#
Answer:.Number.of.new.cases.of.disease.or.injury.during.a.specified.period.divided.by.Size.of.
population.at.start.of.period.
Reference:.https://www.cdc.gov/ophss/csels/dsepd/ss1978/lesson3/section2.html.
Explanation:.Cumulative.incidence.is.defined.as.the.probability.that.a.particular.event,.such.as.
occurrence.of.a.particular.disease,.has.occurred.before.a.given.time..It.is.equivalent.to.the.incidence,.
calculated.using.a.period.of.time.during.which.all.of.the.individuals.in.the.population.are.considered.
to.be.at.risk.for.the.outcome..It.is.sometimes.also.referred.to.as.the.incidence.proportion..
..
..
..
21...Wha#is#the#definition#of#a#null#hypothesis?#
Answer:.A.null.hypothesis".is.a.general.statement.or.default.position.that.there.is.no.relationship.
between.two.measured.phenomena,.or.no.association.among.groups..
Reference:.https://en.wikipedia.org/wiki/Null_hypothesis.
..
##
22...Definition#of#case#control#
Answer:.A.caseKcontrol.study.is.a.type.of.observational.study.in.which.two.existing.groups.differing.
in.outcome.are.identified.and.compared.on.the.basis.of.some.supposed.causal.attribute..
Reference:.https://en.wikipedia.org/wiki/CaseKcontrol_study.
..
23...What#determines#how#precise#the#study#is?#
A........... P.value.
B........... Confidence.interval.
C........... Relative.risk.
D........... Odds.ratio.
Answer:.B.
Reference:.https://www.ncbi.nlm.nih.gov/pmc/articles/PMC2920077/.
..
..
..
24...A#study#of#something#shows#reduced#risk#of#dm#from#10%#to#5%#what#is#called?#
A... Efficacy.
B... Effectiveness.
#Answer:.B.
Reference:.https://www.ncbi.nlm.nih.gov/books/NBK44024/.
..
25...A#scenario#and#asked#about#sensitivity#&#specificity#(in#statistics)#you#should#understand#the#
definition#to#answer#according#to#scenario#
Reference:.First.Aid.
##
#

#
##
##
26...Exposed#group#3#non#exposed#group#2#which#is#correct:#that#the#question#
#
27...Exposed#group#0.02#non#exposed#group#0.09#
#

#
#
#Reference:.First.Aid.
##
28...Most#normograph#used#for#positive#predictive#value:#
Answer:.Roc.curve.
Reference:.https://www.omicsonline.org/evaluatingKmeasuresKofK
indicatorsKofKdiagnosticKtestKperformanceKfundamentalKmeaningsK
andKformularsK2155K6180.1000132.php?aid=4054.
Explanation:.In.a.ROC.curve.the.true.positive.rate.(Sensitivity).is.
plotted.in.function.of.the.false.positive.rate.(100KSpecificity).for.
different.cutKoff.points.of.a.parameter.
..
29...What#is#a#funnel#plot?#
Answer:#A.funnel.plot.is.a.scatter.plot.of.the.effect.estimates.from.
individual.studies.against.some.measure.of.each.study’s.size.or.
precision..
Reference:#http://www.bmj.com/content/343/bmj.d4002.
Explanation:#The.standard.error.of.the.effect.estimate.is.often.chosen.as.the.measure.of.study.size.
and.plotted.on.the.vertical.axis8.with.a.reversed.scale.that.places.the.larger,.most.powerful.studies.
towards.the.top..The.effect.estimates.from.smaller.studies.should.scatter.more.widely.at.the.
bottom,.with.the.spread.narrowing.among.larger.studies.
##
#
30...What#is#the#graph#that#is#usually#used#to#illustrate#a#relation#between#two#variables?#
#Answer:#Scatter.plot.
Reference:#
http://www.stat.ucla.edu/~rgould/m12s01/relations.pdf.
##
E#
Answer:.c..
References:.
https://www.ncbi.nlm.nih.gov/pmc/articles/PMC3737004/#!p
o=20.5882.
.
..
●......Å.... Different.Qs.about.type.of.study.needed.
●......Å.... About.20Q.about.incedince.and.prevalence.and.relative.risk.
●......Å.... Multiple.qs.about.RR.OR.
●......Å.... Memorize.the.equation.of.Specificity..Few.questions.came.asking.about.it..
..
Notes:#
KStudy.selective.every10th.family.in.city.=systematic.study..
K.non.response.bias.=cs.
KLung.ca.90%.30%.smokers.=70%..
KDadycare.another.developed.same.=80%..
Typhoid.fever. . .
KSensitive.=probability.that.a.disease.pt.have.+ve.test.results.
K.specificty.=.probability.thr.non.disease.have.Kve.test.results..
KBalder.ca.=.2.25..
K..IR.1000/8000*100=12.5..
KCohort.study.+ve.mersa.ration.of.exp.on.non.exp.=66:1.
Risk.exp.to.mersa.Kvir=66.
KLung.ca.=.(80/100)/(6/500)=66:1.
K.RR..=a/(a+b)/.c/(c+d)=.20/(20+80)./494/(494+80)=0..2.
.KInsulin.=near.to.treatment.14.
KCongenital.hearts.KGD.
OR=.(20/980)/(80/4930)=1.255.
RR.=(20/1000)/(80+5000)=1.25.
KGDM..K>.OR.=2.
K..new.case.200.old.case.80.total.280.so.(280/8000000)*100000=3.5.
Kodd.ration.75.control.rate.0.5.
Relative.risk.is..12.
K.group.of.breast.ca..mastectomy.with.radiation.and.mastectomy.with.chemo.=.randomized.
controlled.trial...
KEpidemic.investigations.wt.first.=.identiving.population.at.risk...
..Risk.of.un.exp.2.and.risk.of.exp.3..wt.true.=1.
Bladder.ca.cohort.study.=5.
2.groups.disease.and.non.disease.=.RR=.5.
Incident.of.RS.infection.117.in.1000..=.1.2.
KStudy.compared.and.followed.till.birth.type.=cohort.study.
KResearch.about.creatin.disease.in.the.beginning.were.2000.but.after.3y.were.1000.wt.incidence.in.
one.year.=10%.
KMost.determine.study.=positive.predictive.value..
KStudy.used.to.comparing.2.groups.=.cohort.study.
KStudy.used.to.comparing.2.groups.pregnet.spesfic.diet..=.cohort.study.l2nha.follow.up.pregnancy.
K.Epileptic.pt.on.carbamazepine.=prospective.cohort.study..
K.Red.eye.case.à.80.
K.cumulative.incidence.à.2.5.
K.OR.5.2.(.high.fat.diet.pregnant.ladies.comparison.).
K.Lung.ca.à.70%.
K.MERSKCO.study.Relative.risk.à.66:1.
K.Calculate.disabilities.days.of.URTI.à.292.5.(correct).
*.GDM.and.congenital.malformation.OR.or.RR?.
*.Research.question.with.numbers.0.3.and.0.2..
..
. .
. .
..
.
.
.
.
.
.
.
.
.
.
.
.
.

Emergency.Medicine.
.
.
1[#Aspirin#toxicity?#
Respiratory.alkalosis.and.metabolic.acidosis.
.
2[#Acute#asthma#in#ER,#took#SABA#what#to#do#next?#
A.! Thyophilin.
B.! Inhaled.ipratropium.
Answer:B.
.
3[#Elderly#patient#came#to#the#ER#looking#toxic,#comatose,#and#hypotensive?#
A.! septic.shock.
B.! insulin.overdose.
.
Correct.answer.is.A.
.
4[#Case#of#MG#take#neostagmine#then#present#with#worse#symptoms##to#ER#what#is#the#Rx?#No.IVIG.
in.answers.
A.! Plasmapheresis..
.
Correct.answer.is..
Source:.https://emedicine.medscape.com/article/793136Koverview#a9.
.
Emergency#Department#Care#
Patients.with.myasthenia.gravis.who.are.in.respiratory.distress.may.be.experiencing.a.myasthenic.
crisis.or.a.cholinergic.crisis..Before.these.possibilities.can.be.differentiated,.ensuring.adequate.
ventilation.and.oxygenation.is.important...

Inpatient#Care:#
Plasmapheresis.has.been.found.to.be.an.effective.shortKterm.treatment.of.acute.exacerbations.of.
myasthenia.gravis..Clinical.improvement.takes.several.days.to.occur.and.lasts.up.to.3.weeks..
Because.of.the.delayed.onset.of.beneficial.effects,.plasmapheresis.has.limited.utility.in.the.ED.
setting,.but.often.is.used.in.the.ICU.setting..Immunotherapy.with.intravenous.gamma.globulin.
appears.to.diminish.the.activity.of.the.disease.for.unknown.reasons..The.benefit.begins.within.2.
weeks.and.may.last.for.several.months..The.AAN.considers.IVIG.an.effective.therapy.for.moderateK
toKsevere.cases.of.myasthenia.gravis,.as.per.their.2012.guidelines...
.
5[21#years#old#patient#with#known#case#of#depression,#has#been#found#on#the#floor#unconscious#
with#empty#pill#bottle#,#patient#was#obtunded#,#dilated#pupil#and#unreactive#bilaterally#and#other#
symptom(#I#can#not#remember)##
What#is#the#medication#the#patient#most#likely#on?#
A.! Sertraline.
B.! .Fluoxetine..
C.! Other.SSRI.
D.! Amitriptyline.
.
.
Correct.answer:.D.
.
6[#Abdominal#trauma#with#pancreatic#body#injury,#pancreatic#juice#split#anteriorly#,#where#this#
juice#will#collected#?#
A.! Omentum.bursa.
B.! Left.para.colic..
C.! Sub.hepatic..
.
Correct.answer:.A.
.
7[#What#is#most#common#cause#of#death#in#flame#burn#?#
A.! Hypovolemic.shock..
B.! Inhalation.Smoke..
.
Correct.answer:.B.
Source:.
http://msue.anr.msu.edu/news/smoke_inhalation_is_the_most_common_cause_of_death_in_hous
e_fires.
Smoke.inhalation.is.the.most.common.cause.of.death.in.house.fires.

8[#Patient#came#with#snake#bite#what#is#your#first#action?#
A.! Incision.on.the.site.of.bite.
B.! Mechanical.suction.of.venom.
C.! Immobilize.limb..
D.! tourniquet.application.proximal.to.the.site.of.bite.
.
Correct.answer:.C.
Source:.WHO,.Guidelines.for.the.management.of.snakeKbites.
http://apps.searo.who.int/PDS_DOCS/B4508.pdf.
.
Recommended.firstKaid.methods..
1).Reassure.the.victim.who.may.be.very.anxious...
2).Immobilize.the.whole.of.the.patient’s.body.by.laying.him/her.down.in.a.comfortable.and.safe.
position.and,.especially,.immobilize.the.bitten.limb.with.a.splint.or.sling..Any.movement.or.muscular.
contraction.increases.absorption.of.venom.into.the.bloodstream.and.lymphatics..
3).Avoid.any.interference.with.the.bite.wound.(incisions,.rubbing,.vigorous.cleaning,.massage,.
application.of.herbs.or.chemicals).as.this.may.introduce.infection,.increase.absorption.of.the.venom.
and.increase.local.bleeding...
4).Tight.(arterial).tourniquets.are.not.recommended:.Traditional.tight.(arterial).tourniquets.are.not.
recommended..To.be.effective,.these.had.to.be.applied.around.the.upper.part.of.the.limb.so.tightly.
that.the.peripheral.pulse.gets.occluded..This.method.can.be.extremely.painful.and.very.dangerous.if.
the.tourniquet.was.left.on.for.too.long.(more.than.about.40.minutes),.as.the.limb.might.be.damaged.
by.ischaemia..Tourniquets.have.caused.many.gangrenous.limbs.
.
.
9[#Massive#pleural#effusion?#
A.! Thoracocentesis.
B.! Needle.decompression.
C.! Inhalator.in.high.dose..
.
Correct.answer:.A.
Source:.https://emedicine.medscape.com/article/807375Koverview#a10.
.
Thoracentesis,.which.involves.the.removal.of.50 K100.mL.for.laboratory.analysis,.is.the.first Kline.
invasive.diagnostic.procedure.and.can.be.safely.performed.in.most.patients,.including.those.
undergoing.mechanical.ventilation...
.
11[#patient#came#to#ER#he#was#semiconscious#you#give#naloxone#he#became#conscious#what#is#the#
drug#toxicity?#
A.! .Opioids.
.
Correct.answer:.A.
.
12[#Patient#with#drug#overdose#O/E#there#was#pinpoint#pupils#what#is#the#cause?#
A.! Morphine.
Correct.answer.is.A.
.
13[#Antidote#of#digoxin?##
A.! FAB.immune.globulin..
.
Correct.answer:.A.
Source:.emedicine.
https://emedicine.medscape.com/article/154336Ktreatment.
.
Digoxin.immune.Fab.(Digibind).is.an.immunoglobulin.fragment.that.binds.with.digoxin..It.is.currently.
considered.firstKline.treatment.for.significant.dysrhythmias.(eg,.severe.bradyarrhythmia,.secondK.or.
thirdKdegree.heart.block,.ventricular.tachycardia.or.fibrillation).from.digitalis.toxicity..
.
14[#Patient#with#paracetamol#overdose#you#after#doing#ABC#you#measure#the#drug#level#you#found#
it#toxic#what#is#the#proper#management#?#
A.! .Give.NKacetylcystiene.
.
Correct.answer:.A.
source.:.
http://bestpractice.bmj.com/bestKpractice/monograph/337/treatment/stepKbyKstep.html.
.
.

.
.
.
.
15[#Patient#with#cherry#red#skin#what#is#he#toxic#of?##
A.! Carbon.monoxide.
.
Correct.answer:.A.
.
16[#Patient#collapsed#at#clinic,#there#is#no#sign#of#breathing,#u#call#cardiac#arrest#team,#pt#shows#no#
signs#of#breathing#what#is#next#step?####
A.! Start.chest.compression.30:2.
B.! Give.2.rescue.breathing.
C.! Asses.carotid.pulse.
.
Correct.answer:.C.
.
17[#Patient#came#with#bee#sting,#what#to#do#for#him?#
A.! .Remove.stinger,.and.monitor.the.patient..
B.! Antihistamines..
C.! Epi,.antihistamines,.and.IV.fluid..
D.! Forgot.it..
.
Correct.answer:.A.
Source:.uptodate.
https://www.uptodate.com/contents/beeKandKinsectKstingsKbeyondKtheKbasics.
.
INSECT.STING.REACTION:.
After.being.stung,.you.should.remove.the.stinger.from.your.skin.as.soon.as.possible.to.prevent.any.
more.venom.from.being.released.into.the.skin..However,.all.the.venom.is.released.from.the.stinger.
within.the.first.few.seconds,.so.this.is.only.helpful.if.done.quickly..You.do.not.need.to.use.any.special.
technique.(eg,.flicking.or.scraping).to.get.the.stinger.out..
Most.people.who.are.stung.by.an.insect.will.develop.a.local.reaction.(an.area.of.swelling.and.
redness)..Some.people.will.also.develop.a.severe.allergic.reaction,.called.anaphylaxis..

Local#reaction.—.Immediately.after.being.stung,.most.people.have:.

●Sharp.or.burning.pain.
●Skin.swelling.and.redness..

The.swelling.and.pain.usually.improve.within.a.few.hours..

Local#reaction#treatment.—.To.reduce.pain.and.swelling.after.an.insect.sting,.you.can.try.the.
following:.

●Apply.a.cold.compress.(a.cold,.damp.washcloth.or.damp.cloth.wrapped.around.an.ice.pack).to.
the.area..
●If.you.develop.itching,.you.can.take.a.nonprescription.antihistamine,.such.as.cetirizine.
(Zyrtec)..
●A.pain.reliever,.such.as.ibuprofen.(sold.as.Advil,.Motrin,.and.store.brands),.may.help.reduce.
pain..

Allergic#reaction.—.Insect.stings.cause.allergic.reactions.in.some.people..Symptoms.of.an.allergic.
reaction.usually.develop.quickly,.and.include:.

●Skin.symptoms,.such.as.hives,.redness,.or.swelling.of.skin.away.from.the.area.that.was.stung.
(for.example,.the.face.or.lips.may.swell.after.being.stung.on.the.hand).
●Belly.cramps,.nausea,.vomiting,.or.diarrhea.
●Hoarse.voice,.shortness.of.breath,.and.difficulty.breathing.
●Lightheadedness,.dizziness,.passing.out.
Severe.allergic.reactions.are.called.anaphylaxis..You.can.have.an.anaphylactic.reaction.the.first.time.
you.are.stung..The.first.and.most.important.treatment.for.a.severe.allergic.reaction.is.a.shot.of.
epinephrine..

.
18[Patient#has#hemorrhage,#baroreceptor#activated#which#lead#to#increase#heart#rate,#what#of#
those#will#be#decrease?#
A.! Ventricular.contractility.
B.! Venous.capacitance..
C.! Coronary.blood.flow.
.
Correct.answer:.B.
Source:.SMLE.malzama.12..
http://www.cvphysiology.com/Blood%20Pressure/BP031.
.
The.body.can.quickly.sense.a.fall.in.blood.pressure.through.its.arterial.and.cardiopulmonary.
baroreceptors,.and.then.activate.the.sympathetic.adrenergic.system.to.stimulate.the.heart.(increase.
heart.rate.and.contractility).and.constrict.blood.vessels.(increase.systemic.vascular.resistance)..
Sympathetic.activation.has.little.direct.influence.on.brain.and.coronary.blood.vessels,.so.these.
circulations.can.benefit.from.the.vasoconstriction.that.occurs.in.other.organs.(particularly.in.the.
gastrointestinal,.skeletal.muscle.and.renal.circulations).that.serve.to.increase.systemic.vascular.
resistance.and.arterial.pressure..In.other.words,.cardiac.output.is.redistributed.from.less.important.
organs.to.the.brain.and.myocardium,.both.of.which.are.critical.for.survival..Reduced.organ.blood.
flow.caused.by.vasoconstriction.and.reduced.arterial.pressure,.leads.to.systemic.acidosis.that.is.
sensed.by.chemoreceptors..The.chemoreceptor.reflex.further.activates.the.sympathetic.adrenergic.
system.thereby.reinforcing.the.baroreceptor.reflex..When.the.hypotension.is.very.severe.(e.g.,.mean.
arterial.pressures.<50.mmHg).and.the.brain.becomes.ischemic,.this.can.produce.a.very.intense.
sympathetic.discharge.that.further.reinforces.the.other.autonomic.reflexes..
.
19[Patient#swallows#pins#what#to#do?#
#
20[#Patient#came#to#ER#with#hemorrhage,#hypotensive.#What#will#you#do#next?#
A.! Ringer.lactate.
B.! Packed.RBC.
C.! Blood.
D.! Fresh.frozen.plasma...
..
Correct.answer:.A.
source.:.emedicine..
https://emedicine.medscape.com/article/760145Ktreatment#d11.
.
Once.IV.access.is.obtained,.initial.fluid.resuscitation.is.performed.with.an.isotonic.crystalloid,.such.as.
lactated.Ringer.solution.or.normal.saline..An.initial.bolus.of.1K2.L.is.given.in.an.adult.(20.mL/kg.in.a.
pediatric.patient),.and.the.patient's.response.is.assessed..
.
21[#Patient#18#years#old#was#playing#tennis,#he#presented#to#ER#with#abdominal#pain#(left#para#
umbilical#pain),#without#history#of#trauma.#What#is#your#next#action?.
A.! Erect.chest.xray.
B.! Abdominal.ct.
C.! Kidney.us.
D.! .
Correct.answer:.B.
.
Source:.https://reference.medscape.com/article/776871Kworkup#c5.
CT.is.the.diagnostic.test.of.choice.for.rectus.sheath.hematoma.and.is.superior.to.ultrasonography.in.
sensitivity.and.specificity..Patients.who.are.pediatric,.pregnant,.or.have.renal.insufficiency.may.
benefit.from.ultrasonography.as.a.firstKline.test.to.avoid.radiation.and.intravenous.contrast.material.
.
Ultrasonography.should.be.used.as.a.firstKline.diagnostic.test.in.pediatric.patients,.pregnant.
patients,.or.perhaps.in.patients.with.renal.insufficiency..However,.in.other.patients,.its.primary.role.
may.be.to.follow.hematomas.to.maturation.and.resolution.after.definitive.diagnosis.by.CT.scanning.
.
22[#Patient#presented#to#ER#with#congested#neck#veins,#no#chest#sounds,#difficulty#breathing.#What#
is#your#first#action?#
A.! Needle.thoracostomy....
.
Correct.answer:.A.
Source:.http://www.trauma.org/archive/thoracic/CHESTtension.html.
.
Classical.management#of#tension#pneumothorax.is.emergent.chest.decompression.with.needle.
thoracostomy..A.14K16G.intravenous.cannula.is.inserted.into.the.second.rib.space.in.the.midK
clavicular.line..The.needle.is.advanced.until.air.can.be.aspirated.into.a.syringe.connected.to.the.
needle..
.
23[#Patient#presented#to#ER#with#pneumothorax.#What#is#your#next#action?#
A.! Tube.thoracostomy..
....
Correct.answer:.A.
.
.
#24[#Contraindication#for#gastric#lavage?#
A.! Ingestion.of.detergents.
B.! Ingestion.of.aspirin..
.
Correct.answer:.A.
Source:.https://wikem.org/wiki/Gastric_lavage.
.
Gastric#Lavage#
Contraindications:#
●! Corrosive.ingestions.or.esophageal.disease.
●! The.poison.ingestion.is.not.toxic.at.any.dose.
●! The.poison.ingestion.is.adsorbed.by.charcoal.and.adsorption.is.not.exceed.by.quantity.ingestion.
●! Presentation.many.hours.after.poisoning.
●! A.highly.efficient.antidote.such.as.NAC.is.available.
.
Indications:#
●! LifeKthreatening.poisoning.(or.history.is.not.available).and.unconscious.presentation.(eg.
Colchicine).(Protect.airway.(endotracheal.intubation).if.patient.is.stuporous.or.comatose).
●! LifeKthreatening.poisoning.and.presentation.within.1.hour.
●! LifeKthreatening.poisoning.with.drug.with.anticholinergic.effects.and.presentation.within.4.
hours.
●! Ingestion.of.sustained.release.preparation.of.significantly.toxic.drug.
●! Large.salicylate.poisonings.presenting.within.12.hours.
●! Iron.or.lithium.poisoning.
.
25[#Infant#with#abdominal#colics,#vomiting#and#bloody#stool,#other#data#provided,#diagnosis#is#
intussusception,#the#question#asked#about#the#1st#step#in#management?##
A.! .IV.fluids.and.bowel.rest.
B.! CT.abdomen.
C.! Surgery.
D.! Barium.enema.
.
Correct.answer:.A.
Source:.https://emedicine.medscape.com/article/930708Ktreatment#d10.
.
Tailor.treatment.of.the.child.with.intussusception.to.the.stage.at.presentation..For.all.children,.start.
intravenous.fluid.resuscitation.and.nasogastric.decompression.as.soon.as.possible..
.
26[#A#patient#with#depression#on#medications#found#comatose#with#empty#pill#bottle#beside#her#
On#invx;#ABG#showed#Metabolic#acidosis,#what#drug#overdosed?##
A.! Aspirin..
B.! SSRI..
.
Correct.answer:.A.
.
27[#A#teenager#came#with##abdominal#pain#after#a#basket#ball#game,#onExamination#,#there's#
periumbilical#tenderness.##What#is#the#best#action.#?##
A.! CT.abdomen.
B.! Abdominal.US..
C.! Erect.chest.x.ray..
D.! Re.examine.after.24.hours..
.
Correct.answer:.C?.
.
28[#Clear#case#of#pericardial#tamponade(#Hypotension,#distended#neck#veins#and#muffled#heart#
sounds)#
#
29[#RTA#victim#e#closed#head#injury#and#LOC##
What#is#the#best#thing#to#do#?##
A.! Intubation.and.ventilation..
B.! Check.the.pulses..
C.! Check.the.pupils.
D.! Check.the.airway.
.
Correct.answer:.D.
.
#30[#RTA#victim#was#brought#to#hospital#with#multiple#injuries,#transferred#to#ICU.#There,#he#
developed#multi#organ#failure,#the#phosphate#level#in#his#blood##is#raised#,#what#is#the#cause#of#his#
multi#organ#failure?#
A.! Heart..
B.! Lungs.
C.! Kidneys..
D.! Liver..
.
Correct.answer:.A.
.
#31[#2#cases#about#iron#toxicity.#Best#way#to#decontaminate#the#stomach#of#a#patient#with##iron#
poisoning?#
A.! Gastric.lavage,..
B.! Syrup.epicac.,..
C.! Something.called.tacren.I'm.not.sure.about.it.and.last.option.was..
D.! induced.vomiting,..
.there.was.no.activated.charcoal.in.the.choices...
.
Correct.answer:.A.
Source:.https://emedicine.medscape.com/article/815213Ktreatment#d10.
.
Gastric.lavage.with.a.largeKbore.orogastric.tube.may.remove.iron.from.the.stomach..Ideally,.lavage.
should.be.performed.1K2.hours.postingestion,.although.later.use.may.be.appropriate.if.evidence.of.
iron.products.in.the.stomach.is.observed.on.a.radiograph..However,.iron.has.a.gelatinous.texture.
and.may.be.difficult.to.remove.by.lavage...
The.American.Academy.of.Clinical.Toxicology.advises.that.the.routine.administration.of.ipecac.in.the.
emergency.department.should.definitely.be.avoided...
Activated#charcoal#does#not#bind#iron.#However,.it.should.be.utilized.if.coKingestants.are.suspected..
In.acute.or.chronic.iron.toxicity,.chelation.therapy.with.deferoxamine.is.indicated.for.patients.with.
serum.iron.levels.>350.mcg/dL.who.have.evidence.of.toxicity,.or.levels.of.>500.mcg/dL.regardless.of.
signs.or.symptoms..In.patients.with.significant.clinical.manifestations.of.toxicity,.chelation.therapy.
should.not.be.delayed.while.one.awaits.serum.iron.levels..
32.KIron.poisoning,#a#child#swallowed#a#bottle#filled#iron#pills,#presented#to#ER#e#abdominal#pain,#
nausea#and#confusion.#
Best.way.to.manage.is.?#
A.! IV.deferoxamine.
.
Correct.answer:.A.
Source:.https://emedicine.medscape.com/article/815213Ktreatment#d10.
.
In.acute.or.chronic.iron.toxicity,.chelation.therapy.with.deferoxamine.is.indicated.for.patients.with.
serum.iron.levels.>350.mcg/dL.who.have.evidence.of.toxicity,.or.levels.of.>500.mcg/dL.regardless.of.
signs.or.symptoms..In.patients.with.significant.clinical.manifestations.of.toxicity,.chelation.therapy.
should.not.be.delayed.while.one.awaits.serum.iron.levels..

33.KPatient.drank.an.ethylene.glycol.containing.product.,what.renal.damage.is.suspected?##
A.! Acute.tubular.necrosis.(ATN)..
B.! .
Correct.answer:.A.
Source:.https://en.wikipedia.org/wiki/Acute_tubular_necrosis#Toxic_ATN.
.
Toxic.ATN.can.be.caused.by.free.hemoglobin.or.myoglobin,.by.medication.including.antibiotics.such.
as.aminoglycoside,.statins.such.as.atorvastatin,.and.cytotoxic.drugs.such.as.cisplatin,.or.by.
intoxication.(ethylene.glycol,."antiKfreeze")..
.
34[#Trauma#to#abdomen#Omentum#out.#What#is#the#management?#
A.! immediate.laporatomy..
.
Correct.answer:..A.
Source:.uptodate.
http://cursoenarm.net/UPTODATE/contents/mobipreview.htm?23/36/24143.
.
Indications.for.emergent.laparotomy.include:.hemodynamic.instability,.unequivocal.peritoneal.signs.
on.physical.examination,.signs.of.gastrointestinal.hemorrhage,.and.implement.in.situ..Evisceration.of.
bowel.or.mesentery.is.an.indication.for.immediate.laparotomy.at.most.trauma.centers.
.
35[#What#is#the#maximum#duration#that#N#acetylcystein#can#be#administered?.
A.! Given.after.8.hrs..
B.! ..
NAC.is.most.effective.in.managing.paracetamol.toxicity.when.administered.in.the.first.8.hrs...
Source:bmj.
..
..
36[#MVA#injury#to#jugular#Foramen#which#muscle#will#be#still#functioning#?.
A.! sternocleidomastoid.muscle.
B.! hypoglossus.muscle.
C.! stylopharyngus.muscle.
D.! Trapazius..
.
.Correct.answer:.B.
Hypoglossus.muscle.is.supplied.by.XII.nerve,.which.does.not.pass.through.the.jugular.foramen..
..
..
37[#Patient#with#organophosphate#poisoning#what#to#give#?.
A.! Atropine..
B.! Nalxoprine.
C.! NKActylecystein..
D.! .
Correct.answer:.A.
The.antidote.for.organophosphate.toxicity.is.Atropine.&.pralidoxime..
..
38[#Child#swallowed#battery,#presenting#with#increase#salivation.#What#is#the#first#step#in#
management?.
A.! XKray.abdomen,.chest,.and.neck.
B.! Upper.GI.endoscopy.
C.! Watchful.waiting.for.24.hours.
.
Correct.answer:.A.
Source:.http://adc.bmj.com/content/archdischild/61/4/321.full.pdf.
..
39K.What.to.give.after.CPR.in.a.child.with.asystole?.
A.! Atropine.
B.! Epinephrine.
.
Correct.answer:.B..
Source:.AHAK.pediatric.cardiac.arrest.algorithm..
.

40[#Dehydrated#child#(I#think#moderate)#with#severe#vomiting.#Management#?.
A.! Pedialyte.10.ml.over.5.min.
B.! Normal.saline.50ml/kg.
C.! ORS.with.ratio.of.1:1.glocose.to.sodium.
D.! .
Correct.answer:.B.
Source:.clinical.emergency.medicine...
.

41[#Patient#with#tachycardia#and#SOB#after#bee#sting#what#is#the#"OPTIMAL"#treatment:.
A.! Remove.stinger.and.watch.the.pt.
B.! SC.epinephrine.and.IV.antihistamine..
C.! IM.epinephrine..
.
Correct.answer:.C..
The.initial.management.include.IM.or.SC.epinephrine.,.while.the.optimal.treatment.include.
monitoring.the.patient,..H1.antagonist,.H2.antagonist.,.corticosteroids..
Source.:.Toronto.notes..
.
42[#Man#with#aortic#stenosis#develop#syncope#what#is#the#cause#:.
A.! Systemic.hypotension.
.
43[#women#found#unconsious#and#empty#bottle#of#medication#near#to#her#and#she#was#diagnosed#
with#depression#previously#on#exam#dilated#unresponsive#pupils#what#is#the#drug:#
A.! acetylcholine..
B.! Dopamine.
C.! serotonin..
.
Correct.answer:.C.or.TCA..
Dilated.pupils.are.more.diagnostic.of.anticholinergic.syndrome.than.serotonin.syndrome...
Source:.bmj.....
.
44[#Patient#presented#to#ER#with#hx#of#drug#overdose#for#last#8#hours##on#the#examination#the#gag#
reflex#was#absent#.#the#best#management#is?.
A.! Iv.naloxone.
B.! Immediate.endortracheal.intubation*.
C.! Gastric.lavage.

Correct.answer:.B..

Source:.https://www.ncbi.nlm.nih.gov/pmc/articles/PMC1671555/pdf/bmj00153K0032.pdf.

45[#70#Y/O#came#with#urinary#retention#,1st#step#?.
A.! Urinary.catheter.
.
46[##29#year#old#lady,#presented#to#the#ER#with#nausea#vomiting,#generalized#rash#with#
desquamation#and#bilateral#conjunctivitis#and#vaginitis.#No#history#of#travel.#Past#medical#
unremarkable#except#for#vaginitis#a#few#days#back.#Which#of#the#following#is#the#most#likely#
diagnosis?.
A.! Toxic.shock.syndrome.
B.! Meningiococcal.septicaemia.
.
Correct.answer:.A.
.
47[#Elderly#with#fever,night#sweats,##and#loss#of#weight#what#you#will#do#in#the#ER?.
A.! Isolation.in.negative.pressure..
B.! CXR.
C.! Tb.medication.
D.! .
Correct.answer:.A..
.
.
48[#Methotrexate#antidote?.
A.! Folinic.acid.
.
Correct.answer:A..
.
49[#Acetaminophen#antidote?.
A.! NK.actylecyctin..
.
Correct.answer:.A.
.
50[#Alcohol#abuse#antidote?.
naltrexone.or.disulfiram..
.
51[#What#drug#reverses#the#effect#of#Benzodiazepines?.
A.! Flumazanil..
.
Correct.answer:A.
.
52[#30#yrs#old#male#presented#to#you#after##45#minutes#from#ingesting#multiple#paracetamol#,#he#is#
awake#alert#and#not#complaining#of#anything,#what#is#your#next#step?.
A.! N.acetylcysteine.
B.! Activated.charcoal.
C.! Gastric.lavage.
D.! Serum.acetaminophen.concentration.
.
Correct.answer:.D.
When.suspecting.paracetamol.toxicity,.serum.concentration.should.be.checked,.especially.at.the.
earliest.4.hrs.after.ingestion...
Source:bmj.
.
53[#Comatose#patient#came#to#the#ER#the#1st#thing#to#do?.
A.! Intubation.
.
54[#Boy#coming#to#the#ER#complaining#on#the#right#side#,examination#all#normal#except#
hypertensive#,#one#day#before#he#is#fallen#on#right#side##what#is#the#most#organ#involved#?#(not#
clear).
A.! Right.kidney.
B.! Liver.
.
Correct.answer:.A.(.NOT.SURE)..
.

55[##Patient#comes#to#you#with#drooling#saliva#constricted#pupil#same#scenario#almost#as#in#notes#
what#to#do ?.

A.! IV.atropine.
B.! Lavage..
Correct.answer:.A..

Constricted.pupils.and.drooling.are.parts.of.organophosphate.toxicity.syndrome,.which.is.managed.
with.atropine...

.
56[#20#years#old#student#came#to#you#lethargic#change#of#sense#of#time#blood#test#normal#and#
waiting#for#urine#test#.#What#is#the#cause?.

A.! Caffeine ..
B.! Cannabis..
C.! Cocaine..
D.! Opioid.
Correct.answer:.B.
.
57[#Overdosed#drug#caused#resp#depression?#

opioids#

58[#Overdosed#lead#to#comatose?#

59[#Overdosed#lead#to#dilated#pupils?#

Anticholinergic.drugs..

60[##Food#allergy#with#symptoms#of#anaphlyctic#shock...

61[#Pscych#pt#swallowed#safety#pin#reached#dudenum#what#to#do#

62[#.Organophosphate#poisoning#case#.#Tx?.
A.! Praldoxime.
.
63[#.Dilated#pupil,#no#neuro##deficit?.
A.! TCA.
.

64[#Which#of#the#following#is#the#diagnostic#test#for#tension#pneumothorax?.
A.! CXR.
B.! CT.
Correct.answer:.A.
.
65[#Patient#came#with#the#presentation#of#excess#salivation#and#difficult##breathing#due#to#
respiratory#secretions.#The#patient#has#a#history#of#pesticide#exposure#and#now#he#also#have#a#(#
garlic#odor)#What#you#will#give:#(ORAGANOPHOSPHATE#POISINING).
A.! Nesostagmine..
B.! Pralidoxime.
C.! .
Correct.answer:.B..
Atropine.and.pralidoxime.are.antidotes.of.organophosphate.toxicity..
.
67[#How#to#treat#alcohol#withdrawal#(#no#diazepam#in#the#answers)?.
A.! Disulfiram.
B.! Propranolol.
C.! Chlorodizepoxide.
.
Correct.answer.:.C...
Chlorodizepoxide.(it.is.benzo.group).
Disulfiram.is.used.is.alcohol.dependence....
.
68KSign#of#uncomplicated#pneumothorax?#
(.decreased.breath.sounds.and.hyperresonance.over.the.affected.side,.mediastinal.shift.toward.the.
side.of.pneumothorax)..
.

69[#17#years#old#from#MVA,#she#was#intubated#but#then#she#was#severely#hemodynamically#
unstable.#What#sign#will#warrant#you?.
A.! Engorged.IJV.
B.! Shifted.trachea.
. . . . . .
Hypotension.Resistant.ventilation.
. . . . . .
REF:.Deterioration.after.intubation.K.DOPE:.
●! Displaced.ETT. . . . . .. . .
●! Obstruction.(anywhere.along.circuit).
●! Pneumothorax.
●! Equipment.failure.(ventilator.malfunction.or.disconnect).
.
70[#Patient#present#to#ER#intensive#knee#swelling#with#ballottement#pattern#next#step?.
A.! ESR.
B.! Arthrocentesis.
C.! CBC.
Correct.answer:B.
.
70[#23#yo#with#agitation,#dilated#pupil,#diaphorisis#,#tachycardia#,#what#is#the#toxicity?.
A.! TCA.
B.! Digoxin.
C.! Sympathomymetic..
D.! Anticholenergic.
.
Correct.answer:.C.
. . . . . . . . . . .
72[#Female#taking#paracetamol#500mg#bid#daily#presented#with#liver#enzymes#elevated#and#
hepatomegaly?.
A.! Alpha.1.antitrypsin.deficiency.
B.! Liver.damage.due.to.paracetamol. . . . . .
C.! Hepatitis.i.guess.
.
Correct.answer:.B.(not.sure)..
Paracetamol.ingestion.may.cause.hepatocyte.death.which.causes.acute.liver.failure...
Source:.BMJ.
.
.
.
73[##Treatment#of#MS#attack#in#ER:.
A.! Oral.antibiotic.
B.! IV.antibiotic.
C.! Oral.steroid.
D.! IV.steroid.
.
Correct.answer:.D..
Source:.step.up.to.medicine..
.
74[#Patient#take##a#lot#of#sleeping#drugs##unresponsive#pulse#rapid#weak,#show#reflex#gasping#
breathing,#give#2#breathing#by#mask,#next#step?.
A.! Wait.for.blue.team..
B.! Perform.CPR..
C.! Intubate..
.
Correct.answer:.C..
CPR.is.not.indicated.if.there.is.pulse...
Source:.AHA.
.
75[#Case#scenario#about#patient#with#basal#skull#fracture#what#expected#nerve#injury?.
A.! Olfactory.
B.! Optic.
C.! Oculomotor.
.
Correct.asswer:A.
.
76[#Unconscious#patient#with#agonal#gasps?.
A.! check.carotid.pulse.
B.! Start.CPR.
C.! (Rescue.breaths?).
.
Correct.answer:.B..
Agonal.gasping.(.breathing).is.associated.with.cardiac.arrest...
Source:.AHA.
.
77[#Epileptic#pt#with#trauma?.
A.! Posterior.dislocation.of.shoulder.
.
78[#RTA#victim#come#with#closed#head#injury#and#LOC.#What#is#the#best#thing#to#do?.
A.! Intubation.and.ventilation.
B.! Check.pulse.
C.! Check.pupils.
D.! Check.airway.
E.! .
Correct.answer:.D.
Head.injury.with.LOC.requires.immediate.intubation..
Source:.Toronto.notesK.traumatology..
.
.
.
79[#A#child#brought#to#ER#with#barking#cough,#red#epiglottis,#thumb#sign#on#x#ray,#
Best#initial#management?.
A.! examination.of.epiglottis..
B.! Endotracheal.intubation...
C.! Emergent.tracheostomy.
D.! Nasopharyngeal.Tube.
.
Correct.answer:.B..
.
.
80.#What#is#your#concern#with#the#following#when#giving#a#patient#opioids#(patient#on#opioid,#
contraindicated#drug#is)?.
A.! Aspirin.
B.! Antidepressants.
C.! NSAID.
D.! Laxative.
.
Correct.answer:.B.
Source:.https://www.healthline.com/healthKnews/fdaKwarningKinvolvingKopioidKprescriptionsK
antidepressants#2.
.
81.#paracetamol#antidote?.
A.! Acetylcysteine.
.
82.#When#to#give#paracetamol#antidote#(#maximum#or#something)?.
A.! 2h.
B.! 4h.
C.! 6h.
D.! 8h.
.
Correct.answer:D..
NAC.is.most.effective.in.managing.paracetamol.toxicity.when.administered.in.the.first.8.hrs...
Source:bmj.
.
83.#20#yrs#girl#come#to#hospital#after#20#mins#of#complain#of#light#headiness#and#headache,....When#
she#went#to#the#doctor#she#start#complain#of#peri#oral#numbness#and#fingers#tingling.#What#to#do?.
A.! alcohol.leveling.................................................................................. .
.................................................. .
84.#Side#affects#of#atropine:.
A.! Seizure.
B.! Vomiting.
C.! Dry.mouth.
.
Correct.answer:C.
.
85.#SE#of#morphine:.
A.! N/V.
.
86.#1st#symptom#in#hypomagnesemia:.
A.! Decreased.deep.tendon.reflexes..(hypermagnesemia).
B.! Respiratory.depression..(.hypermagnesemia).
C.! .
Hypomagnesemia.:.(muscle.twitch,.weakness,.tremor,.hyperreflexia,.seizure.)..
.
87.#What#drug#is#likely#to#cause#heat[stroke#as#it#inhibits#sweating:.
A.! Hyoscamine.Sulfate.
B.! .
Correct.answer:.A.
https://www.drugs.com/pro/hyoscyamineKsulfateKtablet.html.
.
88.#Usually#if#there#is#fentanyl#overdose#what#will#give?.
A.! naloxone..
.
89..Patient#admitted#to#ER,#she#denies#eating#and#says#she's#not#hungry,#BMI#11.3#What#most#likely#
to#find#in#her#labs:##########................................................... .
A.! inc.K..................................................... .
B.! Dec.Creatinine................................................................ .
C.! Other.labs.
D.! .
Correct.answer:B..
.
GCS#
.

.
.
.
.
.
# APGAR#SCORE
.

.
.
.
. .
.
.
.

OB/GYN.
#
.
1.What#indicates#the#labor#progress?#
a..Descent.of.the.presenting.part.
b..Head.manipulation.
c..Strength.of.uterine.contraction.
d..Frequency.of.uterine.contractions.
Answer:.A.
.
2.#Which#OCP#causes#hyperkalemia#
a..EstradiolKlevonorgestrel.
b..EstradiolKdrosperirenone.(.yasmin.).
Answer:.B.(https://reference.medscape.com/drug/yasminKyazKdrospirenoneKethinylKestradiolK
342768#4).
.
3.#When#to#do#screening#for#bacterial#vaginosis?#
a..All.trimester.
b..First.trimester.
c..Second.trimester.
Answer:.GBS.screening.at.35K37.weeks.gestation..
.
4.#40#y/o#Female#preconception#counseling,#hx#of#fetal#death#after#delivery#with#neural#tube#
defect,#what’s#your#best#advice?#
a..It.could.occur.because.of.your.age.
b..Cvs.at.week.16.
C..Folic.acid.supplementation..
Answer:.C?.
.
5.#Case#about#threatened#abortion.(hint.is:.POC.is.intact,.and.cervix.is.closed)..
.
6.#MgSO4#is#given#in#pregnancy#in#case#of#preeclampsia#for#what?#
Answer:.To.reduce.the.risk.of.seizures..
.
7.#Greenish#foul#smelling#vaginal#discharge,#oE/#cervix#e#dots#and#erythema#(the#q#provided#the#
description#of#strawberry#cx)#
Answer:.trichomoniasis..
.
7[#Same#scenario#(trichomoniasis),#what#is#the#treatment?#
AK.Cerftriaxone..
BK.Metronidazole.
Answer:.B.
.
8.#One#of#the#following#is#seen#in#bacterial#vaginosis#
a..No.itching.or.burning.
b..Yellow.discharge.on.Vaginal.PH.paper.
c...Greenish.discharge.
Answer:.A..
itching.could.be.present,..
They.will.have.Alkaline.pH,.in.pH.paper.the.color.of.the.paper.changes.from.bright.yellow.at.pH.4.5.
and.lower.to.dark.blue.at.pH.7.0.and.higher...
Characteristic.discharge.appearance.is.thin,.gray,.and.homogeneous..Positive.whiff.test.and.clue.
cells.
.
9.#Young#female#with#fishy#odor#vaginal#discharge#and#positive#whiff#test#
a..B.vaginosis.
b..Trichomoniasis.
c..Candidiasis.
d..Chlamydia.infection.
Answer:.A..
.
10.A#female#came#to#GYN#clinic#with#post#coital#bleeding,#Source#of#bleeding#is#
a..cervix.
b..uterus.
c..vulva.
d..vagina.
Answer:.A.
.
11.#Couples#are#said#to#be#infertile#after#marriage#and#should#seek#for#fertility#after#
a..3.M.
b..6.M.
c...9.M.
d..12.M..
Answer:.D.
.
12.#Young#couples#trying#to#conceive#for#6#months,#both#are#healthy#
Answer:.Try.some.more..
.
13.#38#wks,#multiparous,#with#a#trial#of#ECV#on#week#36,#her#amniotic#fluid#index#is#12,#now#her#
fetus#is#in#lateral#position,#her#membranes#ruptured#with#clear#fluids,#What#would#be#the#
indication#for#CS?#
a..Repeated.ECV.
b..amniotic.Fluid.index.
c..ROM.
d..Fetal.position.
e..Failed.first.attempt.
Answer:.D.
.
14.#Patient#with#DUB,#what#to#give?#
Answer:.OCP.(progesterone).
.
15.#Young#age#married#woman#trying#to#conceive#for#1#year#but#failed,#she#has#some#facial#hair#and#
hoarseness#of#voice,#her#husband#semen#analysis#is#normal,#what#is#the#ttt#
a..Laparoscopy.
b..Laparotomy.
c...Clomiphene.citrate.
Answer.:.C.
.
16.#Post[menopausal#woman#presented#with#vaginal#bleeding#cervical#and#vaginal#exam#normal#
except#for#blood#at#the#cervical#os#what#is#ur#next#step:#
a..Endometrial.sampling.
b..Pelvic.ultrasound.
c..Bhcg.
d..Ca.125.
Answer:.A.(https://emedicine.medscape.com/article/258148Kworkup#c7).
.
17.#Which#can#cross#placenta#and#cause#deafness#and#heart#problems?#
a..Rubella.
b..Measles.
c..Mumps.
d..HBV.
Answer:.A.
.
18.#30#year#old,#did#HPV#testing#and#pap#smear.#When#is#it#recommended#for#her#to#be#retested?#
a..1.year.
b..2.years.
c..3.years.
d..every.5.yrs...
Answer:.D.(.correct).(http://www.aafp.org/patientKcare/clinicalKrecommendations/all/cwKpapK
smears.html)..
.
Notes.about.pap.smear.and.HPV.:.
Age.to.start.:.Screening.should.start.at.age.21.
Age.to.stop.:.Women.age.65.and.older.should.stop.getting.screened.if.they.meet.all.3.of.these.
requirements:.
They.have.never.smoked.
They.do.not.have.a.new.sex.partner.since.their.last.Pap.test.
They.had.Pap.tests.done.regularly.until.they.turned.65.
They.had.3.normal.Pap.tests.in.a.row.
They.had.no.abnormal.Pap.
From.21K29.:.pap.test.every.3.years.
30.and.older.:.Pap.test.plus.HPV.test.every.5.years.
Or.Pap.test.every.3.years.
.
.
19.#which#medication#decrease#effect#of#OCP#?#
A.Anticoagulant.
B.Anticonvulsant.
C.Steroids.
Answer:.B..
.
20.#Pregnant#in#labor,#fetal#heart#rate#deceleration#from#140#to#80.#What#is#the#most#appropriate#
anesthesia?#
Answer:.General.anesthesia.
.
21.#Turner#patient#diagnosed#with#premature#ovarian#failure,#which#of#the#following#you#will#
expect#to#find?#
Answer:.High.FSH,.LH,.low.estrogen.
.
37.#41#week#pregnant#women##came#to#u#with#some#fetus#position#and#u#did#External#cephalic#
version,##but#after#some#time#u#found#the#the#fetus##return#to#the#same#previous#position#?#What#
prevent#u#from#doing#another#External#cephalic#version,#?#
a..failed.first.attempt.
b..Polyhedron.ibis.
c..Oligohydramnios..
d..Gestational.Age.40.weeks.
Answer:.A.or.C,.Contraindication.of.ECV:.Fetal.abnormality,.placenta.previa,.oligohydromnios.or.
polyhydromnios,.Hx.of.antepartum.hemorrhage,.previous.c/s,.multiple.gestation,.preKeclampsia.or.
hypertension,.plan.to.deliver.c/s.anyway.
.
23.#Best#way#to#prevent#fracture#in#osteopenic#postmenopausal#lady!?#
a..Daily.Vit.D..
b..Exercise.
Answer.A.(but.if.there.bisphosphonates.chose.it).
We.suggest.adequate.calcium.and.vitamin.D.for.all.postmenopausal.women.with.osteoporosis.and.
bisphosphonates.should.be.considered.as.firstKline.agents.for.the.prevention.of.osteoporosis..
UpToDate.
.
24.#Lady#has#vaginal#discharge,#postcoital#bleed,#and#painful#urination.#Exam#show#red#cervix,#gram#
stain:#gram#negative#diploococci#.What#is#organism?#
a..Gonorrhea..
b..Chlamydia.
Answer:.A.
.
25.#Pregnant#lady,#1st#trimester,#UTI#which#Abx#contraindicated:#
a..Tetracycline..
b..Nitrufornitoin.
Answer:.A.
.
26.#which#of#the#following#exacerbate#symptoms#of#primary#dysmenorrhea!?#
a..copper.IUD..
b..Levinorgasterel.IUD.
c..Mg.
d..NIfidipine.
Answer:.A.
Dysmenorrhea.is.often.worse.in.the.first.few.cycles.after.insertion.of.a.copper.IUD,.and.along.with.
unscheduled.bleeding,.is.one.of.the.primary.reasons.for.copper.IUD.discontinuation..However,.
discontinuation.rates.for.pain.are.low.(0.1.to.2.4.percent).in.both.copper.and.LNg20.IUD.users..
Moreover,.the.LNg20.and.LNg14.have.both.been.found.to.reduce.rates.of.dysmenorrhea...Mild.to.
moderate.dysmenorrhea.can.be.treated.with.nonsteroidal.antiinflammatory.drugs.(NSAIDs).begun.
at.the.onset.of.menses.and.maintained.for.the.first.three.days.of.each.menstrual.cycle..Women.with.
severe.dysmenorrhea.and.a.copper.IUD.should.consider.the.LNg20.or.LNg14.IUD.or.choose.another.
method.of.contraception..
reference:.uptodate.
.
27.#Screening#for#Group#b#Streptococcus#in#pregnancy:*#
Answer:.35.wks.
.
28.#Female#patient#presented#with#symptoms#of#urinary#incontinence,,#it's#associated#with#cough#
and#stress,,#patient#tried#to#avoid#it#by#strengthening#pelvic#muscles#while#accidentally#urinating#
but#it#was#of#no#use...#what#to#do#for#her#?#
a..strengthening.pelvic.
b..slings.
Answer.:.B.
conservative.approaches.to.treatment.of..Stress.urinary.incontinence.include.pelvic.floor.muscle.
training.and.incontinence.pessaries.and.Bladder.training..However,.for.women.who.decline.or.have.
insufficient.improvement.following.conservative.therapy,.there.are.a.variety.of.surgical.treatments..
The.introduction.of.midurethral.slings.has.changed.the.decision.process.for.surgical.treatment.and.is.
likely.a.factor.in.the.increase.in.the.rate.of.antiKincontinence.surgery.in.the.United.States.
https://www.uptodate.com.ezp.uod.edu.sa/contents/surgicalKmanagementKofKstressKurinaryK
incontinenceKinKwomenKchoosingKaKprimaryKsurgicalK
procedure?source=see_link&sectionName=Women%20who%20decline%20or%20have%20persisten
t%20symptoms%20following%20conservative%20therapy&anchor=H4#H4.
.
29.#Which#of#the#following#is#contraindicated#during#pregnancy?#
a..nitrofuranroin.
b..ampicillin.
c..folorquinolones.
Answer.is.C.
nitrofurantoin.Pregnancy.Risk.Factor.B.(contraindicated.at.term).
fluoroquinolones.are.considered.Pregnancy.Risk.Factor.C.
ampicilin.Pregnancy.Risk.Factor.B.
uptodate.
.
30.#Patient#presented#with#whitish#vaginal#discharge,#no#foul#smell,#wet#mount#showed#
psaeudohyphea#what's#your#treatment#
a..Topical.miconzaole.
b..Metronidazole.
c..Ampicillin.
d..Ceftriaxone.
Ans:.A.
Ref:.https://www.cdc.gov/std/tg2015/candidiasis.htm.
.
31.#Pt#presented#with#whitish#cheese#like#vaginal#discharge#no#foul#smell#what's#the#organism#
a..Candida.
b..Trichomonas.
c..Syphilis.
d..Chlamidiya.
Ans:.A.
Ref:.http://emedicine.medscape.com/article/2012015Koverview#a1.
.
32.#16#wk#pregnant#presented#with#severe#vomiting,#abdominal#distention#and#small#for#
gestational#age.#U/s#showed#snow#storm.#Bhcg#6.#What#is#the#dx?#
a..Choriocarcinoma.
b..Complete.mole.
c..Partial.mole.
Ans:.B.
Ref:.https://radiopaedia.org/articles/hydatidiformKmole.
.
33.#Pt#presented#with#mobile#vulvar#cyst#what#is#the#dx?#
AK.Bartholin.cyst.
Ans:.incomplete.case,.read.more.from.the.following.link..
Ref:.https://www.uptodate.com/contents/vulvarKlesionsKdifferentialKdiagnosisKbasedKonK
morphology#H27977424.
.
34.#What#is#the#most#common#cause#of#bleeding#in#postmenopausal#woman?#
a..cervical.polyp.
b..cervical.erosion.
c..atrophic.vaginitis..
d..endometrial.hyperplasia.
Answer:..C.
.
35.#Case#scenario#about#postmenouposal#atrophic#vaginitis#and#ask#about#treatment:#
A.! topical.steroid.
B.! topical.estrogen.
.Answer:.B.
.
36.#39#weeks#pregnant#lady,#without#any#complications#in#her#pregnancy,#came#with#vaginal#
bleeding#(Fresh#blood#per#vaginal#with#minimal#clots)#what#is#the#diagnosis#?#
a..abruptio.placenta..(it.is.painful).
b..placenta.previa..
c..attempted.to.abort.herself..
d..rupture.of.membrane..(.clear.fluid.not.blood).
Answer:..B.
.
38.#mechanism#of#Action#of#drugs#that#inhibit#Conversion#of#estriol#to#estrogen.#(I#forgot#the#exact#
question#but#it#mentioned#about#ovulation#and#who#inhibits#conversion#of#esterone#to#estrogen?)#
Answer:.Aromatase.inhibitors.
.
39.#A#women#wants#babies#but#she#doesn't#wants#to#get#pregnant#for#several#years#based#on#these#
statistics#of#her#condition#what#contraceptive#you#would#give#her#
a...Diaphragm.
b..Progestin.only.pills.
c..Combined.OCP.
Answer:.Question.may.not.including.the.right.answer,.they.want.a.long.acting.contraception!.and.all.
of.them.are.used.regularly.
If.you.find.implant.(Implanon).or.intrauterine.device.(IUD).choose.one.of.them.
Reference:.http://m.acog.org/ResourcesKAndKPublications/CommitteeKOpinions/CommitteeKonK
GynecologicKPractice/IncreasingKAccessKtoKContraceptiveKImplantsKandKIntrauterineKDevicesKtoK
ReduceKUnintendedKPregnancy?IsMobileSet=true.
.
40.#Female#17#year#old,#menarche#at#15,#regular.#Complaining#of#painful#menses.#Examination#
unremarkable.#What#to#give#her?#
Answer:.NSAID..
##
#
41.#lady#with#secondary#dysmenorrhea,#has#bilateral#ovarian#cyst,#US#showed#hypoehoic,#doppler#
showed#poor#perfusion,#what#next#to#confirm:#
a..CT.
b..MRI.
c..Biopsy.
d..CA125.
.
42.#Pregnant#4cm#dilation#cervix#90%#effacement#with#platelets#60#what's#ur#option#for#
anesthesia?#
a..General.
b..Pudendal.
c..Paracervical.
d..Narcotics.
Answer.:.A.
Pudendal.use.in.2nd.stage.of.labor.
Pudendal.and.epidural.are.contraindication.in.low.platelet.pt.
.
43.#38#week#pregnant#LL#swelling#fatigue#(the#hx#suggests#DVT)#
A.bed.rest.
B.LMWH.
C.venography.
Answer:.B.http://www.aafp.org/afp/2008/0615/p1709.html.
.
44.#what#can#u#palpate#in#the#lateral#fornix#
Answer:.Ovary..
.
45.#Hormonal#replacement#therapy#will#prevent#wt#from#the#following#
A.Menopausal.symptoms..
B.Osteoporosis.
Answer:.A.
.
46.#Pregnant#women#30#weeks#came#to#the#clinic#for#the#first#time,#with#symptoms#of#anemia#(#
tachypnea#,#pale#..#Etc#)#and#lower#limb#pitting#edema#+#heart#murmur#,#her#labs.#Showed#Hg#of#6#
and#low#MCV#wt#is#the#diagnosis#;#
A.Heart.disease.
B.Anemia..
Answer:.B.
.
47.#Case#scenario#about#female#patient#with#swelling#in#labia#majora#drainage#was#done#2#time#and#
she#ask#the#doctor#about#treatment#to#prevent#the#recurrence#?#what#will#he#do#?#
A.aspiration.
B.Marsupialization.
C.drainage..
Answer:.B.
There.is.many.options.for.treatment.but.the.best.is.a.surgical.procedure.known.as.marsupialisation.
may.be.used..
*Reference.:*.Gynecologist..
.
48.#Multiparous#lady#with#cervical#dysplasia.#Which#of#the#following#is#the#most#likely#cause#of#her#
dysplasia?#
A.HPV.
B.multiparity.
Answer:.A.
.
49.#Lady#24#weeks#pregnant#with#DM2#with#nephritis#grade#F,#and#chronic#HTN#controlled#on#
meds,#which#one#of#the#following#is#the#most#likely#complication?#
A.Still.birth.
B.Preeclampsia.
C.Shoulder.dystocia.
D.Increased.birth.weight.
Answer:.B.
.
50.Case#of#amenorrhea#,#there#is#breast#,#absent#uterus#and#vagina#?#
A.Mullerian.agenesis..
B.Androgen.insensitivity.
Answer:.A.
.
51.Case#of#pregnant#women#,#sleep#a#lot,#want#to#eat#specific#food..#etc,#what#is#the#responsible#
hormone?#
A.Progesteron.
B.Estrogen.
C.bHCG..
D.Prolactin.
Answer:.C.
.
52.Pregnant#41#week#,#cervix#5#cm#dilated#,#60#%#effacement,#in#the#hospital#since#8#hours,#you#
checked#again#the#same#date#above,#what#to#do?#
(same#q#)Multigravida#came#with#regular#contractions.#Cervix#is#4#cm#dilated#and#60%#effaced.#2[3#
hours#later#her#cervical#dilation#is#5#cm#but#still#60%#effacement.#What#is#the#management:#
A.C/S.
B.IV.oxytocin.and.rupture.of.amniotic.membrane..
Answer:.A.
Ref:.http://www.medscape.com/viewarticle/706359.
.
54.#26#year#old#came#with#foul#smelling#vaginal#discharge#and#irritated#uvula.#Which#one#of#the#
following#is#the#most#likely#diagnosis:#
A..Trichomoniasis.
B..Bacterial.vaginosis.
C..Candida.
Answer:.A.
55.#a#woman#used#IUD#for#contraception,#now#has#vaginal#pain#and#discharge,#What#organism#
responsible?#
Answer:.IUD.causing.PID.by.actinomyces.israelii#
#
56.#Mother#with#Rh#[ve#and#a#father#with#Rh#+ve,#what#the#probability#of#having#Rh#+ve#child?#
A..25%.
B..50%.
C..75%.
D..100%#
Answer:.B..
.
57.What#is#the#most#commonly#injured#organ#in#hysterectomy#
A..Ureter.
B..Colon.
C..Bladder.
Answer:.C.
Ref:.http://laparoscopy.blogs.com/prevention_management_3/2010/07/complicationsKofK
laparoscopicKgynecologicKsurgery.html.
.
58.#vaginal#discharge#watery,#yellow#,#foul#smelling,#and#no#itching#?#
Answer:.Bacterial.vaginosis...(.no.itching.,.watery.,.odour).
Take.care.If.they.mentioned.erythema.of.the.vulva.it.will.be.trichomonus.
.
59.#Case#of#miscarriage#,#the#fetus#has#bilateral#renal##agenesis#,#what#you#expect#also#to#find#!?#
A.polyhydramnios.
B.Oligohydramnios.
C.Renal.hyperplasia.
Answer:B.
.
60.#Dizygotic#twins#which#is#true?#
2.placenta.2.amniotic.fluid.regardless.sex.
2.placenta.1.amontic.same.sex.
1.placenta.1.amontic.same.sex.
1.placenta.1.amontic.regardless.sex.
Answer:A.
.
61.PCOS#have#a#risk#to#cause#which#type#of#tumor?#
AK.endometrial.cancer.
BK.ovarian.cancer.
CK.cervical.cancer.
Answer:.A.
.
62.pregnant#women#already#have#a#history#of#two#miscarriage,#after#that#the#doctors#diagnosed#
her#as#cervical#incompetence,#she's#now#pregnant#and#have#a#bleeding,#what#is#the#cause#of#her#
condition?#"##
AK.cervical.incompetence..
BK.chromosomal.disorders.
CK.ectopic.pregnancy.
Answer:A.
.
63.#premature#menopause#at#which#age#:#
A..30.
B..35.
C..40..
D..45.
Answer:.C.
.
64.#Most#common#risk#factor#for#fibroid#or#(most#important):#
AKage..
BK.African.race..
C.Kmultipara.
DK.Smoking.
Answer:..B.(African.women.tend.to.have.larger.fibroid.and.they.are.3.times.more.likely.to.develop.
fibroid.than.white.women).
Note:.smoking.and.multipara.is.protective.against.fibroid....
.
65.#28#years#old#pregnant#lady#with#tubal#pregnancy,#Which#one#of#the#following#most#common#
risk#factor#of#ectopic#pregnancy#
AK.Induction.by.ovulation.
BK.Previous.pregnancy.(ectopic).
CK.IUCD.
DK.Pelvic.inflammatory.disease..
Answer:.B,D.
.
66.pregnant#lady,#we#want#to#give#her#prostaglandin#F2#alpha,#Which#one#of#the#following#diseases#
should#be#taken#into#consideration?#
AK.Asthma..
BKDM.
CKHTN.
DK.cholelithsis...
Answer:.A.
.
67.#Best#investigation#to#diagnose#ECTOPIC#pregnancy:#
AKUltrasound.
BKLaparoscopy..
CKB.HCG.
Answer:.B.
.
68.pregnant#woman#when#to#take#vaginal#swab#for#meningiococcus#or#something#about#infection#
to#protect#the#fetus#from#this#infection#?#
A.38.weeks..
B.40.weeks..
C.26.weeks..
Answer:.35K37.weeks.
I.think.they.mean.GBS.infection..
.
69.#women#diagnosed#with#trichomonas#and#she#is#asymptotic,#when#to#start#treatment?#
A.! immediate..
B.! when.symptomatic.
Answer:.A#
#
70.#when#birth#consider#premature?#
A.40..
B.38.
C.26.
Answer.:.Less.than.37.weeks..
.
71.#which#ligament#protect#uterus#from#prolapse#?#
A.broad..
B.round.
C.uterosacral..
Answer.:.C..
.
72.#HIV#pregnant#women#something#drop#from#400#to#200#so#how#she#will#deliver?#
Spontaneous.vaginal.(.answer.).
.
73.#Female#with#S&S#of#pregnancy#+right#lower#quadrant#pain#after#doing#US#there#is#ectopic#
pregnancy#and#the#pt#stable#what#will#you#do?##
Treat.medically.(.methotraxate.if.the.patient.stable.).
.
74.#S&S#of#pregnancy#B[HCG#negative,dx?#
Ovarian.torsion(.not.sure,.missing.details.).
.
75.#what#is#the#role#of#OCP#in#protection#against#illegal#pregnancy#?##
Prevent.fertilization..
.
76.#case#of#postpartum#hemorrhage#you#gave#oxytocin#no#stoppage#and#the#diagnosis#was#uterine#
atony,#next#step?..
Bimanual.compression.
.
77.#which#one#is#true#about#Pap#smear#?##
Decrease.the.incidence.of.cervical.ca.dramatically.
.
78.#best#site#to#take#Pap#smear#?##
Transformation.zone.
.
79.#In#Pap#smear#you#find#hyperplasia#in#ectocervix#,#next#step?##
Colposcopy.directed.biopsy.
.
80.#correct#definition#of#recurrent#miscarriage?##
three.or.more.consecutive.pregnancy.losses.
.
81.#pregnant#e#bacterial#vaginosis#what#is#ttt?#
No.clindamycin.only.metronidazole.in.choices.
Metronidazole.is.the.most.common.and.preferred.antibiotic..
.
82.#A#young#woman#came#to#Primary#health#care#for#pregnancy#counseling,#she#had#chickenpox#
when#she#was#a#child.#What#will#you#do#to#her?#
A..Varicella.immunoglobulin..
B..Rubella.antigen..
C..Give.MMR.vaccine.
Answer.:.B.
83.#CASE#about#abruptio#placenta#which#one#of#the#following#sign#and#symptoms#is#the#most#
common#or#the#most#serious#one#(I#am#not#sure)?#
AKvaginal.bleeding.
BKuterine.contraction.
CK.Fetal.distress.(most.serious).
Note:.the.choice.is.not.very.accurate.so,.make.sure.in.the.exam..
Answer.:.A.
.
84.#Patient#in#labor#and#got#complicated,#will#go#for#c/s,#when#to#give#abx?#
A..Preoperative.
B..Intrapartum.
C..Intraoperative.
Answer.:.A..
Note:.To.give.cefazolin.or.ampicillin.
https://www.uptodate.com/contents/cesareanKdeliveryKpreoperativeKplanningKandKpatientK
preparation#H9.
.
85..Multiparous.,38.wks,.in.labour,.90%.cervical.effacement,..4CM.dilatation.,membranes.ruptured.e.
clear.fluid.,On.CTG.,FHR.dropped.from..140to.80,.the.pt.platelets=50,000,.what.type.of.anesthesia.is.
suitable?.#
A..GA.
B..Epidural.
Answer:.A.
.
86.#MOA#of#metformin#in#PCOS?#
Reduces.insulin.resistance..
.
87.#Dose#of#folic#acid#during#pregnancy#in#milligrams?#
A..0.4..
B..0.2..
C..2.
D..4.
No.risk.factor.:.0.4..
Risk.factor.(.NTD,.DM.).=.4..
.
88.#Safest#anti#diabetic#med#during#pregnancy?#
A..metformin..
B..acarbose..
C..sitagliptine..
D..glypuride..
answer:A..If.there.is.insulin.go.for.it...
.
89.#Lady#with#Hx#of#PCO#syndrome#the#best#management#is:.......................................................... .
A..Weight.reduction.*.first.line.*.
B..Metformin.and.clomiphin.(.if.she.wants.to.get.pregnant.)..
C..Weight.reduction,.danazol.and.metformin..
Did.not.find.clear.answer..
.
90.#Lesion#from#vulva#with#tree#like#shape?#
No.choises.
condyloma.acuminatum.valvua.((.i.think.they.mean..cauliflower.o.answers.was.provided.I.tried.to.
search.no.answers.)..
.
91.#Female#come#for#follow#up#with#lesion#in#labia#majora#2#in#2#cm,#biopsy#was#taken,#what#is#the#
histopathology#of#this#biopsy?#
A..Adenocarcinoma.
B..Squamous.cell.carcinoma.
C..Adeno.squamous.cell.carcinoma.........
.Answer.:.B..
92.#Female#come#to#the#clinic#with#plaque#in#labia#majora,#what#most#likely#to#be?..
A..Basal.cell.carcinoma.
B..Melanoma.........................
C..lupus.pernio.
Answer.:.B..
....................
93.#Female#had#2#abortion#in#second#trimester,#last#one#done#for#her#D&C#diagnosed#as#cervical#
incompetence#now#pregnant#in#5#week#present#with#vaginal#bleeding#and#open#cervical#os#what#is#
the#cause#of#bleeding?#
A..Luteal.phase.defect.
B..Cervical.incompetence.(..second.and.third.trimesters)..
C..Asherman.syndrome.
D..Chromosomal.cause.
Answer.:.D.
https://www.uptodate.com/contents/miscarriageKbeyondKtheKbasics.
.
94.#Female#had#2#abortion#in#second#trimester,#last#one#done#for#her#D&C#diagnosed#as#cervical#
incompetence#now#pregnant#in#10#week,#vaginal#bleeding#stopped#and#closed#cervical#os.#what#to#
do#next?#
A..Bed.rest.
B..Cerclage.(.12K14.weeks.).
C..Salicylate.
D..Admission.........................
Answer.:.A.
.
95.#Pregnant#in#37#week#come#complain#of#RUQ#pain,#headache#and#blurred#vision,#they#do#
emergency#c/s,#and#MgSO4#was#given,#one#houre#later#she#develop#tachycardia#130#and#BP#
110/74,#what#to#expect?###................
A..Hemorrhage.
B..Mg.toxicity.
C..Anesthesia.induced.hypotension.
Answer.:..B.
...
96.#Scenario#of#severe#preeclampsia#given#MGSO4#and#reflex#is#+1?#
.magnesium.sulfate.toxicity.
...................
97.#Pregnant#lady#has##preeclampsia#started#on#mg#sulphate#and#hydralazine#then#RR#become#12#
what#to#give?#
A..narcan.
B..atropin............................................................ .
C..ca.gluconate.
D..naloxone.
Answer.:.C..
https://overdoseinfo.com/magnesiumKsulfateKtoxicity/.
98.#Primary#PPH#,#give#the#pt#oxytocin#no#benefit,#what#is#second#step?.
A..uterine.massage..
B..ligation.internal.iliac.A.
C..hysterectomy.
Answer.:.A....
.
99.#PPH#with#low#BP#and#tachycardia#the#1st#thing#to#do?#
A..Ergotamine.
B..Ringer.lactate.
C..Normal.saline.with.something.
.
.
100.#Tumor#associated#with#polycystic#ovarian#syndrome?#
A..Lung.
B..Breast.
C..Endometrium.
D..Ovary......
Answer:.C#
.
101.#A#patient#about#40s#and#had#PCO#since#she#was#young,#now#she#has#vaginal#bleeding.#what#is#
the#most#likely#she#has?#
No.choices..
PCOS.associated.with.endometrial.hyperplasia.and.later.carcinoma.
.
102.#Female#with#PCOS#and#has#very#enlarged#ovaries,#on#examination#how#do#you#differentiate#
between#ascites#and#her#cystic#ovaries?#
A..Fluid.thrill..
B..Dullness.in.the.flanks.and.tymany.in.the.midline..
C..Tympany.in.the.flanks.and.dullness.in.the.midline..
Answre:.B.
.
103.#20#years#old#female#with#hirsutism,#acne,#menstrual#is#regular,#ovaries#US#shows#perl#string#
appearance.#what#is#the#dx:#
A..Prolactinoma.
B..PSOS.
C..hypothyroidism.
Answer.:.B.(.perl.string.appernce.i.think.it.should.be.irregular.typo.)..
............................................................................. .
104.#20#years#old#female#with#hirsutism,#acne,#menstrual#is#irregular,#ovaries#US#shows#perl#string#
appearance.#what#is#the#dx:.
A..PCOS.
B..Congenital.adrenal.hyperplasia.
Answer.:A.
.
105.#Girl#with#normal#menstruation#has#ovarian#mass#7#cm:#............................................................. .
A..Benign.teratoma.
B..pco.
C..something.malignant.
D..Functional.cysts.
Answer.:D.
.
106.#Female#patient#with#gonorrhea.#What#to#rollout?#
A..Herpes.
B..Chlamydia.........................
C..Bacterial.vaginosis.......
Answer.:.B....
................
107.#Gonorrhea#commonly#found#in:#
Urethra.
Cervix.
Annswer.:B.
Diagnosis.is.by.testing.the.urine,.urethra.in.males,.or.cervix.in.females.
.
108.#Watery[gray#vaginal#discharge,#fishy#odor,#some#itching#and#discomfort,#the#husband#uses#
condom.#The#most#likely#organism:#
A..candida.
B..bacterial.
Answer.:.B.
.
109.#60#YO#female#with#bloody#nipple#discharge,#O/E:#1#cm#mass#subaereolar.#What’s#the#best#
management:#
A..excision.and.Bx.
B..mammo.with.FNA..
Answer.:.B.
110.#postpartum#came#with#discharge#with#other#symptoms.#I.
.
111.#30#years#old#female#with#….#in#the#vagina#(i#forget),#she#has#high#FSH#and#LH?#
ovarian.failure.........................
..........
112.#Patient#with#merorraghea#wha#to#give#her?#
Combined.oral.contraception..
.
113.#Inhibition#of#which#of#the#following#is#the#primary#action#of#oral#contraceptives?#
A..Decrease.estrogen.to.prevent.the.ovulation.
B..Decrease.GTRH.spur.at.the.mid.cycle.
C..Increase.prolactin.
D..Suppressing.the.release.of.gonadotropins.
E..Spermatozoa.and.thickening.cervical.mucusa.
Answer.:.D.
.
114.#How##is#the#MOA#for#emergency#contraceptives?#
A..Prevents.fertilization.
B..Prevents.implantation.
C..Prevents.anovulation.
D..Delays.fertilization..
Answer:.B.
.
115.#A#patient#come#after#one#day#of#intercorse#afraid#to#get#pregnant.#What#to#give#her?#
emergency.contraceptives.
117.#How#ectopic#pregnancy#occurs#at#the#cellular#level?#
A..Early.abscess.of.zona.pellucida.
B..Persistence.of.zona.pellucida.
Answer:.A.(.i.think.they.mean.disappearance.=absent.)..
.
118.#Patient#with#constipation,#weight#loss#and#pelvic#mass.#What#will#help#in#reaching#the#Dx?#
A..Pap.smear.
B..Colposcopy.
Missing.information.wide.differential..
Ovarian.cancer.:.CAK125.blood.test..TVUS..
Symptoms.suggestive.of.cervical.cancer.:.colposcopy....
https://www.cancer.org/cancer/cervicalKcancer/detectionKdiagnosisKstaging/howKdiagnosed.html.
.
119.#When#to#consider#prolonged#pregnancy?#
A..40.weeks.
B..41.weeks.
C..42.weeks.
D..43.weeks.
.Anwer.:.C.
120.#OCP#by#increase:#
A..Estrogen..
B..Progesterone..
C..FSH..
D..LH.
Answer.:.B.
.
121.#56#yrs#old#woman#with#uncontrolled#urination#upon#coughing,#sneezing#or#laughing,#Best#Mx#
measure#for#her#is:#
Kegel.Exercise.
.
122.#what#is#true#about#Pap#smear?#
A..21.till.65.yrs.repeated.every.year.
B..21.till.65.yrs.repeated.every.5.yrs.
C..21.till.35.repeated.every.3.years.
D..30.till.65.repeated.every.5.yrs..
Answer.:.D.every.5.year.if.(.pap+.HPV).
.
123.#Female#has#genital#warts#over#the#past#years,#genital#warts#are#associated#with#what#?#
hyperkeratosis..
.
124.#A#woman#in#labour,#everything#is#fine,#she#asked#for#Epidural#analgesia,#what#test#should#be#
requested?#
A..Hb.
B..PLT.count.
Answer.:.B.
.
125.#Best#way#to#diagnose#bacterial#vaginosis?#
gram.stain..
.
126.#23#yrs#old#female,#with#burning#upon#voiding#urine,#she#goes#frequently#to#bathroom#but#with#
little#amount#of#urine#voided,#she#does#not#have#suprapubic#tenderness#OE,#she#had#also#
hematuria.#What#to#do?#
A..Urinalysis.and.culture.
B..testing.for.Gonorrhea/chlamydia.
C..Pylocyctogram.
Answer.:.unclear.question.but.usually.start.simple.and.noninvasive.to.diagnose.or.rule.out.most.
common.disease.such.as.urinalysis.and.culture.
.
127.#most#common#type#of#fibroid#
A..submucous.
B..intramural.
C..subserous.
Answer:.B.
.
128.#multiparous#woman#term,#came#with#Progressive#contractions#and#cervical#dilatation,#
mentioned#that#her#membranes#ruptured#and#fluid#was#clear(..details.of.examination.plus".uterine.
tenderness")..correct.action?#
Multiple.choices.and.one.of.them.was.give.intrapartum.abX. .
Because.this.is.a.case.of.chorioamnionitis.with.possible.GBS.infection.so.treated.e.intrapartum.
antibiotics.
.
129..40..yrs.old.multiparous.with.hx.of.CSCame.e.vaginal.bleeding,.which.was.painless,.she#is#38#
weeks,#US#examination#showed#low#lying#placenta#,#which#of#the#following#would#make#the#doctor#
decide#CS#with#hysterectomy?#
A..age.of.pt.
B..Previous.CS.
C..placenta.accreta..
D..multiparity...
answer:C.
.
130..A.woman.e.Ca.cervix.,e.mets.beyond#uterus#and#cervix,#1st.group.of.LN.receiving.the.lymphatic.
drainage.is:#
A..uterine.
B..external.iliac.
C..common.iliac.
D..para.aortic.
Answer.:.B.
.
131.#Pregnant#lady#on#her#24th#week#of#gestation#with#a#significant#medical#history#of#DM#II#on#
insulin#with#nephritis,#chronic#HTN#controlled#on#medications.#O/E#her#fundal#height#was#25#cm,#
otherwise#unremarkable#PE.#What#complication#is#more#likely?##
A).PreKEclampsia..
B).Shoulder.dystocia................................................................... .
C).Stillbirth..
D).LargeKforKGestationalKAge.infant..
http://emedicine.medscape.com/article/1476919Koverview.
DM.can.increase.the.risk.of.preeclampsia.especially.with.nephritis.
Answer.:A.
.
132.#Preeclampsia#case#what#is#the#first#step#?#
A..Magnesium.sulphate.
B..IV.hydralazine.
C..Methyldopa.
Answer.:.sever/end.organ.damage.A..
.
133.#A#female#with#polycystic#ovarian#syndrome#noticed#hyperpigmented#skin#in#her#neck#and#
axilla,#what#is#this#abnormality#called?#
a..Acanthosis.nigricans.
b..Linea.nigra.
A.
.
134.#Female#patient#pregnant#present#with#abdominal#pain#and#vomiting.#Vital#sign#normal#except#
blood#pressure#160/100.#What#is#the#diagnosis?#
A..Renal.failure..
B..Fetal.distress..
(.no.preeclampsia.in.the.choices.).sever.preKeclamsia.160/110..
.
135.#When#you'll#do#Alpha#fetoprotein#?#(16[18)#.
A..13.
B..15.
C..17.
D..19.
Answer:.C.
.
136.#women#with#DUB#&#Endometrium#biopsy#shows:#endometrial#hyperplasia#This#caused#by?#
a).Adrenal.hyperplasia.
b).Liver.dysfunction.....................................................
c).Local.genetic.mutation.in.the.endometrium............. .
d).Peripheral.percoser.converted.to.estrogen.
Answer.:.C.
.
137.#patient#with#high#grade#hyperplasia#with#atypia#and#total#hysterectomy?#
(I.do.not.know.what.is.the.question)..
.
138.#Patient#had#fibroid#3*4#cm#take#tamox#after#1#years#come#with#vaginal#bleeding#US#show#
fibroid#6cm#with#endometrial#hyperplasia#what#is#the#cause#of#bleeding?#
A..Fibroid.
B..Endometrial.hyperplasia.
Answer.:B.
.
139.#28#years#old#diabetic#female#DM1#controlled#by#insulin#she#is#married#and#wants#to#become#
pregnant.her#blood#glucose#is#well#controlled#and#she#is#asking#about#when#she#must#control#her#
metabolic#state#to#decrease#risk#of#having#congenital#anomalies:#
A..before.conception.
B..1st.trimester.
C..2nd.trimester.
D..3rd.trimester.
Answer.:.A.
140.#First#trimester#.with#H/O#GDM#in#last#pregnancy#3#yrs#back.#When#you#do#GTT?#
16.weeks.
..
If.there.is.a.risk.do.it.in.the.first.trimester.(less.or.equal.16).
All.pregnant.24K28.Weeks..
.
141.#Case#of#GDM#risk#in#future#develop#DM2?#
.
142.#Long#scenario#of#pregnant#women#admitted#due#to#hypertension,#proteinuria,#lower#limb#
oedema#fundal#level#not#corresponding#to#gestational#age.#What#the#risk#to#baby#to#develop#
IUGR???#
A..elevated.maternal.liver.enzymes.(.??.placental.insufficiency)..?.
B..oligohydramnios.
C..polyhydromenous.
D..GDM.
Answer.:B.(.I.asekd.dr.reem.alanzi..about.this.question.)..
SMLE.12.Page.437.Q.97.
.
143.#45#years#old#Female,#on#estrogen.#Complain#of#dysuria#for#the#last#year#frequency#low#steam#
void,#she#had#recurrent#UTI,#take#multiple#antibiotic#without#benefits,#Examination#shows#normal#
vagina/vulva/cervix?#and#tenderness#at#neck#of#bladder.#what#is#the#diagnosis?#
A..Traumatic.urethritis.
B..Interstitial.urethritis...
C..DM.
D..Candida.
Answer.:.B.
.
144.#A#wife#of#a#man#diagnosed#as#having#gonorrheal#infection#she#was#worried#about#
transmission#of#the#infection#from#her#husband#what#is#the#best#investigation#to#exclude#
gonorrhea#infection?##
Gram.stain.
.
145.#Lady#in#labor,#you#can#feel#nasal#bridge#and#orbital#ridge.#Which#one#compatible#with#vaginal#
delivery?#
A.#Mentoanterior.
B..Mentoposterior.
C..Transverse.mentoanterior.
answer:A.
.
146.#contraindications#for#breastfeeding?#
A..Active.HCV..(.IF.WITH.CRACKED.NIPPLE.)..
B..Active.HIV..
C..Varicella.zoster..
Answer.:.b..
147.#disease#increases#the#mortality#in#pregnant#female?#
Pheochromocytoma.
Peripartum.cardiomyopathy.develops.in.the.last.weeks.of.pregnancy.until.6.weeks.postpartum.(.IF.IT.
WAS.one.of.the.option.)..
.
148.#Pregnant,#36#weeks,#present#with#agitation,#BP:#88/60,#fetal#distress,#what#is#the#diagnosis?#
A..Pulmonary.embolism..
B..Amniotic.fluid.embolism.
C..Abruption.(.BV.bleeding+.pain.)..
Missing.information.mostly.A.not.sure.....
.
.
149.#Female#pregnant#with#small#fibroid#what#to#tell#her#about#the#risk:#
A..Risk.of.preterm.labor.
B..Degeneration.is.common..
C..Mostly.asymptomatic.
Answer:.C.
.
150.#34#years#old#woman#,Multiparous#,#complain#of#heavy#menstruation#,#upon#examination#:#
there#is#intramural#fibroid#
.
151.#Picture#of#CTG,#what's#the#dx?#
.
152.#Breastfeeding#mother#(they#gave#history#of#mastitis#and#was#treated#with#antibiotics)#after#3#
weeks,#lump#occurred#fluctuating,#not#tender,#what#is#the#most#likely#diagnosis?#
Breast.abscess.
https://emedicine.medscape.com/article/781116K
clinical?pa=Rwlpdtd60tTvbZzGkjszR4uzFrShLqaKyXFvSgKPRiFt%2BR95nHSjnV9%2FkrGRFHQFfbcwf64
RVLoPX%2FSLPtDaPfEiL5fM42L%2B9xlMlua7G1g%3D.
.
153.#Pregnant#women#with#placenta#previa#what#is#the#risk#factor:#
A..race.
B..age..
C..previous.placenta.previa.
Answer.:.C..
.
154.#Para2#woman#planned#with#her#husband#to#avoid#pregnancy#during#next#3#yrs#she#doesn’t#like#
to#use#IUDS#nor#OCP#What#should#you#tell#her#about#transdermal#contraceptive#
A..It’s.more.likely.to.form.clots.more.than.OCP.
B..It’s.easy.to.forget.changing.it.
C..less.effective.than.IUD/OCP.
Answer.:.A.
.
155.#young#couples#trying#to#conceive#for#6#months,#both#are#healthy#
try.some.more..
.
156.#Women#at#40#week#gestation#suddenly#develope#dyspnea#with#hypoxia#and#chest#pain:#
A..Amniotic.fluid.embolism.
B..Pulmonary.embolism..
C..Myocardial.infarction..
Answer.:.B.
.
157.#Female#g3p2#20#week#ask#you#about#cancer#happen#during#pregnancy,#2#of#his#sister#develop#
cancer#during#pregnancy#and#died,#origin#of#cancer#
A..Ovary.
B..Breast.
C..Cervix.
D..Vulvar.
Answer:.B.
.
158.#Most#common#malpresentation:#
A..Braw.
B..Breech*.
C..Face.
D..Transverse.
Answer.:.B.
.
159.#case#scenario#about#female#with#past#hx#of#PROM#and#now#she#is#pregnant#asymptomatic#and#
ask#about#indication#of#screening#for#bacterial#vaginosis:#
A..no.indication.
B..screen.at.first.trimester.
C..screen.at.second.trimester.
D..screen.at.third.trimester..
answer:A.
.
.
160..MAFP.measure.in.which.week:.
A..13.weeks.
B..15.weeks.
C..17.weeks*.
Answer.:.C.
15–18.weeks.
Not.sure.about.the.correct.answer.what.I.found.is.15K18.week..
.
161.#Female#patient#with#hX#of#multiple#abortion#and#D&C##want#to#get#pregnant#what#is#the#
diagnosis:#
Asherman.syndrome.(.corrrect.)..
.
162.#Patient#with#two#time#ASCUS#pap#smear#what#do#next:#
Colposcopy.
.
163.#42#years#old#female#with#hx#of#hot#flushing#and#night#sweating#and#amenorrhea#what#the#
diagnosis?#
A..Hypothyroidism.
B..Congenital.adrenal.hyperplasia.
C..Hyperprolactinemia..
Correct.:.C.#
164.#Pregnant#women#during#labor#,#Iv#oxytocin#was#given#the#CTG#show#variable#and#acceleration#
what#will#you#do?#
A..Stop.oxytocin..
B..Expectant.delivery.
C..Change.mother.position.
Answer:b.
..
165.#Giving#birth#,#cervix#6#cm#dilated#,#fetus#is#left#occiput#posterior#,#sign#of#molding#can#be#felt#
what#the#stage?#
First.(.correct.)..
..
166.#Daily#requirement#of#iron#in#pregnancy?#
14.7.mg/day.(.requirement.of.iron.).
with.30.mg.to.60.mg.of.elemental.iron.(.dose)..
http://www.who.int/elena/titles/guidance_summaries/daily_iron_pregnancy/en/.
.
167.#On#examination:#her#cervix#is#dilated#by#3#cm#and#effaced#by#70%#and#fetal#presenting#part#at#
0#station.#After#6#hours#or#so,#her#cervix#is#dilated#by#7#cm#and#effaced#by#80%,#but#she#is#
complaining#of#tenderness#when#palpated#the#uterus,#her#temperature#is#38.#
what#is#the#best#management?#
A..Give.intrapartum.antibiotics..
B..Emergency.cs.
C..IV.antibiotics.ƒ.ampicillin.(2.g.IV.q6h).and.gentamicin.(1.5.mg/kg.q8h).ƒ.anaerobic.coverage.(i.e..
clindamycin.if.C/S).
answer:A.
https://www.uptodate.com/contents/groupKbKstreptococcusKandKpregnancyKbeyondKtheKbasics.
.
•.expedient.delivery.regardless.of.gestational.age.torronto.notes..
J‫ﻣﺴﺤﻮ‬$ $‫ﺳﺘﻔﺪﺗﻮ‬$ ‫ ﻣﺎ‬$N$ C‫ ﻣﻨ‬96‫ﺪ‬+‫ )ﻤﻜﻦ ﺗﺴﺘﻔ‬$Nَ R6 SH‫ﻣﺪ‬.:I6H$.^..
.
168.#67#years#female#on#regular#follow#up#for#20#year#for#pap#smear#which#is#negative#regarding#
check#up#
A..every.6months.
B..every.year.
C..every.3years.
D..no.further.check.up.
answeR:.D.
.
169.#pt#hysterectomy#done#co#of#mild#pain#in#incision#site.#Hb#normal,#Temperature#37.5,#HTC#
normal,#Bp#110/70.#What#cause#to#be#still#in#hospital??#
A..low.Hb.
B..high.tempreture.
C..high.HTC.
D..because.not.oral.feeding.
Normal.temp.:.37ºC.(98.6ºF),.but.anywhere.between.36.5ºC.and.37.2ºC..
.Postpartum.fever.is.defined.as.a.temperature.of.38.7.degrees.C.(101.6.degrees.F).or.greater.for.the.
first.24.hours.or.greater.than.38.0.degrees.C.(100.4.degrees.F).on.any.two.of.the.first.10.days.
postpartum..
I.asked.a.fellow.her.answer.was.B.But.I.not.convinced.by.her.answer.I.tried.to.look.for.criteria.of.
discharge.did.not.find.anything.useful.I.think.the.answer.is.D(.Dana.).
.
170.#50#years#old#complain#of#2#months#of#amenorrhea#what#u#will#found#
A..Increase.FSH,increase.LH.
B..Decrease.FSH,decrease.LH.
C..Increase.FSH,decrease.LH.
Answer.:A.
.
171.#Which#one#of#the#Contraceptive#drug#cause#hirsutism#“androgenic”#side#effects?##
progesterone.only.

#
172.#Female#with#developed#secondary#sex#character#sparing#axillary#and#pubic#hair#in#
investigation#there#was#high#testosterone,dx?#
Androgen.insensitivity.syndrome.
.
173.#Women#underwent#bilateral#salpingoophrectomy#and#hysterectomy,#she#developed#hot#
flushes.#You#want#to#give#her#Hormonal#replacement#therapy.##
What#will#you#give?##
A..Cyclic.estrogen.and.progesterone..
B..Continues.estrogen.and.progesterone..
C..Transdermal.estrogen.patches..
D..Levonorgestrel.IUD..
Answer.:c.(.NO.need.for.protective.effect.of.progestron.since.she.had.hysterctomy.).but.estrogen.
patches.is.no.longer.giving.due.risk.of.DVT.and.high.estrogen..Confusing.question.it’s.eather.A.or.C
. .
.
174.#Hormone#replacement#therapy#increase#risk#of##
A..Endometrial.cancer.
B..Breast.cancer.
C..Ovarian.cancer.
Answer.:.B.
.
175.#40#years#old#Patient#she#has#secondary#dysmenorrhea#and#heavy#bleeding#they#decided#to#do#
hysterectomy,#diagnosis#was#confirmed#with#a#histopathological#exam#of#the#tissue#of#
hysterectomy,#which#of#the#following#is#the#best#non#invasive#investigation#to#support#the#
suspected#diagnosis.#
A..office.endometrial.biopsy..
B..pelvic.MRI.
C..pelvic.CT.
D..pelvic.US.
Adenomyosis.:.answer.B.
.
176.#what#is#the#best#non[invasive#method#to#diagnose#Adenomyosis#
A..Office.sampling..
B..Ultrasound..
C..MRI.
Answer.:.MRI.(more.sensitive.and.specific)...................................................................... .
A.definitive.diagnosis.of.adenomyosis.can.only.be.made.from.histological.examination.of.a.
hysterectomy.specimen..The.preoperative.diagnosis.is.suggested.by.characteristic.clinical.
manifestations.(ie,.menorrhagia.and.dysmenorrhea.with.a.uniformly.enlarged.uterus).in.the.absence.
of.endometriosis.or.leiomyomas..................... ............................................. .
Both.transvaginal.ultrasound.(TVUS).and.magnetic.resonance.imaging.(MRI),.especially.T2Kweighted.
images,.are.increasingly.used.for.clinical.decisionKmaking..
.
.
178.#Most#common#cause#to#perform#hysterectomy?##
Uterine.fibroid..
.
179.#Pap#smear#of##ASC[US,#what#to#do#next?#
A..Reassure..
B..Hysterectomy..
C..Repeat...
D..Cone.biopsy..
Answer.:.b.(.if.21K30.year:.repaat.).if.>30.year.do.HPV..
.
180.#Pregnant#prolonged#labour#for#12hrs#asthmatic,#mitral#stenosis#what#is#the#indication#for#
forceps#delivery?#
A..Asthma.
B..Prolonged.labour.
C..Mitral.stenosis.
D..I.forgot.it.
Answer.:.C.(.either.prolonged.2nd.stage.or.maternal.cardiac.condition.,.need.more.details.about.
which.stage.of.labor.)..
https://www.uptodate.com/contents/operativeKvaginalKdelivery.
.
181.#Pregnant#lady#on#20#weeks#has#active#herpes#what#you#gonna#do?#...... .
A..Give.acyclovir.
B..Wait.until.2.week.and.C.S.election.gve.before.delivery.#
Answer#:A.
.
182.#Pregnant#with#UTI#what#is#the#complications#that#might#happen?#
A..Low.birth.weight.
B..Preterm.(.BV.)..
C..Acute.pyelonephritis.
ANSWER.:.c..
183.#Elderly#women#with#vulvovaginal#lichen#planus#lesion#in#posterior#vaginal#fornix,#this#patient#
is#at#risk#to#develop#which#type#of#the#following#vaginal#cancer:#
A..Squamous.cell.carcinoma..
B..Adenocarcinoma.
C..Squamous.adenocarcinoma.
Answer:.A.
.
184.#How#to#screen#for#thalassemia#in#pregnant#women:#
A..US.on.12.weeks.
B..Chorionic.villus.sampling.at.16.weeks..
C..Triple.test.at.16.weeks.
D..quadruple.test.at.15.weeks.
Note:.it’s.by.mniocentesis.–.this.is.done.from.15.weeks.of.pregnancy..I.THINK.TYPO..
https://www.nhs.uk/Conditions/pregnancyKandKbaby/Pages/screeningKsickleKcellKthalassaemiaK
pregnant.aspx.
.
185.#40#yrs#old#pregnant,#history#of#dead#upon#delivery#baby,#is#thinking#of#getting#pregnant;#&#
worries#that#the#same#will#happen#again#,what#will#you#tell#her:#
A..Sampling.of.amniotic.fluids.@.(.X.of.month).
B..Has.the.same.risk.as.the.rest..
C..Higher.risk.because.of.her.age.
D..US.@.X.of.months.
Depend.on.number.A.should.be.more.than.16.weeks..
.
.
186.#OCP#side#effect#dry#skin#muscle#spasm,#deprration#
A..Hypothyroidism.
B..Hyperkalemia.
C..Hypocalcemia.
ANSWER:.b.
187.#Ovarian#cancer#tumor#marker#?#
A..CA125.
B..AFP.
C..PS.
ANSWER.:.A.
.
188.#18#year#old#girl#with#primary#amenorrhea#Normal#pubic#hair#and#breast#development,#
Absence#of#uterus:#
A..Klinefelter.syndrome.
B..Mallrune.agencia.
C..Turner.syndrome.
Answer.:B.
.
189.#Pregnant#at#8#weeks#and#had#previous#baby#with#down#syndrome,#she#wants#to#screen#for#
down#in#this#pregnancy,#you#booked#her#after#2#weeks#for#the#test.#What#should#you#write#in#the#
consent#form#as#a#complication#for#this#test?#
A..Risk.of.miscarriage..(my.answer.b/c.CVS.can.cause.miscarriage).
B..Rupture.amniotic..
answeREr:.A.
.
190.#Case#of#premenstrual#symptoms,#what#to#give?#
.fluoxetine(.ssri.)..
.
191.#Ovarian#mass#measuring#5#cm#not#affecting#menstrual#cycle,#what#dx?#
A..Follicular.cyst.(.simple.cyst)..
B..Theca.lutein.(.lear,.strawKcolored.fluid)..
C..Something.carcinoma.
NOT.SURE.!..
.
192.#What#is#the#most#common#birth#injury?#
A..Clavicular.fracture.
B..Shoulder.dislocation.
C..Hip.dislocation.
D..Femur.fracture.
Answer.:.A..
http://www.birthinjuryguide.org/birthKinjury/types/infantKbrokenKbones/.
.
.
193.#Case#of#G2P1#+0#(not#sure)#and#presented#with#lower#abdominal#pain#that#comes#and#goes#for#
10[15#min#more#in#the#right#iliac#fossa.#Pregnancy#test#was#negative#(although#she#is#G2P1+0).#
What#is#the#diagnosis?#
A..Acute.appendicitis.
B..Ectopic.pregnancy.
ANSWER.:.A.?? #
#
194.#Menorrhagia#define#as?#How#much?#
Menorrhagia.is.defined.as.excessive.uterine.bleeding.occurring.at.regular.intervals.or.prolonged.
uterine.bleeding.lasting.more.than.seven.days..Classically.more.than.80.ml/cycle..
.
195.#Patient#with#elevated#CA125#levels,#what#is#she#in#risk#for?#
A..Ovarian.germ.cell.tumor.
B..Sex.cord.ovarian.tumor.
C..Epithelial.ovarian.tumor.
Answer:.C.
.
196.#Patient#with#ovarian#mass.#What#is#the#best#screening#test:#
A..Ultrasound.
B..CA.125.
Answer:.B.
.
197.#Pregnant#lady#has#hypothyroidism,#how#much#do#you#have#to#increase#her#dose?#
A..10%.
B..20%.
C..30%.
D..40%.
Answer:.25K50%.
.
198.#Amniocentesis#is#best#done#at#what#gestational#age:#
A..10.Weeks.
B..12.Week.
C..14.Week.
D..16.Week.
Answer:.D.
.
199.30#weeks#gestation#came#with#Bp=#162/95#urine#protein#+2.#Which#one#of#the#following#you're#
going#to#give?#
A..Dexamethasone.
B..Magnesium.sulphate.
C..Methyldopa.
D..Labetalol.
Answer:.B.
.
200.#Fetus#with#single#umbilical#artery.#What#could#be#the#cause#?#
Answer:.The.occurrence.of.a.single.umbilical.artery.is.thought.to.be.due.to.secondary.atresia.or.
atrophy.rather.than.primary.agenesis.of.the.artery..the.absence.of.the.left.umbilical.artery.is.much.
more.common.(~70%)..
.
201.#postmenopausal#women#taking#estrogen#and#progesterone,#Increase#risk#of#which#cancer?#
Answer:.Breast.cancer.
.
202.#What#is#the#preferred#pelvis#shape#for#vaginal#delivery?#
A..Gynecoid..
B..Android..
C..Platypelloid..
D..Anthropoid.#
Answer:.A..
.
203.#Hymen#penile#penetration:..
Answer:.6.o’clock..
Note:.The.question.is.not.clear.
.
204.#Pregnant#lady#with#history#of#2#SvD#with#normal#babies#of#3#kg.#The#baby#is#breech#and#the#
head#is#flexed#she#found#to#have#bicornuate#uterus#.#And#the#baby#weight#is#2kg#.#What#is#the#
contraindication#for#external#cephalic#version#?#
A..Baby.weight.
B..Flexion.of.the.head..
C..Bicornuate.uterus...
D..Hyperextended.head..
Answer:.C.
.
205.#Pregnant#with#genital#warts#what#to#do?#
Answer:.Vaginal.delivery..
Source:.https://www.cdc.gov/std/treatment/2010/genitalKwarts.htm.
.
206.#Placenta#previa#and#adherent#what#is#the#noninvasive#test#to#do?#
A..MRI.
B..Transvaginal.US.
C..Doppler.ultrasound.
Answer:.B.
Explanation:.MRI.is.too.expensive..
.
207.#patient#in#postpartum#hemorrhage,#you#want#to#give#methylergonovine.#what's#the#relative#
contraindication#to#use#it:#........................................ .
A..Asthma..
B..Diabetes.
C..Hypertension.............
Answer:.C.................................................... .
.
208.#Most#common#way#of#HIV#vertical#transmission#and#…#?#
A..Placenta..
B..Breastfeeding.
C..Umbilical.cord.blood.contamination.
Answer:.A.
Explanation:.Exposure.through.pregnancy,.birth,.or.breastfeeding.—.Vertical.transmission.of.HIV.can.
occur.at.any.time.during.gestation.and.delivery,.and.through.breast.milk.in.the.postpartum.period..
More.than.95.percent.of.HIVKinfected.children.worldwide.have.acquired.the.virus.via.vertical.
transmission..The.rate.of.vertical.transmission.of.HIV.varies.from.approximately.20.to.30.percent.in.
the.absence.of.antiretroviral.therapy..Although.it.is.known.that.HIV.can.be.transmitted.early.in.
gestation.in.utero,.most.transmissions.(50.to.80.percent).are.believed.to.occur.during.the.time.
period.near.or.during.delivery...
reference:.uptodate.
.
209.#Mother#had#abortion#two#times#last#child#with#45X#monosomy,#recurrence#in#next#pregnancy?#
A..40%..
B..60%.
C..70%.
D..90%.
Answer:.Recurrence.of.TS.is.observed.in.1.4%,.which.represents.a.35Kfold.increased.probability.of.
having.a.second.child.with.TS.compared.to.general.population...
Source:.https://www.ncbi.nlm.nih.gov/pubmed/21648298.
.
210.#Painless#Genital#ulcer#plus#inguinal#lymphadenopathy:#
Answer:.Syphilis..
.
211.#A#patient#with#painless#Genital#ulcer,#how#to#diagnose?#
Answer:.Dark.field.microscopy.
.
212.#Middle#age#women#trying#to#conceive#last#born#1#year#all#normal#what#you#check.##
Answer:.FSH?.
Note:.The.question.is.not.complete.
.
213.#Female#want#to#conceive#after#2#yrs,#what#best#ocs#to#give?.
.
214.#Young#lady#with#previous#hx#of#chlamydial#infection#tried#to#conceive#for#previous#14#months#
but#she#can't,#her#husband#has#normal#semen#analysis#and#she#didn't#have#any#hormonal#diseases,#
the#next#investigation#should#be:#
A)! MRI.
B)! US..
C)! Hysterosalpingography.
Answer:.C.
.
215.#40#year#old#with#heavy#bleeding#everything#excluded.#Even#D&R#normal.#Cause#of#bleeding?#
Answer:.Anovulatory.cycles..
.
216.#Girl#18#years#come#with#no#menstrual#cycle#2#month#ago#she#is#not#sexually#active#and#on#
examination#she#feel#tenderness#on#abdominal#what#you#will#do#?#
A..Ultrasonography........................................ .
B..Do.pregnancy.test.
Answer:.B.
Explanation:.The.initial.laboratory.evaluation.(after.ruling.out.pregnancy).for.women.with.secondary.
amenorrhea.should.include.follicleKstimulating.hormone.(FSH),.serum.PRL,.and.thyroidKstimulating.
hormone.(TSH).to.test.for.POI,.hyperprolactinemia,.and.thyroid.disease,.respectively..
.
217.#In#menopausal#lady,#diagnostic#test#
Answer:.FSH.
.
218.#Type#of#postpartum#hemorrhage#in#atony#of#uterus?#.
Answer:.Primary.
..
219.#Postpartum#bleeding#gave#IVF#and#add#what?#
A..Oxytocin.
B..Ergotamine.
C..Carbetocin.................................................... .
Answer:.A.
.
220.#Regarding#preeclampsia,#first#symptom/#sign?#
Answer:.BP.≥140.mmHg.systolic.and/or.≥90.and.proteinuria..#
#
221.#Pregnant#in#30#gestational#weeks,#pressure#was#high#unlike#previous#visites#was#normal.#No#
visual#disturbances#no#protein#in#the#urine.#What#is#the#diagnosis?#
A..Gastinal.hypertension.
B..Preeclampsia.
C..Eclampsia.
D..Superimposed.hypertension.
Answer:.A.
#
222.#Scenario#of#patient#with#preeclampsia#then#ask#what#is#the#most#important#test#should#be#
done#
A..blood.pressure.monitoring.and.respiratory.rate.
B..LFT.and.platelet. . . . .
C..urinalysis.and.proteinuria.
D..creatinine.and.BUN..
http://emedicine.medscape.com/article/1476919Koverview#a14.
Answer:.C.
Explanation:.since.the.scenario.didn’t.make.a.diagnosis.of.preeclampsia.there.for.urine.for.protein.is.
mandatory.in.addition.that.24.h.protein.in.urine.is.determine.the.severity.of.the.disease#
#
223.#Pregnant#Patient#with#herpes#what#is#best#management#?#
Answer:.if.active.disease.on.40.weeks.gestation.delivers.CS.
...
224.#Best#dx#tool#for#female#with#irregular#massive#bleeding?#
Answer:.The.question.is.not.clear..The.first.to.do.are.pregnancy.test.and.CBC..The.history.will.direst.
the.other.test...
.
225.#22#years#old#irregular#menses#w#abdominal#pain#started#7[8#after#menstruation#,#her#menses#
7[9#days#,#4[5#intercourse#/week#protected#by#condom#,#she#has#high#hygiene#with#Vaginal#douche#
monthly#after#menses#what#is#the#cause#of#symptom?###...........................................................
A..Number.of.intercourse............................................................. .
B..Vaginal.douche.............
C..Condom.
Answer:.B?.
https://www.ncbi.nlm.nih.gov/pmc/articles/PMC2567125/#
#
226.#A#question#about#Tanner#stage#
Answer:.please.google.Tanner.scale.
.
227.#Pt#in#labour#6#cm#dilation,#fetal#CTG#acceleration#and#variable.#Next#step?#
Answer:.Question.is.not.complete,.probably.change.the.maternal.position..
.
228.#Rupture#of#condom,#what#to#use#?#
a)! Contraceptive.
b)! Douching..
c)! Tell.her.to.leave..
Answer:.A.
.
229.#34#period#day,#when#will#she#start#ovulating#?#
Answer:.21.
..
230.#Multipara,#last#delivery#was#assessed#by#vacuum,#presented#with#bulging#mass#from#vagina.#
What#to#do#next:#
A..Speculum.vaginal.examination.
B..Retrograde.cystourethrogram.
Answer:.A.
.
231.#At#which#age#has#the#highest#risk#associated#with#chromosomal#abnormality:#
A..>.30.years.
B..>.35.years.
C..>.40.years.
D..>.45.years.
Answer:.D.
..
232.#Which#of#the#following#is#an#independent#risk#factor#that#will#increase#the#risk#of#infant#
getting#an#early#GBS#infection?#
A..Intrapartum.fever.>39.
B..Family.history.of.infant.with.GBS.
C..Premature.rupture.of.membranes.for.more.than.18.hours.
.Answer:.C.
.
234.#Female#pt#with#symptoms#of#menopause#and#symptoms#of#depression#,#what's#the#
treatment?#
A..Estrogen.
B..Progesterone.
C..Paroxitine.
Answer:.For.post.menopausal.mood.lability/depression.we.can.use.HRT.with.or.without.SSRIs..The.
best.choice.for.this.woman.is.combined.estrogen.and.progesterone....
..
235.#34#wk#+#open#cervix#+#bulging#membrane#+#fetal#HR#..#(forget#no.)#what#is#your#1st#
management#?#
Answer:.question.is.not.complete,.is.the.fetus.in.distress?.
.#
236.#What#is#true#about#liquid#based#cytology#for#Pap#smear:#
A..The.brush.is.used.to.sample.the.endocervical.canal.
B..The.sample.is.taken.from.the.transformation.zone.
C..The.sample.is.taken.by.a.spatula.and.a.brush.
Answer:.A.
.##
237.#A#woman#with#Cervical#cancer,#mets#beyond#uterus#and#cervix,#1st#group#of#LN#receiving#the#
lymphatic#drainage#is:#
A..Uterine.
B..External.iliac.
C..Common.iliac..
D..Para.aortic.
Answer:.A.
..
238.#For#who#should#we#measure#BP#for#24#hours#
A..Confirm.compliance.of.medication.
B..Elderly.with.palpitation.
C..Resistant.HTN.on.medication.
D..Suspected.preeclampsia.
Answer:.D?.
..
240.#The#cause#of#maternal#mortality#if#happened#during#pregnancy#?#
A..Syphilis..
B..Toxoplasmosis..
C..Biliary.cholestasis.
Answer:.?.
..
241.#30#years#old#female#present#to#you#in#the#clinic#after#ROM#before#one#hour#which#was#clear#
fluid#P/E:#no#vesicles#in#the#vagina#medical#report#documents#as#pt#has#hx#of#recurrent#HSV#what#is#
your#management?#
A..Proceed.to.C/S".
B..IV.acyclovir..............................................................................
Answer:.Perform.a.sterile.speculum.ex,.then.if.positive.PROCEED.TO.CS..If.negative.SS.EX,.it's.
controversial...either.SVD.or.CS..
If.pt.is.not.in.labour.and.EX.is.normal,.but.with.known.hx.of.HSV,.obtain.a.weekly.cervical.culture..
If.pt.is.presenting.with.active.lesions.but.not.in.labour,.give.IV.acyclovir.........................................................................
............................................................................................................................................................. .
242.#A#32#week#pregnant#lady#.#By#ur#exam#u#found#her#to#be#off#date.#What#serial#clinical#
assessment#u#will#do#over#the#next#2#weeks#?#
A..Maternal.weight.
B..Cardiographs.
....I.forgot.the.others.
............................................................................. .
Answer:.The.question.is.not.complete..The.3.basic.methods.used.to.help.estimate.gestational.age.
(GA).are.menstrual.history,.clinical.examination,.and.ultrasonography..
............................................................................. .
243.#monochorionic#twins#in#27#week,#one#of#them#died,#wt#to#do#?#
A..wait.for.spontaneous.delivery.at.any.gestation..
B..wait.till.34.week.and.deliver.
C..give.steroid.and.deliver..
Answer:?.
............................................................................................. .
244.#Best#investigation#in#1st#trimester#
A..CBC.
B..Ultrasound.. .
Answer:.Question.is.not.clear.but.probably.CBC..
245.#Women#is#bleeding#after#delivery,#and#can't#stop,#wt#to#do#immediately?#
A..Hysterectomy..
B..Something.to.apply.pressure..
I.forgot.other.choices.
Answer:.Uterine.massage.
Immediately.commence.resuscitation..Raising.the.legs.improves.venous.return.and.is.consistent.with.
the.positioning.used.to.diagnose.and.treat.the.underlying.causes.of.bleeding..Administer.oxygen.and.
obtain.intravenous.access..All.intravenous.lines.started.on.the.labor.ward.for.other.reasons.must.be.
placed.with.cannulas.of.sufficient.gauge.if.PPH.develops..
............................................................................................. .
246.#Oby#surgery#clamb#the#artery#close#to#lateral#vaginal#wall#what#structure#may#be#injured:#
A..Pudendal.nerve.
B..Ureter.
Answer:.B.
.
.
247.#In#pudendal#nerve#block#which#of#the#following#will#not#be#affected?##
A..Rectum.
B..Vulva.
C..Perineal.body.
D..Urogenital.diaphragm.
Answer:.A.
. .
248.#What#are#contraindications#of#instrumental#delivery?#
A..Placenta.abruption.
B..Breech.presentation.
C..Cephalopelvic.disproportion.......... .
D..Face.presentation.
Answer:.C.
. .
249.#Smoking#pregnant#women#what#will#be#changes#in#her#child?#
Answer:.Intrauterine.growth.restriction.(small.baby)..
..
250.#Female#with#mass#protruding#from#vagina:##.... .
Answer:.Utero.vaginal.prolapse............................................................................. .
............................................................................. .
251.#Female#post[delivery#her#child#on#excessively#breastfeeding,#menstruation#resumed#after#
10th#month,#because#of#her#other#child#she#and#husband#ask#for#contraception,#what#will#you#
advise#them?#
A..Depo.provera.
B..OCP.
Answer:.A.
http://patient.info/doctor/postpartumKcontraception.
.
.
252.#Definition#of#recurrence#abortion#
A..2.documented.consecutive..and.more.
B..3.documented.consecutive.and.more.
C..4.documented.consecutive.and.more.
Answer:.A.
.
253.#35#years#old#women#O/E#they#found#10*11#mm#raised#irregular#mass#on#lateral#aspect#of#the#
cervix..#what#to#do:#
A..Excision.
B..Taking.biopsy.from.the.mass.............................................................. .
C..Investigate.for.human.papillomavirus.
D..Reassure.her.and.wait.for.pap.smear.result..
Answer:.B..
254.#Pregnant#in#labor#everything#normal#except#tender#cervix#during#examination#what#to#do?.
A..Intravenous.AB............................................................................. .
B..Intra.labor.antibiotics.
Answer:.?.
............................................. ..............................
255.#Pregnant#with#vaginal#bleeding#and#everything#was#normal#what#do#you#ask#pt#about?#
Last.sexual.intercourse......................................................................... .
Answer:.question.is.not.complete..You.should.obtain.history.about.fetal.movements.and.previous.
US..
.
256.#Female#patient#she#done#tube#ligation#4#years#back,#current#complain#is#she#has#period#which#
was#6#week#back#now#she#intermittent#spotting#of#blood#,#V/E#there#is#no#blood#normal#cervix#and#
close#os#wt#the#next#step#?###.................. .
A..CT..
B..Laparoscopy.................
C..Pregnancy.test............................................................... .
Answer:.Maybe.C.
............................................................................. .
257.#Pregnant#at#the#at#the#time#of#delivery#was#given#epidural#when#she#was#6#cm#dilated#,#later#
she#was#given#fentanyl!!#And#started#pushing#later#she#started#to#have#lightheadedness#and#she#
was#laying#on#her#side#what#will#you#do#for#her:#
A..Ask.the.nurse.to.bring.forceps..
B..Give.fentanyl.again.
C..Give.ephedrine.
D..Do.her.a.position.with.weird.name.
Answer:.D,.change.the.patient.position..
..
258.##First#antenatal#visit#of#a#woman#in#her#10th#week#gestation#what#to#do?#
A..Assess.fetal.size.or.like.that.
B..Assess.risk.factors.
C..Determine.the.fetal.age.or.like.that.
Answer:.B.
.
259.#In#regard#to#vulvar#cancer,#which#of#the#following#is#the#principle#in#diagnosing#it?#
A..Clinical,.through.history.of.HPV..
B..Histopathological,.through.biopsy...
C..Radiological,.through.pelvic.US...
D..Irrelevant.choice..
Answer:.B.
.
260.#Fallopian#tube#embryology:##
A..Mesoderm.intermediate.cell..
B..Lateral.part.of.genital.ridge.
Answer:.A.
..
261.#Case#of#preeclampsia##(weight#gain,#proteinuria,#headache)#with#IUGR,##What#other#clinical#
finding#support#IUGR:#
A..Oligohydramnios.
B..Polyhydramnios.
C..Liver.enzymes.
Answer:.A.
..
262.#Case#of#patient#with#recurrent#abortion#and#she#came#at#20#weeks,#cervix#length#was#30.#what#
to#do:#
A..Circulage.
B..Progesterone.
Answer:.B.
..
263.#Stillbirth#at#which#gestational#age?##
A..20.
B..25.
Answer:.B,.more.than.24.weeks..
.
264.#Female#I#think#in#40#had#2#child#with#45#xo#next#child#percentage#to#have#it?#
A..40.%.
B..60.%.
Answer:.please.see.Q209.
..
265.#31#years#old#Female#came#to#do#pap#smear#and#HPV#test,#result#normal#before#1#years#done#
pap#smear#+#HVS#result#normal#when#to#do#test#again?#
A..6.months.
B..1.years.
C..2.years.
D..3.years.
Answer:.5.years,..
Source:.https://www.cancer.org/cancer/cervicalKcancer/preventionKandKearlyKdetection/cervicalK
cancerKscreeningKguidelines.html.
.
266.#Which#of#following#has#best#survival#on#COPD#patient?#
A..Smoking.cessation.
B..O2.supplement.
Maybe:.B.
.
267.#Female#has#4#intercourse#in#week#use#condom#also#her#husband#use#condom,#also#she#use#
douching#after#that#which#of#the#following#has#risk#to#health?#
A..Use.of.condom.
B..Douching.
C..Number.of.intercourse.
Answer:.B.
.
268.#Female#pt#with#puritosis#and#enuresis,#with#pelvic#strengthening#exercises,#she#was#given#
duxbutine,#what#is#the#mechanisms#of#action:#
A..Inhibit.muscarinic.cholinergic.
B..Activate.acetylene.
C..etc.
Answer:.?.
............................................................................. .
269.#Semen,male#Works#in#batteries#factory,#came#after#1#year#trying#with#his#wife#to#get#pregnant#
analysis:#oligospermia.#CBC#show#heavy#metals#on#RBC,#what#the#cause?###...................................................
A..Nikle........................................................ .
B..Lead.
Answer:.B.
............................................. ..
270.#Regarding#cervical#ca#screening:#
A..Started.at.age.21.regardless.of.sexual.activity.
B..HSV.+.Pap.smears.are.done.together.every.5.years.
C..HSV.screening.is.an.alternative.to.Pap.smear..
Answer:.A.
.
271.#When's#the#best#time#to#do#amniocentesis?#
A..10.weeks.
B..12.weeks.
C..14.weeks..
D..16.weeks..
Answer:.D.
.
272.#Ectopic#pregnancy#of#2.5*3#[#hcg#5000#patient#stable#what#to#do#?#
A..Wait.
B..Laparotomy.
C..Laparoscopy.
D..D&C.
Answer:.Methotrexate,.if.not.available.laparoscopy..
Source:.http://bestpractice.bmj.com/bestKpractice/monograph/174/treatment/stepKbyKstep.html.
.
273.#Female#nullipara#unable#to#conceive,#plethoric#face#and#purple#striae#at#abdomen#and#BP#
160/...,#hypokalemia,#what#also#she#has?#
A..Acropachy.(.graves.).
B..this.is.cushing....
Answer:.B.
.
274.#What#is#the#incidence#of#prolonged#pregnancy:.(.from.3.to.12).
1K5%.
10K15%.
Answer:..As.many.as.10.percent.of.pregnancies.will.deliver.postterm..
Source:.Uptodate.
.
275.#Female#pregnant#(#twin#)#GA#34#doctor#plan#for#CS#because#presentation#of#twin#A##may#cause#
fetal#complication,#what#is#the#presentation#?#
A..Cephalic.K.breech.
B..Transverse.K.cephalic.
C..Breech.K.cephalic.
D..Cephalic.K.cephalic.
Answer:.B.
.
276.#Female#in#her#30s.#P3.#Complaining#of#amenorrhea#for#the#last#year#or#so.#Underwent#D&C#
after#2nd#pregnancy.#Labs#show#high#FSH#and#LH.#Low#estrogen.#
A..Asherman.
B..Premature.ovarian.failure..
Answer:.B.
..
277.#Principle#treatment#of#urge#incontinence?#
A..Medical.
B..Surgical.
C..Medical.and.surgical.
D..Bladder.training.and.something.else.
Answer:.A.
.
278.#A#patient#who's#on#estrogen#therapy#is#complaining#of#dysuria,#urgency#and#frequency#for#a#
few#months.#She#took#antibiotics#multiple#times#but#it#didn't#work.#What#is#the#cause#of#her#
symptoms?.
A..Bacterial.vaginosis..
B..Candida.vaginitis..
Answer:.B.
.
.
279.#50#YO#female#with#urge#incontinence,#dry#vagina.#The#first#step#to#do:#
A..urinalysis.and.culture.
B..cystourethrogram.
C..Bonney's.test.
Answer:.Probably.A..
.
280.#Male#with#pelvic#injury,#1st#thing#to#do:#
Answer:..Suprapubic.catheter..
.
281.#Most#common#ovarian#cyst#is:#
A..theca.lutein.
B..follicular.
Answer:.B.
.
282.#The#day#of#ovulation#is#
A..14.
B..17.
C..22.
Answer:.on.average.day.14.
.
283.#A#Lady#with#heavy#menstrual#bleeding#and#intermenstrual#bleed#with#no#Hx#of#pregnancy#or#
sexual#intercourse...#the#cause#is:#
A..chronic.endometritis.
B..anovulatory.cycles..
Answer:.B.
.
284.#Most#common#side#effect#of#IUD#?#
Answer:.Menstrual.cycle.irregularities..
.
285.#PPH#with#low#BP#and#tachycardia#the#1st#thing#to#do#:. . . .
A..Ergotamine.
B..Ringer.lactate.
C..Normal.saline.with.something.
Answer:.C.
.
286.#Woman#P5,#all#svd,complains#of#(#case#scenario#about#uterine#prolapse#),#what#test#to#do#?#
A..options.of.uro.gyne.tests.
B..speculum..
Answer:.B.
.
287.#Patient#with#normal#menses#since#puberty#,#normal#exam#except#ovarian#mass#
A..Follicular.cyst.
B..Ovarian.carcinoma. . . . . .
C..Endometrial.cancer.
Answer:.A.
#
288.#How#many#Barr#body#in#XXX#female:#
A..1.
B..2.
C..3. . . . . .
D..4.
Answer:.C.The.bar.body.number.is.the.same.as.X..
.
289.#How#much#normal#blood#loss#during#menses?.
Answer:.60.(The.usual.amount.of.blood.loss.per.period.is.10.to.35.ml.).
.
.
290.#Female#patient#in#her#late#40s,#she#has#children#and#does#not#want#to#get#pregnant#any#more.#
Presented#with#abdominal#pain,#she#has#previous#history#of#endometrioma#in#her#right#ovary#and#
it#was#removed.#She#did#imaging#and#it#showed#that#she#has#endometrioma#of#the#left#ovary.#What#
is#the#appropriate#management?##
AK.Bilateral.salpingoKoophorectomy.
BK.removal.of.the.endometrioma..
Answer:.Most.likely.A.since.she.is.not.planning.to.conceive.and.failed.endometrioma.resection.
.
.
291.#Thalidomide#SE#in#pregnancy:##
Answer:.limb.defect.
.
292.#9#week#pregnant,#presented#with#bleeding,#cervix#was#dilated#1#cm#with#visible#tissues#
A..Threatened.abortion.
B..Incomplete.abortion.
C..Complete.abortion.
Answer:.B.
.
293.#benign#ovarian#teratoma. . . .
Answer.:.missed.Q.but.most.likely.benign.ovarian.teratoma.
.
294.#Female#with#HBV#and#had#done#pap#smear#(there#is#results).#What#is#the#cause?#
HBV.
HPV.
Answer:.The.question.is.not.clear.
..................
295.#Pregnant#taking#iron#supplements#and#come#with#anemia#symptoms,#Hb#is#low,#what#is#the#
diagnosis?#
A..Iron.deficiency.anemia..
B..Thalassemia.
Answer:.Needs.further.details..Maybe.B..
.
296.#Pregnant#with#microcytic#anemia,#what#to#give#here?#
Answer:.Iron.
.
297.#When#to#give#antibiotic#for#patient#will#go#for#C/S?#
Answer:.PreKoperation.
..
298.#Pregnant#had#PROM,#2#days#later#she#develop#fever#and#pain.#What#is#the#next#in#
management?#
A..C/S.
B..IV.Antibiotic..
Answer:.B.and.delivery..
.
●! Know.the.age.of.the.fetus.(e.g..The.lungs.are.mature.by.36.weeks.before.that.we.give.
steroid)..
●! Know.the.phases.of.delivery,.stages.and.sections..
●! When.to.give.epidural,.pedundal.and.general.and.what.are.the.parts.blocked.in.each...
●! gene.of.cervical.cancer.→.*see.the.image*.
http://m.cancer.org/cancer/cervicalcancer/detailedguide/cervicalKcancerKwhatKcauses.
.
.
.
.
.
.
.
.
.
.
.
.
.
.
.
.
.
.
.
.
.
.
.
.
.
.
.
.
.
.
.
.

!General.Surgery.
##
##
Breast#and#Endocrine#
. .
Q..Patient.with.solitary.thyroid.nodule,.picture.of.hyperthyroidism,.what.is.the.most.
appropriate.next.step?.
A..US.
B..FNA.
Answer:B.
Ref:.https://emedicine.medscape.com/article/850823K
overview?pa=OsGCL06RS84x%2Bx%2BlWevYeJ1avQalW6vjZFUwUEd8rFafzWuiuDz1vfrlWIllf
KfsX8MwC0EECwzp432Skuf9qw%3D%3D#a7.
..
..
Q..When.FNAC.reveal.fibrocystic.change?.
Apocrine.metaplasia.
..
Q..Female.work.as.an.actress,.present.with.mass.that.increase.before.menses,.after.taking.FNA.the.
result.is.yellow.stain.with.no.refilling.again.what.is.the.diagnosis?.
KANDI.(my.answer).
Kphyllidus.tumor.
Breast.cyst.
(Cystic.change.were.not.provided.in.the.option).
not.sure!.
..
..
Q..yo.came.with.a.history.of.preKmenstrual.multiple.breast.lumps..She.said.it.goes.after.
menstruation.but.on.exam.she.was.found.to.have.multiple.lumps.with.one.dominant.lump.of.3.cm.
and.LN.exam.is.normal..How.will.u.manange.:......
A.....Wait.for.the.the.next.menstrual.cycle.and.reKevaluate................................ .
B.....FNA.
C.....Radilogy.and.mamogram.
Answer:.
..
Q...Radical.mastectomy.with.pectoralis.major.removal,.which.movement.is.lost:.
A..adduction.
B..abduction.
Answer:.A.
..
..
Q..Patient.with.solid.thyroid.nodule.2.cm.size,.euthyroid,.what’s.next?.
A..FNA.
B..CT.scan.
..
Answer:A.
..
Q..40.years.old.female.with.unilateral.cystic.mass.with.no.lymph.node.involvement..Next.
step?.
A..Mammography.
B..FNA.
C..Excisional.biopsy.
..
Answer:.it.is.often.difficult.to.distinguish.a.cyst.from.a.solid.mass..Ultrasonography.or.aspiration.
must.be.used.to.establish.a.definitive.diagnosis..Cysts.require.surgical.biopsy.only.if.the.aspirated.
fluid.is.bloody.
Ref:.http://www.aafp.org/afp/2000/0415/p2371.html.
..
Q..Patient.presented.with.signs.of.hypercalcemia,.his.labs.show.hypercalcemia.+.high.
Alkaline.phosphatase.(from.scenario.it.is.Hyperparathyroidism)..
The.clinical.presentation.in.the.question.will.lead.to.what?.
A..Osteoporosis.
B..Osteomalacia.
C..Hyperparathyroidism.
..
Answer:C.?.
..
..
Q..Women.with.bilateral.breast.nipple.white.discharge.and.vision.disturbance.
Prolactin.level.is.high,.where.is.the.lesion?.
A..Sella.turcica.lesion.
..
Answer:A.
..
. .
Q..Midline.swelling.moves.with.protruding.the.tongue?.
A..Thyroglossal.cyst.
B..Thyroid.nodule.
C..Hygroma.
..
Answer:A.
..
Q..Patient.presented.with.a.hard.Mass.on.the.outer.upper.area.of.the.breast.which.lymphK
. node.you.have.to.examine?
. A..Posterior.axillary
. B..Anterior.axillary
. C..Lateral.axillary
. D..Medial.axillary
. .
. Answer:
. .
. .
Q..Case.of.bloody.nipple.discharge.(probably.asking.about.the.cause)..
A..Intraductal.papilloma.
..
Answer:A.
..
Q..Female.with.left.upper.outer.mass.in.the.left.breast,.which.lymph.node.group.will.be.
examined?.
A..Pectoral.
..
Answer:.
..
Q..21.y/o.Female.with.lump.in.breast,.what’s.the.best.method.for.evaluation?.
A..Mammogram.
B..US.
C..FNA.
D..Ductography.
..
Answer:C.
Ref:.https://www.mayoclinic.org/healthyKlifestyle/womensKhealth/inKdepth/breastK
lump/artK20044839?pg=2.
..
Q..Patient.with.breast.mass,.with.calcification.in.the.MRI,.what.is.the.most.appropriate.
management?.
..
Answer:.not.sure!.
..
Q..Dimpling.of.skin.in.breast.cancer.is.caused.by?.
A..Cooper.ligament.
..
Answer:A.
..
Q..Bilateral.breast.cancer.most.likely.cause?.
A..Lobular.
..
Answer:.
..
Q..Patient.in.her.40s.with.breast.mass,.what.will.be.your.next.step.in.her.management?.
A..Mammogram.
B..US.
C..FNA.
..
Answer:A.
..
(Similar.question).
Q..41KyearKold.Lady.has.breast.mass.since.2.weeks,.upon.examination.it.hard.and.fixed,.
what.is."most.".accurate.diagnostic.modality?.
A..US.
B..Mammography.
C..Lactography.
D..MRI.Breast.
..
Answer:D.?.
..
Q..Superolateral.breast.cancer.with.axillary.tail,.which.lymph.node.group.will.be.involved?.
A..Anterior.group.
..
Answer:.
..
Q..Patient.with.clear.nipple.discharge,.regular.menstrual.cycle,.how.you.will.investigate.her?.
A..Prolactin.level.
..
Answer:.
..
Q..Gene.of.ductal.carcinoma?.
.A..P53.
..
Answer:A.
..
Q..Breast.cancer.gene?.
A..BRCA2.
..
Answer:A.
..
Q..Thyroid.nodule.biopsy.found.medullary.(?),.what.is.the.diagnosis?.
A..Papillary.cancer.
B..Medullary.thyroid.cancer.
..
Answer:B.
..
Q..Patient.with.bloody.stained.nipple.discharge,.you’re.suspecting.intraductal.papilloma..
What’s.the.next.step?.
A..Mammogram.
B..FNA.
C..Excision.
..
Answer:C.
Ref:.https://www.healthline.com/health/intraductalKpapilloma.
..
Q..What’s.the.effect.of.taking.Tamoxifin.in.reducing.breast.cancer.in.high.risk.patients?.
A..High.
B..Low.
C..Intermediate.
D..No.effect.
..
Answer:.not.sure!.
..
..
Q..Initial.investigation.of.a.single.thyroid.nodule?.
A..FNA.
B..CT.scan.
..
Answer:A.
Ref:.https://emedicine.medscape.com/article/850823Koverview#a7.
..
Q..Female.treated.from.mastitis,.after.10.days.presented.with.painless.lump,.what.you.will.
do?.
..
Answer:?.
..
Q..25.years.old.female.asking.about.selfKbreast.exam,.when.it.should.be.done?.
A..6K7.days.after.the.cycle.
B..3K5.days.after.the.cycle.
C..7K10.days.after.the.cycle.
D..14K16.days.after.the.cycle.
..
Answer:B.
Ref:.https://medlineplus.gov/ency/article/001993.htm.
..
Q..Female.with.bilateral.menstrual.breast.pain..In.examination.there.is.bilateral.breast.
nodularity.and.mobile.mass.in.the.right.breast,.axillary.lymph.nodes.are.free,.what.to.do.
next?.
A..Mammogram.then.US.
B..Aspiration.and.cytology.
C..Reevaluate.at.the.next.cycle.
..
Answer:.C.
http://www.aafp.org/afp/2005/0501/p1731.html.
..
Q..Female.with.diffuse.thyroid.swelling.and.dominant.single.nodule,.lab.result.showed.
increased.T4.and.decreased.TSH,.what.will.you.do?.
A..Radionuclide.scan.
B..FNA.
C..US.
D..Thyroidectomy.
..
Answer:A.
##
..
Q..Mobile,.firm.mass.not.related.to.menses,.what.is.the.diagnosis?.
A..Fibroadenoma.
B..Fibrocytic.change.
C..İntraductal.carcinoma.
..
Answer:A.
..
Q..After.aspiration,.when.is.a.breast.lump.safe.to.leave.alone?.
A..There.is.minimum.blood.staining.of.aspirate.
B..Clear.cyst.fluid.and.does.not.refill.
C..Cyst.cells.with.hyperchromatic.nuclei.
D..FNAC.suggests.fibroKcystic.disease.
.
Answer:.B?.
..
(Similar.question).
Q..Patient.with.breast.mass,.which.of.the.following.indicates.that.the.mass.is.cystic.(or.no.
need.for.reevaluation)?.
A..FNA.showing.clear.fluid.on.aspiration.and.disappearance.of.the.mass.after.
B..Bloody.nipple.discharge.
C..Clear.fluid.with.reappearance.of.the.mass.
..
Answer:A??.
..
Q..Breast.cancer.prognosis.depends.on?.
A..Number.of.axillary.lymph.nodes.involved.
B..Estrogen.receptors.
C..The.patient’s.age.
..
Answer:A.
##
Q..Recurrence.of.breast.cancer.depends.on?.
A..Number.of.the.involved.axillary.lymph.nodes.
B..Estrogen.receptors.
C..Progesterone.receptors.
..
Answer:A.
Ref:.https://www.mayoclinic.org/diseasesKconditions/recurrentKbreastKcancer/basics/riskK
factors/conK20032432?p=1.
................................................. .
Q..Treatment.of.multinodular.goiter?.
A..Surgery.
B..Radioiodine.
C..Beta.blockers.
D..Antithyroid.medications.
..
Answer:B??.
Ref:.https://www.healthline.com/health/multinodularKgoiter.
##
..
Q..Patient.with.Lab.findings.of.hypothyroidism,.he.has.hoarseness.and.a.large.neck.swelling..
O/E.lt.lobe.is.swollen.and.is.larger.than.the.right.one.
Best.treatment.is?.
A..FNA.
B..Thyroid.lobectomy.
C..Radio.isotope.scan. .
..
Answer:A.
..
Q..Mammogram.can.detect.breast.cancer.before.clinical.examination.by.how.many.years?.
A..1.
B..2.
C..3.
D..4.
..
Answer:b.
Ref:.https://www.radiologyinfo.org/en/info.cfm?pg=mammo.
..
Q..Tamoxifen.for.breast.cancer.patient,.she.has.metrorrhagia,.US.showed.thick.
endometrium,.what.to.do.next?.

A..Endometrial.biopsy.

B..CA125.

Answer:.not.sure!.

..

Q..Lady.developed.postpartum.mastitis.(red.swollen.right.breast).and.was.give.fluxacillin.for.
10.days..A.few.weeks.later.she.presented.with.a.right.breast.hard.mass,.nonKtender..What.is.
the.Dx?.
A..Antibioma.
B..Duct.ectasia.
..
Answer:.B.
. .
..
Thoracic#Surgery#
. .
Q..Retrosternal.pain.and.barium.swallow.showed.esophageal.corkscrew.appearance,.what.is.
the.diagnosis?.
A..Achalasia.
B..Esophageal.cancer.
C..Diffuse.esophageal.spasm.
..
Answer:.C.
..
..
Q..Hyperthyroidism.presentation.but.with.bradycardia,.what.is.the.cause?.
A..Multinodular.goitre.
B..Hyperthyroidism.
C..Hypothyroidism.
..
Answer:.not.sure!.
..
Q..Patient.smoker,.alcohol.drinking.has.thyroid.nodules,.what.is.the.first.thing.that.you.do?.
......................................................... .
A..Needle.aspiration.
..
Answer:.
. .
..
Q..During.a.surgery.in.the.diaphragm,.phrenic.nerve.is.dissected.at.which.level?.
A..Anterior.to.scalenus.anterior.muscle.
..
Answer:A.
..
Q..Which.organ.will.be.affected.with.injury.to.the.posterior.vegal.truck?.
A..Bladder.
B..Descending.colon.
C..Esophagus.
..
Answer:C.??.
..
..
..
Q..Chronic.smoker,.presented.with.signs.of.lung.cancer,.what.is.the.origin.of.the.cancer?.
A..Clara.
B..Brush.cells.
C..Goblet.
..
Answer:.b?.
..
Q..In.the.OR.the.doctor.cut.the.vagus.nerve.by.mistake.while.doing.NISSIEN.fundoplication.
operation.for.treatment.of.chronic.GERD,.which.organ.will.be.affected.from.cutting.the.
nerve?.
A..Esophagus.
B..The.fundus.
C..Urinary.bladder.
D..Colon.
..
Answer:.
..
Q..Patient.with.hoarseness,.what.is.your.first.next.step?.
A..Laryngoscopy.
B..Inner.ear.MRI.
..
Answer:.not.enough.options.but.I.will.go.with.A.
..
Q..Patient.with.mass.in.the.throat,.what.is.your.management?.
A..Surgery.plus.or.minus.radiotherapy.
..
Answer:.
..
Q..Best.xKray.for.the.ribs?.
A..Oblique.
..
Answer:A.
..
Q..About.SVC.syndrome.and.most.likely.asking.about.the.site.of.the.lesion.in.the.
mediastinum..
..
Q..Patient.came.to.the.ER.with.congested.neck.veins,.difficulty.breathing.and.absent.breath.
sounds,.what.is.the.first.step.in.his.management?.
A..Needle.thoracotomy.
..
Answer:.
..
######### #
Q..What.is.the.management.of.tension.pneumothorax?.
A..Tube.thoracotomy.
B..Needle.decompression.
..
Answer:B.
..
Q..Patient.presented.after.MVA.with.unilateral.hyperresonance.and.mediastinal.shift..XKray.
showed.hyperlucency,.what.is.the.1st.step?.
A..100%.o2.
B..Needle.thoracentesis.
..
Answer:B.
..
..
Q..Lung.cancer.with.the.best.prognosis?.
A..SCC.
B..Adenosquamous.carcinoma.
C..Adenocarcinoma.in.situ.
Answer:.C.?.
..
..
Q..Patient.came.to.ER,.complaining.of.hypotension,.tachycardia.and.hypercapnia,.
ECG.showed.arrhythmia,.what.is.your.immediate.action?.
A..Needle.decompression.
B..Pericardiocentesis.
C..FAST.ultrasound.
D..Thoracotomy.
..
Answer:B.?.
..
..
Q..You.found.nodule.6.cm.on.chest.xray.and.lab.test.show.hypercalcemia,.which.one.of.the.
following.cancers.is.associated.with.this.finding?.
A..SCC.
B..Adenocarcinoma.
..
Answer:A.
..
Q..Cancer.in.the.middle.of.esophagus,.which.type?.
A..Adenocarcinoma.
B..Scc.
..
Answer:B.
Ref:.https://www.cancer.gov/types/esophageal/patient/esophagealKtreatmentKpdq.
..
..
Q..Patient.diagnosed.with.Barrett's.esophagus.will.have?.
A..Adenocarcinoma.
B..Squamous.cell.carcinoma.
..
Answer:A.
Ref:.https://www.cancer.org/cancer/esophagusKcancer/causesKrisksKprevention/riskK
factors.html.
..
Q..In.nonKsmall.cell.lung.CA,.which.of.the.following.is.a.contraindication.to.do.surgery?.
A..Stage.IA.
B..Stage.IB.
C..Peripheral.lung.lesion.
D..SVC.obstruction.
......................................................... .
Answer:.not.sure!.
.Patients.with.stage.IIIB.or.IV.tumors.are.almost.never.surgical.candidates..
..
##############
..
..
Urology#
..
Q..Case.about.epididymo.orchitis,.what.is.the.treatment?.
..
Answer:.
Ceftriaxone.250K500.mg.IM.once.plus..
Azithromycin.1.g.PO.once.or..
Substitute.doxycycline.100.mg.PO.BID.for.7K14.days.for.the.azithromycin..
Ref:.https://emedicine.medscape.com/article/2018356Koverview.
..
Q..27.y.Man.with.progressive.enlarged.testis.no.tenderness.no.urinary.symptoms.no.sexual.hx.for.6.
months.(mostly.tumor):.
KUs.and.surgical.referral.
KAntibiotics................................................... .
KBiopsy.
K.NSAIDs.
Answer:C.
..
Q,.KStone.at.the.level.of.L3KL4.on.xKray.
AK.Ureter.
BK.Minor.calyx.
CK.Major.calyx.
DK.Renal.pelvis.
Answer:A.
Ref:.http://www.startradiology.com/internships/GeneralKSurgery/Abdomen/AbdominalKXKray/.
..
.Q..principle.treatment.of.urge.incontinence?.
AKmedical.
BKsurgical.
CKmedical.and.surgical. .
DK.Bladder.training.and.something.else.
Answer:.C.
Ref:.https://emedicine.medscape.com/article/452289Ktreatment.
..
..
Q..What.is.the.best.management.of.pt.with.urge.incontence.?.
Kegel.exercise.
Urethropexy.
NSAID.
Antibiotic.
Answer:.A.
Ref:.https://familydoctor.org/bladderKtrainingKurinaryKincontinence/.
..
Q..Which.one.of.the.following.cancers.directly.related.to.smoking?.
A..Colon.
B..Bladder.
C..Testicular.
D..Small.cell.lung.cancer.
..
Answer:B??.
..
Q..Man.doing.proctectomy.I.think.and.he.came.later.with.metastatic.cancer,.what.is.the.
most.common.site.of.metastasis?.
A..Vertebral.column.
B..Lung.
C..Kidney.
..
Answer:B.
Ref:.https://www.webmd.com/colorectalKcancer/tc/colorectalKcancerKmetastaticKorK
recurrentKtopicKoverview.
..
Q..Male.with.lumbar.pain.extending.to.the.umbilical.region,.no.urinary.symptoms,.normal.
urine.analysis,.what.is.the.dx?.
A..Epidydimo.orchitis.
B..Pyelonephritis.
C..Lumbar.muscle.spasm.
..
Answer:.not.sure!.
..
Q..Patient.with.sudden.acute.testicular.pain,.what.is.the.most.appropriate.next.step?.
A..Refer.to.surgery.
..
Answer:A.
..
Q..Patient.with.renal.mass.(loin.pain.&.hematuria.&.weight.loss),.best.next.step.is?.
A..CT.scan.
..
Answer:.
..
Q..Which.part.of.the.male.urethra.is.more.likely.to.be.injured.with.catheterization?.
A..Penile.
B..Prostatic.
C..Membranous.
D..Middle.spongy.
..
Answer:C.
..
Q..Patient.is.treating.from.erectile.dysfunction,.which.of.the.following.medications.can’t.be.
given.to.him.during.his.treatment?.
A..Nitrates.
..
Answer:A.
..
Q..Question.about.BPH,.hypertensive.but.already.on.medication..What.is.your.management?.
TURPT.
Open.prostatectomy................................................ .
Alpha.blocker**.
Answer:.C.
..
Q..Old.age.male.complaining.of.acute.fever,.dysuria,.lower.abdominal.pain,.rectal.exam.
showed.anterior.buggy.swilling,.what.is.the.diagnosis?.
A..Acute.prostatitis.
B..Chronic.prostatitis.
C..BPH.
..
Answer:A.
..
Q..Old.age.person.complaining.of.dysuria,.back.pain.,investigations.showed.high.ALP,.very.
high.PSA.what.is.the.diagnosis?.
A..BPH.
B..Prostatic.cancer.
..
Answer:B.
..
..
Q..Man.with.Premature.ejaculation..What.to.give?.
A..SSRI.
..
Answer:A.
..
Q..Question.about.BPH,.hypertensive.but.already.on.medication..What.is.your.
management?.
A..TURPT.
B..Open.prostatectomy.
C..Alpha.blocker.
..
Answer:C.
..
..
Q..Patient.with.filling.defect,.US.showed.echogenic.shadow,.which.type.of.stone?.
A..Uric.acid.stone.
..
Answer:.
..
..
Q..Ureteric.stone,.best.diagnostic.modality?.
A..CT.without.contrast.
B..CT.with.contrast.
C..Renal.US.
..
Answer:A.
..
Q..Old.man.with.difficulty.in.urination.and.back.pain.
ALP.high.
LFT.normal.
Dx?.
A..BPH.
B..Prostate.cancer.
C..Orchitis.
..
Answer:.B.bone.mets.
..
Q..Scrotal.swelling,.abdominal.pain,.exaggerated.bowel.sound,.irreducible.swelling,.next.?.
A..US.
B..Urology.referral.
C..Surgery.referral.
..
Answer:C.
..
..
Q..70.Y/O.came.with.urinary.retention,1st.step.in.the.management?.
A..Urinary.catheter.
..
Answer:A.
..
Q..Most.common.presentation.of.renal.carcinoma?.
A..Abdominal.mass.
B..Hematuria.
..
Answer:B?.
..
Q..Patient.with.recurrent.uti.presented.with.renal.stones,.what’s.the.organism?.
A..Proteus.
B..E.coli.
C..Klebsiella.
..
Answer:.
Ref:.https://www.ncbi.nlm.nih.gov/m/pubmed/3026032/.
..
Q..Patient.was.treated.for.renal.stone.before.....they.attached.picture.of.XKray.which.I.did.
not.get.he.had.to.go.for.some.surgery.so.what.will.you.do ?.

A..send.him.to.vascular.surgeon.for.follow.up.

Answer:A.

Hepatobiliary#
.
Q..Pancreatic.trauma.case,.where.is.the.first.place.for.the.pancreatic.enzyme.to.collect.in?.
A..Omental.bursa.
B..Sub.hepatic.space..
C..Sub.phrenic.space...
Answer:.A.
Note:.lesser.sac.is.the.same.as.omental.bursa..
.
Q2..Patient.with.severe.epigastric.pain.radiates.to.the.back,.labs.high.amylase,.ALP,.GGT,.
direct.bilirubin,.what.is.the.cause?.GGT.was.highly.elevated.more.than.ALP.
A..Gallstones.
B..alcoholic.pancreatitis..
Answer:.B.
.
Q..You#did#DRE#and#found#prostate#gland#swelling,#which#lobe#of#prostate#has#the#
neoplasm!?#
A... Anterior.
B... Posterior.
C... Medial.
Answer:.B..
.
Q3..Migratory.thrombophlebitis.is.seen.in?.
A..Pancreatic.cancer.
Answer:.A.
EXPLANATION:.
Trousseau's.sign.is.associated.with.gliomas,..pancreatic.or.pulmonary.malignancy.
(medscape).
.
Q4..Recurrent.cholecystitis.and.7.stones.treatment?.
A..Cholecystectomy.
.
Answer:.A.
.
..
Q5..Patient.with.upper.right.quadrant.pain.radiating.to.shoulder.with.nausea.and.vomiting.
and.she.had.same.attack.before,.ask.about.investigation?.
.
Answer:.most.likely.cholecystitis.and.US.is.the.investigation.of.choice..
.
Q..Patient.post.cholecystectomy.developed.unilateral.parotid.swelling,.saliva.was.cloudy,.
culture.was.negative,.What.does.he.have?.
A..Sarcoid.granuloma..

B..Bacterial.sialadenitis.

C..Sarcoma.

D..Sjogren's.syndrome.

Answer:.B.

https://www.ncbi.nlm.nih.gov/pubmed/18949350.

Q..New.onset.acute.pancreatitis,.which.type.of.nutrition.should.be.given?.
A..TPN.
B..Nasojejunal.tube.
Answer:.A.(NOT.SURE).
.
.
.
Plastic#Surgery#
.
Q..What.is.the.most.common.cause.of.death.in.flame.burns?.
A..Hypovolemic.shock.
B..Smoke.inhalation.
.
Answer:B.
.
Q..Facial.suture.removal.day..
A.! 3..
B.! 5..
C.! 7..
D.! 10.
Answer:.B.
https://www.emedicinehealth.com/removing_stitches/page2_em.htm.
#
Q..Patient.3.cm.lump.in.his.upper.back.,.slowly.growing.for.years...Physical.examination.:.
compressible.,.no.erythema.,.there.a.punctum.in.the.middle.that.drain.white.foul.smelling.material.,.
what.is.the.management.!?.
K.Cryotherapy.
K.Total.intact.resection✅✅✅ .
K.Antibiotic.and.resection.
Answer.:.Not.sure..
Epidermoid.cyst:.Incision.and.drainage.is.the.recommended.treatment.for.inflamed.epidermoid.
cysts,.carbuncles,.abscesses,.and.large.furuncles.
.
Q..Pt.with.filling.defect.,.us.,.ecoech.shadow.9‫ﻟﺴﺘﻮ‬$ T‫ ﻧﻮ‬R6 V C+‫ﺧﺘﻠﻔﻮ ﻓ‬$ ‫ﻟﻲ‬$ C‫ﻟﻤﻠﺰﻣ‬$ Y$‫ﻧﻔﺲ ﺳﺆ‬.:..
A.! uric.acid.stone..#!
#
Q..When.should.a.facial.suture.be.removed?.
A..3.days.
B..5.days.
C..8.days.
D..10.days..
.
Answer:.B.
.
Q..Most.common.malignant.lesion.need.to.be.removed?.
A..Erythema.gyratum.
B..Erythema.migrans.
C..Erythema.marginatum.
D..Erythema.annulare.
.
Answer:.A..
https://emedicine.medscape.com/article/1081565Kclinical.
.
Q..What.is.the.role.used.in.burns?.
A..Role.of.9.
.
Answer:.A.
.
Q..Patient.with.burn.over.the.anterior.trunk.and.circumferential.burn.on.upper.and.lower.
left.limbs..The.weight.is.70.Kg,.Calculate.parkland’s.formula?.
.
Answer:.same.q.solved.in.13.booklet.with.different.statement.
The.role.is.parkland.formula.=.TBSA.of.burn%.x.weight.(kg).x.4..
.
Q..Tender,.hard.swelling.and.oozing.after.suturing.wound.in.ankle,.what.is.the.diagnosis?..
A..Cellulitis.
B..Abscess.
C..Furuncle.
D..Carbuncle.
.
Answer:.not.sure.
.
Q..What's.the.first.step.in.managing.a.minor.burn?.
A..Apply.butter.to.affected.area.
B..Wash.with.tap.water.
C..Apply.ice..
D..Surgical.debridement.
.
Answer:.B.
First.aid.american.heart.association...
.
Q..Patient.with.lower.lip.lesion,.what.is.the.lymphatic.drainage?.
A..Submental.
B..Submandibular.
.
Answer:.A.
Reference:Snell.anatomy.
#
Q..Signs.of.good.healing?.
A..Epithelium.covering..
.
Answer:A.
.
Q..Flat.reddish.skin.lesion.in.the.left.side.of.the.face.with.dilated.dermal.vessels,.what.is.the.
diagnosis?.
A..Port.wine.stain.
B..Cavernous.hemangioma.not.flat.
C..Melanoma.not.red.
#
Answer:.A.
.
.
Q..Leg.ulcer.with.rolled.outer.edge,.what.is.the.diagnosis?.
A..SCC.
B..Basal.cell.carcinoma.
Answer:.A.
Surgery.Recall..
.
Q..Elderly.has.bedsore.that.invade.through.the.muscles,.what.is.the.stage?.
A..1.
B..2.
C..3.
D..4.
Answer:.D.
https://www.webmd.com/skinKproblemsKandKtreatments/pressureKsoresK4Kstages#2K5.
.
Q..Patient.3.cm.lump.in.his.upper.back,.slowly.growing.for.years..Physical.examination:.
compressible,.no.erythema,.there.a.punctum.in.the.middle.that.drain.white.foul.smelling.
material,.what.is.the.management?.
A..Cryotherapy.
B..Total.intact.resection.
C..Antibiotic.and.resection.
Answer:B.(not.sure).
Solved.before..
.

Q..Child.with.cutaneous.hemangioma,.those.hemangiomas.could.be.found.in.which.organ? .

A..Liver.
B..Spleen.
C..Kidney.
Answer:.A.
.
Q..Describe.neva,.What.is.the.evidence.of.naval.hyperplasia?.
A..Change.in.the.color.
B..Irregular.border.
C..All.of.the.above.
.
Answer:.not.sure.
Colorectal#Surgery#
.
.
Q..Colon.cancer.screen.recommended.grade.A,.which.age.group?.
A..45K65.
B..50K65.
C..50K75.
..
Answer:.C.
https://www.uspreventiveservicestaskforce.org/Page/Document/UpdateSummaryFinal/col
orectalKcancerKscreening.
.
Q..Young.pt.have.severe.pain.during.defection.followed.by.bleeding.and.relief.after.4.h:.
Khemarroid.
Kanal.fissure. . .
Answer:.anal.fissure. (NOT.SURE). . .
http://www.healthline.com/health/analKfissure.
.
Q..Most.common.cause.rectal.bleeding.in.age.50.years?.
A..Anal.fissure.
B..Hemorrhoids..
The.answer.should.be.diverticulosis.BUT.if.the.q.is.only.containing.these.2.choices.
Then.more.information.needed.to.differentiate.between.A.and.B....
.
Answer:(NOT.SURE).
.
Q..What.is.the.best.screening.test.for.colorectal.CA?.(no.colonoscopy.in.choices).
A..Fecal.occult.blood.testing..
B..Sigmoidoscopy.every.5.years.
C..CT.colonography.
D..Stool.DNA.
Answer:(not.sure),.could.be.A.because.sigmoidoscopy.cannot.detect.right.sided.of.the.colon.
where.42%.of.malignancies.are.found..
.
Q..Colon.cancer.stage.B2.means?.
A..No.LN.involvement.
.
Answer:.A.
Search..in.google.picture.for.AJCC.TNM.staging.system..
.
Q..Which.of.the.following.is.at.a.high.risk.for.colorectal.carcinoma?.
A..High.salt.diet.
B..Smoker.
C..Male.on.high.fat.diet.
D..Gardner.syndrome.
.
Answer:D.
.
Q..Which.one.of.these.polyps.have.potential.to.transform.to.malignant?.
A..Hyperplastic.polyp(no.risk).
B..Adenoma(most.malignamt).
C..Peutz–Jeghers(low.malignant).
D..Juvenile.
.
Answer:.B.
.
Q..k/c.case.of.chronic.constipation.complaining.of.pain.and.bleeding.during.defecation.the.
pain.persist.for.many.hours.what.is.the.diagnosis?.
A..Thrombosed.hemorrhoid.
B..Anal.fissure.
.
Answer:.A.or.B.not.clear.q.and.incomplete..
.
Q..k/c.of.chronic.constipation.complain.of.pain.during.defecation.on.examination.there.was.
fluctuating.mass.in.the.perianal.area.what.is.the.treatment?.
A..Sitz.bath.
B..Incision.and.drainage.
.
Answer:.start.with.A.then.if.it.is.large.go.directly.to.B..
.
Q..Case.about.thrombosed.pile.with.severe.pain,.used.warm.press,.sitzs.bath,.antibiotic.but.
still.pain.not.relieved,.what.to.do.next?.
A..Evacuate.the.clot.
B..Sitz.bath.
.
Answer:.A.
medscape.
.
Q..Male.with.changed.bowel.habits,.LLQ.pain,.tenderness,.swelling.and.weight.loss,.what.is.
the.most.likely.diagnosis?.
A..Diverticulosis.
B..Diverticulitis.
C..Cecum.cancer.
D..Sigmoid.cancer.
.
Answer:.NOT.SURE..
.
Q..Enlarge.medial.group.of.horizontal.Kinguinal.lymph.node.(superficial)..What.you.will.
check.1st?.
A..Anal.canal.
B..Muscle.of.thigh.
C..Muscle.of.leg.
.
Answer:.A.(not.sure).
They.receive.lymphatic.afferents.from.the.following:.
●! integument.of.the.penis.
●! scrotum.
●! perineum.
●! buttock.
●! abdominal.wall.below.the.level.of.the.umbilicus.
●! back.below.the.level.of.the.iliac.crest.
●! vulva.
●! anus.(below.the.pectinate.line).
●! the.thigh.and.the.medial.side.of.the.leg.(the.lateral.leg.drains.to.the.popliteal.lymph.
nodes.first)..
wiki.
#
General#
#
24KCase.of.necrotizing.fasciitis,.stain.showed.gram.positive.cocci.in.chains?..
1K.ampicillin.and.gentamicin..
2K.penicillin.and.clindamycin.(.I.think.correct).
3K.pepracillin.and.tazobactam.
4K.imipenem.and.metronidazole.
Answer:.2.
.
Necrotizing.fasciitis.with.Gym.+ve.strept,.what’s.the.best.Abx.combination:.
A..penicillin.and.clinda.
B..piperacillin.and.tazobactam.
ANSWER:.A.
.
Case.of.necrotizing.fasciitis,.stain.showed.gram.positive.cocci.in.chains?.
1K.ampicillin.and.gentamicin.
2K.penicillin.and.clindamycin.
3K.pepracillin.and.tazobactam.
4K.imipenem.and.metronidazole.
Aaswer:.2.
BMJ.
.
23).Stage.of.ulcer.reaching.fascia.and.extends.through.muscle:.
A..I.
B..II.
C..III.
D..IV.
Solved.previously.
.
Patient.with.stab.wound.injury.at.the.level.of.the.right..6.th.intercostal.space..Which.will.be.affected?..
1K.right.superior.lobe.
2K.right.middle.lobe.
3K.right.lower.lobe.
4K.horizontal.fissure.
answer:D.
.
Q..Patient.with.periumbilical.pain.with.anorexia.and.nausea,.labs.Increase.WBC,.dx?..
A..Acute.appendicitis.
.
Answer:.A.
.
Q..Which.organ.will.be.affected.if.you.ligate.internal.iliac.artery?..
A..Ovary.
B..Intestine.
C..Bladder.
Answer:A.
.
Q..Best.investigation.for.detection.of.retroperitoneal.hemorrhage?.
A..Computed.tomography..
.
Answer:.A.
Previous.qs.
.
Q..Patient.with.grade.1.splenic.injury,.what's.the.management?.
A..Surgical.Rx.
B..Observe.in.ICU.
C..Observe.in.surgical.ward.
D..Medical.Rx.
.
Answer:.i.could.not.find.the.CLEAR.answer.could.be..B.OR.C...
.
Q..The.genetics.of.hereditary.chronic.pancreatitis?..
A..Autosomal.dominant.
B..Autosomal.recessive.
Answer:A.
https://www.ncbi.nlm.nih.gov/pmc/articles/PMC1774562/.
.
Q..What.are.the.structures.that.could.be.injured.during.adrenalectomy?.
.A..Right.side.will.injure.IVC,.left.side.will.injure.the.pancreas.
.
Answer:.………....
.
Q..Appendicitis.histopathology?.
A..Neutrophils.in.the.muscularis.layer..
.
Answer:A.
https://goo.gl/images/ktLXvY.
.
Q..Open.cholecystostomy.incision?...
A..Kochar."subcostal".incision..
Answer:A.
.
Q..Irreversible.hernia.with.painful.abdomen,.what.it’s.called?.
A..Strangulated.hernia.
Answer:.A.
.
.
Q..Patient.presented.with.an.ulcer.on.sole.of.foot.surrounded.by.erythema,.what.is.the.best.
action?..
.
A..Oral.Abx.and.discharge.the.patient.
B..Reassurance..
C..Admission.for.debridement.and.deep.tissue.culture.
Answer:C.
..
.
. Q..Pathophysiology.of.acute.appendicitis
. A..Appendicular.obstruction
.
Answer:.A.
Recall.surgery.
.
Q..Patient.has.acute.pancreatitis.labs.shows.mild.elevation.of.TG.what.is.the.most.common.
cause.of.this.presentation?.
A..Gallstones..
B..Alcohol.intake..
C..Hypertriglycerima..
.
Answer:.NOT.SURE.(incomplete).
.
Q..Most.common.cause.of.Meckel’s.diverticulum?.
A..Terminal.duodenum.
B..Terminal.jejunum...
C..Terminal.ilium.
D..Cecum..
.
Answer:C.
MTB.
.
. Q..Pancreatic.cancer.gene?
. A..KKras
answer.:A.
https://www.hopkinsmedicine.org/health/articlesKandKanswers/askKtheKexpert/pancreaticK
cancerKexpertsKanswerKcommonlyKaskedKquestions.
. .
Q..What.is.the.characteristic.sign.of.perforated.duodenal.ulcer.?.
A..Epigastric.pain.(symptom.not.sign).
B..Melena.
C..Vomiting.
D..Steatorrhea.
.
Answer:.A.
.
. Q..Site.of.insertion.of.butterfly.needle?
. A..2nd.intercostal.space.midclavicular.line
.
Answer:..may.be.wrong.q..
.
Q..Patient.hospitalized.after.major.procedure.developed.a.small.pulmonary.embolism.which.
was.confirmed.by.CT..What.is.the.best.drug.to.remove.the.embolus?.
A..Heparin.
B..Warfarin.
C..Aspirin.
D..Streptokinase.
.
Answer:A#
#
##
Q..Patient.with.small.bowel.obstruction.after.small.bowel.surgery.one.year.ago,.what.is.the.
best.diagnostic.modality?.
A..US.
B..Barium.enema.
C..Double.contrast.barium.meal.
D..Small.bowel.barium.follow.through.
Answer:.B.
.
Q..Patient.with.right.iliac.fossa.pain,.developed.vomiting.and.nausea.with.rebound.pain.
during.palpation,.what.is.the.most.likely.cause?.
A..Diverticulitis.
B..Acute.appendicitis.
C..Crohn’s.disease.
.
Answer:.B.
#
Q..Most.common.sign.of.lower.intestinal.obstruction?.
A..Altered.bowel.motion.
B..Absolute.constipation.
C..Flatulence.
D..Diarrhea.
.
Answer:.B.
medscape.
.
Q..RTA.victim.e.closed.head.injury.and.LOC..What.is.the.best.thing.to.do.?.
A..Intubation.and.ventilation.
B..Check.the.pulses.
C..Check.the.pupils.
D..Check.the.airway.
.
Answer:.B.(not.sure).
ATLS.
#
Q..A.case.of.GERD.used.antacid.which.caused.constipation.
A..Calcium.carbonate.
B..Aluminum.hydroxide.
C..Magnesium.hydroxide.
.
Answer:.B.
..
Q..Patient.was.taking.a.drug.for.the.acidity.of.the.stomach,.now.complaining.of.
constipation,.what.is.that.drug?.
A..Aluminum.hydroxide(constipation).
.
Answer:.A.
Magnesium.hydroxide.(diarrhea)..
.
Q..Old.age.patient.smoker.complains.of.white.colored.tongue.with.ulcers,.what.is.the.
diagnosis?.
A..Squamous.cell.carcinoma.
.
Answer:A.
Squamous.cell.carcinoma.is.the.most.common.type.oral.cancer.in.smoker..(wiki).
.
Q..What.to.do.when.there.is.a.problem.in.intubation?.
A..Head.tilt.
B..Jaw.thrust..
C..Cricoid.pressure.
Answer:.incomplete.(is.it.trauma.patient.or.not)..
.
Q..Elderly.has.abdominal.pain.and.vomiting,.vitally.stable..on.abdominal.xKray.there.is.air.in.
rectum.and.dilated.loop..what.is.the.management?..
A..Nasogastric.tube.and.IV.fluid.NS.
B..Narcotic.and.IV.antibiotics..
C..Rectal.decompression.
Answer:.A(not.sure).
http://teachmesurgery.com/general/presentations/bowelKobstruction/.
.
Q..Post.surgical.wound.infection.most.likely.from?.
A..Hands.of.surgeons.
.
Answer:.q.incomplete.may.be.A..
.
Q..Patient.treated.for.peptic.ulcer.with.triple.therapy.now.she.is.free.of.symptoms..what.
you.will.do?.

A..Nothing.
B..H..Pylori.antigen.test.
C..Endoscopy.
.
Answer:.B..
https://www.uptodate.com/contents/helicobacterKpyloriKinfectionKandKtreatmentKbeyondK
theKbasics.
Q..Patient.has.an.upper.abdominal.pain, .(gastrin.and.pancreatic.enzymes.released,.and.
asking.about.the.diagnosis)?.

A..Zollinger.Ellison.syndrome.

Answer:.A.

Medscape..
.
Q..What.is.most.common.type.of.shock.associated.with.MVA:.
A..Septic.
B..Hemorrhagic.
C..Neurogenic.
.
Answer:.B.
.
.
Q..Young.male.have.epigastric.pain,.fullness,.nausea.postprandial.bloating.for.three.years,.
his.symptoms.appeared.after.he.stayed.in.India.for.one.year.for.studying,.symptoms.are.
worse.with.food..What.is.the.diagnosis?.
A..Functional.dyspepsia.
B..H.pylori.infection.
C..Esophageal.ulcer.
.
Answer:B.
. . . . . . . . .
.
Q..Presentation.of.erosion.of.gastric.ulcer.?.
A..Occult.bleeding.
B..Epigastric.pain.with.generalized.peritonitis.
Answer:.not.clear.more.details.needed..
.
Q..Old.patient.came.with.abd.pain.and.bloody.diarrhea.in.radiograph.showed.enlarge.
visceral.with.no.air.fluid.level,.what.is.the.diagnosis?.
A..Ischemic.colitis.
Answer:..........
. . . . . . . . . .
Q..Extradural.hematoma.source.of.bleeding?.
. A..Middle.cerebral.artery
. B..Middle.meningeal.artery
. C..Anterior.cerebral.artery
.
Answer:.B.
Step.up.to.medicine.page.571.
.
. . . . . . . . . . . . .
Q..Patient.with.duodenal.ulcer,.what.is.the.treatment?.
A..Triple.therapy.
B..Omeprazole.
.
Answer:q.is.unclear.if.h.pylori.suspected.go.with.A..
If.not.go.with.B..
.
Q..Patient.with.post.disease.in.the.spine.got.abscess.in.the.anterior.chest.wall,.what.is.the.
route.of.transmission?.
A..Anterior.cutaneous.
B..Posterior.cutaneous.
.
Answer:.A.
Previous.bank.in.12.and.13.booklet..
.
.
Q..Diabetic.patient.with.deep.ulcer.in.the.sole.of.foot.ask.about.management?.
A..Admission.and.debridement.
.
Answer:..
Q..Patient.have.MVA.come.to.ER.with.ulcerative.tissue.with.gas.gangrene.in.the.anterior.leg.
what.is.the.causative.organism?.
A..C..Perfringens..
B..Staph.aureus.
.
Answer:.A.
http://www.msdmanuals.com/professional/infectiousKdiseases/anaerobicK
bacteria/clostridialKsoftKtissueKinfections.
#
#
Pediatric#Surgery#
.
Q..Contraindication.of.circumcision?.
A..Hypospadias..
Answer:.A..
Toronto.note.16.edition..p1352.
.
Q..Which.in.inguinal.canal.is.develop.from.the.external.oblique.muscle?. . .
External.spermatic.fascia. . . . . . .
Internal.spermatic.fascia.
Answer:.A.
Wikipedia.,.just.write.external.spermatic.fascia..
Q...How.to.manage.uncomplicated.sigmoid.vulvulus?. . . . . .
a).flatus.tub. . . . . .
b).Nasal.tube..
c).Surgery.
.
Answer:.A.
Toronto.note.p445.
.
Q..Paraphimosis.picture...
Paraphimosis.occurs.when.the.foreskin.of.an.uncircumcised.or.partially.circumcised.male.is.retracted.
for.an.extended.period.of.time..This.in.turn.causes.venous.occlusion,.edema,.and.eventual.arterial.
occlusion..
.
Q..Complication.of.PDA.ligation?.
A..Injury.to.the.vagus.nerve.
B..Injury.to.the.phrenic.nerve.
C..Injury.to.the.recurrent.laryngeal.nerve.
.
Answer:.C.
https://www.ncbi.nlm.nih.gov/pubmed/16797086.
https://www.hindawi.com/journals/cripe/2017/2647353/.
.
Q..Long.scenario.about.hydrocele.typical.:. . . . . . .
Kfailure.of.obliteration.of.process.vaginalis.
. . . . . .
.
Q..What.is.the.most.common.urological.congenital.male.anomaly?.
A..Hypospadias.
B..Cryptorchidism..
C..Phimosis..
Answer:.B.
Wikipedia..
.
Q..How.to.diagnose.volvulus?.
A..CT..
B..Barium..
C..MRI.
.
Answer:.B.
MTB..
.
Q..Which.ligament.pass.through.inguinal.canal?.
A..Round.
B..Broad.
Answer:.A.
Repeated.in.previous.q.
Teachmeanatomy.info.and.wikipedia..
.
Q..How.to.manage.uncomplicated.sigmoid.volvulus?..
A..flat.tube..
B..Nasal.tube.
C..Surgery.
.
Answer:.the.answer.should.be.flatus.tube..
Toronto.note16.edition.page.445.
.
.
Q..Child.with.hirschsprung's.disease,..asking.about.initial.diagnostic.test?.
A..XKray.
Answer:.A.
MTB.3rd.edition..
.
Q..Paraumbilical.hernia,.playing.basket?.(Maybe.they.were.asking.about.the.diagnostic.
modality).
CT.abdomen.
Answer:.I.think.this.q.is.incorrect..
It.is.in.the.previous.bank.as.(boy.play.football.came.with.abdominal.pain.without.any.injury.
in.match.,.physical.examination.was.tenderness.in.paraumbilical.area.what.do.you.do.next:.
(RECTUS.SHEATH.HEMATOMA)..
Answer:.could.be.recheck.or.CT.abdomen.(not.sure).
Recheck.most.likely.because.it.is.clearly.related.to.exercise.not.something.else.serious...
.
Q...Initial.investigation.in.olive.mass.in.babies:.
A..abdominal.XKray.
B..abdominal.US.
C..CT.
D..barium.
Answer:.B.
MTB.3rd.edition..
.
Q..painless.rectal.bleeding.on.investigation.you.diagnosed.the.case.as.meckel's.diverticulum.,what.is.
the.commonest.site.for.it.?..
Lower.ilium.
*.cecum.
*.duodenum. . . . .
*.jejunum. . . . . . .
Answer.:.ilium.(MD.is.commonly.present.90.cm.from.illioKcecal.valve).
.
.
.
.
.
.
.
.
.
.
.
.
.
.
.
.
.
.
.
.
.
.
.
.
.
.
.

Orthopedics.
..
Q1..Patient.with.history.of.falling.on.an.outstretched.hand,.what.is.the.fracture?.
A.....Clavicle.
B.....Shoulder.
C.....Supracondylar.
D.....Colles.
Answer:.D.
..
Q2..Patient.can’t.do.foot.dorsiflexion,.which.muscle.is.responsible?.
A.....Anterior.tibialis.
..
Answer:.A.
The.foot.and.ankle.dorsiflexors.include.the.tibialis.anterior,.the.extensor.hallucis.longus.(EHL),.and.
the.extensor.digitorum.longus.(EDL).K.Medscape..
Q3..Patient.have.fracture.in.his.tibial.it.is.opened.fracture.<1cm.,.what.you.do.?.
A.....Close.reduction.with.cast.
B.....Debridement.and.open.reduction.by.intramedullary.nail..
C.....Give.him.AB.
..
Answer:.B.
Q4..Unilateral.knee.swelling..and.pain,.knee.tap.labs:.cloudy..yellow.color,..mucoid,.WBCKPMN.15.
(normal.less.than.200),.lymphocytes.80%,.what.is.the.diagnosis?.
A.....Gout.
B.....Septic.arthritis.
C.....Rheumatoid.Arthritis.
..
Answer:.B.
Q5..An.athletic.guy.lifts..heavy.weights.as.usual,.this.time.he.experienced.sudden.pain.while.lifting.
the.weight,.examination.revealed.normal.range.of.motion.in.the.shoulder,.what.to.do?.
A.....Reassure.
B.....Diclofenac.
C.....Physiotherapy.
..
Answer:.B.
Q6..Case.of.septic.arthritis.patient.started.on.oxacillin,.3.days.later.the.sensitivity.shown.up.and.it.
was.staph.aureus.resistance.to.cefoxitin,.what.is.your.management?.
A....Stop.antibiotic.
B....Continue.oxacillin.
C.....Start.vancomycin.
..
Answer:.C.
Q7..Proximal.tibial.fracture.palpable.peripheral.pulse.ABI:..85.what.to.do?.
A.....Angiography.
B.....Doppler.US.
C.....CT.
..
Answer:.A.
An.ABI.less.than.0.90.suggests.a.need.for.further.vascular.imaging:.angiography.in.a.stable.patient,.
and.operative.exploration.in.an.unstable.patient.K.Medscape..
..

Q8..Male.patient.with.pain,.effusion,.erythema,.swelling.in.both.knee.joints,.ask.about.treatment?.
A.....Aspiration.and.antibiotics.
..
Answer:.A.
Q9..Patient.with.multiple.fractures,.O/E.sky.blue.sclera,.X.ray.showed.multiple.healing.fractures.with.
callus.formation?.
A.....Osteogenesis.imperfecta.
..
Answer:.A.
Q10..Old.patient.with.neck.pain.and.loss.of.sensation.in.the.left.arm,.best.step.in.Dx?.
A....Spine.MRI.
..
Answer:.A.
Q11..What.is.the.muscle.responsible.for.unlocking.the.knee?.
A.....Popliteus.
..
Answer:.A.
Q12..Stab.wound.in.the.buttocks.there.is.defect.in.the.lateral.rotation.of.the.leg?.
A.....Gluteus.maximus.
B.....Adductor.longus.
..
Answer:.A.
Q13..Commonest.cause.of.olecranon.bursitis?.
A.....Repetitive.trauma.
..
Answer:.A.
Q14..Patient.complaining.of.pain.in.medial.epicondyle,.he.is.a.golf.player,.what.is.the.management?.
A.....Refine.golfer.
..
Answer:.?.
RICE.(rest,.ice,.compression.and.elevation),.elbow.brace,.and.the.last.option.is.surgery:.
fascial.elevation.and.tendon.origin.resection.(FETOR).
Q15..Pain.in.the.wrist.with.positive.tinel's.sign.which.nerve.is.affected?.
A.....Median.nerve.
Answer:.A.
..
Q16..Patient.with.thenar.muscle.atrophy.what.is.the.nerve.affected?.
A.....Median.nerve.
..
Answer:.A.
Q17..Rickets.vs..hypophosphatemia.
..
Answer:.
Rickets:.low.calcium,.low.vitamin.D,.high.PTH,.low.phosphorus.and.high.ALP..
Hypophosphatemic.rickets:.
calcium.levels.may.be.within.or.slightly.below.the.reference.range;.alkaline.phosphatase.levels.will.
be.significantly.above.the.reference.range,.serum.parathyroid.hormone.levels.are.within.the.
reference.range.or.slightly.elevated,.while.calcitriol.levels.are.low.or.within.the.lower.reference.
range..Most.importantly,.urinary.loss.of.phosphate.is.above.the.reference.range..
..
Q18..Pagets.vs..osteoporosis.vs..vit.d.deficiency.
..
Answer:.
K.Paget’s.disease:.serum.calcium.and.phosphate.=.within.the.reference.range..
Hyperuricemia,.serum.total.acid.phosphatase.is.high,.bone.specific.alkaline.phosphatase.is.high..
..
K.Osteoporosis:.levels.of.serum.calcium,.phosphate,.and.alkaline.phosphatase.are.usually.normal.in.
persons.with.primary.osteoporosis,.although.alkaline.phosphatase.levels.may.be.elevated.for.several.
months.after.a.fracture..
Inadequate.vitamin.D.levels.can.predispose.persons.to.osteoporosis..
DXA.scan.is.currently.the.criterion.standard.for.the.evaluation.of.BMD..
..
K.Vitamin.D.deficiency:.low.25Khydroxyvitamin.D.or.25(OH)D..
Q19..Boutonniere.deformity.description?.
A.....PIP.flexion.and.DIP.hyperextension.
..
Answer:.A.
Q20..Athlete.can’t.plantar.flex.the.foot,.where.is.the.injury?.
..
Answer:.could.be.achilles.tendon.injury,.they.present.with.weakness.in.foot.plantar.flexion..
..
Q21..Patient.can't.extend.wrist.and.finger,.nerve.injured?.
..
Answer:.Radial.nerve.
..
Q22..Where.can.you.palpate.dorsalis.pedis.artery?.
A.....Lateral.to.the.extensor.hallucis.longus.
..
Answer:.A.
Q23..Child.with.hip.pain,.xKray.was.normal.but.US.showed.fluid..Lab.revealed.high.ESR,.and.CRP,.
otherwise.normal..What.to.do?.
A.....MRI.
B.....CT.hip.and.pelvis.
C......Aspiration.
..
Answer:.C.
Q24..Athlete,.Heel.pain.in.the.morning,.what’s.the.dx?.
A.....Plantar.fasciitis.
..
Answer:.A.
They.present.with.sharp.heel.pain.worse.in.the.morning.when.first.getting.out.of.bed..
Q25..Posterior.shoulder.dislocation?.
..
Answer:.pain.with.flexion.+.adduction.and.internal.rotation.of.the.arm.+..posterior.dislocation.may.
leave.humeral.head.in.subacromial.position..
..
Q26..Treatment.of.spinal.stenosis?.
..
Answer:.analgesics,.anti.inflammatory,.physical.therapy,.and.lastly.surgical.decompression..
Surgery.is.indicated.when.the.signs.and.symptoms.correlate.with.the.radiologic.evidence.of.spinal.
stenosis..Generally,.surgery.is.recommended.when.significant.radiculopathy,.myelopathy.
(cervicothoracic),.neurogenic.claudication.(lumbar),.or.incapacitating.pain.is.present..
..
Q26..Perths.2.times.(xray/.scenario)..
..
Answer:.
Legg[Calve[Perthes#Disease#(Coxa#plana):.it.is.an.idiopathic.avascular.necrosis.of.the.proximal.
femoral.epiphysis.in.children..4K8.years.is.most.common.age.of.presentation,.male.to.female.ratio.is.
5:1,.up.to.75%.of.affected.patients.have.some.form.of.coagulopathy..
Bone.age.is.delayed.in.89%.of.patients.
Younger.age.(bone.age).<.6.years.at.presentation.is.most.important.good.prognostic.indicator..
[#Symptoms:#insidious#onset,#may#cause#painless#limp,#intermittent#hip,#knee,#groin#or#thigh#pain.#
K.Signs:.hip.stiffness,.loss.of.internal.rotation.and.abduction,.gait.disturbance;.antalgic.limp.and.
Trendelenburg.gait..
..
[#X[ray:#medial#joint#space#widening#(earliest)#from#less#ossification#of#head,#cresent#sign#
(represents#a#subchondral#fracture).#MRI#is#more#sensitive.#
..
K.Treatment.is.nonoperative;.observation.alone,.activity.restriction.(nonKweight.bearing),.and.
physical.therapy.(ROM.exercises).in.children.<.8.years.of.age.(bone.age.<6.years).(they.don’t.benefit.
from.surgery)..
Operative:.femoral.and/or.pelvic.osteotomy.for.children.>.8.years.of.age..
(Orthobullets)..
..
Q27..Osgood.schlatter.disease..
..
Answer:.
OsgoodKSchlatter.disease.(Tibial.Tuberosity.Avulsion.K.Traction.Apophysitis)..
Anterior.knee.pain,.often.bilateral.(around.25K50%.of.cases),.of.tibial.tuberosities.in.adolescent.
children.(10K11.in.girls.and.13K14.in.boys)..
Caused.by.repetitive.stress.from.the.quadriceps.tendon.pulling.on.the.tibial.tuberosities.during.rapid.
growth.spurts..Sports.with.jumping,.running.and.kneeling.making.it.worse,.and.it.improves.with.rest..
P/E:.pain.with.palpation.over.the.tibial.tuberosities,.and.reproduced.with.resisted.knee.extension..
No.need.for.imaging..
Treatment.is.conservative..
(Master.the.Boards).
..
Q28..Patellar.chondromalacia.
..
Answer:.
Condition.characterized.by.idiopathic.articular.changes.of.the.patella..more.commonly.grouped.
together.with.a.number.of.pathological.entities.known.as."anterior.knee.pain".or."patellofemoral.
syndrome"..
Symptoms:.
K.Diffuse.pain.in.the.peripatellar.or.retropatellar.area.of.the.knee.(major.symptom).
K.Insidious.onset.and.typically.vague.in.nature.
K.Aggravated.by.specific.daily.activities.including:.climbing.or.descending.stairs,.prolonged.sitting.
with.knee.bent.(known.as.theatre.pain),.squatting.or.kneeling..
P/E:.
K.Quadricep.muscle.atrophy.
K.Signs.of.patella.maltracking.
K.Palpable.crepitus.
K.Pain.with.compression.of.patella.with.knee.range.of.motion.or.resisted.knee.extension..
Treatment:.rest,.rehab.and.NSAIDS.(mainstay.of.treatment.and.should.be.done.for.a.minimum.of.
one.year)..
Operative.treatment:.https://www.orthobullets.com/sports/3022/idiopathicKchondromalaciaK
patellae.
(Orthobullets).
..
Q29..Medial.collateral.ligament?.
..
Answer:.you.can.test.it.with.valgus.stress.test..
..

Q30..question.about.a.boy.who.pushed.his.friend.from.the.back.10.hours.later.the.boy.came.to.ER.
complaining.of.severe.pain.what.is.the.ligament.affected?.
A.....Iliofemoral.
B.....Ischio….
..
Answer:.?.
Incomplete.question.
..
Q31..Which.tendon.passes.through.lesser.sciatic.foramen?.
A.....Obturator.Internus.tendon.
..
Answer:.A.
..
Q32..Diagnosis.of.osteoporosis..based.on.DXA.scan.?.
A....K3.5.
B....2.
C....3.5.
D....2.
..
Answer:.less.than.K.2.5.
..

Q33..Treatment.of.elderly.with.osteoporosis?.
A.....Alendronate.
B.....Vit.D.
..
Answer:.A.
..
Q34..Patient.with.lytic.lesion.in.the.femur,.his.prostate.test.came.normal..Does.he.have.Paget’s.
disease.or.not?.
..
Answer:..I.have.no.idea.what.they’re.asking.about.
K.Paget’s.disease:.serum.calcium.and.phosphate.=.within.the.reference.range..
Hyperuricemia,.serum.total.acid.phosphatase.is.high,.bone.specific.alkaline.phosphatase.is.high..
Q35:.Components.of.the.ankle.joint?.
A.....Distal.tibia,.fibula,.talus.
..
Answer:.A.
Q36..Patient.cannot.move.her.hand,.where.is.the.defect?.
A....Anterior.interosseous.
B....Posterior.interosseous.
C....Thenar.muscles.
..
Answer:.
Anterior.interosseous.compressive.neuropathy:.weakness.of.grip.and.pinch,.specifically.thumb,.
index.and.middle.finger.flexion,.patient.unable.to.make.OK.sign.(test.FDP.and.FPL),.pronator.
quadratus.weakness.shown.with.weak.resisted.pronation.with.elbow.maximally.flexed..
(Orthobullets).
..
Posterior.interosseous.compressive.neuropathy:.finger.metacarpal.extension.weakness.and.wrist.
extension.weakness;.inability.to.extend.wrist.in.neutral.or.ulnar.deviation,.the.wrist.will.extend.with.
radial.deviation.due.to.intact.ECRL.(radial.n.).and.absent.ECU.(PIN)..
(Orthobullets).
..
Thenar.muscle.action:.opposes.the.thumb,.by.medially.rotating.and.flexing.the.metacarpal.on.the.
trapezium..
(Techmeanatomy).
..
Q37..Diabetic.patient.experiences.pain.with.stretching.the.second.and.third.fingers.and.she.cannot.
move.her.hand..(picture.attached)..What.is.the.diagnosis?.
.Answer:.depends.on.the.picture.
..
Q38..Post.trauma.patient.cannot.flex.the.distal.phalanx,.which.tendon.is.ruptured?.
.Answer:.Flexor.digitorum.profundus.
..
Q39..When.the.blood.supply.in.the.superficial.palmar.arch.is.weak,.which.artery.is.responsible?.
..
Answer:.Ulnar.artery.
The.superficial#palmar#arch.is.formed.predominantly.by.the.ulnar.artery,.with.a.contribution.from.
the.superficial.palmar.branch.of.the.radial.artery..
(Wikipedia).
..
Q40..Function.of.the.anterior.compartment.forearm.muscles?.
A.....Elbow.flexion.
B.....Elbow.extension.
C.....Hand.and.wrist.flexion.
D.....Hand.and.wrist.extension.
.Answer:.C.
..
Q41..Picture.of.humeral.fracture,.then.the.patient.had.weakness.in.extension..Which.nerve.in.
injured?.
..
Answer:.radial.nerve.
..
Q42..Patient.with.hip.dislocation.and.the.leg.is.externally.rotated,.which.muscle.is.responsible.for.
the.external.rotation?.
..
Answer:.gluteus.maximus.
Q43..13KyearKold.boy.felt.down.and.came.with.forearm.swelling.and.pain..XKray..pic.provided,.
something.special.on.x.ray.that.is.the.growth.plates.are.widely.separated?.
A....Salter.Harris.fracture.
..
Answer:.A.
..
Q44..Septic.arthritis,.culture.shows.gram.positive.cocci.in.clusters,.the.patient.was.given.Cloxacillin.
with.no.improvement.after.5.days,.what.will.you.do?.
..
Answer:.Start.Vancomycin.
Q45..Squash.player.with.elbow.pain,.what.is.the.diagnosis?.
..
Answer:.lateral.epicondylitis.
..
Q46..Old.female.with.back.pain.relieved.with.leaning.forward.and.walking.uphill,.peripheral.pulses.
are.intact,.what.is.the.diagnosis?.
..
A......lumbar.spinal.stenosis.
..
Answer:.A.
Q47..Case.of.carpal.tunnel.syndrome,.the.pt.is.a.typist.working.on.computer,.nerve.affected?.
A.....Median.nerve.
..
Answer:.A.
Q48..Non.athlete.forcefully.plantar.flexed.his.foot.and.heard.a.click.sound.on.his.leg,.which.tendon.is.
affected?.
A.....Calcaneal.tendon.
..
Answer:.A.
Q49..Which.muscle.would.be.completely.paralyzed.by.obturator.nerve.injury?.
A.....Gluteus.Maximus.
B.....Adductor.magnus.
C.....Adductor.longus.
..
Answer:.B&C.!!.(I.think.one.of.the.answers.was.added.by.the.one.who.wrote.the.question).
Obturator.nerve.supplies:.external.obturator,.adductor.longus,.adductor.brevis,.adductor.magnus,.
gracilis.and.the.pectineus.(inconstant)..
(Wikipedia).
Q50..What.is.the.cause.of.carpopedal.spasm.?.
A.....Low.Ca.
..
Answer:.A.
Acute.secondary.hypocalcemia.can.result.in.carpopedal.spasm,.muscle.twitching,.a.prolonged.QT.
interval,.and.positive.Chvostek.and.Trousseau.signs..
(Medscape).
Q51..Drop.foot.&.loss.of.sensation.in.1st.&.2nd.digit,.which.nerve.is.affected?.
A.....Deep.peroneal.nerve.
..
Answer:.A.
Deep.peroneal.(fibular).nerve.supplied.the.skin.of.the.adjacent.sides.of.the.big.and.second.toes..
Q52..Knee.pain.increase.by.walking.on.the.stairs.relieved.by.walking.on.flat.floor.this.pain.
aggravated.by.contracting.quadriceps.muscle,.what.is.the.cause?.
A.....Osteoarthritis.
..
Answer:.#Patellar#femoral#syndrome##
.
Q53..Presentation.of.posterior.hip.dislocation?.
..
Answer:.hip.and.leg.in.slight.flexion,.adduction,.and.internal.rotation..
Q54..Patient.got.numbness.and.electrical.like.pain.of.the.left.foot.up.to.knee,.so.at.which.level.is.the.
disc.?.
A.....L3KL4.
. .
Answer:.L4[L5#
..
Q55..Patient.fall.down.over.his.leg,.xKray.shows.comminuted.fracture.of.the.tibia.and.fibula,.the.
dorsalis.pedis.pulse.is.normal,.sensation.normal.what.is.the.next.step?.
A....Doppler.
B....MRI.
..
Answer:.A.
Q56..Most.common.nerve.injured.in.humerus.fracture?.
A.....Radial.nerve.
B.....Axillary.nerve.
C.....Ulnar.nerve.
D.....Median.nerve.
..
Answer:.B.
Q57..What.is.the.type.of.wrist.joint?.
A.....Pivot.
B.....Hinge.
..
Answer:.Condyloid.joint.
Q58..Humeral.neck/head.fracture,.which.nerve.would.be.affected?.
A.....Axillary.
B.....Radial.
C.....Ulnar.
D.....Median.
..
Answer:.A.
Q59..Lachman.test.assess.which.of.the.following.structures?.
A.....Anterior.cruciate.ligament.
..
Answer:.A.
Q60..Loss.sensation.of.the.anterior.and.posterior.aspect.of.the.hand,.which.nerve.is.affected?.
A....Axillary.
B....Radial.
C....Ulnar.
D....Median.
..
Answer:.C.
..
Q61..Loss.sensation.of.the.little.and.ring.finger,.which.nerve.is.affected?.
A....Axillary.
B....Radial.
C....Ulnar.
D....Median.
..
Answer:.C.
Q62..Patient.complains.of.neck.pain.and.tingling.that.radiates.to.his.left.shoulder.and.arm.with.loss.
of.sensation.of.the.arm.following.the.pain.most.likely.dx?.
A.....Cervical.disc.prolapse.
..
Answer:.A.
Q63..Girl.with.unilateral.knee.pain.and.high.fever,.no.hx.of.trauma,.knee.aspiration.results.showed.
high.wbcs,.what.to.do?.
A....Surgical.consultation.and.IV.antibiotic.
B....Oral.antibiotic.for.5.days.
C....IV.antibiotic.for.2.weeks.
D....Antipyretic.and.wait.for.lab.results.
..
Answer:.A.
..
Q64..onion.skin,.periosteal.elevation,.progressive.pain.for.2.weeks?.
A.....Blood.culture.
B.....IV.antibiotics.
C.....MRI.
..
Answer:.C.
It’s.ewing’s.sarcoma.
Q65..Patient.had.MVA.before.3.day.today.complaining.of.mild.neck.pain.with.stiffness?.Without.
neuro.deficit?.Initial.workup.
A.....XKray.
B.....MRI.
C.....CT.
..
Answer:.A.
..
Q66..How.to.diagnose.scoliosis?.
A.....XKray.
B.....Adam’s.test.
C.....MRI.
..
Answer:.A.
..
Q67..Which.degree.of.scoliosis.you.should.do.surgery?.
A.....10.
B.....15.
C......5.
..
Answer:.cobb.angle.>.45°.
(Orthobullets).
for.surgery.if.more.than.40K45.or.symptomatic..
for.ortho.referral.if.more.than.15K20.
.
Q68..Wrist.pain.with.+ve.finkelstein's.test.and.–ve.phalen.test.how.to.treat?.
A....Split.for.the.thump.
B....Split.for.the.wrist.
..
Answer:.A.
First.line.treatment.of.De.Quervain's.tenosynovitis.is:.rest,#NSAIDS,#thumb#spica#splint,#steroid#
injection.#
(Orthobullets)#
..
Q69..Elderly.male.with.history.of.3.fracture.in.2.years..What.is.the.management?.
A....Alendronate.
B....Teriparatide.
C....Selective.oestrogen.receptor.modulators.
..
Answer:.A.
Q70..Fracture.of.the.head.and.neck.of.the.fibula,.which.nerve.is.injured?.
A.....Common.peroneal.
B.....Deep.perineal.
C.....Superficial.
..
Answer:.A.
..
Q71..a.picture.of.complete.fracture.a.small.wound,.management?.
A.....Surgical.detriment.+.intramedullary.nail.
B.....Closed.reduction.+.external.fixation.
..
Answer:.A.
..
Q72..Patient.post.motor.collision.3.days.back.came.with.neck.pain.which.started.early.as.mild..Now.
he.has.neck.pain.and.stiffness.that.radiates.to.the.arms..Which.one.of.the.following.is.most.likely.
diagnosis?.
A.....Disc.prolapse.
B.....Spondylosis.
C.....Subluxation.
..
Answer:.C.
Q73..Picture.of.fracture.of.both.ulnar.and.radial,.management?.
A....Warm.compression.....
B....Rest.and.reassurance.
C....Reduction.and.casting.
..
Answer:.C.
Q74..Best.way.to.prevent.fracture.in.osteopenic.postmenopausal.lady?.
A.....Daily.Vit.D.
..
Answer:.Bisphosphonate.or.calcium.+.vitamin.D.
http://clinicalevidence.bmj.com/x/systematicKreview/1109/overview.html.
..
Q75..Old.patient.did.DEXA.scan.with.picture.of.osteoporosis.had.generalized.bone.pain.and.bone.
tenderness.and.high.Alkaline.phosphatase,.what.is.Dx?.
A.....Osteoporosis.
B.....Osteomalacia.
C.....Paget’s.disease.
D.....Metastasis.
..
Answer:.A.
Q78..Back.pain.assessment.and.importance.of.onset.and.duration..
A....Short.onset.is.important.for.biological.origin..
B....Location.and.onset.are.important.for.the.biological.origin.
C....Location.and.onset.are.important.for.the.outcome.
..
Answer:.A.(not.sure).
Q79..Features.of.osteoarthritis.in.xKray..
..
Answer:.joint.space.narrowing,.osteophytes,.eburnation.of.bone,.subchondral.sclerosis,.and.
subchondral.cysts..
(Orthobullets).
..
Q80..5KyearKold.girl.diagnosed.perthes.disease,.treatment?.
A....Surgery.
B....No.weight.bearing.for.6.months.
..
Answer:.B.(see.above).
Q81..In.which.position.will.you.apply.a.splint.in.patients.with.carpal.tunnel.syndrome?.
A....Dorsiflexion.
B....Plantarflexion.
C....Extension.
D....Abduction.
..
Answer:.C.
Q82..Young.boy.comes.with.knee.pain..whole.scenario.is.given..is.sporty..diagnosis.of.Osgood.
schlatter.is.made..the.knee.pain.is.due.to?.
A....Traction.
B....Tear.of.medial.ligament.
C....Cruciate.lig.tear.
D....Bursitis.
..
Answer:.maybe.A.
(The.pain.is.caused.by.stress.from.the.quadriceps.tendon.pulling.on.the.tibial.tuberosities.during.
rapid.growth.spurts)..
..
Q83..Child.tripped.and.twisted.his.leg.now.he.can’t.walk,.what.is.the.cause?.
A....Spiral.fracture.of.proximal.tibia.
B....Spiral.fracture.of.femur.
C....Chop.fracture.of.tibia.
..
Answer:.A.
Q84..Tingling.and.numbness.of.the.ring.and.index.finger.worsen.with.elevation.of.the.arm.positive.
stress.test?.
A....Carpal.tunnel.syndrome.
B....Thoracic.outlet.obstruction.
C....Ulnar.artery.thrombophlebitis.
.Answer:.B.
.
Q85..Pain.at.the.midline.of.the.plantar.surface.when.walking.and.standing?.
A....Plantar.fasciitis.
B....Flexor.digitorum.longus.
C....Flexor.hallucis.longus.
.Answer:.A.(not.sure).
..
Q86..Loss.of.adduction.of.the.fingers.which.nerve.injury?.
A...Ulnar.
B....Radial.
C....Median.
D....Axillary.
.Answer:.A.
.
Q87..Patient.cannot.extend.his.leg.at.knee.joint.which.muscle.is.responsible?.
A.....Sartorius.
B.....Quadriceps.
C.....Biceps.femoris..
Answer:.B.
.
Q88..Radius.+.ulna.fracture,.what.to.do.?.
A....Irrigation.
B....Closed.reduction.
..
Answer:.you.need.to.know.the.type.of.the.fracture.+.the.location.of.the.fracture.rather.proximal.or.
distal.+.the.description.of.the.wound..
.
Q89..Case.about.patient.with.osteoarthritis.had.enlargement.of.the..distal.interphalangeal.joint.what.
is.it?.
A....Heberden's.node.
B....Bouchard.node.
..
Answer:.A.
..
Q90..Type.of.hip.joint:.
A.....Ball.and.socket.joint.
..
Answer:.A.
..

Q91..30KyearKold.patient.presents.with.back.pain.and.tenderness.when.palpating.the.paraspinal.
muscle,.neurovascular.was.normal.and.ask.about.treatment?.
A....Physiotherapy.
B....Biofeedback.
C....Surgery.
..
Answer:.A.
..
Q92..Pain.in.snuff.box.most.likely.indicate.fracture.in.which.of.the.following?.
A....Scaphoid.
..
Answer:.A.
..
Q93..Boy.with.hip.pain.x.ray.shows.degenerative.and.collapse.of.femoral.neck.what.the.diagnosis?.
A....Perthes.disease.
..
Answer:.A.
..

Q94..2KyearKold.child.fell.on.stairs,.presented.with.spiral.tibial.fracture.with.multiple.contusions.on.
various.healing.stages.what.you're.going.to.do?.
A....Hospitalization.and.call.the.social.services.
B....Referral.to.orthopedics.
C....Cast.and.discharge.
..
Answer:.A.
Could.be.child.abuse.
..

Q96..Case.of.osgood.schlatter.disease.came.with.bilateral.knee.swelling,.what.is.the.cause?.
A....Osteochondritis.
B....Hemorrhage.in.the.bursa.
C....ACL.tear.
..
Answer:.A.(most.likely,.I.did.my.research.and.I.honestly.don’t.know.the.right.answer).
Osteochondritis.dissecans.is.an.intraKarticular.osteochondrosis.of.unknown.etiology.that.is.
knee,.the.medial.femoral.condyle.is.most.commonly.affected..
characterized.by.degeneration.and.reKcalcification.of.articular.cartilage.and.underlying.bone..In.the.
tenderness.along.the.involved.chondral.surface..A.mild.joint.effusion.may.be.present..
also.may.be.reported..On.physical.examination,.the.patient.may.demonstrate.quadriceps.atrophy.or.
effusion..If.a.loose.body.is.present,.mechanical.symptoms.of.locking.or.catching.of.the.knee.joint.
The.patient.reports.vague,.poorly.localized.knee.pain,.as.well.as.morning.stiffness.or.recurrent.
(American.family.physician).

..

Q97..Which.drug.of.the.following.used.in.the.treatment.of.osteoporosis.and.cause.constipation?.
A....Bisphosphonates.
B....Denosumab.
C....Cinacalcet.
..
Answer:.A.
..
Q98..Worker.with.elbow.pain,.exaggerated.by.hammer.use..What.is.the.cause?.
A....Lateral.epicondylitis.
B....Biceps.tendonitis.
..
Answer:.A.
..
Q99..Female.wearing.a.high.heel.falls.her.leg.moved.outward.(.or.something),.what.is.the.injured.
ligament?.
A....Lateral.collateral.ligament.
B....Deltoid.ligament.
C....Anterior.cruciate.ligament.
..
Answer:.B.
An.eversion.ankle.sprain.is.a.tear.of.the.deltoid.ligaments..
.
Q100..Nerve.supply.of.the.calf.muscle?.
A....Tibial.nerve.
B....Femoral.
.Answer:.A.
..
Q101..Type.of.joint.between.vertebra?.
A....Cartilaginous.joint.
..
Answer:.A.
..
Q102..Superficial.layer.of.the.sole.of.the.foot.had.transverse.cut,.which.structure.affected?.
A....Tibialis.posterior.
B....Abductor.hallucis.
..
Answer:.B.
..
Q103..Injury.to.neck.of.fibula,.loss.of.dorsiflexion.and.there's.slight.eversion,.which.nerve.is.
affected?.
A....Deep.peroneal.nerve.
B....Common.peroneal.nerve.
..
Answer:.B.
..
Q104..Long.scenario.about.median.nerve.injury..What.is.the.abnormality.you.expected.to.see.in.
hand.?.
A....Ape.hand.
B....Claw.hand.
C....Wrist.drop.
..
Answer:.A.
..
Q105..Injury.leads.to.loss.of.sensation.in.the.medial.2.fingers..What.is.the.nerve.injured.?.
A....Ulnar.nerve.
B....Median.nerve.
C....Brachial.nerve.
Answer:.A.
..
Q106..Female.typing.on.computer.giving.hx.of.carpal.tunnel.syndrome,.what’s.the.accurate.test?.
A....Tinel’s.test.
B....Compression.test.
..
Answer:.B.
..
Q107..Patient.presented.with.bilateral.knee.pain.and.on.examination.you.found.joint.crepitus.and.
muscle.wasting.around.knee?.
A....Rheumatoid.Arthritis.
B....Osteoarthritis.
C....Chronic.pain.syndrome.or.something.like.that.
..
Answer:.B.
..
Q108..Player.develop.pain.with.movement,.got.better.with.rest..What.is.the.diagnosis?.
A....Plantar.fascitis.
B....Calcaneus.bone.spur.
C....No.tarsal.tunnel.in.the.choices.
..
Answer:.A.(could.be).
You.cannot.judge.based.on.the.given.information.only..
..

Q109..Relation.of.femoral.vein.to.artery.
A....Medial.
Answer:.A.
Medial.to.lateral:.vein,.artery,.nerve..
..
Q110..Which.of.these.joint.is.not.hinge.joint?.
A....Knee.joint.
B....Ankle.joint.
C....Elbow.joint.
D....Hip.joint.
..
Answer:.D.
A.hinge.joint.is.a.common.class.of.synovial.joint.that.includes.the.ankle,.elbow,.knee,.knee..Jaw,.
Finger,.and.toe.joints..Hinge.joint.are.formed.between.two.or.more.bones.can.only.move.along.one.
axis.to.flex.or.extend..
Q111..Boy.presented.with.limping.which.was.initially.painless.the.became.painful.relieved.by.
rest+xray?.
A....AVN.
B....Slipped.capital.femoral.fracture.
..
Answer:.A.
..
Q112..Patient.fell.on.outstretched.arm.4.weeks.ago.presented.by.weak.abduction.and.external.
rotation,.xray.normal,.they.gave.a.picture.of.an.MRI?.
A....Shoulder.dislocation.
B....Impingement.syndrome.
C....Rotator.cuff.injury.
..
Answer:.C.
The. presentation. suggests. rotator. cuff. injury. although. the. most. common. injuries. with. falling. on.
outstretched. arm. is. scaphoid. and. distal. forearm. fractures. +. the. MRI. attached. should. gives. you. a.
hint..
..
Q113..Blood.supply.of.post.compartment.of.the.leg?.
..
Answer:.popliteal#artery#is.the.continuation.of.the.femoral.artery.in.the.popliteal.fossa..It.descends.
across.the.popliteus.muscle.and.at.its.lateral.border,.divides.into.the.anterior.and.the.posterior.tibial.
arteries..
..
The.posterior#tibial#artery.gives.off.the.fibular.(peroneal).artery.and.then.descends.in.the.deep.
posterior.compartment.of.the.leg,.accompanied.by.the.tibial.nerve..It.passes.behind.the.medial.
malleolus.of.the.ankle.and.into.the.sole.of.the.foot.where.it.divides.into.the.medial.and.lateral.
plantar.arteries..(Similarly,.the.tibial.nerve.divides.into.medial.and.lateral.plantar.branches.here.).
..
The.fibular#(peroneal)#artery#is.the.muscular.artery.of.the.fibular.side.of.the.leg..It.descends.near.the.
fibula,.within.the.substance.of.hallucis.longus.muscle,.in.the.deep.posterior.compartment..It.also.
serves.as.a.large.collateral.vessel,.for.near.the.ankle.it.is.connected.by.a.horizontal.communicating.
branch.with.the.posterior.tibial.artery.and.by.a.perforating.ramus.with.the.anterior.tibial.artery..
..
Q114..70.years.old.female.with.left.hip.pain.while.using.stair,.on.examination.all.range.of.motion.is.
normal.except.abduction,.what.is.the.next.step:.
A....Radiographs.
B....NSAIDs.
C....physiotherapy.and.analgesia.
..
Answer:.A.
..
Q115..Patient.came.after.bone.fracture.how.to.check.the.bone.density.after.treatment?.
A....Vitamin.D.
B....Calcium.................................................. .
C.....XKray.Pelvic.and.spine................................................... .
D....High.energy.X.ray.(DEXA).
..
Answer:.D.
..
Q116..50.years.Female,.tall,.thin.with.no.comorbidity.her.mother.died.during.orthopedic.
surgery(replace.of.hip.joint),.she.asked.you.about.the.risk.?.UNCLEAR.Q.
A....Bone.something.
B....Bone.something.with.exogenous.estrogen.
C....Exogenous.estrogen.
..
Answer:.I.don’t.know.
..
Q117..Athletic.run.3.k.and.suffered.from.leg.pain.not.relieved.by.NSAID.or.ice.bag,.x.ray.normal.?.
A....Stress.fracture.
B....Osteosarcoma.
..
Answer:.A.
..
A118..How.to.diagnose.rib.fracture?.
A....PA.Chest.xKray................................................... .
B....AP.Chest.xKray... ................................. .
C....Oblique.chest.xKray.
D....Erect.position.xKray.
..
Answer:.C.
..
Q119..Patient.have.back.pain,.lumbar.pain.and..tenderness,.have.high.renal.function.test,.high.IgG,.
low.IgM.and.IgA..What.is.the.diagnosis?.
A....Multiple.myeloma.
..
Answer:.
..
Q120..Case.of.ESTABLISHED.OSTEOPOROSIS,.what.to.give.to.prevent.osteoporotic.fractures?.
A.....Calcitonin.............................................. .
B.....Bisphosphonate.
..
Answer:.B.
Bisphosphonates.are.of.proven.benefit.in.the.prevention.of.fragility.fractures...In.addition.to.reduced.
fracture.risk,.interventions.can.improve.quality.of.life.and.reduce.mortality.in.patients.with.fragility.
fractures.Recommendations.for.administration.of.bisphosphonates.include.postmenopausal.women.
aged.65–75.years.if.osteoporosis.is.confirmed.by.dualKenergy.xKray.absorptiometry.scan,.and.older.
women.without.the.need.for.a.scan..
(Medscape).
..
Q121..8.y/o.child.presented.with.hip.pain,.limping,.stiffness.for.the.past.5.months..He.denied.any.
history.of.trauma..O/E.there.was.a.fixed.flexion.deformity.of.30.degrees.with.limited.internal.
rotation..Vital.signs..were.normal..What.is.most.likely?.
A....Developmental.Dysplasia.of.the.Hip.
B....Slipped.Capital.Femoral.Epiphysis.
C....LeggKCalveKPerthes.disease.
D....Missed.septic.arthritis.
..
Answer:.C.
..
Q122..Runner.felt.pain.anterior.to.the.heel.which.worsens.in.early.morning.but.gets.better.
throughout.the.day,.what.is.your.Dx?.
A....Plantar.fasciitis.
B....Calcaneal.heel.spur................................
C....Calcaneal.Fracture.
..
Answer:.A.
..
Q123..Atrophy.of.thenar.muscle,.sensation.is.intact,.which.nerve.is.affected?.
A....Radial .
B....Ulnar.
C....Median .
Answer:.C.
..
Q124..Osteoporosis.most.commonly.due.to?.
A....Aging.
.Answer:.A.
..
Q125..Patient.can't.extend.wrist.and.finger,.nerve.injured?.
A....Radial.nerve.
Answer:.A.
..
Q126..Child.had.trauma.then.developed.knee.pain.and.swelling.with.tenderness.on.passive.
movement,.next.step.?.
A.....Blood.culture.
B.....Empirical.Abx.
C.....Knee.x.ray.
D.....Examination.of.synovial.fluid.
..
Answer:.C.
..
Q127..Male.complaining.of.back.pain..Imaging.shows.Bony.lytic.lesions.on.multiple.levels.with.moth.
eaten.appearance..Diagnosis.by?.
A.....Serum.protein.electrophoresis.
B.....Bone.scan......................................................... .
Answer:.A.
..
Q128..Weakness.in.gluteal.area,.the.artery.affected.is?.
A....Femoral.
B....Internal..iliac.
C....External.iliac.
..
Answer:.B.
..
Q129..Carpal.tunnel.syndrome,.cannot.move.his.fingers,.what.is.affected.muscle.?.
A.....Palmar.interossei.
B.....Thenar.muscle...
..
Answer:.stupid.question.
The.nerve.affected.in.carpal.tunnel.syndrome.is.median.nerve,.which.innervates.the.thenar.muscles.
responsible.for.the.movement.of.the.thumb,.and.the.median.nerve.IS.THE.ONLY.NERVE.PASSING.IN.
THE.CARPAL.TUNNEL!.
+.Carpal.tunnel.patients.do.not.present.with.inability.to.move.the.fingers..
The.palmar.interosseous.muscles.adduct.the.fingers.towards.the.middle.finger.+.they.are.supplied.
by.the.deep.branch.of.the.ulnar.nerve..
Q130..Closed.humeral.fracture.with.hand.drop,.what.is.the.type.of.nerve.injury.?.
A....Neuromatosis.
B....Neuropraxia.
C....Axonotmesis.
D....Avulsion.of.radial.nerve.
..
Answer:.B.
..
Q131..Case.of.winging.of.scapula,.what.is.the.origin.of.the.affected.nerve?.
A....Root.of.brachial.plexus.
..
Answer:.A.
The.most.common.cause.of.scapular.winging.is.serratus.anterior.paralysis..This.is.typically.caused.by.
damage.(i.e..lesions).to.the.long.thoracic.nerve..
This.nerve.characteristically.arises.from.the.anterior.rami.of.three.spinal.nerve.roots:.the.fifth,.sixth,.
and.seventh.cervical.nerves.(C5KC7)..
(Wikipedia).
..
Q132..Child.with.fever.and.bilateral.hand.joint.stiffness.along.with.tenderness,.Dx?.
..
Answer:.could.be.rheumatic.fever.
http://emedicine.medscape.com/article/236582Kclinical#b1.
Q133..Most.common.shoulder.dislocation.in.epileptic.patients?.
..
Answer:.posterior.
..
Q134..Stab.wound.at.post.triangle.of.neck..Nerve.damage.presentation?.
A.....Unable.to.raise.arm.
..
Answer:.A.
I’m.not.sure,.but.with.injury.to.the.long.thoracic.nerve,.the.patient.won’t.be.able.to.raise.the.arm.
above.the.head..
..
Q135..What.is.the.most.common.bone.to.get.fractured.in.the.hand?.
A.....Scaphoid.
..
Answer:.A.
..
Q136..Boy.complaining.of.leg.pain.and.externally.rotated,.scenario.typical.of.SCFE,.XKray.provided,.
clearly.showing.SCFE..What.is.the.diagnosis?.
..
Answer:.Slipped.capital.femoral.epiphysis.
Presentation:.groin.and.thigh.pain,.limp.(antalgic.gait),.sometimes.knee.pain,.loss.of.hip.internal.
rotation,.abduction,.and.flexion,.abnormal.leg.alignment.(externally.rotated.foot.progression.angle),.
weakness.(thigh.atrophy)..
..
(Orthobullets) .
..

Q137..Diabetic.presented.with.sudden.onset.of.middle.and.ring.finger.pain..On.examination,.passive.
extension.radiates.the.pain.to.the.wrist..No.history.of.trauma..What's.the.treatment?.
A.....Colchicine.
B.....Cefazolin.
C.....Ibuprofen.
D.....Prednisolone.
..
Answer:.C.(not.sure).
..
Q138..Typical.Scenario.of.cervical.stenosis.with.an.MRI.picture.asking.about.the.diagnosis..
Look.up.MRI.pic.of.cervical.stenosis.
..
Q139..Patient.keeps.his.right.arm.extended.and.Lt.flexed.to.his.waist,.Dx?.
A.....Catatonia.
B.....Dystonia.
..
Answer:.B.(not.sure).
..
Q140..PAtient.with.25K30%.lumbar.spinal.stenosis,.what's.the.treatment.?.
A....Surgery.
B....Physiotherapy.
C....Epidural.steroid.injection.
..
Answer:.B.(I.think).

Treatment.of.lumbar.stenosis:.nonoperative.(first.line):.oral#medications,#physical#therapy,#and#
corticosteroid#injections..

Modalities.include:.

●! NSAIDS,.physical.therapy,.weight.loss.and.bracing.
●! Steroid.injections.(epidural.and.transforaminal).effective.and.may.obviate.need.for.surgery..
Operative:.wide#pedicle[to[pedicle#decompression.#Indications:.

●! Persistent.pain.for.3K6.months.that.has.failed.to.improve.with.nonoperative.management..
●! Progressive.neurologic.deficit.(weakness.or.bowel/bladder)..
Wide#pedicle[to[pedicle#decompression#with#instrumented#fusion.#Indications:.
●! Presence.of.segmental.instability.(isthmic.spondylolisthesis,.degenerative.spondylolisthesis,.
degenerative.scoliosis)..
..
Q141..Pronator.teres.syndrome?.................................................... .
Answer:.medial.nerve.compression.at.elbow.
.
Q142..Nerve.involved.in.tarsal.tunnel.syndrome?.
Answer:.tibial.nerve.
..
Q143..years.old.lady.known.case.of.hypothyroidism.present.with.painful.movement.of.the.right.
shoulder.and.can't.raise.the.shoulder.due.to.this.pain,what's.the.most.likely.diagnosis?.
A....Rotator.cuff.tear................................... .
B....Adhesive.capsulitis. .
C....Impingement.syndrome.
Answer:.B.
I.think.more.details.are.needed,.If.they.mentioned.having.weakness.I.would.choose.rotator.cuff.tear..
..................................................................................................................... .
Q144..If.you.remove.the.pectoralis.major.muscle,.what's.gonna.happen?.
A....Loss.of.arm.adduction.
B....Loss.of.arm.abduction.
C....Loss.of.arm.adduction.and.internal.rotation .................................................. .
D....Loss.of.arm.abduction.and.external.rotation.
.Answer:.C.
..
Q145..Old.bilateral.deltoid.muscle.weakness.and.neck.stiffness.and.limiting.of.movement?.A....
Brachial.plexus.neuropathy.
B....Neck.strain.
.Answer:.cervical.spondylosis.
..
..
.

!ENT.
##
1:#A#little#girl#with#ear#discharge,#bulging#tympanic#membrane,#decreases#TM#vibration#and#
erythema#behind#the#ear.#External#ear#pinna#is#pushed#forward#and#inferior.#What#is#the#Dx?#
AKMastoiditis.
BKOtitis.Media.
CKOtitis.externa.
Answer:.A.(Key.words,.Pinna.is.pushed.forward.with.erythema.behind.the.ear).
##
2:#Mastoditis#treatment?#
Answer:.Vancomycin.+.ceftriaxone.
..
4:#Patient#with#parotiditis#and#have#pain#with#eating#that#radiates#to#the#ear,#pain#by#which#
nerve"?#
AK.Auriculotemporal.
Answer:.A,.If.not.choose.facial.nerve.
..
5:#Most#common#parotid#tumor#in#pediatric:#
Pleomorphic.adenoma.
..
Notes:#
Benign.K.pleomorphic.adenoma.then.hemangioma,.
Malignant.K.mucoepidermoid.tumor.
Head.and.neck.tumor:.HPV.is.a.risk.factor.
..
6:#Child#diagnosed#as#serous#OM,#what#is#the#best#management#to#relieve#his#symptoms?#
AK.Myringotomy.
BK.Myringotomy.+.Gommet.insertion.
..
Answer:.B.(https://emedicine.medscape.com/article/858990Ktreatment).
..
7:#Epiglottitis#treatment#
Answer:.Ceftriaxone.
..
8:#Thumb#sign#on#x[ray#what#is#the#organism#?#(Epiglottitis)#
Answer:.Haemophilus.influenza.
##
9:#Nerve#to#behind#ear#and#...#part#of#auricle#
Answer:.Greater.auricular.nerve.
(It.supplies.the.skin.of.the.face.over.the.parotid.gland,.the.skin.over.the.mastoid.process.and.on.the.
posterior.surface.of.the.auricle..Small.branch.pierces.the.auricle.to.supply.the.lobule.and.concha).
https://radiopaedia.org/articles/greaterKauricularKnerve.
..
10:#Tonsillitis#case,#which#antibiotics#will#you#use?#
AK.Amoxicillin/.clavilonic.
BK.Vancomycin.
CK.Ciprofloxacin.
Answer:.A.
..
Notes:.Tonsillitis.presents.with.fever,.sore.throat,.foul.breath,.odynophagia,.dysphagia,.tender.
cervical.lymph.nodes.
..
11:#Loss#of#sensation#of#anterior#2/3#of#the#tongue?#
AK.5.
BK.7.
CK.9.
DK.12.
..
Answer:.A.
Loss.of.sensation.on.anterior.2/3.à..Trigaminal.(lingual.branch).
Loss.of.taste.on.anterior.2/3.à.Facial.(chorda.tympani).
Loss.of.sensation.and.taste.on.posterior.1/3.à.Glossopharyngeal.
Loss.of.motor.function.of.the.tongue.à.Hypoglossal.
Posterior.part.of.the.root.of.tongue.à..vagus.
..
12:#Man#had#accident#and#loss#of#test#in#the#anterior#two#third#of#the#tongue,#which#nerve#is#
affected?#
Answer:.Facial.nerve.
..
13:#Child#with#left#ear#pain#for#7#days,#had#previous#ear#infection#which#was#treated#with#
antibiotics,#otoscope#shows#bulging#of#tympanic#membrane.#What#is#the#treatment?#(otitis#media)#
AK.Amoxicillin.
BK.Amoxicillin.with.clavulinic.acid.
..
Answer:.B.
..
14:#20#Year[old#patient#is#complaining#of#hearing#loss#in#one#ear,#she#also#mentions#that#she#can#
hear#her#own#voice#louder#in#the#affected#ear.#What#is#the#possible#cause?#
AK.Otitis.media.
BK.Otitis.externa.
CK.Furuncle.in.ear.canal.
DK.Superior.canal.dehiscence.
..
Answer:.D.(https://emedicine.medscape.com/article/857914Kclinical).
..
15:#Most#common#cause#of#tinnitus #
AK.Chronic.salicylate.use.
BK.Salicylate.poisoning.
CK.Chronic.otitis.media .
DK.Sensorineural.hearing.loss .
Answer:.D.(http://www.aafp.org/afp/2014/0115/p106.html).

16:#Clear#case#of#atopic#patient#with#allergic#rhinitis,#symptoms#and#signs#were#given,#then#asked#
about#the#cells#responsible#for#his#allergy?#
AK.Monocytes.
BK.Macrophages.
CK.Mast.cells.
DK.Neutrophils.
.Answer:.C.

17:#Unilateral#nasal#obstruction,#with#foul#smelling#discharge#and#vestibule#erosion,#his#nasal#exam#
was#normal#,what#to#do?#
AK.XKray.head.and.chest.
BK.Nasal.exam.under.GA.
CK.ABx.
..
Answer:.A.(most.likely.foreign.body.insertion).
..
18:#Which#one#of#the#following#drugs#causes#Vertigo?#
AK.Streptomycin.
BK.INH.
CK.Ethambutol.
..
Answer:.A.(ototoxicityKvestibular).
..
19:#Cellulitis#in#the#face#of#infants#between#6#and#24#months#with#purple#discoloration#is#caused#
by: #
AK.GAS .
BK.SA.
CK.HIB .
DK.Strep.pneumonia.
..
Answer:.C.(https://emedicine.medscape.com/article/214222Koverview#a4).
..
20:#Most#common#cause#of#hearing#loss#?#
AK.SNHL.
BK.Conductive.
..
Answer:.A.(http://www.aafp.org/afp/2012/0615/p1150.html).
..
21:#Patient#has#loss#of#sensation#of#upper#lip#and#inferior#eye#lid,#what#nerve#is#affected?#
Answer:.Infra.orbital.nerve.
(Its.branches.supply.the.skin.of.the.medial.cheek,.lateral.nose.and.upper.lip,.the.mucosa.of.the.
anteroinferior.nasal.septum.and.oral.mucosa.of.upper.lip.).
https://radiopaedia.org/articles/infraorbitalKnerveK2.
..
22:#Patient#underwent#dental#procedure,#then#he#develop#numbness#on#upper#lip#what#is#the#
nerve:#
Inferior.orbital.nerve.
..
23:#child#brought#to#ER#with#barking#cough,#red#epiglottis,#thumb#sign#on#x#ray,#
Best#initial#management?#
AK.Examination.of.epiglottis.
BK.Endotracheal.intubation..
CK.Emergent.tracheostomy.
DK.Nasopharyngeal.Tube.
Answer:.B.(https://emedicine.medscape.com/article/763612Ktreatment).
..
24:#Signs#of#epiglottitis,#what#is#the#diagnostic#test#?#
A.KNasopharyngeal.soap.
B.KXKray.
CK.Direct.visualization.of.the.epiglottis.
..
Answer:.C.(https://emedicine.medscape.com/article/763612Kworkup#c8).
..
25:#Patient#with#submandibular#mass#that#becomes#painful#with#eating#and#chewing,#and#
disappears#after.#on#examination#there#is#a#hard#submandibular#mass.#Diagnosis?#
AK.Acute.sialadenitis.
BK.Submandibular.duct.calculus.
..
Answer:.B.(submandibular.sialadenosis).(https://emedicine.medscape.com/article/882358K
clinical#b5).
..
26:#30#years#old#pregnant#with#hearing#loss#and#vertigo,#her#mother#had#the#same#presentation.#
What#is#the#cause:#
AK.Acoustic.neuroma.
BK.Otosclerosis.
..
Answer:.B.(https://emedicine.medscape.com/article/859760Koverview#a10).
..
27:#Most#common#cancer#originates#from#which#paranasal#sinus?#
AK.Ethmiod.
BK.Frontal.
CK.Sphenoid.
DK.Maxillary.
..
Answer:.D.(https://emedicine.medscape.com/article/847189Koverview#a8).
..
28:#Child#has#pale#edematous#nasal#what#to#do#(treatment#of#allergic#rhinitis)?#
Answer:.Intranasal.corticosteroids.
##
29#:#Most#appropriate#treatment#for#nasal#obstruction#?#
AK.antihistamine..
BK.Steroid.
. .

.
.
30:#Patient#after#a#surgery#in#the#posterior#triangle#of#neck,#developed#loss#of#sensation#on#the#skin#
of#lower#mandible#and#Lower#part#of#the#ear,#what#nerve#is#injured?#
Answer:.Great.Auricular.nerve..
..
31:#Patient#with#bad#breath#smell,#his#teeth#are#ok#and#he#expectorates#seed#like#particles,#what#is#
the#most#likely#cause?#
Crypt.tonsillitis.
..
32:#Patient#with#tonsillitis,#what#is#the#duration#of#antibiotics?#
AK.5.
BK.7.
CK.10.
..
Answer:.10.days.
..
33:#Facial#nerve#affection,#normal#lacrimation,#abnormal#tympani#reflex#and#loss#of#taste,#site#of#
injury:#
AK.Distal.to.tympani.chordi.
BK.Proximal.to.tympani.chordi.
CK.Distal.to.gbs.nerve.
DK.Proximal.to.gbs.nerve.
..
Answer:.B.(If.the.lesion.is.proximal.to.the.geniculate.ganglion,.the.motor.paralysis.is.accompanied.by.
gustatory.and.autonomic.abnormalities..Lesions.between.the.geniculate.ganglion.and.the.origin.of.
the.chorda.tympani.produce.the.same.effect,.except.that.they.spare.lacrimation..If.the.lesion.is.at.
the.stylomastoid.foramen,.it.may.result.in.facial.paralysis.only.).Medscape.
..
34:#Patient#complain#of#vertigo#for#22[46#minutes#+#tinnitus#+#hearing#loss,#audiogram#show#low#
frequency#hearing#loss,#diagnosis?#
AK.Meniere's.disease.
BK.Acoustic.neuroma.
CK.Vestibule.neuritis.
Answer:.A.
..
35:#Child#with#history#of#URTI#presented#now#with#bilateral#bulging#TM#and#pain#
AK.observe.
BK.Amoxicillin.45/kg.for.5.days.
CK.Amoxicillin.90/kg.for.10.days.
Answer:.C.
..
36:#Moved#pinna#and#narrowed#external#canal,#what#is#the#diagnosis?#
AK.Choleteatoma.
BK.Otitis.externa.
CK.Otitis.media.
Answer:.B.
..
37:#Old#patient#with#a#history#of#vertigo,#imbalance,#tinnitus#with#hearing#loss#what#is#the#next#
investigation?#
AK.CT.temporal.
BK.MRI.of.cerebelloponitine.angle.
CK.Auditory.brainstem.response.
Answer:.B.(Acoustic.neuroma).
........................................................................................
38:#Scenario#about#hearing#loss#of#gradual#onset#in#one#ear#and#vertigo#.pic#of#bell's#palsy.#What#is#
the#diagnosis?#
Answer:..Acoustic.neuroma.
..
39:#young#boy#with#head#trauma#came#with#painful#swelling#in#nose.#on#examination#cystic#
swelling#on#both#sides#of#nasal#septum#was#seen,#what#will#be#your#action?#
AK.Decongentant.
BK.Antihistamine.
CK.XKray.nasal.septum.
DK.Inscision.and.drainage.
Answer:.D.(septal.hematoma).
..
40:#Symptoms#of#allergic#rhinitis#and#asked#about#the#diagnosis?#
Answer:.sneezing,.nasal.obstruction,.tearing.and.itching..
..
41:#What#is#the#most#common#cause#of#epistaxis#in#children?#
Answer:.Nose.picking,.foreign.body.
..
42:#Child#with#painless#hearing#difficulty#,#tympainc#membrane#not#cleary#seen#
Answer:.Serous.otitis.media.
..
43:#Bilateral#mass#post#auricular#what#is#the#complications:#
AK.Encephalitis.
BK.Hearing.loss.
Answer:.A.
..
44:#40#years#old#man#complaining#of#vertigo#and#tinnitus#associated#with#nausea#and#vomiting,#the#
attack#of#vertigo#last#30#to#45#minutes#(not#sure#about#headache)#
Auditory#test#showed:#low#frequency#sensorineural#hearing#loss.#
What#is#the#most#likely#diagnosis?#
AK.Meniere’s.disease.
BKacoustic.neuroma.
CKvestibular.disease.
DKParoxysmal.benign.vertigo.disorder.
..
Answer:.A..
45:#Child#with#barking#cough#what#finding#on#auscultation?#
Wheezing.because.of.secretion.
..
46:#Dysphagia#,swollen#tonsils,#what#will#decrease#the#swelling#in#pharynges?#
AK.Prednisolone.
BK.Aspirin.
..
Answer:.A.
..
47:#Child#with#fever#and#swelling#of#the#ear#canal:#
Answer:.Otitis.externa.
..
48:#What#is#the#most#common#cause#of#hearing#loss#in#children?#
AK.Eustachian.tube.dysfunction.
BK.Prenatal.maternal.infection.
CK.Otitis.media.with.secretion.
..
Answer:.C.
..
49:#17#year#boy#with#nasal#bleeding#several#minutes#ago.no#previous#h/o#bleeding#disorder,#now#
there#is#slow#bleeding#from#posterior#septum,#initial#action?#
AK.Put.him.in.left.recumbent.position.
BK.Posterior.nasal.packing.
CK.Analgesic.and.vasoconstrictor.nasal.spray.
Answer:.B.,.in.SMLE.12.the.answer.was.C..
..
50:#Patient#with#epistaxis.#On#examination#only#bleeding#point#was#Posterior#inferior#nasal#
septum,#which#vessel#is#affected?#
Answer:.Sphenopalatine.
..
51:#First#step#epistaxis?#
AK.Nasal.packing.
BK.Lying.side.ways.
CK.Pinching.fleshy.part.together.
Answer:.C.
..
52:#Pt#woke#up#with#aphonia,#but#can#cough?#
AK.Functional.aphonia .
BK.Laryngeal.cord.paralysis.
.Answer:.A..
53:#Patient#with#squamous#cell#carcinoma#on#his#lower#lip,#in#which#lymph#node#will#be#drained?#
AK.Submandibule.
BK.Submental.
Answer:.B.
..
54:#Patient#had#occipitofrontalis#paralysis#which#branch#of#facial#nerve#is#affected?#
AK.Temporal.
BK.Buccal.
Answer:.A.
..
55:#Case#of#bell’s#palsy,#which#nerve#will#be#intact?#
AK.Massater.
BK.Bucceneter.
CK.Orbicularis.oris.
DK.Orbicularis.oculi.
Answer:.A.(supplied.by.trigeminal).(https://radiopaedia.org/articles/masseterKmuscle).
..
56:#Child#swallowed#caustic#material#and#presented#with#drooling#of#saliva,#what#is#the#initial#step?#
AK.Secure.airway.
BK.Emergent.endoscope.
CK.Charcol.
Answer:.A.
..
57:#Loss#of#forehead#muscle#action,#which#nerve#is#affected?#
Answer:.Fascial.nerve.
##
58:#Loss#of#gag#reflex#but#normal#uvula,#which#nerve#is#affected?#
AK.glossopharyngeal.
BK.Vegas.
Answer:.A.
##
59:#Lymph#node#of#the#forehead?#
Answer:.Submandibular.
##
60:#Complications#of#mastoiditis?#
Hearing.loss,.Facial.nerve.palsy,.Cranial.nerve.involvement,.osteomyelitis,.Petrositis,.Labyrinthitis,.
Gradenigo.syndrome.(otitis.media,.retroKorbital.pain,.abducent.palsy).Intracranial.extension.
(meningitis,.cerebral.abscess,.subdural.empyema).sigmoid.sinus.thrombosis,.abscess.formation.
##
61:.Nasal#&#and#palate#dryness#due#to#obstruction#of?........................................................ .
K.submandibular.gland.
K.Submental.
K.Ptergoplatine.
..
62:#Elderly#with#vasomotor#rhinitis#?............................................................................. .
●! Antihistamines.
●! Decongested.
●! Corticosteroid.
Mild.:.Antihistamine.and.saline.drop.
Sever.:.Steroid.and.anticholinergic.
Usually.as.the.optimal.tx.is.steroid.
http://www.mayoclinic.org/diseasesKconditions/nonallergicK.rhinitis/diagnosisK
treatment/treatment/txcK20179179.
.
63:#New#born#delivered#by#forceps#and#affected#on#sternomastoid#foramen#:#
decrease.blood.supply.to.ear.,.loss.sensation.of.face.same.side.,.eye.opining.,.loss.sensation.if.
anterior.2/3.
.
64:#Baby#,#during#delivery#,#there#was#stylomastoid#foramen#trauma#,#what#features#will#be#during#
exams:#
AK.loss.of.eye.close.
BK.Loss.of.facial.sensation.
CK.Loss.of.mastication.function.
Answer:.A.
..
65:#Which#nerve#supplies#biggest#part#of#the#tongue#and#covers#it#the#most?#
A.K.trigeminal.
B.Kvagus.
C.Kglossopharyngeal.
Answer:.A.
##
66:#In#SNHL#where#is#the#pathology:#
Answer:.Hair.cells.
..
67:#Treatment#of#cholesteatoma?#
Answer:.Surgery..
##
Notes:........................................................................ .
K.Otosclerosis:.AD.
K.Recurrent.URTI.with.psuedomonas.and.atypical.organism:.CF.
K.Compression.on.maxillary.artery:.sphenopalatine.artery.affected.
K.Thyroid.fascia:.pretrachial.fascia,.part.from.deep.cervical.fascia.
K.Accessory.nerve:.in.post..Triangle.
K.triangle.between.hyoid.bone.and.two.ant..Belly.of.digastric:.submental.
K.External.laryngeal.nerve:.supply.cricothyroid.muscle.KK>.adduct.cord.to.phone.the.voice.
K.Facial.N.injury.proximal.to.greater.petrosal.nerve.
K.Loss.of.sensation.in:.lower.eyelid,.maxilla,.zygomatic.areas.KK>.maxillary.N.injury.which.give.inferior.
orbital.N.
K.Upper.and.lower.lips.LN.drainage:.submandibular.
K.LN.of.middle.of.face:.submandibular.
K.LN.of.lateral.side,.includes.lateral.side.of.eyelid.and.eyebrow:.parotid.gland.
K.Tip.of.the.tongue.and.center.of.the.lips:.submental.
..
.
.
.
.
.
.
.
.
.
.
.
.
.

Anesthesia.
#
1......A#diabetic#patient#on#oral#hypoglycemic#drugs#is#going#into#surgery#what#to#do?#
a... Give.insulin.
..
Answer:#IV#Insulin;.in.general.the.literature.suggests.keeping.glucose.levels.between.150.and.
200 mg/dL..Both.DM.I.and.II.will.be.given.IV.insulin.with.different.rates.of.infusion..
Reference:.https://www.ncbi.nlm.nih.gov/pmc/articles/PMC4452499/.
..
2... Multiparous,#38#wks,#in#labour,#90%#cervical#effacement,#4CM#dilatation,#membranes#
ruptured#e#clear#fluid,#On#CTG,#FHR#dropped#from#140#to#80,#the#pt#platelets=50,000,#what#type#of#
anesthesia#is#suitable?#
a......General.Anesthesia.
b......Epidural.
c......Pudendal.
..
Answer:.General.Anaesthesia;.Regional.anaesthesia.is.absolutely.contraindicated.in.case.of.low.
platelet.count.and.severe.coagulopathy..However,.risks.and.benefits.of.regional.anaesthesia.should.
individually.be.assessed.in.patients.who.had.low.platelet.count.but.no.clinical.signs.of.coagulation.
disorders..
References:.https://www.ncbi.nlm.nih.gov/pmc/articles/PMC4894094/.
..
3... Local#anesthesia#SE#first#sign?#
a... perioral.numbness..
..
Answer:.Perioral.numbness;.SE.appears.1K5.min.after.the.injection.which.begins.as.a.CNS.excitatory.
symptoms.“.circumoral/tongue.numbness,.Metallic.taste,.dizzy,.lightheadedness,.visual/audiotary.
disturbances,.Disorientation”..but.with.higher.doses.it.follow.a.rapid.CNS.depressing.status.“.
convulsion,.Coma,.Respiratory.arrest,.
References:.https://emedicine.medscape.com/article/1844551Koverview#a1.
..
4... Anesthesia#in#asthmatic#
a... ketamine.
Answer:#Ketamine.is.used.in.Asthma,.Hypovolemia,.major.trauma.because.it’s.a.sympathomimetic..
References:.Toronto.note.2016.
5... #What#anesthetic#drug#can#act#as#analgesic#if#given#in#a#low#dose#
a... Ketamine.
Answer:.Ketamine;.works.on.NMDA.receptor.and.subanesthetic.dose.will.potentiate.Opioid.
analgesia,.it.has.antitumor,.neuroprotective.and.antiKinflammatory.properties..
References:.https://www.ncbi.nlm.nih.gov/pmc/articles/PMC4258981/.
..
6... #Which#anesthetic#agent#is#100#times#stronger#than#Morphine?#
a... Fentanyl.
..
Answer:.Fentanyl;.in.the.literature.its.said.the.Fentanyl.is.80.times.more.potent.than.morphine.and.
hydromorphone.is.20.times.more.potent.than.morphine..
References:.https://palliative.stanford.edu/opioidKconversion/equivalencyKtable/.
##
7... Fentanyl#in#patient#with#Absence#seizure:#increase#stimulation#of#GABA#leads#to#seizure#
##
Answer:.its.well.known.that.fentanyl.produces.an.epileptiform.activity.in.patients.with.epilepsy.
disorders;.the.proconvulsant.properties.of.it.affect.mu.and.delta.opioid.receptor.and.by.inhibiting.
GABAergic.interneurons.which.mediate.an.excitatory.effect.on.neurons.especially.in.the.epileptiform.
focus.zone..
References:.http://onlinelibrary.wiley.com/doi/10.1046/j.1528K1157.2001.18600.x/full.
##
8... Drug#given#with#analgesia#to#decrease#its#side#effect#
a... Metoclopramide.
..
Answer:#Metclopromide;.Dopmaine.2.Receptor.antagonist.and.act.on.the.chemoreceptor.zone.and.
gastric.motility..
References:.https://www.ncbi.nlm.nih.gov/pmc/articles/PMC2464411/.
9... Multiparous#woman#everything#was#fine,#except#the#FHR#dropped#from#140#to#80,#also#
mentioned#that#her#platelet#count#is#low,#what#type#of#anesthesia#to#give?#
a... GA.
Answer:.General.Anaestheia..
References:.https://www.ncbi.nlm.nih.gov/pmc/articles/PMC4894094/.
..
10....A#pregnant#lady#was#provided#a#pudendal#nerve#block#as#an#analgesia,#which#structure#would#
be#fully#sensitive#and#not#blocked#by#the#analgesia?#
a......Rectum.
b... Perineal.body.
c... Urogenital.diaphragm.
..
Answer:..Rectum;.Pudendal.nerve.block:.blocks.sensory.and.motor.innervation.to.the.Clitoris.“.
dorsal.nerve”,.skin.of.perineum.[labia.major/minora.and.vestibule].“.perineal.branch”,.External.anal.
sphincter.and.perianal.skin.“.via.inf..Hemorrhoidal.N.”.
References:.https://emedicine.medscape.com/article/83078Koverview#a1.
..
11...Side#effect#of#morphine#?#
a... nausea.and.vomiting.
Answer:#nausea,#emesis,.constipation,.dizziness..S/Sx.of.overdose.includes.[.meiosis,.fever,.HTN,.
decrease.responsiveness,.muscle.cramps/pain/stiff/spasms,.Sleepiness].
References:.https://www.drugs.com/sfx/morphineKsideKeffects.html.
..
12...Investigation#to#be#ordered#before#giving#epidural#analgesia?#
a... Platelet.count.
..
Answer:.low.platelet.count.is.considered.as.a.CI.to.epidural.“.neuroKaxial.block”.and.it.must.be.R/O,.
having.plt..Count.<.80,000.with.bleeding.is.considered.as.a.CI.but.if.without.bleeding.its.considered.
as.a.relative.CI..
References:.https://www.ncbi.nlm.nih.gov/pmc/articles/PMC3417963/.
13...In#order#to#increase#hyperbaric#in#cephalid#position#
a......trendelenburg.position.
b... AntiKtrendelenberg.
..
Answer:#Trendelenburg#position.is.used.to.extend.the.level.of.nerve.block.distribution..
References:.https://academic.oup.com/bja/article/98/3/396/372828/TrendelenburgKpositionKwithK
hipKflexionKasKa.
14...pregnanat#women#during#labor#,#spontaneous#rupture#of#membrane#,#suddenly#baby#is#stress#
from#140#to#80#what#type#of#Anaesthesia#whould#you#use?#
a... GA*.
b... Pedundal.
Answer:.GA.is.usually.the.1st.choice.in.case.of.fetal.compromise..
References:.
https://books.google.com.sa/books?id=L0shAwAAQBAJ&pg=PA187&lpg=PA187&dq=sudden+fetal+heart+rate+drop+type+of+anaesthesia
.&source=bl&ots=q0EutJSyzk&sig=YWMcEK
hz75I06BkEOFiTyyFmv90&hl=ar&sa=X&ved=0ahUKEwjxmae244nXAhVEvBoKHa5iC9cQ6AEILzAB#v=onepage&q=sudden%20fetal%20heart
%20rate%20drop%20type%20of%20anaesthesia.&f=false.

..
15...[Problem#in#intubation#
a......Head.tilt.
b......Jaw.thrust.
c......Cricoid.pressure.
Answer:.The.Q.doesn’t.have.sufficient.information...Head.tilt.is.avoided.in.cervical.injuries.and.Jaw.
thrust.is.used.instead..Cricoid.pressure.is.applied.in.case.of.risk.of.aspiration..
References:.4th.year.med.school.course..
..
16...The#anesthetic#agent#of#choice#in#bronchial#asthma:#
a... Nitric.oxide.
b... .Halothane.
c... .Cyclopropane.
d... Chloroform.
Answer:.Halothane;.Propofol.and.ketamine.inhibit.bronchoconstriction,.decreasing.the.risk.of.
bronchospasm.during.anaesthesia.induction..Propofol.yields.central.airway.dilation.and.is.more.
reliable.than.etomidate.or.thiopental..Halothane,.enflurane,.and.isoflurane.are.potent.
bronchodilators.and.can.be.helpful.even.in.status.asthmaticus.
..
References:.https://www.ncbi.nlm.nih.gov/pubmed/17115010.
..
.
.
.
.
.
.

Dermatology.
..
..
1......[First#line#treatment#of#tenia#pedis:#
AKSystemic.ketoconazole..
BK.Topical.terbinafine..
CKTopical.antiKfungal.
..
Answer:.B/C.are.both.the.same.choose.which.ever.they.put.in.the.choices.
..
Most. dermatophyte. infections. can. be. managed. with. topical. treatments.. For. patients. with. limited.
tinea.pedis,.tinea.corporis,.or.tinea.cruris,.we.suggest.treatment.with.a.topical.antifungal.drug.with.
antidermatophyte. activity. rather. than. systemic. therapy. (Grade. 1A).. Examples. of. effective. topical.
antifungal. agents. are. azoles,. allylamines,. ciclopirox,. butenafine,. and. tolnaftate.. Oral. antifungal.
therapy. is. used. for. extensive. infections. or. infections. refractory. to. topical. therapy.. Nystatin. is. not.
effective.for.dermatophyte.infections..(See.'Tinea.pedis'.above.and.'Tinea.corporis'.above.and.'Tinea.
cruris'.above.).Up.to.date.
..
..
2...... [Patient#developed#cutaneous#lichen#planus#lesion#on#flexor#surfaces#of#the#elbow#bilaterally,#
with# no# mucosal# lesion# on# the# mouth# or# scalp,# which# one# of# the# following# statement# is# correct#
regarding#this#scenario:#
AK.There.is.risk.for.malignancy.transformation..
BK.Automatic.resolution.over.time..
CKpersistent.lesion..
DK.Relapsing.and.remitting.lesion..
Answer:.B.
..
As.classical.cutaneous.lichen.planus.is.a.selfKlimiting.disease,.routine.monitoring.is.usually.not.
required..However,.patients.with.erosive.oral.or.genital.lichen.planus.need.to.be.monitored.
regularly.for.possible.squamous.cell.carcinoma.occurring.at.these.sites..
..
In.cutaneous.disease,.lesions.typically.resolve.within.6.months.(>50%).to.18.months.(85%);.chronic.
disease. is. more. likely. oral. lichen. planus. or. with. large,. annular,. hypertrophic. lesions. and. mucous.
membrane.involvement..http://emedicine.medscape.com/article/1123213Koverview.
In.contrast.to.cutaneous.LP,.which.is.selfKlimited,.lichen.planus.lesions.in.the.mouth.may.persist.for.
many.years,.and.tend.to.be.difficult.to.treat,.with.relapses.being.common..Atrophic/erosive.lichen.
planus.is.associated.with.a.small.risk.of.cancerous.transformation,.and.so.people.with.oral.LP.tend.to.
be.monitored.closely.over.time.to.detect.any.potential.change.early..Sometimes.oral.LP.can.become.
secondarily.infected.with.Candida.organisms.Wiki.
..
..
3......[Which#one#of#the#following#patients#has#worst#prognosis#to#develop#Steven#Jonson:#
AKHIV.patient.receiving.sulfa.group.medication.with.unknown.history.
BKgouty.patient.with.known.sensitivity.to.allopurinol#
Answer:.B.
..
..
4.. . . . . [A# Woman# diagnosed# previously# with# lichen# sclerosis# in# vulva,# she# has# mass# in# vulva# and#
biopsy#taken#,#what#suspect#to#has#?#
A......Squamous.cell.carcinoma.
B......Adenocarcinoma.
C......Squamoadenocarcinoma.
D.....Melanoma.
.... Answer.:A.
## LS# and# cancer. —. Women. with. LS. affecting. the. vulva. are. at. a. slightly. increased. risk. for.
developing..squamous.cell.skin.cancer..Up.to.Date.
..
5......[Which#of#the#following#is#the#most#malignant#and#needs#to#be#removed#
A......Erthyma.migrans..lyme.disease.
B......Erythyma.marginatum..rhueamtic.fever.
C......Erythema.gyratum.sign.of.underlying.lung.cancer.
D.....Erythema.annulare...
Answer:.C.
..
Erythema.gyratum.repens.often.precedes.the.detection.of.malignancy..The.skin.eruption.is.present.
an. average. of. 9. months. prior. to. the. diagnosis. of. malignancy,. with. a. range. of. 1K72. months.. In. a.
minority. of. patients,. erythema. gyratum. repens. occurred. simultaneously. with,. or. up. to. 9. months.
after,.the.detection.of.the.neoplasm..
..
6......[Patient#using#sulfa#drug#there#is#lesion#in#glans#of#penis#ask#about#descriptions?#
..
Answer:.The.characteristic.presentation.is.a.pruritic.or.burning,.sharply.circumscribed,.roundKtoKoval.
patch.with.violaceous#or#dusky#erythema..Scaly,.erythematous.and.ulceration..AAFP.
..
7.. . . . . [Patient#came#with#lesion#in#the#labia#majora#0.5cm#x#0.5cm,#they#took#biopsy,#what#will#the#
histopathology#result#will#be?#
AKAdenocarcinoma.
BK.Squamous.cell.carcinoma.
CKAdeno.squamous.cell.carcinoma.......... .
Answer:B.
..
8.. . . . . [# Patient# came# with# plaque# in# the# labia# majora,# what# is# the# diagnosis?# (The# question# is#
incomplete#)#
AKBasal.cell.carcinoma.
BKMelanoma....................... .
CKLupus.pernio.
..
Lupus.Pernio.–.no.because.its.Sarcoidosis..
Melanoma.a.lump.or.growth.in.the.vagina.
Basal.cell.is.unlikely.as.its.in.sun.exposed.area.
..
9......[#Patient#you#gave#her#retin#A#for#acne,#you#will#warn#her#from#what?#
AKIncreased.breast.tissue.
BKExposure.to.the.sun.
..
Answer:.B.
..
10...[#Rosacea#treatment?#
Answers:.Doxycycline."tetracycline"..is.the.first.option.if.not.available.erythromycin.
..
11.. . [female# has# genital# warts# over# the# past# years,# genital# warts# are# associated# with# what?#
(Incomplete)#
AKhyperkeratosis.à.is.the.description.of.the.wart.not.an.association.
..
The. most. common. type. of. eruption. is. a. morbilliform. (resembling. measles). or. erythematous. rash.
(approximately. 90%. of. cases).[4]. Less. commonly,. the. appearance. may. also. be. urticarial,.
papulosquamous,.pustular,.purpuric,.bullous.(with.blisters).or.lichenoid.[3].Angioedema.can.also.be.
drugKinduced.(most.notably,.by.angiotensin.converting.enzyme.inhibitors)...
..
..
12...[##An#athlete#with#itchy#pink#rash#between#his#thighs,#what#to#prescribe?#
AKTobical.Abx.
BKTopical.steroid.
CK.Topical.antifungal,.selenium.sulfide.(tinea.cruris).
Answer:.C.
..
Tinea.cruris.is.usually.treated.with.topical.antifungal.agents..Sometimes.hydrocortisone.is.added,.for.
faster. relief. of. itch.. Topical. steroids. should. not. be. used. on. their. own.. If. the. treatment. is.
unsuccessful,. oral. antifungal. medicines. may. be. considered,. including. terbinafine. and. itraconazole..
Dermnetz.
..
13...[#Patient#on#multiple#drugs,#developed#violaceous#papule#and#eczematous#pattern#in#the#trunk,#
no#mucus#membrane#involving,#what#dx?#
AKLichenoid.eruption.
BKActinic.eruption.
CKTEN.
DKerythema.multiforme.
Answer:.A.
Extensive. rash. distributed. symmetrically. over. the. trunk. and. limbs. Photodistribution. –. the. rash. is.
predominantly. in. areas. exposed. to. the. sun. Rash. may. be. scaly. resembling. eczema. or. psoriasis.
Wickham. striae. are. usually. absent. Nail. and. mucous. membrane. (e.g.,. mouth). involvement. is.
uncommon.(oral.lichen.planus)More.likely.to.resolve.leaving.marked.pigmentation.
..
14...[Leg#ulcer,#with#rolled#out#edge?#
AKSCC.
BKBasal.cell.
..
Answer:.A.
..
15.. . [Case# of# skin# disease# covering# the# head# and# extensor# surface# of# the# arms# with# silver# scaling#
that#bleeds#when#scratched.#Covering#around#15%#of#her#body#with#nail#pitting#(Psoriasis).#What#is#
the#most#appropriate#management?#
AKTopical.steroid.
BKPUVA.phototherapy.
CKMethotrexate.
Answer:.C.
..
16...[Most#common#affected#site#in#lichen#planus?#
AKMouth.
BKHands.
Answer:.A.
..
17...What#is#the#first#immune#responsible#agent#in#the#skin?#
AKKeratinocyte.
..
Answer:A.
..
They.are.in.the.outer.most.part.of.the.epidermis.the.stratum.cornea.they.act.as.physical.barriers.and.
produce. antimicrobial. peptides. and. the. deeper. layer. keratinocytes. initiate. an. early. immune.
response.–.Journal.of.clinical.and.experimental.dermatology.
..
18.. . [Pt# with# large# painful# nodule# Over# the# nose# with# telangiectasia# over# the# face# (Picture# of#
rosacea)?#
a)Doxycycline.
b)Clindamycin.
c)topical.retinoic.acid.
d)topical.steroid.
Answer:.A.
..
19.. . [19# y/o# female# with# vitiligo# in# the# face# and# hand# the# size# is# increasing# despite# medical#
treatment#now#the#patient#wants#to#get#married#and#ask#you#what#to#do?#
AKcontinue.medical.treatment..
BKstop.Medical.treatment.and.wait.
CKskin.graft..
DKmelanocyte.transfusion..
..
Answer:.Most.Probably.A..
As.for.C.and.D.the.disease.has.to.be.stable.for.6K12.months..DermNet.org.
..
20.. . [picture# of# patient# with# wheal# in# fore# arm,# and# complain# of# hand# pain# after# bee# sting#
associated#with#enlarged#axillary#lymph#node#what#is#the#diagnosis?#
AKUrticaris.
BKlymphangitis.
.Answer:B.
.
.
21...[Acnes#with#pustules#and#nodules#what#type#of#acnes?#
AKInflammatory.
BKobstructive.
Answer:.A.(100%).
Obstructive.=.Comedonal.
Inflamatory=.All.other.types.
..
22...[#Most#common#type#of#eczema?#
AKCold.
BKSoles.
CKAtopic.Dermatitis.
DK.Dermatographism.
Answer:.C.
Note.that.D.is.for.Urticaria.and.not.Eczema..
..
23.. . [A# patient# is# complaining# of# a# skin# mole,# his# father# had# them# as# well# and# remove# them# but#
didn't#know#if#its#benign#or#malignant,#the#patient#is#concerned#that#his#mole#is#malignant,#what#
will#suggest#the#malignancy?#
..
AKIrregular.border.that.fades.in.color.to.be.normal.
BKEquality.of.the.color.
Answer:.A.
..
24.. . [DM# patient# with# rash# on# thigh# folds,# diagnosed# as# eczema,# treated# with# azelic# acid,# topical#
steroid#and#Tacrolimus#with#no#improvement,#your#management?#
AKtopical.corticosteroid.again..
BKtopical.anti.fungal..
CKoral.steroid..
DKpowder..
Answer:.B.
Patients. with. moderate. to. severe. atopic. dermatitis. that. is. not. controlled. with. optimal. topical.
therapy.may.require.phototherapy.or.systemic.immunosuppressant.treatment.to.achieve.adequate.
disease.control..These.treatments.are.not.suitable.for.infants.and.young.children..In.older.children.
and.adolescents,.they.should.be.used.when.other.management.options.have.failed.and.the.disease.
has.a.significant.impact.on.the.quality.of.life..reference:.uptodate.
..
25...[Treatment#of#non[inflammatory#acne#
AKretinoid.acid.
BKclindamycin.
CKisosertonin.
DKazelaic.acid.
..
Answer:A.
Source:.uptodate.
..
26...[#What's#the#treatment#of#cold#induced#urticaria#?#
Answer.:.Antihistamine.
..
27...[#Old#lady#came#with#bilateral#lower#limb#erythema#and#crusting#(Incomplete)#
.Stasis.dermatitis.could.be.the.answer.if.there.is.history.of.venous.statis.or.its.risk.factors.
..
28... KObese#man#develop#painful#lesion#on#gluteus#region,#with#initially#in#the#form#of#mild#lesions#
then#becomes#large#red#painful#(incomplete).#There#was#sinus#on#Examination?#
AKFurunculosis.
BKHidradenitis.suppurativa.
..
Answer:.B.given.the.Obesity.and.location.
..
29... [46#year#old#female#came#to#the#clinic#complaining#of#hair#loss#on#the#central#area#of#the#scalp#
and#thinning#of#her#hair#over#the#past#few#years,#Dx?#
AKFemale.pattern.hair.loss.
BKandrogenic.alopecia.
Answer:.A.
..
30...[#A#clinical#Description#of#Psoriatic#lesions#without#Psoriasis#in#the#options#
AKSLE.
..
Subacute.cutaneous.Lupus.has.a.variant.called.papulosquamous.which.may.resemble.psoriasis.–Up.
to.Date.
..
31...#[Melanocyte#Producing#Cell#?#
..
Corticotropes..
..
32...[Itching#over#the#face,#Anti[cubital##and#popliteal#fossa#?#
Eczema.
..
33...[Nevus#creptus#something#like#this#(child#with#scalp#swelling#with#hair#loss#above#it)#Incomplete#
question#DDX#?#
In.an.infant.or.young.child,.sebaceous.naevus.presents.as.a.solitary,.smooth,.yellowKorange.hairless.
patch,.often.oval.or.linear.in.shape.Sebaceous.naevi.become.more.pronounced.around.adolescence,.
often.appearing.bumpy,.warty.or.scaly..
..
34...[#Description#of#Herpes#simplex#genital#lesions################.. .
Answer:.multiple.painful.fluid.filled.vesicles.
..
35.. . [#Scenario#of#patient#used#a#medication#for#depression#and#developed#rash#in#all#of#his#body?#
(Incomplete)#
.Answer:.Toxic.Epidermal.Necrolysis.(TEN).
..
36...[Diaper#rash#with#satellite#lesion?#
Answer:.Candida.
..
37.. . [Typical# case# seborrheic# dermatitis# Greasy# lesion# on# baby# head# +# Picture# of# seborrheic#
dermatitis#
..
38...[Lesion#from#vulva#with#tree#like#shape#?#
Answer:.Probably.Wart/HPV.
..
39...[Nodular#acne#with#pustule#type#of#acne?#
Answer:.Inflammatory.
..
..
40... SLE+#rash#(papule,#multiple,#itchy,#burning#sensation,#erythematous)#resolve#after#24h#leaving#
hyperpigmentation?#
A......Kdiscoid.rash.
B......Kpityriasis.rosarea.................................................
C......Kurtecarial....
Answer.is.acute.Cutaneous.Lupus.Erythematosus.(ACLE).
http://emedicine.medscape.com/article/1065292Koverview.
Cutaneous.SLE.menifestation:.
Å.... Acute.cutaneous.lupus.erythematosus.(ACLE):.characterised.by.a.typical.butterfly.
pattern.malar.rash.involving.the.central.portion.of.the.face.and/or.a.more.generalised.
maculopapular.eruption..ACLE.is.strongly.associated.with.SLE..
Å.... Subacute.cutaneous.lupus.erythematosus.(SCLE):.characterised.by.a.nonKpruritic,.
nonKscarring.dry.rash..SCLE.may.be.drug.induced.or.can.occur.in.patients.with.SLE,.
Sjogren's.syndrome,.and.complement.C2.deficiency..
Å.... Chronic.cutaneous.lupus.erythematosus.(CCLE):.lesions.are.confined.to.the.skin.and.
oral.mucosa;.DLE.is.the.most.common.form..
..
41...Describe#neva,#What#is#the#evidence#of#neval#hyperplasia?############################################ #
A......Change.in.color.
B......Irregular.border.
C......All.of.the.above....................................................................... .
Answer:.C.
Asymmetry:.Unlike.common.moles,.atypical.moles.are.often.asymmetrical:.A.line.drawn.through.the.
middle.would.not.create.matching.halves..
Border:.While.common.moles.usually.have.regular,.sharp,.wellKdefined.borders,.the.borders.of.
atypical.moles.tend.to.be.irregular.and/or.hazy.—.the.mole.gradually.fades.into.the.surrounding.
skin..
Color:.Common.moles.are.most.often.uniformly.tan,.brown.or.fleshK.colored,.but.atypical.moles.
have.varied,.irregular.color.with.subtle,.haphazard.areas.of.tan,.brown,.dark.brown,.red,.blue.or.
black..
Diameter:.Atypical.moles.are.generally.larger.than.6.mm.(1⁄4.inch),.the.size.of.a.pencil.eraser,.but.
may.be.smaller..
Evolution:.Enlargement.of.or.any.other.notable.change.in.a.previously.stable.mole,.or.the.
appearance.of.a.new.mole.after.age.40,.should.raise.suspicion..
......................................................................................................... .
http://www.skincancer.org/skinKcancerKinformation/atypicalK.moles/warningKsignsKandKimages.
..................................................................... .
42...Diabetic#pt#complain#of#multiple#discharging#sinuses#in#the#back.#Dx:#
AKInfected.lymphoma.
BKLymphangitis.................................................... .
CKCarbuncle........................ ................................. .
DKFuruncle.
Answer:.C.
Carbuncle.:.is.an.abscess.and.its.associated.with.DM,.obesity.and.poor.hygeine..essentially.,.it’s.a.
bunch.of.furuncles.connected.under.the.skin..
..
43... Female#with#polycystic#ovarian#syndrome#noticed#hyperpegminted#skin#in#her#neck#and#axilla,#
what#is#this#abnormality#called?#
a..Acanthosis.negricans.
b..Linea.nigra......................................................... .
Answer:.A.
##
44...A#case#of#xeroderma#pigmentosum#,#what’s#the#mechanism#of#action#?#
..
characteristic.dry,.pigmented.skin..Xeroderma.pigmentosum.is.a.rare.disorder.transmitted.in.an.
autosomal.recessive.manner..It.is.characterized.by.photosensitivity,.pigmentary.changes,.premature.
skin.aging,.and.malignant.tumor.development.,.These.manifestations.are.due.to.a.cellular.
hypersensitivity.to.ultraviolet.(UV).radiation.resulting.from.a.defect#in#DNA#repair..
http://emedicine.medscape.com/article/1119902Koverview.
..
45.. . Patient# 3# cm# lump# in# his# upper# back# ,# slowly# growing# for# years# .# Physical# examination# :#
compressible# ,# no# erythema# ,# there# a# punctum# in# the# middle# that# drain# white# foul# smelling#
material#,#what#is#the#management#!?#
AKCryotherapy.
BKTotal.intact.resection.
CKAntibiotic.and.resection.
Epidermoid.cyst:.Incision.and.drainage.is.the.recommended.treatment.for.inflamed.epidermoid.
cysts,.carbuncles,.abscesses,.and.large.furuncles.
..
46...koebner#phenomenon#is#a#skin#lesions#appearing#on#lines#of#trauma.#
Induced#by:#skin#trauma.#
Associated.with:.psoriasis,.vitiligo.and.lichen.planus.
..
..
47... Female#patient#will#get#married#had#bilateral#vitiligo#in#hands#since#3#years#lesion#increasing#in#
size,#want#to#get#rid#of#it,#most#appropriate#step?#
AKGraft.
BKMelanin.transfer.
CKContinue.medication.
DKStop.medication.
Answer:.C.
..
48...Most#common#type#of#physical#urticaria:#
AKWater.
BKCold.
CKDematographisim.
DKCholinergic.
Answer:.C.(100%).
..
49...Smoker#developed#ulcer#lateral#to#his#tongue#rolled#out#edge#:################################### ################
a).SCC........................................................ .
b).basal.
Answer:.A....
..
50... Old#patient#smoker#complains#of#white#colored#Plaque#over#the#tongue#with#ulcers#what’s#the#
diagnosis?#
Squamous.cell.carcinoma.
..
51...Contraindicated#vaccine#in#dermatological#conditions#(eczema,#psoriasis)?#
AK.Polio.
BK.Measles.
CK.Smallpox.
DK.German.measles.(rubella).
Answer:.C.(not.sure).only.live.vaccine.of.the.lot.
##
52...Eczema#with#topical#steroid#what#to#add:#
Answer:.Tacrolimus.
..
53...Greasy#Scaly#rash#at#edge#of#forehead#and#over#cheeks#not#sparing#folds.#Treatment:#
Answer:.mupirocin.topical.
..
Topical.antibacterial:.indications.include.toxic.shock,.syndrome,.impetigo,.secondery.bacterial.skin.
infections.,,....u.may.see.it.under.the.name.bactroban.
..
54...91[Patient#came#with#buttock#lesion:#
AKBasal.cell.carcinoma.
BKSmall.cell.carcinoma.
CKMycosis.fungoides.
Answer:.C.
..
Review.cutaneous.Tcell.lymphoma.
The.signs.and.symptoms.of.CTCL.vary.depending.on.the.type..The.two.most.common.types.are.
mycosis.fungoides.and.Sézary.syndrome..
Classic.mycosis#fungoides.is.divided.into.the.following.3.stages:.
Å....Patch.(atrophic.or.nonatrophic):.Nonspecific.dermatitis,.patches.on.lower.trunk.and.
buttocks;.minimal/absent.pruritus.
Å....Plaque:.Intensely.pruritic.plaques,.lymphadenopathy.
Å....Tumor:.Prone.to.ulceration.
Sézary#syndrome.is.defined.by.erythroderma.and.leukemia..Signs.and.symptoms.include.the.
following:.
Å....Edematous.skin.
Å....Lymphadenopathy.
Å....Palmar.and/or.plantar.hyperkeratosis.
Å....Alopecia.
Å....Nail.dystrophy.
Å....Ectropion.
Å....Hepatosplenomegaly.may.be.present.
..

55... 92[#Patient#with#Behçet#disease#with#erythema#nodosum#and#mouth#ucelrs,#not#responding#to#
topical#steroids#and#steroid#paste.#What#is#the#treatment?#
AKazathioprine.
BK.interferon.a.
CK.immunoglobulin.
DK??.
Answer:.A..
..
For.oral.and.genital.ulcerations,.topical.steroids.or.sucralfate.solution.are.firstKline.therapy.for.mild.
isolated.ulcerations..Colchicine.has.also.been.used.to.prevent.mucocutaneous.relapse..For.severe.
mucocutaneous.lesions,.systemic.corticosteroids,.azathioprine,.pentoxifylline,.dapsone,.interferonK
alfa,.colchicine,.and.thalidomide.have.demonstrated.benefit..
For.ocular.disease,.azathioprine.is.widely.accepted.as.the.initial.agent..For.severe.eye.disease.
(significant.drop.in.visual.acuity,.retinal.vasculitis,.or.macular.involvement),.either.cyclosporine.A.or.
infliximab.may.be.used.in.combination.with.azathioprine.and.corticosteroids..InterferonKalfa,.alone.
or.in.combination.with.corticosteroids,.appears.to.be.a.second.choice.in.eye.disease..
For.GI.lesions,.based.on.expert.opinion,.5KASA.derivatives,.including.sulfasalazine.or.mesalamine;.
systemic.corticosteroids,.azathioprine,.tumor.necrosis.factor–α.(TNFKα).antagonists,.and.thalidomide.
can.be.used..
Arthritis.may.respond.to.prednisone,.local.corticosteroid.injections,.and.nonsteroidal.antiK
inflammatory.drugs.(NSAIDs),.and.colchicine..InterferonKalfa,.azathioprine,.and.TNFKα.blockers.may.
be.tried.in.rare.cases.of.patients.with.resistant,.prolonged,.and.disabling.attacks..
Cutaneous.disease.with.erythema.nodosum.is.a.special.circumstance.and.may.be.treated.with.
colchicine.or.dapsone..
CNS.disease.is.usually.treated.with.systemic.corticosteroids,.interferonKalfa,.azathioprine,.
cyclophosphamide,.methotrexate,.and.TNFKα.antagonists..
MajorKvessel.disease.with.thrombotic.events.are.treated.with.systemic.anticoagulation.in.addition.to.
corticosteroids,.azathioprine,.cyclophosphamide,.or.cyclosporine.A..Pulmonary.arterial.aneurysms.
are.treated.with.cyclophosphamide.and.corticosteroids..
TNFKα.antagonists.are.increasingly.used.and.have.become.standard.treatment.of.Behçet.disease.that.
is.inadequately.controlled.by.standard.immunosuppressive.regimens..Infliximab.has.been.most.
widely.studied,.but.adalimumab.has.proved.successful.in.cases.refractory.to.both.conventional.
therapy.and.infliximab...Etanercept.is.the.only.TNF.inhibitor.with.data.from.a.short.term.randomized.
controlled.study.with.proven.efficacy.in.suppressing.most.of.the.mucocutaneous.manifestations.of.
Behcet.disease.
Medscape.
..
56... Patient#taking#sulfa#drug#developed#lesion#on#the#penis.#What#is#the#most#suitable#description#
of#the#lesion?#
Answer:.Erythematous.plaque,.blister.and.violaceous.lesion.
..
57...After#sulfa#drug#the#patient#developed#penile#lesion#or#rash#(fixed#drug#eruption)#what#you#will#
find#(best#description#is)?#
AKUlceration.
BKBlister.
CKVesicles.
Answer:.B.
..
58.#Patient#known#case#of#DM#come#with#postule#on#the#inner#fold#in#gluteal#region#they#did#
drainage#after#that#there#is#sinus##and#after#a#period#of#time#he#came#with#the#same#thing#what#is#
the#cause#?#
A...............1K.furuncle.
B...............2KCarbuncle.
C...............3K.hidradenitis.suppurativa.
Answer:.C.
The.diagnosis.of.HS.is.based.upon.the.characteristic.clinical.manifestations;.biopsy.is.neither.
required.nor.diagnostic..Specific.diagnostic.criteria.do.not.exist..
There.are.three.main.diagnostic.features:.
●! Typical.lesions.—.multiple.deepKseated.nodules.(blind.boils),.comedones,.and/or.fibrosis.
●! Typical.locations.—.bilateral.involvement.of.axillae,.groin,.inframammary.areas.
●! Relapses.and.chronicity.
.....
59.#child#with#skin#rash#honey#cluster#(probably#means#crust)#Dx:#
impetigo.
..
60.#scenario#of#patient#used#a#medication#for#depression#and#developed#rash#in#all#of#his#body?#
(question#incomplete)?#Depends#on#what#manifestation#it#takes#
Answer:.Toxic.Epidermal.Necrolysis.(.TEN.).
..
StevensKJohnson.syndrome.(SJS).and.toxic.epidermal.necrolysis.(TEN).are.severe.idiosyncratic.(not.
dose.dependant.and.variable).reactions,.most.commonly.triggered.by.medications,.which.are.
characterized.by.fever.and.mucocutaneous.lesions.leading.to.necrosis.and.sloughing.of.the.
epidermis..SJS.and.TEN.are.distinguished.chiefly.by.severity.and.percentage.of.body.surface.
involved..
up.to.10.%.SJS.10K30.overlap,.over.30.TEN.
..
61.#description#of#Herpes#simplex#genital#lesions#
Answer:.multiple.painful.fluid.filled.vesicles.
Remember:.S1.oral.S2.genital.
..
63.#pt#with#vitiligo#on#medications#for#3#yrs#no#response,#she#requested#a#surgical#TX#What#would#it#
be,#
Answer:.Melanocytes.transfer.
Remember:.it’s.the.treatment.of.choice.in.segmental.vitiligo.
N.B.could.be.correct.if.the.disease.is.stable.for.6K12.months.
..
64.#What#is#the#tt#of#seborrhoeic#dermatitis?#
..
Several.randomized.trials.indicate.that.topical.corticosteroids.and.topical.antifungal.agents.are.
effective.for.treating.seborrheic.dermatitis.and.that.intermittent.topical.antifungals.may.prevent.
relapse..However,.the.high.response.in.the.placebo.groups.suggests.that.frequent.shampooing,.or.
regular.use.of.emollients.may.also.be.beneficial.
..
65.#A#mother#brought#her#child#to#ED#with#itchy#skin#lesions#that#appeared#after#intake#of#some#
food.#The#lesions#appear#in#a#certain#area#and#stay#for#2#hours#then#disappear#then#appear#in#
another#area.#What#is#it#called?#
A......Burrow.(rabies,.so.not.it).
B......Wheal.(urticaria.lesion).
Answer:.B.
..
66.#19#y/o#female#with#vitiligo#in#the#face#and#hand#the#size#is#increasing#despite#medical#
treatment#now#the#pt#want#to#get#married#and#ask#you#what#to#do:#
A)continue.medical.treatment..
B)stop.Medical.treatment.and.wait...
C)skin.graft..
D)melanocyte.transfusion..
Answer:.A.
..
67.#derma:#wheal,#what#is#the#type#of#urticaria#?................................................... .
AKcold.urticaria.
BKsolar.urticaria.
CK.two.other.type.of.urticaria.I.forgot.
..
For.acute.urticaria,.the.main.consideration.involves.possible.precipitants,.such.as.the.following.:.
Å....Recent.illness.
Å....Medication.use.
Å....IV.radiocontrast.media.
Å....Travel.
Å....Foods.
Å....New.perfumes,.hair.dyes,.detergents,.lotions,.creams,.or.clothes.
Å....Exposure.to.new.pets.(dander),.dust,.mold,.chemicals,.or.plants.
Å....Pregnancy.(usually.occurs.in.last.trimester.and.typically.resolves.spontaneously.soon.
after.delivery).
Å....Contact.with.nickel,.rubber,.latex,.industrial.chemicals,.and.nail.polish.
Å....Sun.or.cold.exposure.
Å....Exercise.
Å....Alcohol.ingestion.[3].
Physical.urticaria.is.characterized.by.the.following:.
Å....Blanchable,.raised,.palpable.wheals,.which.can.be.linear,.annular.(circular),.or.
arcuate.(serpiginous);.can.occur.on.any.skin.area;.are.usually.transient.and.migratory;.
and.may.coalesce.rapidly.to.form.large.areas.of.erythematous,.raised.lesions.that.
blanch.with.pressure.
Å....Dermographism.or.dermatographism.(urticarial.lesions.resulting.from.light.
scratching).
..
68.#Treatment#of#non[inflammatory#acne#
AKretinoid.acid.
BKclindamycin.
CKisosertonin.
DKazelaic.acid.
Answer.:.topical.retinoid.acid.
Current.consensus.recommends.a.combination.of.topical.retinoid.and.antimicrobial.therapy.as.firstK
line.therapy.for.almost.all.patients.with.acne..[3].The.superior.efficacy.of.this.combination,.compared.
with.either.monotherapy,.results.from.complementary.mechanisms.of.action.targeting.different.
pathogenic.factors..Retinoids.reduce.abnormal.desquamation,.are.comedolytic,.and.have.some.antiK
inflammatory.effects,.whereas.benzoyl.peroxide.is.antimicrobial.with.some.keratolytic.effects.and.
antibiotics.have.antiKinflammatory.and.antimicrobial.effects..
..
69.#17#year#old#football#player,#hypopigmented#lesions#on#trunk#and#arm.#Treatment:#
AKtopical.abx.
BKoral.abx.
CKtopical.steroid.
DKselenium.sulphate.
Answer.:.D.
..
70.#School#boy#with#itchy#scalp,#10#other#classmates#affected,#diagnosis?#
AKPediculosis.capitis.
Answer.:.Pediculosis.capitis.(.i.e.head.louse)...
The.head.louse.(Pediculus+humanus+capitis).is.an.obligate.ectoparasite.that.lives.on.human.beings.
and.feeds.on.human.blood..Head.lice.infestation.(pediculosis.capitis).mainly.affects.those.who.are.
socially.active,.particularly.young.children..BMJ.
..
71.#patient#complains#of#skin#mole,#his#father#had#them#as#well#and#remove#them#but#didn't#know#
if#its#benign#or#malignant,#the#patient#is#concerned#that#his#mole#is#malignant,#what#will#suggest#
the#malignancy#?#
AKirregular.border.that.fades.in.color.to.be.normal.
BKEquality.of.the.color.
No.other.choice.:.A.
ABCDE=.Assymetry..Irregular.Borders,.abnormal.Color,.Diameter.and.Evoltion.
The.prototypical.melanoma.is.readily.diagnosable.by.the.ABCDE.approach,.based.on.its.asymmetry,.
irregular.border,.irregular.color,.large.diameter,.and.evolution..However,.these.clinical.parameters.
are.largely.useless.in.three.instances,.as.follows:.
Å....For.amelanotic.melanomas,.in.which.pigmentation.is.largely.or.entirely.absent.
Å....For.desmoplastic.melanomas,.which.sometimes.manifest.without.an.associated.in.
situ.component.and.may.also.lack.clinical.pigmentation.
Å....For.ulcerated.and.inflamed.melanomas,.especially.nodular.lesions,.which.may.
clinically.simulate.common.lesions.(eg,.basal.cell.carcinoma,.pyogenic.granuloma).
because.of.masking.by.the.presence.of.ulceration.and/or.inflammation..Medscape.
..
72.#Child#with#scaly#erethematous#plaques#with#follicular#hyperkeratosis#over#elbows#and#knees#.#
what#is#other#area#in#the#body#more#likely#to#be#affected#?#
aK.Eye......................................................... .
bK.Adrenal.
cK.Heart.
dK.Kidney.
Answer:A.
..
73.#Picture#(#look#like#vesicle#)#start#as#1#only#then#spread#to#arm#legs#and#..#with#lymph[node#
enlargement#?#
AKherpes.simplex.virus.
BKDermatitis.herpetic.form............................................................... .
CKVaricella.zoster................................................................... .
INCOMPLETE.Q,.BUT.Verecila.zoster.is.the.closest.
.Answer:C.
.
74.#Patient#with#hx#if#allergy#and#pain#in#the#back#went#to#clinic#they#give#him#drug#(#i#forgot#the#
name#i#think#paracetamol)#then#he#came#next#day#with#viscles#in#back#from#midline#to#the#lateral#
side#?#
A..Xanthosis............................................................ .
B..Tinia.corporus.
C..Herpes.zoster.
Answer:.C.
................
75.#Pinpoint#papule#in#the#face#of#baby#how#to#manage?#
Answer:.Reassure.
THIS.IS.KNOWN.AS.KERATOSIS.PILARIS???.
Incomplete.question….can.be.milia,.baby.acne.or.miliaria.
..
76.#Baby#have#depigmentation#since#birth#and#photophobia#refractory#error#nystagmus#,#What#the#
most#likely#complications?#
A......Kbrain..tumor.
B......skin.cancer.
C......renal.dysfunction.
Answer:.B.
This.a.case.of.Albinism,.complications.are:.
Skin.cancer,.sunburn...................... .
Reduced.visual.acuity......................................................... .
Social.stigma.
http://emedicine.medscape.com/article/1200472Kfollowup#e2.
..
77...Treatment#of#moderately#severe#acne#vulgaris:#
A..topical.retinoid.
B..tetracycline.
C..isotretinoin.
Answer:.C.
In.2016,.the.American.Academy.of.Dermatology.(AAD).issued.new.evidenceKbased.guidelines.for.
treatment.of.both.adolescents.and.adults..Recommended.treatments.include.topical.therapy,.
antibiotics,.isotretinoin,.and.oral.contraceptives.].The.key.recommendations.include.the.following:.
Å....Benzoyl.peroxide.or.combinations.with.erythromycin.or.clindamycin.as.
monotherapy.for.mild.acne;.benzoyl.peroxide.with.a.topical.retinoid.or.systemic.
antibiotic.therapy.for.moderateKtoKsevere.acne.
Å....Topical.antibiotics.(eg,.erythromycin,.clindamycin).are.not.recommended.as.
monotherapy.because.of.the.risk.of.bacterial.resistance.
Å....Topical.retinoids.as.monotherapy.in.primarily.comedonal.acne,.or.in.combination.
with.topical.or.oral.antimicrobials.for.mixed.or.primarily.inflammatory.acne.
Å....Topical.adapalene,.tretinoin,.and.benzoyl.peroxide.can.be.safely.used.to.treat.acne.
in.preadolescent.children.
Å....Topical.dapsone.5%.gel.for.inflammatory.acne,.particularly.in.adult.females.
Å....Systemic.antibiotics.are.recommended.for.moderate.and.severe.acne.and.forms.of.
inflammatory.acne.that.are.resistant.to.topical.treatments;.doxycycline.and.
minocycline.are.both.more.effective.than.tetracycline.
Å....Topical.therapy.with.benzoyl.peroxide.or.a.retinoid.should.be.used.with.systemic.
antibiotics.and.for.maintenance.after.completion.of.systemic.antibiotic.therapy.
Å....Monotherapy.with.systemic.antibiotics.is.not.recommended.
Å....Systemic.antibiotic.use.should.be.limited.to.the.shortest.possible.duration;.to.
minimize.the.development.of.bacterial.resistance,.reevaluation.at.3K4.months.
Å....Use.of.oral.erythromycin.and.azithromycin.should.be.limited.to.those.who.cannot.
use.the.tetracyclines.(ie,.pregnant.women.or.children.aged.<8.y);.erythromycin.use.
should.be.restricted.because.of.its.increased.risk.of.bacterial.resistance.
Å....Isotretinoin.is.recommended.for.severe.acne.or.moderate.acne.that.does.not.
respond.to.other.therapy;.lowKdose.isotretinoin.can.be.used.to.effectively.treat.acne.
and.reduce.the.frequency.and.severity.of.medicationKrelated.adverse.effects,.but.
intermittent.dosing.is.not.recommended;.all.patients.treated.with.isotretinoin.must.
adhere.to.the.iPLEDGE.risk.management.program;.patients.should.receive.routine.
monitoring.of.liver.function.tests,.serum.cholesterol,.and.triglycerides.at.baseline.and.
again.until.response.to.treatment.is.established,.but.routine.monitoring.of.complete.
blood.count.is.not.recommended;.patients.should.be.educated.about.the.potential.
risks.and.monitored.for.any.indication.of.inflammatory.bowel.disease.and.depressive.
symptoms.
..

78.#What’s#your#advice#to#SLE#pts#
A..avoid.sun.exposure.
..
.
.
.
.
.
.
.
.
.
.
.
.
.
.
.
.
.
.

Ophthalmology.
..
1.## Patient#with#follicular#keratosis.#What#will#you#check?#
aKEyes.
Answer:.A.
Reference:.http://disorders.eyes.arizona.edu/category/clinicalKfeatures/keratosisKpilaris.
..
2.## Eye#endothelium#layer:#dry#cornea#(Incomplete)#
Explanation:.corneal.endothelium.is.responsible.for.removing.excess.water..
##
3.## AIDS#retinitis:#CMV#
Explanation:.CMV.is.the.most.common.ocular.opportunistic.infection.in.patients.with.AIDS..
Refrence:.Wills.Eye.Manual,.page.377.
..
4.## With#viral#conjunctivitis:#follicles#+#epithelial#nummular#keratitis#
Explanation:.
Ten.percent.of.patients.with.adenoviral.conjunctivitis.go.on.to.develop.corneal.involvement.
towards.the.end.of.week.2..This.is.an.immune.keratitis.with.white.deposits.in.the.cornea,.
causing.significant.reduction.in.vision,.as.well.as.prominent.photophobia..This,.‘nummular.
keratitis’.requires.treatment.with.a.weak.topical.steroid.such.as.fluorometholone.(FML),.
and.therefore.requires.referral.to.an.ophthalmologist..
Refrence:.https://www.foresteyesurgery.com.au/blog/files/viralKconjunctivitis.php.
https://en.wikipedia.org/wiki/Nummular_keratitis.
..
.......... 5.#####Orbital#pseudotumor#treatment:#
1..Prednisone.80.to.100.mg.p.o..q.d..as.an.initial.dose.in.an.adult,.along.with.gastric.
prophylaxis.(e.g.,.ranitidine.150.mg.p.o..b.i.d.)..Pediatric.dosages.typically.begin.with.1.
mg/kg/day.of.prednisone..All.patients.are.warned.about.potential.systemic.side.effects.and.
are.instructed.to.follow.up.with.their.primary.physicians.to.monitor.blood.sugar.and.
electrolytes..
2..LowKdose.radiation.therapy.may.be.used.when.the.patient.does.not.respond.to.sysK.
temic.corticosteroids,.when.disease.recurs.as.corticosteroids.are.tapered,.or.when.
corticosteroids.pose.a.significant.risk.to.the.patient..Radiation.therapy.should.only.be.used.
once.orbital.biopsy.has.excluded.other.etiologies..
Refrence:.Wills.Eye.Manual,.page.158.
..
6.## Nystagmus:#mid#brain#
..
Peripheral)vestibular:)Horizontal)or)hori9)zontal)rotary)nystagmus.+May+be+accomB+panied+by+
vertigo,+tinnitus,+or+deafness.+May+be+due+to+dysfunction+of+vestibular+end+organ+(inner+ear+disease),+
eighth+cranial+nerve,+or+eighth+nerve+nucleus+in+brainstem.+Destructive+lesions+produce+fast+phases+
opposite+to+lesion.+Irritative+lesions+(e.g.,+Meniere+disease)+produce+fast+phase+in+the+same+direction+
as+the+lesion.+Vestibular+nystagmus+associated+with+interstitial+keratitis+is+called+Cogan+syndrome.+
++
Downbeat:+The+fast+phase+of+nystagmus+is+down+and+most+prominent+looking+down+and+to+the+right+
and+left.+Most+commonly,+the+lesion+is+at+the+cervicoB+medullary+junction+(e.g.,+Arnold–Chiari+
malformation)+or+a+manifestation+of+cerB+ebellar+degeneration.+
++
Upbeat:+The+fast+phase+of+the+nystagmus+is+up.+If+present+in+primary+gaze,+the+lesion+typically+
involves+the+brainstem+or+anterior+vermis+of+the+cerebellum.+If+present+only+in+upgaze,+the+most+likely+
etiology+is+drug+effect.+
++
See9saw:+One+eye+rises+and+intorts+while+the+other+descends+and+extorts.+Lesion+typically+involves+the+
parasellar+region+and+chiasm.+Typically+pendular+when+chiasmal+region+involved,+and+jerk+if+involving+
the+midB+brain.+One+proposal+suggests+a+unilateral+lesion+of+the+interstitial+nucleus+of+Cajal+or+its+
connections+are+responsible+for+this+nystagmus+subtype.+May+have+a+bitemporal+hemianopia+
resulting+from+chiasmal+comB+pression.+May+be+congenital+or+associated+with+septoBoptic+dysplasia.+
++
Gaze)evoked:+Absent+in+primary+gaze,+but+appears+as+the+eyes+look+to+the+side.+Nystagmus+increases+
when+looking+in+the+direction+of+fast+phase.+Slow+frequency.+Most+commonly+the+result+of+alcohol+
intoxication,+sedatives,+cerebellar+or+brainB+stem+disease.+
++
++
Convergence)retraction:+ConvergenceBlike+eye+movements+accompanied+by+globe+retraction+when+
the+patient+attempts+an+upward+saccade.+May+be+associated+with+limitation+of+upward+gaze,+eyelid+
retraction,+and+bilateral+midBdilated+pupils+that+react+poorly+to+light+but+constrict+better+with+
convergence.+Papilledema+may+be+present.+Usually,+a+pineal+region+tumor+or+other+dorB+sal+midbrain+
abnormality+is+responsible.+
))
Periodic)alternating:)In+primary+position,+fast+eye+movements+are+in+one+direction+for+60+to+90+
seconds+and+then+reverse+direcB+tion+for+60+to+90+seconds.+The+cycle+repeats+continuously.+Patients+
may+attempt+to+minimize+nystagmus+with+periodic+head+turning.+May+be+congenital.+Acquired+forms+
are+most+commonly+the+result+of+lesions+of+the+cervicomedullary+junction+and+posterior+fossa.+Other+
causes+include+MS,+medication+side+effects,+and+rarely+blindness.+
++
Refrence:#Wills.Eye.Manual,.page.282.+
..
7.## vertical#and#horizontal#nystagmus:#phencyclidine#(No#refrence#was#found)#
..
..
10.##Glaucoma#pt#+#COPD#has#cough:#
a... because.of.timolol.drops.
Answer:.
Explanation:.Timolol.is.contraindicated.in.COPD.and.may.worsens.symptoms..
..
11.##Drug#contraindicated#in#glaucoma:#
a... Epinephrine.
Explanation:.Selective.alpha.agonist.(phenylepherine).and.non.selective.alpha.
agonists.(epinephrine).can.precipitate.angle.closure.glaucoma.in.patients.with.
narrow.angles.and.both.are.contraindicated.in.closed.angle.glaucoma..
..
12.##Drugs#used#for#glaucoma#
Explanation:.
Common.drugs.used.in.glaucoma.include:.
Topical.beta.blockers,.topical.carbonic.anhyrdrase.inhibitors,.alpha.agonists.and.
prostaglandins.analougs..
Prostaglandins.analougs.(like.latanoprost.“XalatanTM”).are.usually.prescribed.as.first.line.
therapy.as.they.are.the.newest.with.the.least.side.effects...
http://www.glaucoma.org/gleams/glaucomaKmedicationsKandKtheirKsideKeffects.php.
..
13.##Child#with#30#degree#esotropia,#whats#your#manegment?#
Accomodative.esotropia.is.managed.by.correction.of.hyperopia.by.glasses..
Non.accomodative.esotropia.is.commonly.managed.surgically.
Refrence:.https://www.rcophth.ac.uk/wpKcontent/uploads/2014/12/2012KSCIK250K
GuidelinesKforKManagementKofKStrabismusKinKChildhoodK2012.pdf.
..
..
15.##Child#with#unilateral#eye#pain#and#inflammation#and#white#structure:#
........... Answer:.Retinoblastoma.
........... Explanation:.
........... A.malignant.tumor.of.the.retina.that.appears.as.a.white,.nodular.mass.that.breaks...........
through.the.internal.limiting.membrane.into.the.vitreous.(endophytic),.as.a.yellowish.subretinal.
mass.lesion.often.underlying.a.serous.retinal.detachment.(exoK.phytic),.or.as.a.diffusely.spreading.
lesion.simulating.uveitis.(diffuse.infiltrating)..Iris.neovascularization.is.common..PseudohypoK.pyon.
and.vitreous.seeding.may.occur..Cataract.is.uncommon,.and.the.eye.is.normal.in.size..May.be.
bilateral,.unilateral,.or.multifoKcal..Diagnosis.is.usually.made.between.12.and.24.months.of.age..A.
family.history.may.be.elicited.in.about.10%..
Refrence:.Wills.Eye.Manual,.page.177..
..
16.##Abscent#red#eye#reflex#is#seen#in#===>#retinoblastoma#
Causes.of.leukocoria.in.pediatrics.group:.congenital.cataract,.retinoblastoma,.retinopathy.of.
prematurity.(ROP),.toxocariasis,.coats.disease,.PFV/PHPV,.retinal.astrocytoma,.retinochoroidal.
coloboma,.retinal.detachment,.familial.exudative.vitreoretinopathy.(FEVR),.myelinated.nerve.fibers,.
uveitis,.incontinentia.pigmenti,.toxoplasmosis..
Refrence:.Wills.Eye.Manual,.page.178.
..
##
17.##Characteristic#sign#of#retinoblastoma?#
Refrence:.https://emedicine.medscape.com/article/1222849Kclinical#b4.
..
18.##Retinoblastoma#case#that#affecting#the#child's#vision#but#MRI#should#intact#optic#
nerve#what#is#the#management#
a... Chemotherapy.
b... Radiation,.
c... enncuulation.
............ Answer:.A.
.Explanation:.mainstay.treatment.of.retinoblastoma.without.optic.nerve.extension.is.
chemotherapy...Read.about.retinoblastoma:.
https://www.aao.org/pediatricKcenterKdetail/retinoblastomaK2016.
..
https://emedicine.medscape.com/article/1222849K
treatment?pa=8dtgBnfXs1A%2FLKJCYR4hr6MZiyqrlkdpQit9AxXgBnl%2BpH%2Bze2zxoyiiMU
mOyvMQJyGvMX%2Fu%2BWdIXoARf%2FT0zw%3D%3D.
##
21.##Pediatric#e#purulent#eye#discharge,#Culture#showed#gram#negative#diplococcic#How#to#
treat?#
a... Iv.cephalosporin.
b... Steroid.
c... Topical.Abx.
Answer:..A..
Explanation:.Neonatal.Neisseria.Gonorrhoeael.conjunctivitis:.In.severe.cases.this.
can.cause.corneal.perforation..Systemic.complications.include.rhinitis,.stomatitis,.
arthritis,.meningitis.and.septicaemia..Due.to.increasing.resistance.to.penicillin.a.
systemic,.thirdK.generation.cephalosporin.(ceftriaxone).is.used.to.treat.the.
condition..The.eye.must.be.kept.clean..Topical.bacitracin.ointment.can.also.be.
given.but.sysK.temic.treatment.is.the.most.important..Refer.parents.to.a.sexually.
transmitK.ted.diseases.clinic...Refrence:.Lecture.notes,.11th.edition..Page.114..
..
..
23.##A#pt#on#anti#TB#drugs#developed#eye#pain#
a... Ethambutol.
Explanation:.A.
Retrobulbar.neuritis.resulting.in.blurred.vision.and.loss.of.redKgreen.vision.occurs.
commonly.with.ethambutol.therapy.and.requires.careful.monitoring.of.visual.acuity.and.
color.discrimination..Optic.neuritis.occurs.more.frequently.at.dosages.greater.than.15.
mg/kg/day..Drug.therapy.should.be.discontinued.at.the.first.sign.of.vision.defects..
https://www.drugs.com/sfx/ethambutolKsideKeffects.html.
..
24.##The#effect#of#anti#TB#drugs#on#the#eye#is#one#of#the#following#
a... Bacterial.conjunctivitis,.
b... Viral.conjunctivitis.,.
c... Glucoma.,.
d... .Uveitis.
ANSWER:.D.
Explanation:.Rifabutin.can.cause.uveitits..
http://eyewiki.aao.org/Drug_Induced_Uveitis.
Question.probably.written.wrong..
If.asking.about.ocular.manifestations.of.TB>.tuberculous.uveitis..
If.asking.about.ocular.complications.of.anti.TB>.ethambutol.causes.optic.neuritis..
..
.25.##Case#with#conjunctivitis#and#the#eyelash#turned#inwards.#
a......entropion.
Explanation:.
1KTrichiasis.is.a.condition.in.which.eyelashes.grow.in.a.posterior.direction.toward.the.corneal.
surface..
2KDistichiasis.is.the.growth.of.lashes.from.the.meibomian.gland.orifices.(extra.line.of.eyelashes).
3KEntropion.is.an.inward.turning.of.the.eyelid.margin.and.appendages.such.that.the.pilosebaceous.
unit.and.mucocutaneous.junction.are.directed.posterior.towards.the.globe..
..
26.##Water#discharge#+#red#eye?#
a... Viral.conjunctivitis.
Explanation:.Watery.discharge.can.be.a.sign.for.dry.eyes.or.viral.conjunctivitis..In.
Viral.KC,.follicular.reaction.can.be.seen.on.slit.lamp.examination,.and.a.pre.
auricular.lymph.node.might.be.present..A.history.of.upper.respiratory.tract.
infection.might.preceed.ocular.sumptoms..
Read.about.dry.eye:.https://emedicine.medscape.com/article/1210417Kworkup#c8.
..
..
27.##Post#eye#surgery#complication#
a... endophthalmitis.(Infection.in.the.eye).
Answer:.The.most.serious.and.feared.complication.after.ocular.surgeries.is.
endophthalmitis..
..
28.##[Patient#presented#with#eye#redness#and#problem#in#vision#just#during#exams#his#
visual#acuity#is#6/6#and#normal#eye#exam#what#does#he#have:#
............ a......Astigmatism.
............ b......Myopia.
............ c......Hypermetropia.
............ d......Anisometropia.
............ Answer:.C.
............ Refrence:.http://eyewiki.aao.org/Hyperopia.
..
29.##[Patient#presented#with#unilateral#esotropia#what's#the#management#
........... a......Glasses.
........... b......Corrective.surgery.
..
If.the.case.was.accomodative.esotropia,.it.is.treated.by.glasses.(Hypermetropia.correction).
Most.cases.of.non.accomodative.esotropia.are.treated.by.surgical.correction..
For.further.information:.https://www.rcophth.ac.uk/wpKcontent/uploads/2014/12/2012K
SCIK250KGuidelinesKforKManagementKofKStrabismusKinKChildhoodK2012.pdf.
..
30.##[Patient#with#increase#ICP#what#nerve#would#u#examine#before#the#CT#scan#
a... Optic.nerve.
b... Abducens.nerve.
c... Facial.nerve.
Answer:.A.
..
..
31.##Pt#came#to#you#after#trauma#c/o#loss#of#the#abduction#of#(left#or#right#eye).#So#which#cranial#
nerve#affected#
a).III.
b).IV.
c).V.
d).VI.
Answer:.D.(Abducense.nerve.innervates.LR.muscle.which.is.responsible.for.abduction).
..
32.##pt#eye#drop#and#eye#goes#to#medial#side#what#nerve#injury#when#try#to#close#the#eye:#
A...3.
B...7.
C...4.
Explanation:.3rd.nerve.palsy.presents.with:.ptosis,.lateral.and.downward.gaze..
.
.

.
33.##Lacrimal#gland#mass#causes#proptosis#in#which#direction?#
a... In.and.down.
b... Out.and.down.
Answer:.A.
Explanation:.lacrimal.gland.masses.displace.the.eyes.inferonasally..
http://webeye.ophth.uiowa.edu/eyeforum/cases/235KAdenoidKcysticKcarcinomaK
lacrimalKgland.htm.
..
.34.##[Patient#with#conjunctivitis,#which#one#of#the#following#will#make#you#refer#to#
ophthalmology?#
a......Bilateral.conjunctivitis.
b......Photophobia.
c......Mucopurulent.discharge.
d......Itching.
Answer:.B.(Couldn’t.find.a.source).
..
35.##[Clear#ophtha#question#about#herpetic#keratitis#(dendrites#with#fluoroscien)#
..
36.##Man#has#eye#pain,#seeing#halos,#headache,#many#GI#symptoms,#eye#examination:#red#and#
injected#vessels.#What#is#the#diagnosis?#
a......Digoxin.toxicity.
b......Angle.closure.Glaucoma.
Answer:.B.
Explanation:.Acute.angle.closure.glaucoma.presents.with.an.acute.onset.of.red.injected.painful.eye,.
blurry.vision,.halos.around.light,.midKdilated.pubil,.N\V.and.headache..
https://emedicine.medscape.com/article/1206956Kclinical.
..
37.##[Case#of#foreign#body#in#the#eye,#successfully#remove,#what#is#next#?#
a......Topical.steroid.
b......Topical.antibiotic.
Answer:.B.
Explanation:.
Following.removal.of.the.conjunctival.foreign.body,.two.drops.of.a.topical.broadKspectrum.antibiotic.
drop.should.be.placed.in.the.affected.eye..If.there.is.no.corneal.abrasion.or.significant.inflammation.
of.the.eye,.no.further.treatment.or.followKup.is.necessary..The.patient.should.be.instructed.to.return.
if.the.foreign.body.sensation.returns.or.if.any.symptoms.of.pain,.redness,.or.visual.changes.occur..
https://emedicine.medscape.com/article/1844102Koverview#showall.
..
38.##HIV#pt#came#for#ophthalmology#follow#up#you#found#cotton#wool#spots#in#the#eye,#what#is#the#
cause#of#this#condition#in#this#pt:#
a......HIV.
b......EBV.
Answer:.A..
Explanation:.HIV.retinopathy..Source:.(Wills.Eye.Manual,.Page.300).
..
40.##(Pic)#of#swollen#upper#eyelid,#no#discharges,#hx#of#eye#redness,#tearing#in#the#morning,#
decrease#vision,#no#itching,#temp:#38.1#
a......Stye.
b......Chalazion.
c......Vernal.conjunctivitis.
d......Orbital.cellulitis.
Answer:.D.
##
##
##
Q.#What#is#the#treatment#of#orbital#cellulitis?#
IV.ceftriaxone.with.Vancomycin.
Answer:.BroadKspectrum.intravenous.antibiotics.to.cover.GramKpositive,.GramKnegative,.and.
anaerobic.organisms.are.recommended.for.48.to.72.hours,.followed.by.oral.medication.for.at.least.1.
week..Ee.currently.recommend:.
..
AmpicillinKsulbactam.3.g.i.v..q6h.in.adults;.300.mg/kg.per.day.in.four.divided.doses.in.children,.
maximum.daily.dose.12.g.ampicillinKsulbactam.(8.g.ampicillin.comK.ponent)..
or.
PiperacillinKtazobactam.4.5.g.i.v..q8h.or.3.375.g.q6h.in.adults;.240.mg.of.piperacillin.
component/kg/day.in.three.divided.doses.in.children,.maximum.daily.dose.18.g.piperacillin..
For.adults.who.are.allergic.to.penicillin.but.can.tolerate.cephalosporins,.use.vancomycin.as.dosed.
below.plus:.
Ceftriaxone.2.g.i.v..q.d..and.metronidaK.zole.500.mg.i.v..q6–8h.(not.to.exceed.4.g.per.day)..
Source:.Wills.Eye.Manual.(Page.182K183).
..
41.##Patient#has#painful#big#mass#in#the#lower#eyelid#beside#upper#nose,#what#is#your#management?#
a......topical.steroid.
b......Surgical.drainage.
c......oral.antibiotics.
Answe:.B.
Explanation:.In.general,.dacryocystitis.is.a.surgical.disease..Surgical.success.rates.in.the.treatment.of.
dacryocystitis.are.approximately.95%..Acute.cases.are.best.treated.surgically.after.the.infection.has.
subsided.with.adequate.antibiotic.therapy..
For.acute.dacryocystitis,.an.external.dacryocystorhinostomy.is.preferred.after.several.days.of.
initiating.antibiotic.therapy..Rarely,.dacryocystorhinostomy.must.be.performed.during.the.acute.
phase.of.the.infection.to.facilitate.clearing.of.the.infection..Reference:.Medscape..
..
42.##Blue#corneal#picture.=.stained.by.fluorescein.stain.to.detect.corneal.abrasion.or.foreign.bodies.
in.the.eye..Treatment.of.abrasion.is.topical.cycloplegic,.topical.antibiotic.and.topical.analgesia..
Reference.http://www.aafp.org/afp/2013/0115/p114.html.
..
..
43.##Eye#nail#abrasion,#rx:#
a.......anti.viral.drops.
b.......antiviral.ointment.
c.......contact.lens.
d.......steroids.(no.sure).
Explanation:.Treatment.of.abrasion.is.topical.cycloplegic,.topical.antibiotic.and.topical.analgesia..
Reference.http://www.aafp.org/afp/2013/0115/p114.html.
..
..
44.##Corneal#abrasion?#
a... Fitting.lens.
b... Steroids.
c... Antiviral.
.Question.is.incomplete..
Steroids.can.be.given.with.severe.keratitis.but.herpes.keratitis.should.be.ruled.out.first..
In.abrasions,.steroids.are.not.given..Lenses.are.not.used.in.cases.of.abrasions.and.ulcers..
Treatment.of.abrasion:.topical.cycloplegic,.topical.antibiotic.and.oral.analgesia..
..
More.information.for.corneal.abrasion:.
..
1..Antibiotic.
—Noncontact.lens.wearer:.Antibiotic.ointK.ment.(e.g.,.erythromycin,.bacitracin,.or.
bacitracin/polymyxin.B.q2–4h).or.antibiK.otic.drops.(e.g.,.polymyxin.B/trimethoprim.or.a.
fluoroquinolone.q.i.d.)..Abrasions.secK.ondary.to.fingernails.or.vegetable.matter.should.be.covered.
with.a.fluoroquinolone.drop.(e.g.,.ciprofloxacin,.moxifloxacin).or.ointment.(e.g.,.ciprofloxacin).at.
least.q.i.d..
—Contact.lens.wearer:.Must.have.antipseuK.domonal.coverage..May.use.antibiotic.ointK.ment.or.
antibiotic.drops.at.least.q.i.d..
..
2..Cycloplegic.agent.(e.g.,.cyclopentolate.1%.to.2%.b.i.d.or.t.i.d.).for.traumatic.iritis.which.may.
develop.24.to.72.hours.after.trauma..Avoid.steroid.use.for.iritis.with.epithelial.deK.fects.because.it.
may.retard.epithelial.healing.and.increase.the.risk.of.infection..Avoid.use.of.longKacting.cycloplegics.
for.small.abraK.sions.to.allow.for.faster.visual.recovery..
..
3..Patching.is.rarely.necessary..Patching.may.be.helpful.for.comfort,.but.DO.NOT.patch.if.the.
mechanism.of.injury.involves.vegK.etable.matter,.fingernails,.or.if.the.patient.wears.contact.lenses..
Be.careful.that.the.patch.is.properly.placed.so.that.the.upper.lid.is.totally.prevented.from.opening.as.
this.can.cause.a.serious.abrasion..
..
4..Consider.topical.nonsteroidal.antiKinflammaK.tory.drug.(NSAID).drops.(e.g.,.ketorolac.0.4%.to.0.5%.
q.i.d..for.3.days).for.pain.control..Avoid.in.patients.with.other.ocular.surface.disease.and.in.
postoperative.patients..Oral.acK.etaminophen,.NSAIDs,.or.narcotics.(in.severe.cases).can.also.be.
used.for.pain.control..
..
5..Debride.loose.or.hanging.epithelium.beK.cause.it.may.inhibit.healing..A.cottonKtipped.applicator.
soaked.in.topical.anesthetic.(e.g.,.proparacaine).or.a.sterile.jewelers.forceps.(used.with.caution).may.
be.utilized..
..
6..No.contact.lens.wear..Some.clinicians.use.bandage.contact.lenses.for.therapy..We.rareK.ly.do.
unless.the.size.of.the.abrasion.and.disK.comfort.warrants.it.and.there.is.poor.healK.ing.in.the.absence.
of.infection..If.a.bandage.contact.lens.is.used,.patients.should.use.prophylactic.topical.antibiotics.
(e.g.,.polyK.myxin.B/trimethoprim.or.a.fluoroquinolone.q.i.d.).and.should.be.followedKup.daily.for.
evaluation.and.contact.lens.replacement..Refrence:.Wills.Eye.Manual,.page.17..
..
45.##Picture#of#corneal#ulcer#that#is#showing#dendritic#lesion.#What#other#feature#is#associated#with#
it?#
a......Scleritis.and.episcleritis.
b......Chorditis.and.retinitis.
c......Optic.neuritis.
d......Hypoesthetic.cornea.(decrease.the.corneal.sensation.of.pain).
Answer:.D.
Explanation:.Herpes.keratitis.results.in.decreased.sensation.over.the.cornea.
..
46.##How#is#corneal#ulcer#diagnosed?#
a... Slit.lamp.
b... Fluorescein.dye.
Answer:.B.
..
47.##What#dye#is#used#in#corneal#ulcer#(pic#of#blue#color):#
a... Fluorescein.dye.
b... Rose.Bengal.dye.
c... Geimsa.
d... Lissman.Green.B.stain.
Answer:.A.
..
48.##Corneal#ulcer#treatment:#
a.......oral.ABx,.cycloplegic.
b......ABx.ointment,.
c......cotton.bud.debridement.....
d......(Not.sure.about.choices).
Answer:.B.
https://emedicine.medscape.com/article/1195680Ktreatment.
..
49.##Female#with#corneal#abrasion,#normal#visual#acuity#normal#IOP,#by#examination#the#lower#
eyelid#rolled#in,#diagnosis:#
a)Uveitis.
b)Glaucoma.
c)Entropion.
d)Ectropion.
Answer:.C.
51.##History#of#URTI#then#developed#watery#eye#
a... Viral.conjunctivitis.
Answer:.A..
Explanation:.URTI.caused.by.adenovirus.infection,.followed.by.adenovirus.
conjunctivitis..
..
52.##[Redness#in#the#eye#with#presence#of#cat#in#the#home#
a... allergic.conjunctivitis.
Answer:.A.
53.##Complication#of#cataract#surgery#
a... endophthalmitis.
https://www.webmd.com/eyeKhealth/cataracts/extracapsularKsurgeryKforK
cataracts.
Answer:.A..Endophthalmitis.is.the.most.serious.and.feared.side.effect.post.occular.surgeries..
Other.early.complications.post.cataract.surgery:.cystoid.macular.edema,.retinal.detachment,.corneal.
edema,.hyphema..
Late:.Glaucoma,.dislocated.IOL,.staining.of.the.lens.capsule,.retinal.detachment,.ptosis..
..
54.##Pediatric#patient#with#strabismus,#what#is#the#complication#
a... amblyopia.
Answer:.A.
..
55.##Superior#oblique#muscle#movement#
a......Medially.down.
b......Medially.up.
Explanation:.Superior.oblique.action:.intorsion,.abduction.and.downward.gaze.
The.righ.answer.shouold.be.Lateral.and.down..
..
56.##Infant#since#birth#unilateral#red#eye#associated#with#continuous#tearing#
a......Congenital.glucoma.
b......Chlamydia.
c......Gonorrhea.
Answer:.A.
Explanation:.The.primary.symptoms.of.primary.congenital.glaucoma.are.epiphora,.photophobia,.and.
blepharospasm..Reduced.vision.can.also.occur.from.corneal.edema.or.progressive.myopia.and/or.
astigmatism..
Signs.include:.epiphora,.conjunctival.erythema,.corneal.enlargement,.corneal.clouding,.Haab.striae,.
abnormally.deep.anterior.chamber,.myopia.and/or.astigmatism,.and.enlarged.optic.nerve.cupping..
http://eyewiki.aao.org/Glaucoma,_Congenital_Or_Infantile.
..
58.#####Case#of#acute#angle#closure#glaucoma,#what#is#the#first#step#in#management?#
a......Acetazolamide.
b......Timolol.
c......Topical.steroids.
Answer:.A.
Explanation:.
.Acetazolamide.should.be.given.as.a.stat.dose.of.500.mg.IV.followed.by.500.mg.PO..A.dose.of.a.
topical.betaKblocker.(ie,.carteolol,.timolol).will.also.aid.in.lowering.IOP..
https://emedicine.medscape.com/article/798811Ktreatment#d10.
..
60.##Ophthalmology#ex#for#DM2#patient#every#
a......6.months.
b......12.months.
c......24.months.
d......36.Months.
Answer:...B......................... .
..
62.##Regarding#distichiasis#definition?#(multiple#defintions#and#u#choose#the#most#accurate)#
Explanation:.Distichiasis.is.the.growth.of.lashes.from.the.meibomian.gland.orifices,.which.can.irritate.
and.damage.the.corne.(it.results.in.an.extra.raw.of.eyelashes).
..
.
.
.
.
.
.
.
.
.
64.##Lady#with#mucopurulent#eye#discharge#when#she#woke#up#she#her#eye#was#stuck#she#couldn't#
open#it.#On#exam,#cornea#was#normal,#red#conjunctiva.#What#is#the#Dx?#
a......Viral.
b......Bacterial.(correct).
Answer:...B,.in.viral.it.is.watery.discharge.......................... .
..
65.##Visual#acuity#200#of#one#eye#+#Esotropia,#other#normal.#What#is#the#intial#thing#to#do?#
a... Fundoscopy.
b... Muscle.resection.
Answer:.this.is.a.case.of.Amblyopia,.patching.should.be.started..
..
66.##Progressive#painless#vision#loss.#Drusen#spots#and#central#scotoma,#what#is#the#diagnosis:#
a... Macular.degeneration.
Answer:..A......................... .
..
Explanation:.In.age.related.macular.degenaration,.extracellular.breakdown.deposits.called.“drusen”.
develop.in.Bruch’s.membrane.causing.blockage.of.the.drainage.to.the.choroidal.bed..Findings.
include:.localized.retinal.atrophy.and.pigmentary.changes.in.the.macula.that.correlate.with.poor.
central.vision..The.visual.loss.occurs.slowly,.however,.and.takes.many.years.to.progress..
Source:.Wills.Eye.Manual.(Page.322K323).
..
67.###Picture#of#one#eye#conjunctivitis,#how#to#prevent#the#infection#for#the#other#eye?#
a... Eye.drop.
b... Eye.ointment.
c... Hand.washing.
Answer:.C.
..
..
73.##Patient#has#painful#big#mass#in#the#lower#eyelid#beside#the#upper#nose,#what#is#your#
management?#(Dacrocystitis)#
a... Topical.steroids.
b... Surgical.drainage.
c... Oral.antibiotics.
Answer:...C...
Explanation:.surgical.intervention.is.made.when.the.acute.episdode.has.resolved..
Source:.Wills.Eye.Manual.(Page.145K146)....................... .
##
74.##Patients#has#decreased#vision#loss#and#went#to#optometrist#and#got#new#glasses,#one#
week#he#comes#back#with#decreased#vision#with#the#new#glasses#he#was#referred#to#
ophthalmologist#due#to:....................................... ...................................................... .
a......Cataract.formation.
b......Increase.glucose.in.lens.
c......Increase.sorbitol.in.lens.
Answer:...C.. .
Explanation:..Rapid.changes.in.refraction.indicates.osmotic.changes.in.the.lens.by.
sorbitol..(Couldn’t.find.a.source)................... .
..
75.##Pt#wear#contact#lens#then#c/o#pain#in#rt#eye#and#discharge#what#is#the#organism#?#
a... H.influenza.
b... Acanthameba.
c... Nesseria.meningitis.
d... Pnumococcal.Chlymedia...
Answer:...B.
Explanation:.
Pseudomonas.aeruginosa.is.the.most.commonly.recovered.causative.organism.in.contact.lensK
related.disease,.followed.by.GramKpositive.bacteria,.fungi.and.Acanthamoeba.......................... .....................................
https://www.ncbi.nlm.nih.gov/pmc/articles/PMC3272197/.
##
76.##Pt#with#myopia#and#chorioretinal#degeneration,#this#is:############################## ####### #
a... Physiological.
b... Pathological.
Answer:.B.
Explanation:.myopia.severe.enough.to.cause.retinal.complications.is.called.
pathological.myopia..
Source:.http://eyewiki.aao.org/Pathologic_myopia_(myopic_degeneration).
##
77.##Picture#of#upper#eyelid#swelling,#and#they#gave#history#of#eye#pain#and#tearing#what#
else#could#be#found:#
a......Uveitis.
b......Discharge.
c......Scleritis.
Answer:.Uveitis.and.scleritis.don’t.cause.upper.eye.lid.swelling..Discharge.could.be.the.answer.if.this.
was.a.case.of.orbital.cellulitis........................................................................... .
78.##Pt#was#taking#anti#TB#meds#and#now#he#presented#with#red#eye#and#pain#along#with#
photophobia#what#the#dx?#
a......Bacrtial.conjunctivitis.
b......Viral.conjunctivitis.
c......Uveitis.
d......Optic.neuritis.
Answer:...D.................................................................................................................. .
https://www.drugs.com/sfx/ethambutolKsideKeffects.html.
...... ..........................................................................
79.##An#old#man,#presents#to#the#ER#with#the#complaint#of#painful#Rt#eye#associated#with#
nausea.#O/E#the#Rt#eye#was#injected,#with#semi[#dilated#pupil.#Lt#eye#was#normal.#What#is#
the#correct#Dx#of#these?#
a... Uveitis..
b... Primary.openKangle.glaucoma..
c......Acute.angleKclosure.glaucoma..
d... Irrelevant.choice..
..............................
Answer:...C.. .
..
80.##Child#exposed#to#fingernail#injury,#what#this#patient#may#have?#
a......Purulent.tearing.
b......Photophobia.
c......Vision.loss.
Answer:...B.
..
81.##Same#as#previous#scenario,#but#asking#about#management:#
Answer:...Topical.antibiotics.
..
Q.#Mother#have#a#photophobia#and#severe#eyes#pain#after#she#received#accidentally#
fingernail#trauma#by#her#daughter#how#to#manage?#
Answer:...Topical.antibiotics.
..
82.##Best#treatment#for#ptosis:#
a......Muller.resection.
b......Levator.muscle.resection.
c......Frontalis.suspension.
Answer:...C....
If.there.is.levator.dysfunction>.frontalis.suspension..
Answered.by.a.resident..
..
83.##Old#age#HTN,#DM#had#painless#vision#loss#examination#show#drusen#diagnosis?#
a......Central.retinal.vein.occlusion.
b......Central.retinal.artery.occlusion.
c......Macular.degeneration.
Answer:...C.
..
84.##Patient#with#red#eye+#history#of#cough,(picture#of#viral#conjunctivitis#attached).How#
to#prevent#transmission?#
Answer:...Hand.washing.
................................. .
85.##Patient#with#red#eye,#cillary#flush,#photophobia,#dendritic#lesion#in#the#cornea:######################################
# #
a......HSV.of.eye.
b......dentric.lesion...
............Answer:...A.
........................................................................................
86.##Long#scenario#about#glaucoma,#pain,#congestion#of#cornea#[#vomiting,#photophobia,#
what#is#the#treatment?#
a......acetazolmide..
b......Corticosteroids.
c......Antibiotics.
Answer:...A.
..
87.##Decrease#vision#&#painful#eye#movement?#
Answer:...Optic.neuritis.
..
88.##What#cause#color#vision#loss?#
Answer:...Optic.neuritis.
..
89.##Young#healthy#pt#e#red#eye#circumcorneal#congestion,#IOP#30,#dilated#pupil,#what#best#
drug?#
a......Latanoprost,.
b......something.b.blocker,.
c......acetazolamide,.....
d.....
Answer:...C.
........... Acute.angle.glaucoma.Treatment:.
beta.blocker.
pilocarpine.eye.drop.
Mannitol.
azetolazmide.
These.all.used.for.glaucoma.
..
91.##Eldely#Patient#with#progressive#vision#loss#for#6#m,#came#to#ER#with#painfull#red#eye,#with#
tearing#and#white#color.#IOP#is#high#What#is#the#cause:#(Scenario.not.complete).
a......Malignant.glaucoma.
b......Adcanced.rubeosis.iridis.
Malignant.glaucoma:.AKA.aqueous.misdirection.syndrome..Happens.post.op..Presents.with.acute.
angle.closure.and.high.IOP.that.is.resistant.to.treatment..
Rubeosis.iridis:.history.of.retinal.ischemia.(PDR,.CRAO,.CRVO).and.vessels.on.iris.causing.angle.
closure..
92.##Treatment#of#severe#ptosis#no#other#details?#
A..Frontalis.suspension.
B..Levator.resection.
Levator.dysfunction.>.frontalis.suspension..
.
93.##Patient#with#decrease#color#vision?#
A... Optic.atrophy.
B... Iridocyclitis.
C... Forget.other.options.
Answer:...A.
Color.vision.function.of.optic.nerve.so.look.for.optic.nerve.pathology.like.optic.neuritis..
Cataracts/diabetic.retinopathy.DO.NOT.cause.color.blindness..
##
Q.Patient#presented#with#loss#of#color#vision..#what's#the#diagnosis?#
Answer:...Optic.atrophy.
http://emedicine.medscape.com/article/1217760Koverview.
..
94.##Child#with#30#degree#esotropia,#whats#your#management?#
A...............Orthoptics.
B...............surgery.
C...............Prism.
30.degrees.of.esotropia.is.a.large.angle.so.surgery.(but.only.if.it.says.medial.rectus.recession)..
Answered.by.resident..
..
#################95.#Brown#dicoloration#of#eye#then#disappeared.#What#is#the#drug#:#
A... Vanco.
B... Metacloprmide.
C... Lanatoprost.
Answer:.C.
..
96.##Patient#with#COPD#has#glaucoma.#What#to#give?#
A... Topical.temolol.
B... Topical.something.olol.
C... Oral.Acetazolamide.
Answer:..C.
..
97.##[#Drugs#used#for#glaucoma:.Page.500.schedule.
..
98.##pt#came#for#ophthalmology#check#up,#has#optic#disc#cupping.#
################### Tonometry#showed#high#IOP.#What#would#u#tell#this#pt:#(Incomplete)#
A... 1/.tonometry.is.sufficient.
B... 2/do.check.up.for.blood.related.members.
C... 3/interventions.may.reverse.these.changes.
Tonometry.is.not.sufficient.(may.need.fundoscopy)..Screening.relatives.not.typically.recommended..
Glaucoma.changes.are.irreversible..Answered.by.resident..
..
99.## #Four#to#five#cases#about#bacterial#and#viral#conjunctivitis#
Viral>.watery.discharge.with.history.of.URTI..
Bacterial>.mucopurulent.discharge..
Allergic>.watery.discharge.and.itching..
..
100.##############Case#of#Wilson#disease#with#eye#manifestations.#What#to#give?#
Answer:.Penicillamine.
Eye.manifestation.in.Wilson’s.>.Kayser.Fleischer.ring..
..
101.###Patient#with#headache#and#his#eyes#pushed#outward.#What’s#the#management?#
A... Head.MRI.
B... Corticosteroids.
Scenario.incomplete..
If.suspecting.orbital.cellulitis.(with.fever,.visual.disturbance.and.restricted.motility).>.CT.scan.is.
sufficient..
If.suspecting.intracranial.tumor.>.MRI..
..
102.#SCENARIO#of#ptosis#(no#other#information#available)#there#is#elevator#muscle#dysfunction,#
what#is#the#management?#
A... 1Kmullurectomy.
B... 2Klevtor.resection.
C... 3Kfrontalis.suspension.
D.. 4Klevator.muscle.aponeurosis.
Answer:.C.
Levator.dysfunction>.frontalis.suspension..Answered.by.resident..
Note:.To.answer.this.question.we.have.to.know.three.variables.which.are:.
1K.Age.of.the.patient.(congenital,.pediatric.or.older).
2Kunilateral.or.bilateral.
3Kseverity.of.the.symptoms.(mild,.severe).and.here.is.the.summary:.
If.it.congenital.mild.unilateral,.no.severe.ptosis.or.symptoms.first.option.is.Levator#resection.BUT.if.
the.patient.is.old.or.the.ptosis.is.bilateral.or.severe.ptosis.go.and.do.frontalis#suspension#
Please.go.and.
..
103.#Patient#k/c#of#HF#&#schizophrenia#on#medication,#came#with#brownish#discoloration#
in#retina#&#brown#vision..#what's#the#drug?#
Answer:.Thioridazine.
..
106.#Pseudotumor#of#the#eye##>#steroids#
Pseudotumor.cerebri:.usually.obese,.presenting.with.headache.and.visual.disturbance..
Associated.with.6th.nerve.palsy..Findings:.High.ICP.and.papilledema..Treatment:.steroids..
..
109.###picture#of#HSV#on#cornea:#
A... episcleritis.
B... scleritis.
C... uveitis.
D.. Corneal.sensation.
Answer:D.
..
111.#Side#effect#of#topical#steroids:#
A... cataract.
B... glaucoma.
Both.correct.but.glaucoma.more.concerning..
..
112.#Child#came#to#eye#evaluation#the#right#eye#20/20#the#left#eye#20#/200#,#the#left#eye#
turne#in#and#the#eye#seems#crossed,#no#restricted#movement#in#the#eye#examination#of#
extraocular#muscles#normal.#what#is#the#diagnosis:#
A... Congenital.cataract..
B... Nystagmus.
C... strabismus.
Answer:.C.
..
113.#Long#scenario#of#open#angle#glucoma#asking#waht#is#the#mechanism#due#to#
obstruction#of#aqueous#drainage#within#the#trabecular#meshwork#and#its#drainage#into#
the#Canal#of#Schlemm??#
..
114.#45#y#farmer#presented#complaining#of#dry#eye#he#has#smoked#2#pack/day#for#20yrs#
what#is#the#management#besides#artificial#tearing#
A... a)advise.him.to.exercise.(.blinking.exercises).
B... b)stop.smoking.
C... c)wear.glasses.
D.. d)all.of.the.above.
Answer:.B.
Smoke.is.irritant.to.eye..Smokers.twice.as.likely.to.experience.dry.eye..
..
115.##############Trauma#to#the#face#that#lead#to#enucleation#of#one#eye.#Patient#came#later#
with#pain#in#the#other#eye.#What#it#the#pathophysiology?#
Answer:.Release.of.sequestered.antigen.
case.of.sympathetic.ophthalmia.
..
116.##############Patient#with#ptosis#and#eye#looking#down#and#out.#What#is#affected?#
A... 3rd.
B... 4th.
C... 3th.and.6th.
D.. 3rd.and.4th.
Answer:.A.
..
117.##############Case#of#angle#closure#glaucoma#and#asked#about#the#diagnosis#
The#same#case#exactly#but#asked#about#treatment?#IV#acetazolamide#and#4%#pilocarpine#
Complication#of#cataract#surgery?#
Answer:..Endophthalmitis.
..
118.##############Which#one#of#the#following#drugs#is#contraindicated#to#be#given#in#Acute#
closure#angle#glaucoma?#
.Any.mydriatic.(ex:.atropine).will.exacerbate.crisis..
..
Q.#In#open#angle#glaucoma,#which#drug#is#contraindicated?#
..
119.###############Bronchocostrictive#disease#with#acute#angle#closure#glaucoma.#What#are#you#
going#to#give?#
A..Timolol.
B..Acetazolamide.
Answer:.B.
..
120.##############Adult#Case#of#eye#watery#discharge,#associated#with?#
A... Dust.and.pollen.will.increase.symptoms.
B... Retinal.exam.will.show.diabetic.and.hypertension.changes.
C... Something.about.glaucoma.
Answer:.A.If.asking.about.allergic.conjunctivitis..
..
121.##############Picture#of#glaucoma,#What#to#do#with#it#?###
Inform.blood.relative.
..
122.##############Retnoblastoma#causes?#
A......Strabismus.
B......Squent.
C......Leucocoria.(white.pupil).
Answer:.C.
..
123.##Man#has#Eye#pain,#seeing#halos#,#headache#,#many#GI#symptoms#,#Eye#examination#:#
red#and#injected#vessels#.#What#is#the#diagnosis?#
aK.digoxin.toxicity.
bK.Angle.closure.Glaucoma.
answer.B.
..
124.##############Clinical#Features#of#Angle#Closure#Glaucoma:#
•.red,.painful.eye.=.RED.FLAG.
•.unilateral,.but.other.eye.increased.risk.
•.decreased.visual.acuity,.vision.acutely.blurred.from.corneal.edema.
•.halos.around.lights.
•.nausea.and.vomiting,.abdominal.pain.
•.fixed,.midKdilated.pupil.
•.corneal.edema.with.conjunctival.injection.
•.marked.increase.in.IOP;.may.be.noticeable.even.to.palpation.(>40.mmHg).
•.shallow.anterior.chamber.±.cells.in.anterior.chamber.
Toronto.Note.
..
126.##############Patient#presented#with#history#of#right#eye#pain#and#decrease#vision#..#flurocin#
staining#showed#(dendrites)..#wht’s#the#diagnosis?#
Answer:.herpes.keratitis.
http://emedicine.medscape.com/article/1194268Koverview.
..
128.#Target#diastolic#Bp#in#HTN#retinopathy:#
80K90.in.2.days.
..
129.#Patient#came#to#ER#due#to#car#accident#had#DM#and#HTN##and#loss#of#vision#in#the#
peripheral:#
A... Cataract.
B... Glaucoma.
C... RD.
Answer:.C.
..
130#A#question#with#a#picture#attached#showing#the#everted#lower#eyelid,#what#is#it#called?#
Answer:.Ectropion.
..
131.##############Which#of#the#following#can#be#managed#with#trabeculectomy?#
A...............Angle.closure.glaucoma.
B...............Open.angle.glaucoma.
Answer:B.
..
132.#Patient#with#bilateral#exophtalmous,#pulsatile,#with#bruit#on#auscultation:#
A... hyperthyroidism.
B... cavernous.sinus.thrombosis.
C... carotidKcavernous.fistula.
Answer:.C.
..
133.#picture#of#corneal#ulcer#that#is#showing#dendritic#lesion.#What#other#feature#is#associated#with#
it?#
A... Scleritis.and.episcleritis.
B... Chorditis.and.retinitis.
C... Optic.neuritis.
D.. Corneal.hypoasthesia.
Answer:.D.
..
134.#Pt#with#unilateral#viral#conjunctivitis#then#became#bilateral.#What’s#your#advice#to#avoid#
spread#of#the#infection?#
A..Isolate.yourself.from.others..
..
135.#Patient#on#neostigmine#but#further#evaluation#showed#drop#eyelid#?########## #
Stop.neostagmin.(.my.answer.).Add.pyrdostagmin................................................................... .
This.a.case.of.ocular.MG.so.the.tx.is.Immune.suppression.with.steroids.is.often.the.main.therapy..
https://www.ncbi.nlm.nih.gov/pubmed/22037997.
..
136.#One#with#eye#trauma,#the#first#thing#you#do#is#to#r/o:#
A..foreign.body.
B..keratitis.
C..conjunctivitis.
Answer:.A.
..
137.#Hypertensive#patient,#showing#change#in#the#optic#disc#what#is#your#next#step#in#
management?#
A... Referral.to.ophthalmology.
B... Treat.her.
Answer:.A.
..

Psychiatry.
##
..
1[Father#with#Schizophrenia#have#child#percentage#of#affected?#
AK5%.
BK10%.
CK15%.
DK.20.
Answer.Is.B.10%.
Reference.FA.step.one.2016.
..
2[ADHD#child#,#can't#involve#for#anything#for#long#time#,#like#in#school#or#even#when#he#
playing#a#video#games#he#can't#finish#it#and#searching#for#something#else#to#do#and#so#
on,#which#type#of#ADHD#he#has?#
AK.inattentive.
BK.impulsive.
CK.hyperactive.impulsive..
Answer.is.A.
Three.types.of.ADHD:.
1Kinattention:.getting.distracted,.having.poor.concentration.and.organizational.skills.and.have.
trouble.focusing.on.a.single.task.
2Kimpulsivity:.interrupting,.taking.risks.
3Khyperactivity:.never.seeming.to.slow.down,.talking.and.fidgeting,.difficulties.staying.on.task.
..
3[Patient#have#depression#and#came#to#the#clinic#complaining#of#decreased#libido#which#
drugs#you#will#shift#your#patient#to#?#
AK.paroxtein.
BK.TCA.
CK.amytriptaline.
Correct.answer.most.likely.one.of.the.three.highlight.with.red.:.
..
Consistent.evidence.shows.that,.with.the.exception.of.bupropion.(Wellbutrin),.trazodone.(Desyrel).
and.nefazodone.(Serzone),.antidepressant.medications.may.cause.a.decline.in.libido.or.sexual.
functioning.despite.improvement.of.depression.
Ref:.http://www.aafp.org/afp/2000/0815/p782.html.
..
4[questions#about#post#traumatic#stress#disorder,#I#can't#remember#exactly#what#was#about#
PostKtraumatic.stress.disorder.(PTSD).is.a.mental.health.condition.that's.triggered.by.a.terrifying.
event.—.either.experiencing.it.or.witnessing.it..Symptoms.may.include.flashbacks,.nightmares.and.
severe.anxiety,.as.well.as.uncontrollable.thoughts.about.the.event.
PTSD.symptoms.are.generally.grouped.into.four.types:.intrusive.memories,.avoidance,.negative.
changes.in.thinking.and.mood,.and.changes.in.physical.and.emotional.reactions..Symptoms.can.vary.
over.time.or.vary.from.person.to.person.
..
..
5[21#years#old#patient#with#known#case#of#depression,#has#been#found#on#the#floor#
unconscious#with#empty#pill#bottle#,#patient#was#obtunded#,#dilated#pupil#and#
unreactive#bilaterally#and#other#symptom(#I#can#not#remember)#
What#is#the#medication#the#patient#most#likely#on?#
AKSertraline.
BK.Fluoxetine.
CK.Other.SSRI.
DK.Amitriptyline.
Answer.is.D.
.Ref:.https://emedicine.medscape.com/article/819204Kclinical#b4.
..
6[which#one#of#the#following#SSRI#has#the#greatest#risk#in#pregnancy?#
AK.Escitalopram.
BK.Fluoxetine.
CK.Paroxetine.
DK.Sertraline.
Answer.C.
Ref:.https://www.drugs.com/pregnancy/paroxetine.html.
..
..
7[2#year#and#half#kid#with#long#scenario#about#impulsive#behavior#and#communication#
impairment#with#other#and#the#distinct#point#of#the#question#is#echolalia#(which#is#
repetition#of#other#words)#what#is#the#diagnosis:#
AK.ADHD.
BKASPENGER.SYNDROME.
CK.AUTISTIC.DISORDERS.
DK.……………………………...
B.is.a.part.of.C.,.need.more.information...
..
8[#there#was#question#about#ADHD#diagnostic#criteria#according#to#ICD10#CLASSIFICATION,#
but#I#cannot#remember#the#answers:#
But.it.was.like.this.:(.number.of.feature.and.the.description.of.it).for.example:.
AK.3.hyperactive,.3.inattentive,.and.other.
BK.2.hyperactive,.3.inattentive.
CK.2.hyperactive,.2.inattentive.
..
Answer:.
At.least.6.inattentive.+3.hyperactive.+.1.impulsive.
..
Ref:.https://adhd.org.sa/en/adhd/resources/diagnosingKadhd/diagnosticKcriteria/icdK10Kcriteria/.
..
9[#pt#suddenly#feel#the#environment#is#strange#?#
.AK.depersonalization.
BK.derealization.
Answer.B.
..
Derealization.(sometimes.abbreviated.as.DR).is.an.alteration.in.the.perception.or.experience.of.the.
external.world.so.that.it.seems.unreal..Other.symptoms.include.feeling.as.though.one's.environment.
is.lacking.in.spontaneity,.emotional.colouring,.and.depth..It.is.a.dissociative.symptom.of.many.
conditions.
https://www.mayoclinic.org/diseasesKconditions/depersonalizationKderealizationK
disorder/symptomsKcauses/sycK20352911.
..
..
10[#child#always#alone#,#dose#not#have#toys#,#doesn’t#play#with#others#,#what#is#the#
relation#cause#or#something?#
AK.intelligence.
BK.interpersonally.
Answer:.B.?.
..
11[23#yrs#old#pt#has#hallucinations,#delusions,#for#1#month,#the#doctor#diagnosed#him#as#
a#case#of#schizophrenia,#Which#of#the#following#is#against#the#diagnosis#of#schizophrenia?#
A......1/.age.of.the.pt.
B......2/hallucinations.
C......3/.delusions.
D.....4/.duration.of.symptoms.
Answer.is.D.
To.diagnose.schizophrenia#at#least#duration#should#be#6#month#
..
12[pt#think#he#has#cancer#went#to#6#doctors#and#examination#was#normal,#but#still#he#think#
he#got#cancer,Dx?#
Hypochondriasis.
..
13[#7#yrs#old#boy#with#clear#manifestation#of#ADHD#which#of#the#following#drugs#might#be#
given?#
AKAtomaxatine.
Answer.is.A.
..
Drugs.can.be.used.:.
amphetamine..and.methylphenidate.Stimulants.usually.reduce.hyperactivity.and.impulsivity.and.
improve.focus..
Or.Atomoxetine.
Or.Clonidine.and.guanfacine.
https://www.webmd.com/addKadhd/tc/attentionKdeficitKhyperactivityKdisorderKadhdKmedications.
..
14[#definition#of#brief#psychosis#and#Schizophrenia/#schizoaffective(#duration)#
Schizophrenia#:.People.with.this.illness.have.changes.in.behavior.and.other.symptoms.KK.such.as.
delusions.and.hallucinations.KK.that.last.longer.than.6.months..It.usually.affects.them.at.work.or.
school,.as.well.as.their.relationships..
Schizoaffective#disorder:.People.have.symptoms.of.both.schizophrenia.and.a.mood.disorder,.such.as.
depression.or.bipolar.disorder..
Schizophreniform#disorder:.This.includes.symptoms.of.schizophrenia,.but.the.symptoms.last.for.a.
shorter.time:.between.1.and.6.months..
Brief#psychotic#disorder:.People.with.this.illness.have.a.sudden,.short.period.of.psychotic.behavior,.
often.in.response.to.a.very.stressful.event,.such.as.a.death.in.the.family..Recovery.is.often.quick.KK.
usually.less.than.a.month..
##
15[Case#of#paralysis#of#the#upper#limbs#(and#I#guess#lower#limbs)#..#Dx:#conversion#
disorder.#
Conversion.disorder.is.a.mental.condition.in.which.a.person.has.blindness,.paralysis,.or.other.
nervous.system.(neurologic).symptoms.that.cannot.be.explained.by.medical.evaluation.
..
16[#A#young#woman#came#with#low#mood,#insomnia,#and#features#of#depression#with#heavy#
menses.#These#symptoms#occur#9#days#prior#to#her#menses#and#disappear#the#second#day#of#her#
menses.#How#would#you#manage#her?#
A .SSRI..
B .Estrogen..
C .Progesterone.
Answer.is.A.
..
.The.core.symptoms.of.premenstrual.syndrome.(PMS).include.affective.symptoms.such.as.
depression,.irritability,.and.anxiety,.and.somatic.symptoms.such.as.breast.pain,.bloating.and.
swelling,.and.headache..The.symptom(s).must.impair.functioning.in.some.way.and.must.remit.at.
menses.or.shortly.thereafter..Premenstrual.dysphoric.disorder.(PMDD).is.a.more.severe.form..
We.recommend.selective.serotonin.reuptake.inhibitors.(SSRIs).as.firstKline.therapy.for.women.with.
premenstrual.symptoms..Ref:.uptodate.
https://www.uptodate.com/contents/treatmentKofKpremenstrualKsyndromeKandKpremenstrualK
dysphoricK
disorder?source=search_result&search=premenstrual%20syndrome&selectedTitle=1~150#H101868
128.
..
17[define#ADHD#:#
##
Attention.deficit.hyperactivity.disorder.(ADHD).is.a.disorder.that.manifests.in.childhood.with.
symptoms.of.hyperactivity,.impulsivity,.and/or.inattention..The.symptoms.affect.cognitive,.
academic,.behavioral,.emotional,.and.social.functioning.
..
18[female#pt#with#depression#and#suicidal#attempt,#stable#for#3#months#on#paroxetine#and#
now#she#is#pregnant,#what's#next?#
AKcontinue.paroxitine.and.monitor.depression.
BKstop.paroxitine.due.to.premature...
CKstop.paroxitine.due.to.it.damage.the.baby.
DKcontinue.and.add.valium.
Most.likely.C.
..
Paroxetine.actually.carries.a.category.D.rating.from.the.U.S..Food.and.Drug.Administration.(FDA).for.
safety.in.pregnancy.
Some.SSRIs.are.generally.considered.a.safe.option.for.pregnant.women,.but.Paxil.already.carries.a.
warning.label.cautioning.against.use.during.pregnancy.due.to.the.potential.for.fetal.heart.defects..
http://www.aafp.org/afp/2012/0415/p747.html.
..
19[#pt#with#symptoms#of#schizophernia#for#3#months#and#then#return#to#normal.#what's#the#
Diagnosis?#
AKSchizophernia.
BKSchizoaffective.
CKSchzioniform.
DKbreif.psychosis.
Answer.is.C.
..
20[Anti#psychotic#causes#weight#gain?#
##
Weight.gain.and.metabolic.effects.are.the.most.prominent.side.effects.of.SGAs..Clozapine.and.
olanzapine.are.especially.associated.with.these.problems.
Olanzapine.is.the.most.common.one....Uptodate.
..
[21[##most#common#anti#psychotic#causing#dystonia?#
Among.the.FGAs,.the.highKpotency.drugs.fluphenazine,.haloperidol,.loxapine,.pimozide,.and.
thiothixene.are.usually.associated.with.the.highest.risk.of.extrapyramidal.symptoms.
..
Haloperidol.is.the.most.common.one...uptodate.
..
22[#Antipsychotic#drug#causing#QT#prolongation?#
Prolongation.of.the.QT.interval.tends.to.be.mild.with.SGAs.but.somewhat.greater.with.iloperidone.
and.ziprasidone.than.with.other.agents..uptodate.
..
[23[Atypical#Antipsychotic#least#to#cause#EPS?#
##
.risperidone.carries.the.highest.risk.of.EPS.(8.to.25.percent.in.adults).
least.risk:.Quetiapine,.iloperidone,.pimavanserin,.and.clozapine.are.the.preferred.agents.in.patients.
at.high.risk.for.EPS..uptodate.
##
..
24[#Antiparkinsonism#which#is#hepatotoxic:#
Answer:.tolcapone.
##
Tolcapone.has.been.reported.to.cause.serum.aminotransferase.elevations.above.3.times.the.upper.
limit.of.normal.in.1%.to.5%.of.patients..
Ref:.https://livertox.nih.gov/Tolcapone.htm.
..
25[TCA#side#effect?#
3.C.(.cardiotoxicty.+.Coma+.convulsions).+.Anticholinergics.
..
26[TCA#most#associated#with#weight#gain?#
Answer:.Amitriptyline.
The.cyclic.antidepressants.block.histamine.receptors.and.cause.sedation,.increased.appetite.leading.
to.weight.gain,.confusion,.and.delirium..The.most.potent.antihistaminic.drugs.are.maprotiline.and.
the.tertiary.tricyclics.amitriptyline,.doxepin,.and.trimipramine.
Ref:.
https://www.uptodate.com/contents/tricyclicKandKtetracyclicKdrugsKpharmacologyKadministrationK
andKsideK
effects?source=search_result&search=tricyclic%20antidepressants%20and%20weight%20gain&selec
tedTitle=1~150#H26?.
..
..
27[Q#about#MAOI#phenelzine,#HTN#with#cheese?#
MAOI.if.mixed.with.tyramine.conaiants.products.like.“.cheese.“.it.may.cause.HTN.crisis.
..
28K.Nocturnal#enuresis#and#depression?#
Imipramine.(TCA).
..
29[T[half#of#SSRI?#
Uptodate.says:.
The.halfKlife.for.fluoxetine.ranges.from.1.to.3.days,.and.for.its.metabolite.norfluoxetine,.4.to.16.days..
Fluvoxamine.has.a.halfKlife.of.approximately.15.hours....
..
https://www.uptodate.com/contents/selectiveKserotoninKreuptakeKinhibitorsKpharmacologyK
administrationKandKsideKeffects?source=search_result&search=TK
half%20of%20SSRI&selectedTitle=1~150#H399779781.
..
30[A#pt#with#depression#on#medications#found#comatose#with#empty#pill#bottle#beside#her#
On#invx;#ABG#showed#Metabolic#acidosis,#what#drug#overdosed?#
AKAspirin.
BK.SSRI.
Answer.is.A.
Ref:.https://emedicine.medscape.com/article/1009987Kclinical.
..
31[clear#scenario#about#depression#(low#mood#lack#of#sleep,,,,#what#is#deficient#in#this#
patient?#
serotonin.
..
32[a#pt#presented#to#psychiatry#department,#the#pt#says#that#some#people#know#what#he#is#
thinking#about#and#his#thoughts#are#known#by#others#as#if#distributed.#This#type#of#thought#is?#
Answer:.thought.broadcasting.
..
.In.psychiatry,.thought.broadcasting.is.the.belief.that.others.can.hear.or.are.aware.of.an.individual's.
thoughts..This.differs.from.telepathy.in.that.the.thoughts.being.broadcast.are.thought.to.be.
available.to.anybody..Thought.broadcasting.can.be.a.positive.symptom.of.schizophrenia..
..
33K18#yrs#old#female,#e#hx#of#amenorrhea#for#6#months,#low#BMI,#but#she#thinks#that#she#is#
fat#and#has#to#lose#weight.#OE#(#increased#hair#distribution#in#her#body#plus#other#
findings)#
What#is#the#diagnosis?#
Answer:.Aneroxia.Nervosa.
..
34[best#and#rapid#management#for#specific#anxiety?#
.A..Benzodiazipines.
.B..Sertraline.
.C..Imipramine.
.D..Bupropion.
Answer.is.A.
..
Long.Term.CBT.or.SSRI.
Short.term.BENZO.or.beta.blocker..... Ref:.FA.step.one.2016.
..
35[Best#drug#for#short#term#Mx#of#GAD#without#causing#dependence#or#addiction;#
1/alprazolam.
2/sertraline.
3/flouxetine.
4/Bupropion.
Answer:.D.(.I.think.it.is.a.misspelling.and.it.should.buspirone..).
BUSPIRONE.—.The.azapirone.buspirone.has.been.shown.in.clinical.trials.to.reduce.symptoms.of.
anxiety.in.patients.with.generalized.anxiety.disorder.(GAD),.offering.similar.efficacy.to.
benzodiazepines.without.the.risk.of.dependence..Buspirone.is.thought.to.affect.the.serotonergic.
system.via.blockade.of.5HT1A.autoreceptors..Buspirone.can.be.used.as.monotherapy.(in.the.
absence.of.comorbid.major.depression).or.for.augmentation.at.doses.of.10.to.60.mg/day.
Reference:.Uptodate..
..
36KA#child#always#kicking#his#mother,#shouting#at#her,#not#responding#to#her#commands#at#
all,#he#was#separated#from#her#after#she#was#divorced,#what#is#the#problem#here?#
1/depersonalization.
2/derealization.
3/anxiety.from.separation.
..
answer:C.“By.resident”.
correct.answer.is.Adjustment.
Adjustment.disorder.is.an.unusually.strong.or.longKlasting.reaction.to.an.upsetting.event..The.
triggering.event.might.be.a.divorce,.a.death.in.the.family,.moving.to.a.new.home,.starting.a.different.
school,.a.break.up,.or.a.big.life.disappointment..A.child.with.the.disorder.will.have.a.hard.time.
coping.with.his.emotions.and.may.become.depressed.or.anxious,.exhibit.hostility,.pick.fights,.or.
refuse.to.go.to.school.
Ref:.https://childmind.org/guide/guideKadjustmentKdisorder/whatKisKadjustmentKdisorder/.
..
37KFemale,#known#case#of#schezophrenia,#came#to#ER#with#suspeciousness,#upon#examination,#
she#was#starring#at#a#person#and#saying,#*you#can't#kill#me*#,#what#does#she#have#?#
*Derealisation.
*Concrete.thinking.
*hallucination.
Most.likely.Delusion.
..
38[An#elderly,#known#case#of#Alzheimer's#disease,#developed.hallucinations,bizarre.
behaviors#and#became#aggressive,#what#drug#to#add?#
1/Haloperidol.
2/Resperidone.
3/Chloropromazine.
4/Amytreptline.
Answer.is.B.
..
Atypical.antipsychotics.are.the.most.widely.used.class.of.psychotropic.medications.in.the.treatment.
of.AD.psychosis.and/or.behavioral.and.psychological.symptoms.of.dementia..A.metaKanalysis.of.
atypical.antipsychotics.for.aggression.and.psychosis.in.AD.suggests.that.both.risperidone.and.
olanzapine.help.reduce.aggression.and.risperidone.helps.reduce.psychosis.9.
http://www.psychiatrictimes.com/geriatricKpsychiatry/managingKpsychosisKpatientsKalzheimerK
disease/page/0/3.
..
39[pt#came#with#symptoms#of#depression.#when#the#doctor#asked#him#about#psycosis#symptoms#
he#left#the#room.#what#does#this#represent?#
AK.acting.out.
BK.Intellectual.
CK.Siblimation.
DK.Interjection.
Answer.is.A.
..
40[scenario#of#anorexia#nervousa,#admitted#to#ER,#she#
she#denies#eating#and#says#she's#not#hungry,#BMI#11.3#
What#most#likely#to#find#in#her#labs#:#
K.inc.K.
K.Dec.Creatnin.
K.Other.labs.
..
Read.about.it.here.:.https://emedicine.medscape.com/article/912187Kworkup#c9.
..
41[Young#age#female#known#case#of#Bulemia#nervosa#came#to#the#ER#after#vomiting,#what#do#
you#expect#to#see#in#her#lab:#
AKHypokalemia.
BKElevated.liver.enzymes.
Answer.is.A.
..
•.Hypokalemic.hypochloremic.metabolic.alkalosis:.Observed.with.vomiting.
•.Acidosis:.Observed.in.cases.of.laxative.abuse.
..
42[Patient#admitted#to#er#,#upon#examination:#dental#smth?#
AKAnorexia.nervosa.
BKbulemia.nervosa.
Answer.is.B.
..
43[anti[psychotic#cause#constipation?#
AntipsychoticKrelated.constipation.is.a.common.and.serious.adverse.effect.,.especially.for.people.
taking.clozapine.
Ref:.http://www.cochrane.org/CD011128/SCHIZ_drugKtreatmentsKconstipationKcausedK
antipsychoticKmedications.
..
44[Dementia#rx?#
Cholinesterase.inhibitors..These.medications.—.including.donepezil.(Aricept),.rivastigmine.(Exelon).
and.galantamine.(Razadyne).—.work.by.boosting.levels.of.a.chemical.messenger.involved.in.memory.
and.judgment.
..
45[Repetitve#idea#scenario#pt#Knows#this#is#wrong?#
.obsession.
..
46[psychotic#pt#on#neuroleptic#drug#the#side#effect?#
AKseizure...
BKakathisia.
Answer.is.B.
..
SE:.Acute.dystoniaK.AkathsiaKparkinsonismKTardive.dyskinesia....FA.step.one.
..
..
47[pt#with#Poor#self[hygiene#,#echolalia#what's#the#treatment?#
A.....Carbamazepine.
B.....SNRI.
C......Lithium.
Answer:.
Incomplete.scenario,.most.likely.antipsychotic,.
..
48[Female#fear#of##elevator#,#which#of#the#following#is#the#best#treatment#?#
AKFlooding.
BKimipramine.
CKpropranolol.
DKpsychoanalysis.
Answer.is.D.
..
49[Pt#with#hallucination#and#low#self#esteem#,#what's#the#treatment#?#
AKAntipsychotic.
BKCBT.
CKAntipsychotic.and.CBT.
answer:C..
..
50[Female#pt#with#symptoms#of#menapose#and#symptoms#of#depression#,#what's#the#treatment?#
A.....Estrogen.
B.....Progesterone.
C......Paroxetine.
.Answer:.
..
51[Impaired#focus#on#the#current#function#with#disruption#of#vitals#physical#activities.#
What#condition#have#these#characteristics?'#
1K.OCD.
2K.dissociative.disorder................................ .
3K.major.depression.
4K.psychosis.
Answer.:.most.likely.major.depression.
..
52[Prevention#of#dementia?#
AKleisure.exercises.
BKmedication.
CKjust.rehab.like.option.
..
Answer.is.A.
We.encourage.all.patients,.especially.those.with.early.dementia.and.those.with.risk.factors.for.
dementia,.to.maintain.or.increase.physical.activity.and.exercise.as.long.as.there.are.no.
contraindications..Similarly,.we.encourage.cognitive.leisure.activities.and.social.interaction.for.as.
long.as.these.are.feasible..However,.we.recognize.that.these.lifestyle.factors.remain.unproven.as.a.
means.of.preventing.dementia.
Ref:.
https://www.uptodate.com/contents/preventionKofKdementia#H18.
..
53[autism#case#the#child#eats#papers.#what#do#?#
Answer:.Behavioral.therapy.
..
54[Elderly#man#admitted#for#pneumonia,#developed#fluctuating#level#of#consciousness#and#
severely#disturbed#sleep/wake#cycle,#how#to#treat?#
a..Regular.Haloperidol.until.symptoms.abate.
b..Regular.Risperidone.until.symptoms.stop.
c..Isolate.him.in.a.dark.quite.room.
d..Allow.a.relative.to.stay.
Answer:.Most.likely.C.
..
..
55[#Leaden#paralysis#in?#
A..conversion.disorder..
B..personality.disorder..
C..somatization.
D..schizophrenia..
Regarding.these.options,.could.be.A.
K.... Leaden.paralysis.is.one.of.the.feature.of.atypical.depression.
K.... Conversion.disorder.is.a.mental.condition.in.which.a.person.has.blindness,.paralysis,.or.
other.nervous.system.(neurologic).symptoms.that.cannot.be.explained.by.medical.evaluation.
..
56[CAGE#questionnaire?#
Have.you.ever.felt.you.needed.to.Cut.down.on.your.drinking?.
Have.people.Annoyed.you.by.criticizing.your.drinking?.
Have.you.ever.felt.Guilty.about.drinking?.
Have.you.ever.felt.you.needed.a.drink.first.thing.in.the.morning.(EyeKopener).to.steady.your.nerves.
or.to.get.rid.of.a.hangover?.
..
57[Disease#associated#with#clinical#dementia:#
A..b12.deficiency.
B..Parkinson’s.
Answer.is.A.
..
58[Teenage#Pt.#Got#a#bad#grade#in#an#exam#and#feels#guilty,#he#try#to#explain#to#everyone#
he#meets/#talk#with#that#the#exam#was#poorly#written&#its#not#his#fault#that#he#got#a#
bad#grade.#what#does#this#represent?#
K.intellectualization.
K.sublimation.
Answer:.Both.choices.are.wrong..I.think.the.right.answer.is.one.of.the.missing.choices,.if.
rationalization.was.one.of.the.choices.then.I.think.it.would.an.appropriate.answer...
..
KIntellectualization:.avoiding.negative.feeling.by.excessive.use.of.intellectual.functions.and.by.
focusing.on.irrelevant.details.(e.g..physician.dying.from.colon.cancer.describes.the.pathophysiology.
of.the.disease.in.detail.to.his.12.y/o.son.).
K.sublimation:.replacing.an.unacceptable.wish.with.a.course.of.action.that.is.similar.to.the.wish.but.
does.not.conflict.with.one's.value.system..(e.g..Teenager's.aggression.toward.his.father.is.
redirected.to.perform.well.in.sports).
KRationalization:.proclaiming.logical.reasons.for.actions.performed.for.other.reasons,.usually.to.
avoid.selfKblame...(e.g...after.getting.fired,.claiming.that.the.job.was.not.important.anyway)..
Reference:.FA,.FA.psychiatry..
..
Feeling#hopeless#is#another#form#for#expressing#(or#indicates#or#assess)#which?#
K.suicidal.ideation.
K.Depression.
K.Anxiety.
Answer:.A.
Reference:.SMLE12.
..
what’s#the#most#common#sign#(presentation)#of#depression?#
A..late.morning.awaking.
B..loss.of.eye.contact.
Answer:.Could.not.find.a.source.about.the.most.common.sign.of.depression..Probably.the.question.
is.not.written.correctly,.see.the.next.question..
..
What#is#associated#with#major#depression?#
K.loss.of.eye.contact.
K.Late.awakening.from.sleep.
K.Hallucination.
Answer:.
MDD.DSMK5:.
Must.have.at.least.five.of.the.following.symptoms.(must.include.either.number.1.or.2).for.at.least.a.
2Kweek.period:.
1K.....Depressed.mood.of.the.time.
2K.....Anhedonia.(loss.of.interest.in.pleasurable.activities).
3K.....Change.in.appetite.or.weight.(increase.or.decrease).
4K.....Feelings.of.worthless.or.excessive.guilt.
5K.....Insomnia.or.hypersomnia.
6K.....Diminished.concentration..
7K.....Psychomotor.agitation.or.retardation.(i.e.restlessness.or.slowness.).
8K.....Fatigue.or.loss.of.energy.
9K.....Recurrent.thoughts.of.death.or.suicide.
..
Hypersomnia:.refers.to.symptoms.of.excessive.quantity.of.sleep,.reduced.quality.of.wakefulness.and.
sleep.inertia/sleep.drunkenness.(i.e..impaired.performance.and.reduced.alertness.after.awakening)....
They.often.complain.of.nonKrestorative.sleep,.automatic.behaviors.(routine.behavior.performed.
with.little.to.no.recall),.and.difficulty#awakening#in#the#morning..
Reference.:.FA.psychiatry.
..
schizophrenic#complaint#on#medication#came#with#metabolic#acidosis#what#is#the#drug?#
KOlanzapine.
KClozapine.
Khaloperido.
Answer:..C.(Not.sure)..
The.only.thing.I.found.is.that.patients.receiving.haloperidol.had.significantly.higher.blood.lactate.
levels.than.patients.receiving.olanzapine.
..
Probably.the.question.was.not.written.correctly.and.original.question.was.asking.about.metabolic.
syndrome.instead.of.metabolic.acidosis,.and.in.that.case.the.answer.would.be.Olanzapine.(.both.
clozapine.and.olanzapine.are.known.to.cause.it.but.it.is.much.more.common.with.olanzapine)..
..
Haloperidol.has.the.least.potential.to.cause.metabolic.syndrome..Clozapine.and.risperidone.also.
have.the.potential.to.cause.metabolic.syndrome.but.have.a.lower.potential.to.do.so.as.compared.
with.olanzapine...References:.https://www.ncbi.nlm.nih.gov/pmc/articles/PMC3136014/.
https://www.ncbi.nlm.nih.gov/pubmed/21328719.
##
Antipsycotic#drug#causes#eye#pigmentation?#
Chlorpromazine.causes.corneal.and.lenticular.deposits.
Thioridazine.causes.Retinal.deposits.
Reference:.FA.psychiatry.
..
2#cases#about#panic#disorder#(ttt#and#dx)#
DSMK5:.
K.Recurrent,.unexpected.panic.attacks.without.identifiable.trigger..
K.One.or.more.panic.attack.followed.by.>1.month.of.continuous.worry.about.experiencing.
subsequent.attacks.or.their.consequences.and/or.a.maladaptive.change.in.behavior(e.g..avoidance.
of.possible.triggers).
K.Not.caused.by.another.mental.disorder,.substance.abuse.or.medical.condition..
Reference:.FA.psychiatry.
..
In#panic#disorder,#how#to#manage?#
Answer:.If.a.single.panic.attack.is.the.diagnosis,.a.
benzodiazepine.is.the.treatment..If.panic.disorder.is.the.diagnosis,.then.pick.the.SSRI..
Reference.:.MTB2.
..
definition#of#delusion?#
Delusions.are.fixed,.false.beliefs.that.remain.despite.evidence.to.the.contrary.and.cannot.be.
accounted.for.by.the.cultural.background.of.the.individual..
Reference:.FA.psychiatry.
..
adjustment#disorder?#
DSMK5:.
1K.....Development.of.emotional.or.behavioral.symptoms.within.3.months.in.response.to.identifiable.
stressful.life.event..These.symptoms.produce:.
KMarked.distress.in.excess.of.what.would.be.expected.after.such.an.event..
K.significant.impairment.in.daily.functioning..
2K.Symptoms.are.not.those.of.normal.bereavement.
3K.symptoms.resolve.within.6.months.
“.in.adjustment.disorder,.the.stressful.event.is.not.life.threatening..In.postKtraumatic.stress.disorder,.
it.is.”.
Reference:.FA.psychiatry.
..
..
2#q#about#SE#of#antidepressant#Tca#and#SSri?#
TCA:#
KAntihistamine:.weight.gain.and.sedation.
K.Antiadrenergic:.orthostatic.hypotension,.reflex.tachycardia,.arrhythmia.(Wide.QT,QRS.and.PR.
interval).
KAnticholinergic:.Dry.mouth,.blurred.vision,.urine.retention.,.constipation,.exacerbation.of.narrow.
angle.glaucoma..
SSRi:#
GI.disturbances:.Nausea.and.diarrhea..
Insomnia.and.vivid.dreams.
Sexual.dysfunction.:.decreased.libido.,.anorgasmia.,.delayed.ejaculation..
Restlessness..
Reference:.FA.psychiatry..
..
what#is#the#least#fatal#dose#of#Tca#
A.....500.mg.
B.....1g.
C......2g.
D.....5g.
..
Answer:.B.
Reference:.Comprehensive.Emergency.Mental.Health.Care.By.Joseph.J..Zealberg,.Alberto.B..Santos,.
Jackie.A..Puckett.
..
Patient#had#history#of#pancreatic#cancer#on#chemotherapy#then#improved#completely#came#to#
doctor#concerning#about#recurrence#of#cancer#and#a#history#of#many#hospital#visits.#This#
patient#has:#
a).Malingering.
b).somatization.
c).Factitious.
d).Conversion.
Answer:.A.
History.of.many.hospital.visits.in.a.patient.with.cancer.(in.this.case.pancreatic.cancer).and.the.
patient.is.claiming.that.he.has.recurrence.would.let.us.think.that.he/she.is.seeking.narcotics..Based.
on.this.information.the.answer.would.be.A...“This.is.my.personal.explanation”..
..
Patient.consciously.fakes,.profoundly.exaggerates.or.claims.to.have.a.disorder.in.order.to.attain.a.
specific.2ndary.(external).gain.(e.g..avoiding.work,.obtaining.compensation)..Poor.compliance.with.
treatment.or.followKup.of.diagnostic.tests..Complaints.cease.after.gain.(vs..factitious.disorder)..
..
Factitious:.patients.with.factitious.disorder.intentionally.falsify.medical.or.psychological.signs.and.
symptoms.in.order.to.assume.the.role.of.sick.patient..
Malingering:.Intentional.reporting.physical.or.psychological.symptoms.in.order.to.achieve.personal.
gain....Reference:.FA.,.FA.psychiatry.
..
patient#with#irregular#menses#BMI#was#16#she#think#that#she#is#fat#and#want#to#lose#
weight#what#the#diagnosis?#
K.bulimia.nervosa.
K.anorexia.nervosa.
Answer:.B.
Reference:.FA.psychiatry.
..
18#yrs#old#female,#e#hx#of#amenorrhea#for#6months,#low#BMI,#but#she#thinks#that#she#is#
fat#and#has#to#lose#weight.#OE#(#increased#hair#distribution#in#her#body#plus#other#
findings) What#is#the#diagnosis?#
anorexia.nervosa.
“repeated”.
Reference:.FA.psychiatry.
..
what#is#best#to#describe#post#traumatic#syndrome?#
A).disorientation.and.dissociation.
B).Flashback.and.arousal.
Answer:B.
Reference:.FA.psychiatry.
..
outcome#of#45#y/o#pt#with#recurrent#schizophrenia#on#maintenance#therapy?#
A).complete.remission.
B).70%.will.have.good.quality.of.life..
C).one.third.of.patients.will.have.good.prognosis.
..
Answer:.C.
Even.with.medication.40K60%.of.patients.remain.significantly.impaired.after.their.diagnosis,.while.
only.20K30%.function.fairly.well.in.the.society..About.20%.of.patients.with.schizophrenia.attempt.
suicide..
..
Reference:.FA.psychiatry.+.BMJ/schizophrenia.prognosis.
..
Child#with#rapid#blinking#of#the#eye#communicate#with#his#parents#during#it#what#is#the#
diagnosis#all#are#normal?#
A)TIC.
B).petit.mal.seizure.
C).tourette.syndrome..
Answer:.A.
Reference.:.FA.psychiatry.
..
4#years#old#have#decrease#in#head#growth#,#has#weird#hand#movement#(#wiringing)#,#lost#
expressive#end#receptive#language#skills#,#lost#his#interest#in#his#social#environment#
.#what#is#the#diagnosis?#
K.Autism.
K.Mental.retardation.
K.Rett.syndrome.
Answer:C.
Reference:.FA.psychiatry.
..
..
4y/o#child#doesn’t#speak#except#mama#papa#response#will#to#verbal#command#his#older#
brother#had#the#same#problem#when#he#was#same#age#he#is#shy#what#is#the#diagnosis:#
A)Autsim.
B).development.language.disorder.
Answer:.B.
..
Reference:.
https://www.uptodate.com/contents/expressiveKlanguageKdelayKlateKtalkingKinKyoungKchildren.
..
http://www.childspeech.net/u_iv_e.html.
..
Cancer#associated#with#depression:#
A).pancreas.
B).prostate.
C).kidney..
D).Liver..
.Answer:.A.
Reference:.FA.psychiatry.p.37.
https://www.ncbi.nlm.nih.gov/pmc/articles/PMC4356432/.
..
What#is#the#percentage#of#a#child#to#have#schizophrenia#if#his#father#has#the#disease?#
A.! 5%,.
B.! .10%,.
C.! .15%,.
D.! .20%.
Answer:.B.
10%#FA#step1,#12%#in#FA#psychiatry,#10%#in#http://schizophrenia.com/family/FAQoffspring.htm#
.the#average#risk#for#first#degree#relatives#of#a#person#with#schizophrenia#is#9%.(#The#Encyclopedia#
of#Schizophrenia#and#Other#Psychotic#Disorders).#
##
A#female#patient#was#hearing#voices#coming#out#of#refrigerator#and#microwave#telling#
the#food#is#poisoned#what#does#she#has?.(.auditory.hallucinations).
Reference:.Toronto.notes..
..
best#and#rapid#management#for#specific#anxiety?#
.A..Benzodiazepines.
.B..Sertraline.
.C..Imipramine.
.D..Bupropion.
Answer:.A.
Reference:.FA.psychiatry.
..
Best#drug#for#short#term#Mx#of#GAD#without#causing#dependence#or#addiction;#
1/alprazolam.
2/sertraline.
3/fluoxetine.
4/Bupropion.
..
Answer:.D.(.I.think.it.is.a.misspelling.and.it.should.buspirone..).
BUSPIRONE.—.The.azapirone.buspirone.has.been.shown.in.clinical.trials.to.reduce.symptoms.of.
anxiety.in.patients.with.generalized.anxiety.disorder.(GAD),.offering.similar.efficacy.to.
benzodiazepines.without.the.risk.of.dependence..Buspirone.is.thought.to.affect.the.serotonergic.
system.via.blockade.of.5HT1A.autoreceptors..Buspirone.can.be.used.as.monotherapy.(in.the.
absence.of.comorbid.major.depression).or.for.augmentation.at.doses.of.10.to.60.mg/day.
Reference:.Uptodate..
..
Married#man#feeling#anxiety,#irritability,#weight#loss#and#trouble#sleeping.#He#has#no#
marital#problems#and#no#external#cause#for#his#anxiety.#What#is#the#most#likely#
diagnosis?#
GAD.
MDD.
Schizophrenia.
Answer:.A.
Reference:.Toronto.notes.
..
Newly#Married#couple#came#to#psychiatry#clinic.#The#wife#is#complaining#that#her#
husband#goes#back#home#multiple#times#to#check#if#they#locked#the#door#and..#(other#OCD#
symptoms).#The#husband#does#not#deny.#Diagnosis?#
OCD.
Answer:.A.
.Reference:.Toronto.notes.
..
Man#with#Premature#ejaculation.#What#to#give?#
SSRI.
Reference:.Toronto.notes.
..
Anxiety#medication#that#does#not#cause#dependence?#
KOne.of.the.benzodiazepines.
KAn.SSRI.
KBuspirone.
Answer:.C.
Reference:.
.BUSPIRONE.—.The.azapirone.buspirone.has.been.shown.in.clinical.trials.to.reduce.symptoms.of.
anxiety.in.patients.with.generalized.anxiety.disorder.(GAD),.offering.similar.efficacy.to.
benzodiazepines.without.the.risk.of.dependence..Buspirone.is.thought.to.affect.the.serotonergic.
system.via.blockade.of.5HT1A.autoreceptors..Buspirone.can.be.used.as.monotherapy.(in.the.
absence.of.comorbid.major.depression).or.for.augmentation.at.doses.of.10.to.60.mg/day.
Reference:.Uptodate..
Man#with#symptoms#of#sadness,#grief,#wife#died#two#months#ago.#Dx?#
KMDD.
KBereavement.
Answer:.B.
.Reference:.Toronto.Notes.
..
Treatment#of#OCD?#
Increase..sertonine.availability.
Decrease.sertonine.availability.
Increase.sertonine.secretion.
Decrease.sertonine..secretion.
Answer:.A.
Reference:.FA.psychiatry.
..
Elderly#take#haloperidol,#and#came#to#ER#afebrile#rolled#eye.#what's#the#Dx?#
Neuroleptic.syndrome.
Dyskinesia.
Antonia.
..
Answer:.The.correct.answer.is.dystonia.
Reference:.Toronto.notes.
..
Clozapine#used#for#which#childhood#psychiatric#disorder?#
aK.Schizophrenia.
bK.Bipolar.
cK.Mania.
.Answer:.A.
..
Reference:.https://www.ncbi.nlm.nih.gov/pubmed/18220495.
http://www.jaacap.com/article/S0890K8567(09)61687K7/abstract.
..
Female#diagnosed#with#depression#on#medication#and#has#constipation#Which#of#the#
following#is#the#drug#used#
aK.TCA.
bK.SSRI.
cK.SSNRI.
Answer:.A.
Reference:.FA.psychiatry.
..
what's#the#first#line#treatment#of#depression?#
SSRI.
..
Reference:.Toronto.notes.
..
What#is#the#first#line#in#the#management#of#depression#in#children?#
AK.Fluoxetine.
BK.Imipramine.
CK.MOAI.
Ans:.A.
..
Reference:.Toronto.notes.
..
58#years#old#man#is#complained#of#tiredness#and#fatigue,#and#after#his#wife#died#with#MI#when#she#
was#67#years#old,#his#symptom#increased#and#new#symptoms#are#(insomnia,#there#is#death#
thoughts#and#lack#of#concentration#in#work#and#loss#of#interest),#no#
loss#of#appetite.#Your#diagnosis#?#
Kdysthermia..
Kmajor.depressive.disorder..
Kbereavement..
Answer:.B.
5.symptoms.(including.loss.of.interest.“anhedonia”).
MDD.DSMK5:.
Must.have.at.least.five.of.the.following.symptoms.(must.include.either.number.1.or.2).for.at.least.a.
2Kweek.period:.
1K.....Depressed.mood.of.the.time.
2K.....Anhedonia.(loss.of.interest.in.pleasurable.activities).
3K.....Change.in.appetite.or.weight.(increase.or.decrease).
4K.....Feelings.of.worthless.or.excessive.guilt.
5K.....Insomnia.or.hypersomnia.
6K.....Diminished.concentration..
7K.....Psychomotor.agitation.or.retardation.(i.e.restlessness.or.slowness.).
8K.....Fatigue.or.loss.of.energy.
9K.....Recurrent.thoughts.of.death.or.suicide.
..
Reference:.FA.Psychiatry. .
..
Drug#that#maintain#treatment#of#bipolar#disorder#?#
Lithium.
SSRI.
Answer:.A.
Reference:.Toronto.notes.
..
..
..
..
Patient#when#presenting#a#topic#in#a#meeting#started#to#have#palpitations,#anxiety,#she##avoids#to#
give#presentations#so#not#to#experience#the#previous#feeling?#
Social.phobia.
PostKtraumatic.stress.disorder.
Specific.phobia.
Panic.disorder.
Answer:.A.
Reference:.FA.psychiatry.
..
Best#treatment#for#hallucinations#is?#
antipsychotics..
Reference:.Toronto.notes.
..
Kid#having#problem#with#eye#contact#and#has#weird#behiavor?#
Autistic.disorder.
Autism.spectrum.disorder.is.characterized.by.impairments.in.social.communication/interaction.and.
restrictive.behaviors/interests..Problems.with.social.interaction.and.communication.like.(inability.to.
hold.conversation.and.decreased.eye.contacts)..Restricted.repetitive.patterns.of.behavior,.interest.
and.activities.like.(.preoccupation.with.unusual.objects.,.rigid.thought.patterns,.hand.flapping..etc).
Reference:.FA.psychiatry.
..
Psychiatric#drug#cause#dystonia#?#
Olanzapin.
Clozaoin.
Respiridone.
..
Answer:.If.the.missing.choice.is.1st.generation.antipsychotic.then.it.is.the.right.answer..
Between.these.choices,.C.is.the.most.appropriate.answer..
Reference:.Manual.of.Basic.psychiatry..
..
#Patient#complaining#of#multiple#symptoms#for#2#yrs#examination#and#labs#normal#?#
A).Somatization.
B).Conversion.disorder.
C).Ocd.Schizophrenia.
..
Answer:.A.
Reference:.FA.psychiatry.
..
Pt#came#to#ER#he#said#(his#neighbor#want#to#kill#him)#and#he#hearing#his#neighbor#voice,#what#is#dx?#
Schizophrenia.
Reference:.Toronto.notes.
..
Parents#of#child#noticed#that#he's#sitting#alone,#not#developing#well,#playing#with#his#toy,#and#have#
repetitive#behavior#what's#ur#dx:#
AK.Global.developmental.delay.
BK.Autism.
Ans:.B.
Reference:..FA.psychiatry.
##
What#is#the#treatment#of#generalized#anxiety#disorder?#
1Ktricyclice.antidepressants.
2Kbenzodiazepines.(used.in.acute.attack).
3Kssri.
4Kbuspirone.
Answer:.C.
Reference:.FA.psychiatry.
..
Patient.with.dysmenorrhea,.breast.tenderness.and.mood.symptoms..what's.the.treatment?.
Progesterone.
SSRI.
Reference:.MTB2.
..
Patient#his#family#member#died#from#3#weeks#he#has#hallucinations#and#acting#lost.#
what's#the#Dx?#
schizophrenia .
brief.psychotic.disorder.
schizoaffective.disorder.
Answer:.B.
Brief.psychotic.disorder:.
Patient.with.psychotic.symptoms.as.in.schizophrenia,.however.the.symptoms.last.up.to.1.month..It.
may.be.seen.in.reactions.to.extreme.stress.such.as.bereavement,.sexual.assault…..etc..
..
DSMK5.schizoaffective.disorder:.
KMeet.the.criteria.for.either.Major.depressive.or.manic.episode.which.psychotic.symptoms.
consistent.with.schizophrenia.are.also.met..
K.delusions.or.hallucinations.for.2.weeks.in.the.absence.of.mood.disorder.symptoms.(this.criterion.is.
necessary.to.differentiate.schizoaffective.disorder.from.mood.disorder.with.psychotic.symptoms)..
K.mood.symptoms.present.during.the.majority.of.psychotic.illness..
..
Reference:.psychiatry.FA.
..
Obseesive#complussive#disorder,#what#is#the#mechanism#of#action#of#the#drug#used#to#treat#it?#
Serotonin.reuptake.inhibitor/increase.serotonin.availability.
Reference:.Psychiatry.FA..
..
A#female#pt#had#been#isolating#herself,#and#thinks#people#are#chasing#her,plus#she#avoids#eye#
contact#?#
.Psychosis/.schizophrenia.
Reference:.Toronto.notes.
..
Pt#visits#psychiatrist#and#the#do#noticed#depressive#symptoms,#while#taking#hx,#she#was#
not#responding,#then#stopped#visiting?#
There#is#another#form#of#the#question:#patient#came#with#symptoms#of#depression,#when#doctor#
asks#him#questions#about#psychosis,#the#patient#left#the#hospital,#what#does#that#mean?#
A).Acting.out.
B).intellectual.
C).sublimation.
D).interjection.
Answer:.A.
Acting.out:.Expressing.unacceptable.feelings.and.thoughts.through.actions..
Reference:.FA.
..
Your#friend#avoids#you,#what#type#of#personality?..
Answer:.Avoidant.
Avoidant.personality:.Patient.with.avoidant.personality.disorder.have.a.pervasive.pattern.of.social.
inhibition.and.an.intense.fear.of.rejection..They.will.avoid.situations.in.which.they.may.be.rejected..
Their.fear.of.rejection.is.so.overwhelming.that.it.affects.all.aspects.of.their.lives..They.avoid.social.
interactions.and.seek.jobs.in.which.there.is.little.interpersonal.contact..
Reference:.FA.psychiatry.
..
A#5#yr#old#boy#who#is#shy#and#can#only#say#mama,#baba,#his#older#brother#was#like#him?#
A)Autsim.
B).development.language.disorder.
Answer:B.
Reference:.
https://www.uptodate.com/contents/expressiveKlanguageKdelayKlateKtalkingKinKyoungKchildren.
..
http://www.childspeech.net/u_iv_e.html.
##
Most#common#risk#factor#in#schizophrenia#?#
both.parents.have.it.
..
I.could.not.find.a.source.about.the.most.common.risk.factor.of.schizophrenia.and.I.think.that.the.
question.was.asking.about.the.most.significant.risk.factor.instead.of.most.common,.in.which.the.
answer.will.be.Monozygotic.twins.or.if.the.both.parents.have.schizophrenia.
KGenetic:.40%.concordance.in.monozygotic.(MZ).twins;.46%.if.both.parents.have.
schizophrenia;.10%.of.dizygotic.(DZ).twins,.siblings.....Reference:.Toronto.notes.
..
Man#with#erectile#dysfunction,#but#in#morning,#normal#erectile#fuction,#where#to#refer#?#
cardio,.
neuro,.
psychological.
Answer:.C.
..
Reference:.Toronto.notes.
..
Lady#with#low#mood,#insomnia,#poor#appetite#for#5#months.#Her#symptoms#get#worse#during#
the#premenstrual#period.#The#symptoms#do#NOT#resolve#after#menstruation.#What#is#the#
management?#
AKReassure.
BKGive.fluoxetine.for.premenstrual.syndrome.
CKGive.antidepressant.for.postmenstrual.syndrome.
DKRefer.for.psychological.evaluation.
Answer:.Mostly.D.by.exclusion..Definitely.not.reassurance..the.symptoms.usually.resolve.after.
menstruation.in.PMS.and.there.is.nothing.called.postmenstrual.syndrome..
..
Most#potency#antipsychotic#drug?#
Haloperidol.
I’m.not.sure.if.it.is.the.most.potent.or.not..But.1st.generation.antipsychotics.are.classified.based.on.
their.potency:.High.potency.antipsychotics.include.haloperidol.,.fluphenazine.,.trifluoperazine.and.
pimozide.... Reference:.FA.psychiatry.
..
Patient#complain#of#chest#pain,#vertebral#pain,#a#lot#of#complain#with#pain#also#has#past#history#of#
complain#of#pain#and#nausea#and#vomiting#with#irregular#menses#weakness#in#left#leg.#Malingering#
and#factious#ruled#out,#diagnosis?#
A.....Pain.disorder.
B.....Somatization.disorders.
Answer:.B.
Pain.disorder.removed.from.DSMK5..
Reference:.FA.psychiatry.
##
Patient#had#social#phobia,#avoid#gathering,#on#work#had#palpitation#chest#pain#before#
meeting#since#5#years#treatment?#
A......Sartoline.
B......Buporin.
C......Azla.(.benzo).
D.....Beta.blocker.
Answer:.Sertraline.
Reference:.FA.psychiatry.
..
Women#is#anxious#,#irritable#,#insomina#for#2#months?#
A......Major.depression.
B......GAD.
C......Social.phobia.
Answer:.B.
..
Erectile#dysfunction?.Options>.multiple.drugs.including.Mertazepine.
Answer:.Mertazepine.does.not.cause.erectile.dysfunction.,.choose.SSRI.instead..
..
Cancer#patient#on#SSRI,#anti#cancer#drug,#.#Feverish.#Agitated#but#alert.#(+Mydriasis?)#
and#splenomegaly#on#examination?#
A......1.Neuroleptic.malignant.syndrome.
B......2.Serotonin.syndrome.
C......3.Side.efffect.of.anti.cancer.drug.
Answer:.B.
Reference:.FA.psychiatry.
..
Which#of#the#following#has#a#poor#prognosis#for#schizophrenia?#
A......acute.onset.
B......adolescents.age.onset.
C......family.history.
D.....panic.attack.during.psychosis.
Answer:.C.
Reference:.FA.psychiatry.
..
A#patient#with#chronic#back#pain#for#which#she's#taking#NSAID#for#more#than#1#year.#No#history#of#
trauma#and#her#sphincter#function#is#normal.#She's#also#complaining#of#mild#epigastric#discomfort.#
She's#concerned#about#her#financial#status,#What#would#you#give#her?#
1).escitalopram.
2).amitryptaline.
3).gabapentin.
Answer:.B.
Reference:.BMJ/Back.pain.
..
A#patient#came#with#aggressive#hostile#and#violent#behavior.#What#are#the#neurotransmitters#
responsible#for#his#behavior?#
1).high.serotonin.
2).low.serotonin.
3).high.endorphins.
4).low.endorphins.
Answer:.2.
A.deficit.in.serotonin.has.been.theorized.to.have.a.primary.role.in.causing.impulsivity.and.
aggression..
Reference:.https://en.wikipedia.org/wiki/Aggression.
..
.
.
.
.
.
.
.
.
.
.
.
.
.
.
.
.
.
.
.
.
.
.
.
.
.
.
.
.
.
.
.
.
.
.
Basic.Science..
..
1......Child#with#lactic#acidosis#+#hyperammonemia#?#
a).. Kpyruvate.dehydrogenase.deficiency..
b).. Kpyruvate.carpoxylase.deficiency..
ANSWER:#A#
REFERENCES:.https://www.ncbi.nlm.nih.gov/pubmed/4050860.
..
Å.......Child#with#high#lactic#acid?#
Answer:.Pyruvate.dehydrogenase.deficiency.
https://emedicine.medscape.com/article/948360Koverview.
..
.●.Scenario#of#a#child#with#hyperammonemia#
and#lactic#acidosis.#Which#enzyme#deficiency#
results#in#this#disorder?#
1......Pyruvate.dehydrogenase.
2......Pyruvate.carboxylase.
Answer:.A.
.
..
#
#
#
#
#
#
#
#
#
#
#
#
#
#
#
2......Dizygotic#twins?#
a).....Dichorionic.diamniotic.regardless.to.the.sex.
b).....Dichorionic.monamniotic.same.sex..
ANSWER:.A.dizygotic.twin.pregnancy.always.results.in.dichorionic.K.diamniotic.pregnancies...
REFERENCES:.https://radiopaedia.org/articles/dizygoticKtwinKpregnancyK1.
..
3......Oxybutynin#mechanism#of#action?#
a).....antagonist.muscrinic.
b).. agonist.nicotinic..
ANSWER:##A.#.anticholinergic.K..antimuscarinic.“.M1,M2,M3”.antagonist..
REFERENCES:#https://en.wikipedia.org/wiki/Oxybutynin#
..
4......Bisphosphonate#drugs#mechanisms,#and#then#asked#which#one#?#
.ANSWER:.Bone.stabilizer—bisphosphonate;.pyrophosphate.analog;.reduces.hydroxyapatite.crystal.
formation,.growth,.and.dissolution,.which.reduces.bone.turnover.
REFERENCES:.step.up.to.step.1..
..
5......Which#one#of#the#choice#secrets#gulcocorticod#hormones?#
AKReticularis.
BKGlomerulosa.
CKMedulla.
DKFaciculata.
..
ANSWER:#D..Glucocorticoids.are.chiefly.produced.in.the.zona.fasciculata.of.the.adrenal.cortex,.
whereas.mineralocorticoids.are.synthesized.in.the.zona.glomerulosa..
REFERENCES:.https://en.wikipedia.org/wiki/Glucocorticoid.
..
6......case#scenario#about#patient#with#hypertention#,#labs#result#hypernatremia#hypokalemia#
and#asking#about#cell#responsible#for#that?#
a).....Reticularis.
b).....Glamerulosa.
ANSWER:#B.
REFERENCES:#https://en.wikipedia.org/wiki/Mineralocorticoid.
..
..
7......Patient#complain#of#abdominal#pain#I#think#with#diarrhea#,#also#his#wife#noticed#he#had#
SOB#and#tightness.#Doctor#order#5[hydroxyindoleacetic#acid#in#urine#Which#cell#
responsible?#
A.chromaffin.cell.
B.Enterocell.
C.Lympho.cell.
D.Goblet.cell.
ANSWER:#A.#[carcinoid.syndrome.].
#5KHIAA.is.the.main.metabolite.of.serotonin.and.is.frequently.produced.by.carcinoid.tumors.of.the.
enterochromaffin.cells.of.the.small.intestine..
REFERENCES:#https://en.wikipedia.org/wiki/5KHydroxyindoleacetic_acid.
..
8......UTI#case#with#gram#negative#indole#positive:#
A).....E..coli.
.
.
.
.
9.#female#on#anti#dyslipidemia#drug#present#with#flushing#what#is#the#drug#:#
A..Niacin.
B..Atorvastatin.
Answer:#A#
References:#https://www.healthline.com/nutrition/niacinKflush#section2.
##
##
10.#what#is#the#MOA#of#glipizide#?#
A..Increase.insulin.secretions.from.pancreas.
..
Answer:#
Sulfonylureas.likely.bind.to.ATPKsensitive.potassiumKchannel.receptors.on.the.pancreatic.cell.
surface,.reducing.potassium.conductance.and.causing.depolarization.of.the.membrane..
Depolarization.stimulates.calcium.ion.influx.through.voltageKsensitive.calcium.channels,.raising.
intracellular.concentrations.of.calcium.ions,.which.induces.the.secretion,.or.exocytosis,.of.insulin..
References:.https://www.drugbank.ca/drugs/DB01067..
..
11.when#the#body#exposed#to#ionized#radiation#what#the#effect#of#radiation#on#DNA?#
a).....break.DNA.strains.
b).....affecting.DNA.pyrimidine.
.Answer:.
..
..
.13.#which#of#the#following#is#synthesized#in#nucleolus?#
a).....rRNA.
b).....mRNA.
c).....tRNA.
Answer:#rRNA#
References:.https://www.ncbi.nlm.nih.gov/pubmed/18712681.
..
14.#what’s#the#duration#of#Nitrofurantoin#regimen#in#recurrent#UTI?#
Answer:#.If.unrelated.to.sexual.intercourse,.consider.a.sixKmonth.course.of.lowKdose.nitrofurantoin..
References:.https://patient.info/doctor/recurrentKurinaryKtractKinfection.
..
15.#which#diuretic#can#cause#renal#stone?#
##
..

. .
.
..
16.lip#swelling#for#3#years#deficiency#in#which#enzyme?#
a).....C1.esterase.inhibitor.
b).....Acute.urticaria..
c).....Ascher.syndrome..
##
Answer:#.C1.esterase.inhibitor.deficiency..
References:#https://emedicine.medscape.com/article/1048887Kdifferential#1.
..
17.old#man#with#abnormal#hand#and#head#movement#he#toke#drug#1#day#before#what#is#the#drug:#
Answer:#Metoclopramide#
References:.https://www.drugs.com/metoclopramide.html.
..
18.pt#with#neck#rigidity#and#abdominal#muscle#regidity#(clear#extraparamidal#
manifestations),#which#drug#can#cause#these#symptoms?#
##
Answer:#Metoclopramide;.extrapyramidal.syndrome.(EPS)/acute.dystonic.reactions.(at.least.25%)..
References:.https://www.drugs.com/sfx/metoclopramideKsideKeffects.html.
..
19.#which#drug#if#you#stopped#abruptly#it#will#cause#hypertensive#crisis?#
Answer:.Sympatholytic.antihypertensive.are.commonly.known.to.cause.rebound.HTN;.beta.blockers,.
alpha.blockers,.Clonidine,.labetalol..
References:.http://www.ld99.com/reference/old/text/2878909K814.html.
..
20.Which#cell#produce#melanocyte#releasing#hormone??#
Answer:.Melanocyte.stimulating.hormones.is.secreted.from.the.pars.intermedia.from.the.
melanotrophs.cells.which’s.a.type.of.chromophobes.cells..
..
..
22.which#one#of#the#following#is#found#in#the#nucleus?#
.mRNA.in.nucleus.
.
23.Which#of#the#following#made#in##nucleolus#?#
.rRNA.in.Nulcleolus..
.
#.for.Q22K23;.no.options.were.provided.but.check.the.table.for.
relevant.
Information.related.two.both.structures..
..
..
..
..
..
Q.#bond#between#r[RNA#&#m[RNA#?#
##
Q.#what#is#the#bond#between#mRNA#and#tRNA?#
.Hydrogen.bond.
..
24[what#is#the#longest#phase#in#the#cellular#division?#
a).....Anaphase.
b).....Prophase.
c).....Interphase..
Answer:#Interphase.The.G1.phase,.S.phase,.and.G2.phase.are.
collectively.known.as.interphase..Cells.spend.90.to.95.percent.of.the.
time.in.interphase,.where.DNA.is.synthesized.and.the.cell.doubles.in.
mass.before.mitosis.begins..
Reference:.https://www.thoughtco.com/understandingKtheKcellK
cycleK373391.
..
25[source#of#energy#that#transfer#molecules#in#and#out#nucleus?#
a).....GTP.in.cytoplasm.
b).....ATP.cytoplasmic..
c).....GTP.nuclear..
..
Answer:#ATP#Cytoplasmic#..Molecule.movement.requires.energy.in.order.to.move.solute.against.its.
electrochemical.gradient..There.are.two.ways.in.which.a.flow.of.energy.can.be.coupled.to.
transporters:.Primary.active.transport.requires.energy.is.provided.by.cytoplasmic.atpase.and.
Sodium,.potassium—atlases.(Na,.K—atpase).is.present.in.plasma.membranes.
References:.http://www.biologyKonline.org/9/3_movement_molecules.htm.
..
26.on#OR#the#doctor#cut#vagus#nerve#by#mistake#while#doing#NISSIEN#fundoplication#
operation#for#ttt#of#chronic#GERD,#what#organ#will#be#affected#from#cutting#the#nerve?#
AKesophagus,.
BKfundus,.
CKurinary.bladder..
DKcolon..
Answer:#A.#
..
27.Postroir#vagal#trunk#supplies?#
.Answer:.posterior.vagal.trunk.are.branches.of.the.posterior.vagal.trunk.which.supply.the.stomach..
References:.https://en.wikipedia.org/wiki/Posterior_gastric_branches_of_posterior_vagal_trunk.
..
28.what#is#the#organ#affected#if#you#ligate#anterior#iliac#artery?#
a).....ovary.
b).....intestine,.
c).....bladder..
Answer:#bladder#necrosis;..It.has.been.reported.that.there.exist.some.complications.after.IIAL.such.
as.urinary.bladder.necrosis,.peripheral.nerve.ischaemia.or.gluteal.compartment.syndrome..
References:.https://www.ncbi.nlm.nih.gov/pmc/articles/PMC4105656/.
..
29.#Which#OCP#causes#hyperkalemia?########################### ######## #
a).....EstradiolKlevonorgestrel......................................... ....... .
b).....EstradiolKdrosperirenone.
..
Answer:.EstradiolKdrosperirenone.;.is.an.oral.contraceptive.(OC).that.possesses.unique.
antimineralocorticoid.activity..
References:.https://www.ncbi.nlm.nih.gov/pmc/articles/PMC3265420/.
..
..
31.difference#between#Hb#and#HbA1C#?#
Glycosylation.of.something.
##
32.which#type#of#cells#will#be#high#in#viral#infection?#
Lymphocyte.
.

.
..
..
..
..
33.Which#muscle#would#be#completely#paralyzed#by#obturator#nerve#injury?#
A......Gluteus.maximus.
B......adductor.magnus.
C......adductor.longus.
..
Answer:#adductor#longus.;.obturator.nerve.motor.supply.is.gracilis,.adductor.longus.and.brevis..
References:.https://www.gpnotebook.co.uk/simplepage.cfm?ID=x20120114173923202976.
..
36.action#of#metformin#on#muscles?#
a).....Decrease.glucose.uptake,.
b).....increase.glocuneogenesis,.
c).....increase.release.of.glucose,.
d).....increase.acid.oxidation.
..
Answer:.increase.acid.oxidation.
;.the.MOA.of.metoformin.is.not.fully.understood.yet.but.in.vivo.studies.have.showed.that.
Extracellular.accumulation.of.the.drug.in.the.muscles.is.responsible.for.its.hypoglycaemic.affect..
suppresses.lipid.accumulation.in.skeletal.muscle.by.promoting.fatty.acid.oxidation..
References:#
.https://www.ncbi.nlm.nih.gov/pubmed/7988785..
https://en.wikipedia.org/wiki/Metformin#Mechanism_of_action..
https://www.ncbi.nlm.nih.gov/pubmed/25016691..
..
..
40.Patient#has#hemorrhage,#baroreceptor#activated#which#lead#to#increase#heart#rate,#what#
of#those#will#be#decrease?#
a).....Ventricular.contractility.
b).....Venous.capacity.
c).....Coronary.blood.flow.
Answer:#Venous#capacity.#
References:.
https://www.google.com.sa/url?sa=t&rct=j&q=&esrc=s&source=web&cd=1&cad=rja&uact=8&ved=
0ahUKEwi09cfWhJvXAhVCExoKHUxJBK
MQFggtMAA&url=http%3A%2F%2Fmediwikis.com%2Fwiki%2Findex.php%2FBaroreceptor_Reflex_a
nd_Response_to_Haemorrhage&usg=AOvVaw0Rxp86Y_cyoZsJzQjV2CTs.
..
41.#RTA#victim,#hypotensive,#tachycardic,#tachypnic,#activation#of#baroreceptors#will#
result#in#which#of#the#following?#
##
.

.
..
References.:http//:advan.physiology.org/content/40/3/283.
..
42.How#hyperglycaemia#affect#wound#healing?#
a......Decrease.immune.system..
b......Dec.phagocytosis..
c......Stimulates.bacteria.growth..
Answer:.C..
References:.http://www.ncbi.nlm.nih.gov/pubmed/16006275.
..
43.Where#can#you#palpate#dorsalis#pedis#artery?#
Answer:.lateral.to.the.extensor.hallucis.
longus..
..

. .
..
..
45.S&S#of#anxiety#and#ask#about#deficiency?#
The.answer.was.Serotonin....
##
#

#
#.
..
46.Non[steroidal#and#Anti[inflammatory?#
.Answer:.Ibuprofen.
..
47.pudendal#nerve#block#which#area#is#not#affected?#
Answer:#Pudendal.nerve.block:.blocks.sensory.and.motor.innervation.to.the.Clitoris.“.dorsal.nerve”,.
skin.of.perineum.[labia.major/minora.and.vestibule].“.perineal.branch”,.External.anal.sphincter.and.
perianal.skin.“.via.inf..Hemorrhoidal.N.”.
References:.https://emedicine.medscape.com/article/83078Koverview#a1.
..
49.which#nerve#is#responsible#for#sensation#(not#taste)#of#anterior#two#
thirds#of#the#tongue?#
a).....5.
b).....7.
c).....9.
d).....12.
Answer:.Trigeminal.nerve..
References:.
..
.
50.Loss#of#anterior#2/3#tongue#taste?#
Answer:.Facial.nerve.
..
51.Loss#of#posterior#1/3#tongue#sensation#what#is#the#nerve?#
Answer:.Glossopharyngeal.
..
53.disease#of#spine#got#anterior#chest#abscess#which#nerve#carry#the#
infection:#
AK.lateral.cutaneus.
BK.anterior.cutaneus.
CK.posterior.cutaneus.
DK.diaphragmatic.
Answer:#Anterior#cutaneous..Tuberculous.exudates.from.the.spine.generally.follow.the.course.of.
anterior.cutaneous.or.lateral.cutaneous.branch.of.the.intercostal.nerve.to.produce.cold.abscess.in.
the.parasternal.line.or.midaxillary.line,.respectively..
References:.http://www.atmph.org/article.asp?issn=1755K
6783;year=2012;volume=5;issue=2;spage=142;epage=144;aulast=Das.
..
54.in#patient#post#mastoctomy#they#do#for#her#reconstrtion#from#the#rectus#muscle#what#is#
the#vesles#that#they#should#take#with#it?#
a).....1.superior.epegastric.artery.
b).....2.inferior.epigastric.artery.
#Answer:##Inferior#epigastric#artery.#
References:.https://www.ncbi.nlm.nih.gov/pmc/articles/PMC3015273/.
..
55.which#of#the#following#when#it#doesn't#undergo#Heterogeneity#it#causes#gastric#cancer?#
Answer:#E#cadherine###gene.;.loss.of.function.of.EKcadherin.(CDH1).has.been.linked.with.diffuse.
gastric.cancer.susceptibility..
References:.https://www.ncbi.nlm.nih.gov/pubmed/14630673............... .
..
56.which#of#the#following#has#a#Side#effect#of#causing#angioedema?#
.
..
.

.
..
..
57.#which#is#responsible#for#apoptosis?#
Answer:#P53#pathway.#
.

.
..
58.Protein#enter#in#proteasome#by#what#?#
AK.N.terminal.single.
BK.N.terminal.double.
CK.S.terminal.single.
DK.S.terminal.double.or.C.
..
Answer:#N#terminals#single.#
..
59.How#does#the#protein#enter#the#peroxisomes?#
a).....Folded.with.the.help.of.CKterminal.
b).....Unfolded.with.the.help.of.TKterminal.
c).....Folded.with.the.help.of.TKterminal.
d).....Unfolded.with.the.help.of.CKterminal.
Answer:#D.#
..
60.which#decrease#the#risk#of#some#cancers?#
a).....Vit.D.
b).....Fibers.................... .
c).....Salt.
Answer:.Vitamin.D..
..
61.scenario#of#pt.#who#his#freind#pushed#him#from#behind,#what#is#the#ligament#that#prevent#
overextension#of#hip?#
Answer;#Iliofemoral.ligament.
..

. .
.62.How#ectopic#pregnancy#occurs#at#the#cellular#level?#
A...early.Disappearance.of.zona.pellucida..
B...Fertilization.at.ampulla.tube..
C...Persistence.of.Zona.pellucida..
D...Fast.division.of.blastomere.
Answer:#A.#
References:.https://www.ncbi.nlm.nih.gov/pmc/articles/PMC3276879/.
..
63.which#Muscle#is#passing#through#the#lesser#sciatic#foramen?#
Answer:.Obturator.internus.muscle..
..

. .
..
..
64.CSF#is#circulating#in#which#space?#
Answer:.Subarachnoid.space..
..
65.which#vitamin#help#in#absorbtion#of#non#heme#iron?#
Answer:.Vitamin.C..
..
65.pt#with#vomiting#and#hyperventilation.#what#do#expect#to#find#in#him?#
Answer:.vomiting.causes.metabolic.alkalosis,.hyperventilation.causes.respiratory.alkalosis.4.
..
66.#What#is#usually#given#with#analgesics#to#reduce#side#effects?#
Answer:.Metoclopramide.
..
67.What#moves#the#molecules#in#cell#in#and#out#of#the#nucleous?#
a..ATP.cytoplasmic.
b..ATP.nuclear.
c..GTP.cytoplasmic.
d..GTP.nuclear.
Answer:#A#
##
68.about#action#or#Side#effects#of#atropine?#
Answer:#These.include.palpitation,.dilated.pupils,.difficulty.in.swallowing,.hot.dry.skin,.thirst,.
dizziness,.restlessness,.tremor,.fatigue.and.ataxia..
References:##https://www.rxlist.com/atropineKdrug.htm####################################################### #
..
69.#what's#the#mechanism#of#insulin#resistance#in#obesity?#
Answer:#.Mitochondrial.Over.activation..

. .
References:.https://www.ncbi.nlm.nih.gov/pmc/articles/PMC3936017/.
..
70.Insulin#resistance#in#type#2#DM,#how#does#it#occur?#
Answer:.Evidence.is.provided.showing.that.increased.plasma.FFA.levels.produce.insulin.resistance.
dosage.dependently,.acutely,.and.chronically..
References:.https://www.ncbi.nlm.nih.gov/pubmed/11727400.
..
..
71.what#forms#the#ankle#joint?#
Answer:..Tibia,.Fibula.and.Talus.bones...

. .
..
72.#the#blood#in#the#superficial#palmar#arch#is#low.#the#problem#in#which#artery?#
Answer:.Ulnar.Artery..
..
73.pt#can't#move#her#hand.#where#is#the#problem?#
A).....ant..interossi.
B).....or.post..interossi.
C).....or.theaner.
..
Answer:.the.patient.is.presenting.with.Radial.nerve.injury.and.the.post.interosseous#nerve.supplies.
the.post.compartment.of.the.forearm”.extension”.
..
74.diabetic#pt#with#pain#when#she#tries#to#stretch#2nd#and#
3rd#finger#(and#i#think#she#
can't#move#her#hand).#there#was#pic#in#the#exam.#what's#the#
Daignosis?#
A).....diabetic.dactylitis.
B).....dupturen.contracture.
..
Answer:.Diabetic.Dactylitis..
.
75.pt#had#trauma#then#can't#flex#the#DIP.#where#is#the#lesion?#
ANSWER:#.Flexor.digitorm.profundus.injury..“.in.PIP.
immobility.the.FD.superficilias.is.injuried”.
..
76.Action#of#Anterior#compartment#of#the#forearm#muscle#?#
Answer:.In.general,.muscles.in.the.anterior.compartment.of.the.forearm.perform.flexion.at.the.wrist.
and.fingers,.and.pronation..
References:.http://teachmeanatomy.info/upperKlimb/muscles/anteriorKforearm/.
..
77.#pic#of#humerus#bone#fracture#with#weakness#in#wrist#extension,#which#nerve#is#injured?#
Radial.nerve.injury..
..

. .
..
..
..
78.pt#can't#extend#wrist#and#finger#,#nerve#injured?#
Answer:#Radial#nerve.#
..
79.Patientwith#trauma,#femoral/hip#fracture.#Left#leg#short#and#laterally#rotated.#What#muscle#is#
responsible#for#lateral#rotation:#
AKgluteus.maximus.
BKrectus.femorus.
.CKgracilus.
DKadductor.magnus.
Answer:#a.The.lateral.rotators.are:.the.superior.gemellus,.inferior.gemellus,.obturator.externus,.
obturator.internus,.quadratus.femoris,.gluteus.maximus.and.the.piriformis..References:.
Http://radiopaedia.org/articles/glutealKmuscles.
..
##
80.pt#has#BP#of#160#over#something.#what's#the#cause?#
A).....change.in.paroreceptor.sensitivity.
B).....increase.peripheral.vascular.resistance.
Answer:.B.#
#
#
.
..
81.pt#in#shock.#what#will#happen?#
a).....decrease.coronary.perfusion.
b).....decrease.in.venous.capacitance.
.Answer:.insufficient.information..

. .
..
..
83.#pt#with#erectile#dysfunction#and#taking#a#medication#for#that.#which#of#the#following#
is#contracindicated#for#this#patient?#
Answer:.nitrate.
..
84.pt#with#hyperthyroidism#came#with#red#eye.#which#medication#can#cause#this?#
Answer:#Propylthiouracil.#
References:#https://www.drugs.com/sfx/propylthiouracilKsideKeffects.html.
..
85.Blood#supply#to##AV##and##SA##node#in##the#heart#?#
Answer:.Right.coronary.artery,.
##
. .

.86.Hinge#joint?#
..
. .
..
.
87.Most#common#SE#of#atropine?#
Most.common.side.effects.from.Rxlist.com..
●! dry.mouth,.
●! blurred.vision,.
●! sensitivity.to.light,.
●! lack.of.sweating,.
●! dizziness,.
●! nausea,.
●! loss.of.balance,.
●! hypersensitivity.reactions.(such.as.skin.rash),.and.
●! rapid.heartbeat.(tachycardia)..
..
88.#Inhaled#antiviral#works#by?#
..
..
. .
..
..
..
89.Bacteria#sexual#behaviour?#
Answer:.Bacterial.conjugation..
..
90.Ribavirin#SE?#
A).anemia.
B).renal.damage.
C).Hepatic.damage..
..
Answer:#Haemolytic#anaemia;.haemolytic.anemia,.decreased.hemoglobin,.insomnia,.dyspnea,.lack.
of.concentration,.emotional.lability,.and.irritability...
..
91.#Side#effects#of#Anti[TB.#
K.Anti.TB,.causing.seizure:.isoniazid.
K.Anti.TB,.causing.parasthesia#or#nerve#problem:.isoniazid.Rx:#pyrodixine.#
.K.Anti.TB,.causing.visual#problem:.ethambutol.
K.Anti.TB,.causing.vertigo#or#ear#problem:.streptomycin.
K.Anti.TB,.causing.red#urine:.rifampicin.
..
92.#Thalidomide#SE#in#pregnancy?#
Answer:.limb.defect..
References:..http://onlinelibrary.wiley.com/doi/10.1002/bies.200900103/pdf.
..
93.Anti#thyroid#causing#pancytopenia#(Agrnanulocytosis)?#
Answer:.Propylthiouracil.and.methimazole.both.can.produce.thrombocytopenia..
References:.https://www.ncbi.nlm.nih.gov/pubmed/22049174.
..
94.Anti#thyroid#used#in#pregnancy?#
Answer:.Both.methimazole.(MMI).and.propylthiouracil.(PTU).may.be.used.during.pregnancy;.
however,.PTU.is.preferred.in.the.first.trimester.and.should.be.replaced.by.MMI.after.this.trimester..
References:.https://www.ncbi.nlm.nih.gov/pubmed/21389085.
..
95.Drug#contraindicated#in#glaucoma?#
Antihistamines..H2.receptor.agonists(cimetidine),.adrenergic.agonists..Epinephrine,.ipratropium.
bromide..
References:.https://www.reviewofoptometry.com/ce/medsKthatKdontKmixKwithKglaucomaKpatients.
..
96..Drug#for#HTN#reduces#heart#rate#and#peripheral#resistance?#
Answer:#Carvediolol;...is.a.betaKadrenergic.receptor.blocking.ability.decreases.the.heart.rate,.
myocardial.contractility,.and.myocardial.oxygen.demand..Carvedilol.also.decreases.systemic.vascular.
resistance.via.its.alphaKadrenergic.receptor.blocking.properties..
..
97.Drug#for#HTN#reduced#preload#and#cause#vasodilatation?#
Answer:#ACE’I,#
References:.http://cvpharmacology.com/vasodilator/ACE.
..
98.drug#associated#with#Hyperpigmentation?#
..
.Answer:.amiodarone..
..
.

.
.
99.#Orlistat#mechanism#of#action?#
.Answer:.Orlistat.works.by.inhibiting#gastric#and#pancreatic#lipases,.the.enzymes.that.break.down.
triglycerides.in.the.intestine..When.lipase.activity.is.blocked,.triglycerides.from.the.diet.are.not.
hydrolyzed.into.absorbable.free.fatty.acids,.and.instead.are.excreted.unchanged..Recently.it.was.
found.to.inhibit.the.thioesterase.domain.of.fatty.acid.synthase..
References:.https://en.wikipedia.org/wiki/Orlistat#Mechanism_of_action.
..
..
100.Aspirin#SE?#
.Answers:.frequently.reported.side.effects.include:.dyspepsia,.epigastric.distress,.heartburn,.and.
nausea..
References:.https://www.drugs.com/sfx/aspirinKsideKeffects.html.
..
..
KHALID.ALOMAR.
..
K.Aspirin#in#kids?#(fulminant#hepatitis#and#cerebral#edema)?#
Answer:.Reye.syndrome.
..
[#Post#menopausal#hormonal#therapy#SE?#
Answer:.breast.cancer.
..
[#Bisphosphonate#inhibits#osteoclast#activity?#
Answer:.alendronate.
Reference:.https://en.wikipedia.org/wiki/Alendronic_acid.
..
K.Bisphosphonate#works#by#ADP?#
Answer:.clodronate,.etidronate.and.tiludronate.
Reference:..https://en.wikipedia.org/wiki/Bisphosphonate#Mechanism_of_action.
..
[#Med#for#osteoprosis#causes#heartburn?#
Answer:.risedronate.
Reference:..http://reference.medscape.com/drug/actonelKrisedronateK342835#4.
..
[Drug#for#asthma#decreases#secretion#more#than#bronchodilatation?#
.Answer:.ipratropium.bromide.
.Reference:.https://en.wikipedia.org/wiki/Ipratropium_bromide.
##
[#Hg#F#inducer?#
aK.hydroxyurea,.
bK.Na.butyrate,.
cK.decitibine.
dK.5Kazacytidine.
.Answer:..A.
..
drug#that#cause#vertical#and#horizontal#nystagmus?#
Answer:.phencyclidine.
Reference:..https://www.uptodate.com/contents/phencyclidineKpcpKintoxicationKinadults?.
source=search_result&search=Phencyclidine&selectedTitle=1~62.
..
Perinoeal#artery?#branch.from.post..Tibial.artery,.a.branch.from.popliteal.artery.
.Answer:.??.
Perineal.artery.is.a.branch.from.internal.pudendal.artery.which.is.a.branch.from.internal.iliac.artery..
..
[##facial#veins#(via#the#superior#and#inferior#ophthalmic#veins)#as#well#as#the#
sphenoid#and#middle#cerebral#veins.#They,#in#turn,#empty#into#the#inferior#petrosal#
sinuses,#then#into#the#internal#jugular#veins#and#the#sigmoid#sinuses#via#the#superior#
petrosal#sinuses#
.Answer:.??.
..
K.Radial#pulse:.
Answer:.lateral.to.flexor.carpi.radialis.
..
K.Fingers#adduction:.
Answer:.deep.ulner.nerve.
..
K.Fibger#abduction:.
Answer:.deep.radial.nerve.
..
K.Fingers#adductors:#
Answer:.palmar.interosseous.
..
K.finger#abductors:.
Answer:.dorsal.interosseous.
..
[#Blood#supply#of#hand:#
Answer:.superficial.arch;.ulnar.mainly.+.radial..
............. Deep.arch;.radial.mainly.+.ulnar.
..
[#Pain#during#hand#typing:.decrease.blood.in.ulnar.artery."carpal.tunnel.syndrome;.
median.Nerve".
Answer:.median.nerve.
..
[#Nail#fold#area#supplied.by:.
answer:.median.N.
..
K.Blood#of#distal#palmar#aspect:.
Answer:.proper.palmar.digital.artery,.from.common.palmar.
digital.artery..A.
..
K.Pronator#teres#syndrome:.
Answer:.median.N.
..
K.Can't#flex#distal#phalanx:#
Answer:.rupture.profundus.tendon.
..
[#Pain#with#rising#arm#and#numbness#in#ulnar#distribution:#
Answer:.thoracic.outlet.syndrome.
..
[#Falling#on#outstretched#hand:.
Answer:.colle's.fracture.
..
K.Hip#and#knee#flexion:#
Answer:.sartorius.muscle.
..
[#Gluteal#artery.branch#from#
Answer:.internal.iliac.
..
K.Obturator#nerve:.
Answer:.Adductor.longus.
supplies.all.adductors;.adductor.longus,.brevis,.gracilis....etc.+.
sensation.in.medial.aspect.of.thigh.
..
K.Internal#iliac#artery:.
Answer:.Bladder?..
gives.the.inferior.and.superior.gluteal.artery.to.gluteus.
muscles,.and.obturator.artery.to.adductor.groups..Otherwise,.other.muscles.in.the.lower.
limb.from.the.femoral.artery.
..
K.Strongest#hip#ligament:#
Answer:.iliofemoral.
..
K.Patient#after#MVA,#walking#bending#to#unaffected#side#(positive#trendelburg#test):#
Answer:.injury.to.sup..Gluteal.nerve.which.supplies.gluteus.medius.and.minimums.
..
[#Post.#Compartment#of#leg#BV:#
Answer:.posterior.tibial.artery.
..
K.Ant.#Compartment#of#leg#BV:.
Answer:.anterior.tibial.artery..
..
K.Posterior#leg#compartment:.
Answer:.tibial.N.
..
K.Anterior#leg#compartment:.
Answer:.common.peroneal.which.gives.deep.perpneal.
..
K.Foot#Dorsal#flexion:.
Answer:.tibial.muscle.
..
[#Foot#Plantar#flexion#"standing#on#toe":#
Answer:.plantaris.
..
[#Patient#can't#planter#flex#his#foot#after#injury:#
Answer:.calcaneal.tendon.rupture."achilles.
tendon".
..
[#Football#player#cant'#extend#big#toe:#
Answwer:.deep.muscle.under.foot.(turf.toe).
..
[#Fibular#head#fracture:#
Answer:.common.peroneal.nerve;.drop.of.foot.
..
[#First#thing#to#be#injured#in#palmar#foot:#
Answer:.abductor.hallucis.longus.M.
..
K.Flat#foot:#
problem.with.tibialis.posterior.tendon.
..
K.Tongue#muscle:.originates.from.occipital.
Answer:.
Mandible,.
hyoid.bone,.
styloid.process.
of.temporal.
bone..
..
.
.
..
#

#
#
#
#
[#3rd#layer#of#skull:#
Answer:.aponeurosis.
Reference:.Medscape..
..
[#Compression#on#maxillary#artery:#
Answer:.sphenopalatine.artery.affected.
..
K.Thyroid#fascia:.
Answer:.pretrachial.fascia,.part.from.deep.cervical.fascia.
..
[#Accessory#nerve:#
Answer:.in.post..Triangle.
..
[#LP#puncture:#
Answer:..injury.to.ligamentum.flavum.
..
[#triangle#between#hyoid#bone#and#two#ant.#Belly#of#digastric:#
Answer:.submental.
..
[#External#laryngeal#nerve:#
supply.cricothyroid.muscle.KK&gt;.adduct.cord.to.phone.the.
voice.
Answer:.supply.cricothyroid.muscle.
..
[#Loss#of#smell#sensation;.
Answer:.temporal.lobe.
..
K.Facial#N#injury#
Answer:.proximal.to.greater.petrosal.nerve.
(Q.from.SMLE13:.Loss.of.taste.sensation.of.the.anterior.2/3.of.the.one.side.of.the.tongue.and.
decreased.hearing.on.the.ear.on.the.same.side).
..
K.Loss#of#sensation#in:#lower#eyelid,#maxilla,#zygomatic#areas.KK&gt;.maxillary.N.
injury.which.give.inferior.orbital.N.
Answer:.inferior.orbital.N.
..
K.Foramen#between#3rd#and#4th#ventricle:.
Answer:.aqueduct.of.sylvius.
..
K.Upper#and#lower#lips#LN#drainage:.
submandibular.
Answer:.submental.and.submandibular.

. .
.
K.LN#of#middle#of#face:.
Answer:.submandibular.
..
.
.
.
.
.
.
.
.
K.LN#of#lateral#side,#includes#lateral#side#of#eyelid#and#eyebrow:.
Answer:.parotid.gland.
..
K.Tip#of#the#tongue#and#center#of#the#lips:.
Answer:.submental.
Reference:.http://teachmeanatomy.info/neck/vessels/lymphatics/.
..
K.Follicular#cell#of#ovary:.
Answer:.cortical.cord.
References:.Reference:.Langman’s.Medical.Embryology.
https://en.wikipedia.org/wiki/Sex_cords.
..
[#Phrenic#nerve#injury:#
increase.heart.rate.
Answer:.phrenic.nerve.injury.causes.hiccups.problem.and.difficulty.breathing..
..
[#Cardiac#notch:.
Answer:.4th.rib.
..
[#Injury#in#forth#intercostal#space:#
Answer:.horizontal.fissure.
..
K.Thoracocentesis:.
between.posterior.axillary.line.and.midline.;.between.7.and.9.intercostal.
Answer:.between.7th.and.9th.intercostal.space.and.posterior.and.mid.axillary.line..
Reference:.Medscape..
..
K.Needle#thoracostomy:.
Answer:.2nd.intercostal,.mid.axillary.line.
..
[#Chest#tube:#
Answer:.4th.intercostal,.anterior.axillary.line.
..
K.Adrenalectomy:.
Answer:.right.side.will.injure.IVC,.left.side.will.injure.the.pancreas.
..
K.Internal#iliac#artery#injury:#
Answer:.bladder.is.affected.
..
[#Best#x[ray#for#rib:#
Answer:.oblique.view.
..
[#Meckel's#diverticulum:#
Answer:.in.lower.ilium.
..
[#Crohn:#
Answer:.distal.ilium.
..
K.UC:#
Answer:.rectum.
..
[#Celiac:#
Answer:.proximal.small.intestine.
..
[#Appendicitis#histopathology:#
Answer:.neutrophils.in.muscularis.layer.
..
K.Tenia#coli:.
Answer:.in.muscularis.externa.layer.
..
[#H.#Pylori:#
Answer:.stimulate.gastrin.cell.
..
[#Intrinsic#factor#for#vit#B12:#
Answer:.from.parietal.cell.
..
K.in#pt#with#congested#neck#vein#and#tumor#(SVC.syndrome):.
in.middle.mediastinum.
Answer:.could.be.superior.or.middle.mediastinum..
..
[#Sugar#pentose:#
Answer:.arabinose.
..
[#Melanin#pigment:#
#corticotrope.
Answer:.Corticotrophs.mainly.secrete.melanocyteKstimulating.hormone..
Melanocytes.secrete.melanin..
..
[#Rhesus#autoimmunity#in#pregnancy:#
Answer:.hypersensitivity.type.2.
..
[#phosphoenolpyruvate#to#pyruvate#and#generation#of#ATP:#
Answer:..by.pyruvate.kinase.
..
[#From#pyruvate#to#lactic#acid:#
.Answer:.lactic.dehydrogenase.
..
K.Accumulation#of#lactic#acid:.
Answer:.pyruvate.carboxylase.deficiency.
..
[#Phenylketonuria:#
Answer:.phenylalanine.hydroxylase.deficiency.
..
[#Strongest#layer#of#vein:#
answer:.adventitia.
..
[#Juxtaglomerular#apparatus#origin:#
Answer:.macula.densa.cell..
..
[#Alveoli#part#contacting#blood:#
Answer:.saccular.phase..
..
[#Phase#of#alvuloendothelium#part#get#matured:#
Answer:.canalicular.phase.
..
[Amyotrophic#lateral#sclerosis:#
Answer:.anterior.Horn.
..
[#Minimal#change#glomerulonephritis:#
Answer:.Light.micro:.normal,.electron:.fusion.of.foot.
process.
..
[#Hepatic#cell#secrete#extra#matrix#component:#
Answer:.stellate.cell.
..
[#Ever#smoker:#
Answer:.100.sig.per.life.
..
[#Biopsychological#therapy:#
Answer:.Holistic..
..
[#Wild#mushroom:#
Answer:.inhibits.RNA.polymerase.II.
..
[#X#fragile#syndrome:#
Answer:.macrogonadism.
..
[#Comes#with#turner:#
Answer:.hypothyroidism.
..
[#Vit#C#def#affect#which#stage#of#wound#healing:#
Answer:.collagen.synthesis.or.proliferative.stage..
Reference:.http://www.surgerysupplements.com/theKroleKofKvitaminKcKinKwoundKhealing/.
..
[#of#embriology#i#cant#remember#(failure#of#spiral#)#??#
Answer:.Transposition.of.great.vessels.
..
Which#of#the#following#results#from#malrotation#of#septum,#truncus#arteriosus?#
Answer:.Transposition.of.great.vessels.
..
Protein#after#transfer#sequence#what#is#the#last#destination#??#
Answer:.endoplasmic.reticulum.
..
an#athlete#came#with#xanthametaous#lesion#on#his#Achilles'#tendon,#otherwise#normal,#
Genetic#defect#on#which#of#the#following#
Answer:.LDL.receptors. .
..
pt#heared#about#incretin#on#TV#and#wishes#to#know#more#about#its#action#
Answer:.increases.insulin.secretion.
..
a#pt#with#muscle#weakness,#apart#from#hypotension#and#bradycardia,#his#examination#was#
normal,#
His#lab#tests#(#high#K+,#low#NA,#low#Chloride,#high#urea)#
What#is#the#etiology#behind#his#presentation?#
aK.hyponatremia.
bK.hyperkalemia.
cK.low.chloride.
dK.uremia.
Answer:..b.
..
pt#after#a#surgery#in#the#posterior#triangle#of#neck,#developed#loss#of#sensation#on#
the#skin#of#lower#mandible#and#Lower#part#of#the#ear,#
Nerve#injured#is#
Answer:.Great.Auricular.nerve..
..
clear#case#of#Carpal#tunnel#syndrome,#the#pt#is#a#typist#working#on#computer,#nerve#
affected?#
Answer:.Median.nerve .
..
During#a#surgery#in#the#diaphragm,#
Phrenic#nerve#is#dissected#at#which#level#
Answer:.anterior.to.sclaneous.anterior.muscle.
..
non#athlete#forcefully#planter#flexed#his#foot#and#heard#a#click#sound#on#his#leg#,#
What#tendon#affected?#
Answer:.calceneal.tendon.
..
The#effect#of#anti#TB#drugs#on#the#eye#is#one#of#the#following:.
aK.....bacterial.conjunctivitis.
bK.....viral.conjunctivitis..
cK.....glaucoma..
dK.....uveitis.
Answer:.d.
..
an#RTA#victim#was#brought#to#hospital#with#multiple#injuries,#transferred#to#ICU.#
There,#he#developed#multi#organ#failure,#
The#phosphate#level#in#his#blood#is#raised,#what#is#the#cause#of#his#multi#organ#
failure?#
aK.heart.
bK.lungs.
cK.kidneys.
dK.liver.
Answer:.a.
..
the#transmission#of#maternal#antibodies#to#the#fetus#in#pregnancy#is#a#way#of#
aK.active.artificial.Immunity.
bK.passive.artificial.immunity.
cK.passive.natural.immunity.
dK.active.natural.immunity.
.Answer:.c.
..
clear#case#of#atopic#pt#e#allergic#rhinitis,#symptoms#and#signs#were#given,#then#asked#
about#the#cells#responsible#for#his#allergy#
aK.....monocyte.
bK.....macrophage.
cK.....mast.cells.
dK.....neutrophils.
Answer:.c.
..
lymphatic#drainage#of#teste#
Answer:.para.aortic.LN.
..
75.pt#had#trauma#then#can't#flex#the#DIP.#where#is#the#lesion?#
ANSWER:..Flexor.digitorm.profundus.injury..“.in.PIP.immobility.the.FD.superficilias.is.injuried”.
##
76.Action#of#Anterior#compartment#of#the#forearm#muscle#?#
Answer:.In.general,.muscles.in.the.anterior.compartment.of.the.forearm.perform.flexion.at.the.wrist.
and.fingers,.and.pronation..
References:.http://teachmeanatomy.info/upperKlimb/muscles/anteriorKforearm/.
##
77.#pic#of#humerus#bone#fracture#with#weakness#in#wrist#extension,#which#nerve#is#injured?#
Radial.nerve.injury..
#.
Q.#A#woman#e#Ca#cervix,#e#mets#beyond#uterus#and#cervix,#
1st.group.of.LN.receiving.the.lymphatic.drainage.is:#
aK.....uterine.
bK.....external.iliac.
cK.....common.iliac.
dK.....para.aortic.
Answer:.a.
(para.uterine.K>.obturator.K>.internal.iliac.K>.common.iliac.K>.external.iliac.K>.para.aortic).
References:..https://www.uptodate.com/contents/invasiveKcervicalKcancerKepidemiologyKriskK
factorsKclinicalKmanifestationsKandK
diagnosis?source=see_link&sectionName=ROUTES%20OF%20SPREAD&anchor=H1087317#H108731
7.

. .
.
K! Flouroquinolones#mode#of#action:#
Answer:.DNA.gyrase.inhibitors.
..
Typical#scenario#of#Acute#appendicitis,#(periumbilical#pain#radiating#to#RIF,#pt#has#
aneroxia,#nausea#and#fever,#what#is#the#pathophysiology#?#
Answer:.Appendiceal.obstruction.
..
Side#effect#of#morphine#is#
Answer:.nausea.and.vomiting.
..
What#is#the#most#common#part#of#urethra#could#be#injured#during#catheterization#?#
aK.....Membranous.urethra.
bK.....Prostatic.urethra.
cK.....Penile.urethra.
Answer:.a.
..
Baby#,#during#delivery#,#there#was#stylomastoid#foramen#trauma#,#what#features#will#be#
during#exams:#
aK.loss.of.eye.close.
bK.Loss.of.facial.sensation.
cK.Loss.of.mastication.function.
.Answer:.a.

. .
Q.#Which#of#the#following#integrates#glucose#and#fatty#acid#metabolism#?#
aK.pyruvate.
bK.Citrate.
cK.Carboxylase.
.Answer:.??.acetylKCoA..
References:.https://en.wikipedia.org/wiki/AcetylKCoA.
https://www.docsity.com/en/answers/biochemistry/whichKfollowingKmetaboliteKgratesKglucoseK
fattyKmetabolism/193124/.
..
patient#with#CAD#,#prescribe#for#her#drug#lower#lipid#.#Present#with#facial#flush#,#
what#is#the#drug#?#
aK.statin.
bK.Nicain.
cK.Fibrate.
Answer:.b.
Reference:..https://en.wikipedia.org/wiki/Niacin.
..
pt#on#antithyroid#meds,#then#developed#fever.#What#does#he#use?#
Answer:.Methimazol.
..
How#to#exclude#fictitious#fever?#
a.KBlood.culture.
b.KUrine.analysis.
c.KHeart.rate.
d.KRheumatoid.factor.
.Answer:.c.
..
pt#with#Carpopedal#spasm?#
Answer:.low.ca.
..
Good#healing?#
Answer:.epithelium.covering.
..
KMetformin#MOA?#
Answer:..decrease.liver.glucose.production.
..
[PTU#MOA?#
.Answer:.block.release.of.thyroid.hormones.
..
Drop#foot#&#loss#of#sensation#in#1st#&#2nd#digit.#which#nerve#is#responsible?#
Answer:.Deep.pernoneal.nerve.
..
pt#with#End#Stage#Liver#Disease#and#yeast.#Which#antifungal#you#should#use?#
aK.....fluconazole.
bK.....capsufungin.
.Answer:.b.
Reference:.http://www.medscape.com/viewarticle/775172_4.
..
Overlflow#incontinance#Rx#Moa?#
Answer:.activate.muscarinic.receptors.(muscarinic.agonist.).
Reference:..https://en.wikipedia.org/wiki/Overflow_incontinence.
..
which#one#will#protect#aganst#dental.carries?.
Answer:.Fluride.supplmenets.
..
which#is#true#about#Parkinson#case?#
Answer:.low.dopamine.
..
which#is#true#about#case#of#Parkinson#
Answer:.problem.in.substantia.nigra.
..
..
how#Ocp#prevents#pregnancy?#
aK.....Relase.estrogen.that.prevent.ovulation.
cK.....relase.prolactin.that.prevent.ovulation.
dK.....Stop.spermatozoa.and.i.increase.thickness.of.cervical.mucosa.
Answer:.a.
Reference:.https://en.wikipedia.org/wiki/Combined_oral_contraceptive_pill.
..
Hormone#replacement#therapy?#
Answer:.increase.risk.of.endometrial.ca.
..
Pt#got#numbness#and#electrical#like#pain#of#the#left#foot#up#to#knee#,#so#at#which#
level#is#the#disc?#
Answer:.L3KL4.
. .
..
Gout#meds#MOA:#
Answer:.
KAllopurinol.+.febuxostat.=.xanthine.oxidase.inhibitor.
Kpegloticase.=.recom..uricase.that.catalyzes.uric.acid.to.a.more.water.sol..Matter.
Kprobenecid:.inhibits.reabsorption.in.pct,.but.can.lead.to.uric.acid.stones.
Kcolchicine.=.microtubule.polymerization.inhibitor.
..
MVA#injury#to#jugular#Forman#which#muscle#will#be#still#functioning#?#
aKsternoclecomastiod.
bKsternoglossal.
cKsternpphygenas.
dKtrapazius.
.Answer:.b.Styloglossus.
..
Q.#You#gave#a#patient#a#drug#and#ask#him#to#come#back#to#check#PT#and#INR#what#was#the#drug#?.
aKAsprin.
bKWarfarin.
cKenxoparin.
dKUF.heparin.
Answer:.b.
..
MOA#of#emergency#contraceptives?#
aK.....delay.fertilization.
bK.....delay.implantation.
.Answer:.a.
Reference:.https://en.wikipedia.org/wiki/Emergency_contraception.
..
[#Which#one#of#the#following#GIT#cell#secret#defensins:#
aKparatial.cell.
bKchief.cell.
cKpaneth.cell.
dKplasma.cell.
Answer:.c.
Reference:..https://www.ncbi.nlm.nih.gov/pmc/articles/PMC4073591/.
..
type#of#wrist#joint?#
aKPIVOT.
bKHINGE.
Answer:..ellipsoid.(condyloid)..
..
BRUNNER's#gland#location?#
Answer:.submucosal.duodenum..
Reference:..https://en.wikipedia.org/wiki/Brunner%27s_glands.
..
anti#diabetic#medication#working#through#activation#enzyme#AMP#protein#kinase?#
AKglipizide.(sulfonylurea).
BKtolbutamide.
CKpioglitazone.
DKbiguanide.or.metformin.
Answer:.d.
Reference:.https://en.wikipedia.org/wiki/Metformin.
..
loss#of#sensation#of#posterior#&#anterior#aspect#of#the#hand#;#which#never#is#affected?#
aK.radial.
bK.Medial.
cK.Ulnar.
dK.Axillary.
Answer:.d.
..
#Loss#sensation#of#the#little#&#ring#finger#which#nerve#is#affected?#
aK.radial.
bK.Medial.
cK.Ulnar.
cK.....Axillary.
Answer:.c.
..
Q#about#OCP#and#side#effects#of#dry#skin,#muscle#spasm#,#and#depression?#
aK.....hypothyroidism.
bK.....Hyperkalemia.
cK.....Hypocalcemia.
Answer:..a.
..
Q.#which#contraceptive#has#a#side#effect#of#increasing#facial#hair#and#causing#(i#guess#
acne?)?#
.Answer:.no.choices..
..
Co[receptor#of#HIV#?#
Answer:.CXCR4.
..
To#collect#blood#sample#we#need#how#much#blood#for#one#sample?#
aK.....3K5.ml.
bK.....8K.10.ml.
Answer:??.Differ.according.to.the.test.and.the.age.of.the.patient..
..
What’s#minimum#volume#of#blood#to#do#a#culture:#
10_20.ml.
Answer:.??.
5.ml.for.adult,.1.ml.for.for.pediatric..
References:.http://d2xk4h2me8pjt2.cloudfront.net/webjc/attachments/71/6e4b1cfKbloodKcultureK
collectionKandKvolumeKhuddleKtool.pdf..
https://www.slideshare.net/doctorrao/roleKofKspecimenKcollectionKinKmicrobiology.
..
vit#c#difeciency#which#affected#in#stages#of#wound#healing?#
Answer:.Collagen.synthesis.
.
case#scenario#regarding#patient#known#case#of#epilepsy,#he#take#anti[epileptic#drug#
for#long#time#and#now#he#complain#of#increased#hair#in#his#body#and#gum#bleeding,#what#
do#you#think#of#the#following#medication#he#is#on?#
Answer:.phenytoin.
..
CD40L#defect?#
Answer:.hyper.IgM.syndrome.
Reference:..https://en.wikipedia.org/wiki/Hyper_IgM_syndrome.
..
Blouting#and#gases#after#eating#ice#cream#and#dairy#product.#Enzyme#deficint?#
aK.....Lastase.
bK.....Pancreatic.alpha.amylase.
cK.....Sacrase.
dK.....Isomaltose.
Answer:.lactase.
..
Dideoxynucletide,#used#in#what#?#
aK.PCR.
bK.Western.blot.
cK.DNA.sequencing.
dKSouthern.blot.
.Answer:.c.
..
Which#one#of#the#following#follows#a#mendellian#inheritance?#
Answer:.Thalassemia.
..
What#will#increase#in#ancylostoma#duodenale#?#
Answer:.eosinophils.
Reference:..https://emedicine.medscape.com/article/218805Kworkup.
..
scenario#of#mother#how#had#a#previous#pregnancy#with#down#syndrome#and#I#think#it#asked#about#
why#or#how#it#happen?#
Answer:.nondisjunction.
..
loss#of#sensation#over#shoulder#which#part#of#pleura#will#be#affected?#
a.. ....... Mediastinal.
b.. ....... Pleural.
c.. ....... Vesceral.
Answer:.a.
..
Why#aspirin#causes#hypererythema#?#
Answer:.no.choices.
..
example#of#hinge#joint?#
Answer:.elbow.
..
example#of#Condyloid#joint?#
Answer:.wrist.
..
type#of#joint#between#vertbra?#
Answer:.Cartilaginous.joint.
..
which#layer#of#scalp#contain#bv#and#nerves?#
Answer:.no.choices...
2nd.layer,.connective.tissue=.superficial.fascia..
..
Case#scenario#with#Lactic#acidosis#and#Hyperalinen.#Multivitamin#B#was#given#with#no#
response:#
What.to.give:.
a..Thiamin.
b..Pyruvate.kinase.
c..Pyruvate.dehydrogenase.
d..Pyruvate.carboxylase.
Answer:.d.
..
Which#one#of#the#following#is#associated#with#malignancy?#
a..Streptococcus.bovis.
b..Staph.aureus.
c..Pseudomonas.
Answer:.A.
References:.https://www.hindawi.com/journals/ijmicro/2011/792019/.
..
Q.#Male#got#bee#sting#and#his#wife#is#searching#for#epinephrine#pin.#What#is#the#mechanism#of#
action#of#this#drug#(No#mast#cell#or#histamin#in#the#options):#
Answer:.no.choices..Cross.reactivity.with.the.cardiac?.
..
Q.#Scenario#of#a#patient#whose#thirsty#with#low#urine#output#because#of#ADH#secretion.#
(They#attached#a#picture#of#a#nephrons#with#number#labels#on#each#segment#and#asked#to#
choose#which#area#does#ADH#work#on).#
Answer.is.collecting.duct.(which.was.labeled.by.number.5.in.the.picture).

. .
..
..
What#is#the#first#immune#responsible#agent#in#the#skin?#
Answer:.Keratinocyte.
..
loss#of#adduction#of#the#fingers#which#nerve#injury:#
aK..ulner.
bK..Radial..
cK..Median.
dK.Axillary..
Answer:.a.
..
contracting#muscle#compared#to#relaxing#muscle:#
aK..more.pyruvate.converted.to.lactate..
bK.
Answer:.a.
..
which#enzyme#deficiency#will#result#in#converting#pyruvate#to#lactate:#
A).Pyruvate.kinase..
B).pyruvate.carboxylase..
C).Pyruvate.dehydrogenase.
Answer:.b.
..
the#maximum#accepted#level#of#LDL#in#mmol/l?#
Answer:..4.12.

. .
patient#can#not#extend#his#leg#at#knee#joint#which#muscle#responsible#?#
aK.....sarturius.mucsle.
bK.....quadricbles.
cK.....biscepts.femoris.
Answe:.b.
..
posterior#triangle#trauma,#pt#complain#of#hearing#loss#which#nerve#is#affected:#
aK.....greater.occipital.
bK.....lesser.occipital.
cK.....great.auricular.
Answer:.??.
..
OCP#work#by#increase:#
aKestrogen.,.
bKprogesterone,.
cKFSH.
dKLH.
Answer:.most.likely.b..
..
Inhibition#of#which#of#the#following#is#the#primary#action#of#oral#contraceptives?#
AKSpermatozoa.and.thickening.cervial.mucous.
BKImplantation.
CK.Gondatropic.hormone.release.and.ovulation.
Answer:.C.
..
pt#on#opioid,#contraindication#with#these#drug#is?#
aKNSAID.
bKaspirin.
cKlaxative.
dKcoxen....
Answer:.?.
..
Patient#with#dm,#which#one#of#the#following#is#contraindicated?#
aK.losartan.
bK.Nifedipine.
cK.Hydrazine.
Answer:.c?.
Reference:.https://en.wikipedia.org/wiki/Hydrazine_sulfate.
..
How#heparin#in#high#dose#cause#hyperthermia#
aK.inc.metabolism.
bK.uncupling.oxidation.phosphrylation..
cK.Increase.liver.metabolism.
dK.Increase.energy.
Answer:.b?.
..
contraindicated#hypoglycemic#drug#in#diabetic#patient#with#heart#disease?#
aK.pioglitazone.
bK.sitagliptin.
Answer:.a?.
Reference:.https://www.uptodate.com/contents/drugsKthatKshouldKbeKavoidedKorKusedKwithK
cautionKinKpatientsKwithKheartKfailure.
https://www.medscape.com/viewarticle/579822_2?pa=9SK0rtUK%2BCkGjM0fP6yZGm5ZplE3h%2Bx
vCZ7C5bPWTGIqqtp5ScqVT7b0XmUJeZrIs7CF3wx2Tu1U792SxywYLg%3D%3D.
https://www.ptcommunity.com/news/20170422/sitagliptinKjanuviaKlinkedKincreaseKheartKfailureK
hospitalizations.
..
case#scenario#about#patient#after#dental#extraction#c/o#of#loss#of#sensation#below#the#
eye#and#upper#lip#,#I#think#also#cheek#which#nerve#was#injured#?#
Answer:.infraorbital.
..
which#medication#decrease#the#effect#of#OCPs?#
Answer:.anti.epileptic.
..
case#scenario#about#patient#with#basal#skull#fracture#what#expected#nerve#injury?#
aK.Olfactory.
bK.optic.
CK.oculomotor.
Answer:.a.
..
type#of#hip#joint?#
Answer:.ball.and.socket.joint.
..
case#scenario#regarding#breast#mass#and#asking#about#which#lymph#node#must#be#assist#
first?#
aK.anterior.axillary.lymph.node.
bK.posterior.
cK.medial.
dK.lateral.
Answer:.a.
..
child#present#with#malaise,#history#of#meningitis#last#week#treated#with#iv#ab.#
Lab#:#hb:#low#
Rbc:#low#
What#antibiotic#he#used#?#
aK.chloramphinicol.
bK.Erythromycin.
Answer:.a.
..
Hepatotoxic#alzehaimer#drug?#
Answer:.Tacrine.
..
Differentiate#between#types#of#gram#positive?#with#catalase#
.Answer:?.
..
Forceps#delivery,#Truma#behind#ear,#What#happen#to#him?#
aKloss.sensation.
bKhearing.loss.
Answer:.?.facial.nerve.
..
Role#of#metformin#in#PCOS?#
Answer:.reduces.insulin.resistance..
..
pt#complain#recurrent#renal#stone#on#htn#medication#to#reduced#calcium#excretion#and#
cause#gout?#
aKHydrochlorithiazide.
bKfursemide.
Answer:.a.
..
main#lateral#rotator#muscle#of#hip?#
Answer:.Gluteus.maximus.
..
Patient#k/c#of#HF#&#schizophrenia#on#medication#,came#with#brownish#discoloration#in#
retina#&#brown#vision..#what's#the#drug#?#
Answer:.Thioridazine.
*.Chlorpromazine—(C)orneal.deposits;.
*.Thioridazine—re(T)inal.deposits.
..
..
Qs#about#alternative#cell#reservoir#for#HIV?#
aKCD8.
bKMonocyteKMacrophage.
Answer:.b.
Reference:..https://www.ncbi.nlm.nih.gov/pmc/articles/PMC4947377/.
..
Latency#period#in#HIV,#cell#responsible?#
Answer:.CD4.Cells.
Reference:..http://www.canfar.com/news/stagesKhivKinfection.
..
rRNA#in#which#cell#?#
aKBasophils.
bKMonocyte.
.Answer:.a?.
..
injury#in#the#sole#,1st#structure#injured?#
Answer:.Abdactor.hallusis.longus.
..
side#effect#of#amitryptaline?#
aKDystonia.
bKDiarrhea.
Answer:..a.
..
typical#scenario#of#mysthenia#Gravis#(fatigabality#and#weakness#worsen#at#the#end#of#
the#day,What#is#the#pathophysiology?#
aK.viral.
bKbacteria.
cKdrug.induced.
dKAutoimmune.
Answer:.d..
..
another#scenario#about#Mysthenia#graves,Part#affected?#
Answer:.Acetylcholine.receptors.
..
a#physician#wants#to#decrease#the#recurrence#of#UTI,#
Best#measure#is#?#
Answer:.(decrease.PH.,.increase.urea.and.increase.urine.osmolarity).
..
relation#of#saphinous#vien#to#mallelus?#
Answer:.anteromedial.
..
Function#of#anterior#compartment#of#forearm#muscles#?#
aK.flexion.of.elbow.
bK.extension.of.elbow.
cK.flexion.of.hand.and.wrest.
dK.extension.of.hand.and.wrest.
Answer:.c.
..
A#case#of#GERD#used#antacid#which#caused#constipation#
aKCalcium.carbonate.
bKAluminum.hydroxide.
cKMagnesium.hydroxide.
Answer:.b.
..
Patient#presented#to#er#with#agitation,#poor#hygiene#and#suspicions#er#dr#referred#him#
to#psychiatry#what's#the#dx#
aKAcute.confusional.state.
bKSchizophrenia.
cKPsychosis.
Answer:.b?.
..
Patient#with#increase#ICP#what#nerve#would#u#examine#before#the#CT#scan?#
aKII.
bKIII.
cKIV.
dKX.
Answer:.a.
..
Patient#on#TB#medication#developed#numbness,#what#would#u#give#him?#
Answer:.Vitamin.B6.(pyridoxine).
..
adult#male#had#an#accident#to#the#lateral#side#of#his#fibula,#next#day#the#patient#is#
complaining#of#drop#of#his#foot#and#no#inversion,#diagnosis?#
aK.deep.peronial.injury.
bK.common.peronial.injury.
Answer:.if.no.eversion.the.answer.is.b..
..
Which#one#of#the#following#drugs#causes#Vertigo?#
aKStreptomycin.
bKINH.
cKEthambutol.
Answer:.a.
..
Gout,#which#of#the#following#will#be#inhibited#by#the#drug#you#will#give?#
aKXanthine.oxidase.
bKPRPP.
Answer:.a.
..
Which#of#the#cells#is#responsible#for#tissue#destruction#in#IBD?#
a.B.lymphocytes.
b.T.lymphocytes.
c..NK.cells.
Answer:.b.
..
Virulence#factor#of#streptococcus#?#
aKMK.protein.
bKCapsule.
cKEndotoxin.or.exotoxin.
dKphosphatase.
Answer:.a..
Reference:..https://en.wikipedia.org/wiki/Virulence_factor.
..
What#is#the#mechanism#of#diarrhea?#
aKDecrease.fluid.intake.
bKDecrease.fluid.absorption.
cKIncrease.fluid.intake.
dKIncrease.fluid.absorption.
Answer:.b.
..
Embryo#origin#of#right#atrium#?.(no.options).
Answer:.Rt.sinus.veinous.
..
Strongest#part#of#vein.(no.options).
Answer:.Tunica.adventitia.
..
Relation#of#femiral#vein#to#artery.(no.options).
Answer:.Medial.
..
KNerve#injury#with#humerus.fracture.(no.options).
Answer:.Radial.N.
..
Ligament#pass#through#inguinal#canal#
aKRound.
bKBroad.
Answer:.a.
..
When#you#examine#superior#inguinal#LN#You#should#examine#?#
Answer:.anal.canal.
..
Enlarge#medial#group#of#horzintal#[inguinal#lymphnode#(#superficial).#What#you#will#
check#1st?#
Answer:.no.choices..
..
garlic#odor#organism?#
a).Anal.canal.
b).Muscle.of.thigh.
c).Muscle.of.leg.
Answer:.??.
..
a#man#after#MVA#,#can#not#raise#his#arm#above#horizontal#plane#and#has#winging#scapula#
.#Localise#the#injury#in#the#brachial#plexus?#
aK.roots.
bK.Posterior.cord.
cK.Lateral.plexus.
dK.Medial.
Answer:.a.
..
What#is#the#enzyme#used#glycolysis?#
aK.pyrovate.kinase.
bK.Pyrovate.carboxylate.
cK.Glucose.phosphatase.
Answer:..a.
..
Nerve#responsible#for#posterior#compartment#of#the#leg?#
a..Sciatic.nerve.
b..Tibial.nerve.
c..Fibular.nerve.
Answer:.b.
..
Loss#of#forehead#muscle#action#what#is#the#nerve?#
Answer:.Fascial.nerve.
..
Loss#of#gag#reflex#but#normal#uvula:#
a..glossopharyngeal.
b..Vegas.
Answer:.a.
..
Pt#had#occipitofrontalis#paralysis#which#branch#of#facial#nerve#is#affected?#
a..Temporal.
b..Buccal.
.Answer:.a.
..
Known#asthma#has#whitish#rash#easly#removed#in#mouth#.#which#antiasmatic#cuse#this?#
a).Cromolyn.sodium.
b).Betamethasone.
c).Albutmerol.
Answer:.b.
..
carpal#tunnel#syndrome#,#can’t#move#his#fingers#.#what#is#affected#muscle?#
aK...........palmar.interosssi.
bK...........thenar.muscle.
Answer:.b.
..
Closed#humerou#fracture#with#hand#drop#.#what#is#the#type#of#nerve#injury?#
aK.neuromatosis.
bK.neuropraxia.
cK.axonotmesis.
dK.avulsion.of.radial.nerve.
Answer:.b.
..
Eldery#has#bed#sore#that#invade#through#the#muscles#,#what#is#the#stage?#
A)1.
B)2.
C)3.
D)4.
Answer:.d.
..
Female#taking#paracetamol#500mg#bid#daily#presented#with#liver#enzymes#elevated#and#
hepatomegaly.*.
aK.alpha.1.antitrypsin.deficiency.
bK.liver.damage.due.to.paracetamol.
cK.hepatitis.
dK.Glutathione.depletion.
Answer:.d.
..
nerve#supply#of#calf#muscle?#
aK*.tibial.nerve.
bK*.Femoral.
Answer:.a.
..
Which#nerve#supplies#biggest#part#of#the#tongue#and#covers#it#the#most#?#
A.K.trigeminal.
B.K.vagus.
C.Kglossopharyngeal.
Answer.:.A.
..
which#of#these#components#is#acidophilic#
aKcorticotrophic.
bKtrophoyrophic.
cKlactotrophic.
dKgonadotrophic.
Answer:.c?.
..
Acidophil:.one.of.the.hormone.K.producing.acidophilic.cells.of.the.adenohypophysis;.type.include.
corticotrophic,.lactotrophic,.lipotrophs,.and.somatotrophs..also.called.alpha.cell.and.
A.cell..
Reference.:.Dorland's.medical.dictionary.
..
Patient#have#MVA#come#to#ER#with#ulceraiton#tissue#with#gas#gangrene#in#the#anterior#
leg#what#is#the#MO?#
aKC..Perfiringes.
cK.....staph.aures.
Answer:.a.
..
Superior#oblique#muscle#movement?#
aKMedially.down.
bKMedially.up.
Answer:.a.
..
child#pt.#came#with#scenario#of#chest#infection#,#first#day#of#admission#he#treated#
with#cefotaxime,#next#day,#pt#state#became#bad#with#decrease#perfusion#and#x[ray#show#
complete#rt.#Side#opcifaction#+#hydrothorax##,#causative#organism#:#
aKStrepto..Pnem.
bKStaph..Aureus.
cKHemophilus.influenza.type.b.
dKPseudomonas.
Answer:.d.
..
COPD#exacerbation#by#Infection,#patient#has#fever#and#greenish#sputum,#what#is#the#most#
likely#microorganism?.(.no.pseudomonas.in.the.answers).
aKstaph..Aureus.
bKstreptococcus.pneumonie.
cKmycoplasma.pneumonia.
dKhomophiles.influenza.
Answer:.d.
..
What#drug#is#likely#to#cause#heat[stroke#as#it#inhibits#sweating?#
Answer:.Hyoscamine.Sulfate.
..
foul#smelling#urine#in#a#7#y/o#child#with#fever#and#lower#abdominal#pain?#
aKE.Coli.
bKProtues.
cKKlebsiella.
Answer:.a?.
..
[#Gram#[ve,#lactose#non[ferminting#oxidase#+ve?#
Answer:.Psudomonas.
..
most#common#electrolyte#disturbance#in#a#patient#with#digoxin?#
Answer:.K.
..
patient#with#superficial#temporal#artery#bleeding,#blood#will#be#collected#in?#
Epicranial.aponeurosis.
Answer:.?.
..
Drug#which#decreases#HR#and#Pre#and#after#load?#
aKCarvidilol.
bKNifidepine.
cKAmlodepine.
Answer:.a?.
Reference:.https://en.wikipedia.org/wiki/Carvedilol.
..
[#Long#sinario#about#median#nerve#injury.#What#is#the#abnormality#you#expected#to#see#
in#hand#?#
A).ape.hand.
B).Clwe.hand.
C).Wrist.drop.
Answer:.a.
..
[#Injury#leads#to#loss#of#sensation#in#the#medial#2#fingers.#What#is#the#nerve#injured#
?.
A).Ulnar.nerve.
B).Median.nerve.
C).Brachial.nerve.
Answer:.a.
..
KWhat#is#biochemical#deficiency#in#respiratory#distress#syndrome#in#neonate!?(no.
surfactant.in.answers).
aK.shingomyelin.
bKPhosphate.
cKdodecylbenzenesulfonate.
Answer:.c.
Refrence:.https://en.wikipedia.org/wiki/Sodium_dodecylbenzenesulfonate.
..
History#of#URTI#then#developed#watery#eye?#
.Answer:.Viral.(adeno).
..
Weakness#in#gluteal#area#,#the#artery#affected#is#?#
aKFemoral.
bKIntrenal.iliac.
cKExternal.
.Answer:.b.
..
Old#pt#take#antiviral#that#taken#by#inhalation?#
Answer:.**Zanamivir.
..
Culture#showed#non#fermenting#gram#negative#bacilli#what#is.organism.(no.options).
***correct.answer.one.of.the.following.
1K...........Moraxilla.
2K...........Legionella.
3K...........Pseudomonas.
4K...........Bordetella.
..
which#of#these#joint#is#not#hinge#joint?#
a).Knee.joint.
b).Ankle.joint.
c).Elbow.joint.
d).Hip.joint.
Answer:.d.
..
[#Patient#in#ICU#had#onset#of#ventilator#associated#pneumonia#culture#showed#gram#
negative,#oxidase#positive#and#non#lactose#fermenting#organism#
AK.Pseudomonas.
BK.Ecoli.
CK.Klebsiella.
Ans:.a.
..
Patient#with#recurrent#uti#presented#with#renal#stones#whats#the#organism#
AK.Proteus.
BK.Ecoli.
CK.Klebsiella.
Answer:.A.
..
Long#scenario#about#patient#come#to#ER#with#chest#pain#radiat#to#left#arm#then#get#
coma#and#death#what#you#will#find#in#the#brain#?#
aK.necrosis.because.of.left.middle.artery.
bKred.nearon.degeneration.in.hippocampus.
Answer:.a??.
..
During#bypass#heart#surgery#the#assistance#asking#about#origin#of#right#coronary#
DOMINANT#artery#?#
aK.posterior.interseptal.
bKanterior.septal.
cK.marginal.
dK.cercumflex.
Answer:.a.
Refrence:.https://en.wikipedia.org/wiki/Right_coronary_artery.
..
old#patient#known#to#have#acquired#immunodeficiency#presented#with#cough#and#night#
sweat#,#he#did#mentoux#test#which#was#negative#but#culture#was#positive#for#
tuberculosis#what#is#most#likely:#
aK.culture.is.false.positive.
bK.mentoux.is.false.negative.
cK.mentoux.is.not.a.screening.test.for.TB.
dK.the.patient.should.be.screened.with.heaf.test..
Answer:.b.
..
What#is#the#pathology#in#Huntington#syndrome#?#
1Ksomething.related.to.stratum.whith.nerotransmitter.deffect.
Answer:.?.
Early.damage.is.most.evident.in.the.striatum,.
Reference:.https://en.wikipedia.org/wiki/Huntington%27s_disease.
..
Case#of#side#effect#of#ACEI?#
Answer:.cough.
..
You#did#DRE#and#found#prostate#gland#swelling#,#which#lobe#of#prostate#has#the#
neoplasm!?#
aK.anterior.
bK.Posterior.
cK.Medial.
Answer:.b.
..
food#poisoning#case,#4#family#members#ate#from#a#restaurant,#they#developed#diarrhea#
and#vomiting#and#remit#after#24#hrs#Culture#showed#gram#positive#bacilli?#
aKsalmonella.
bKshigella.
cK.Staph.Aureus.
dKBacillus.ceres.
Answer:.d.
..
where#surgeon#well#do#vagus#nerve#crush#?#
Answer:.
..
heart#duct#emperionic#origination?.(Specific.name.not.Mesoderm).
Answer:.
..
picture#of#ring#cell#stage#of#malaria.#Asked#the#stage#ang#malaria#type?#
Answer:.
..
picture#of#histopath#of#tumor#with#stary#sky#pattern?#
Answer:.EBV.
..
case#of#wegeners#but#asked#about#the#basic#pathology#of#disease?#
Answer:.
..
A#patient#ate#a#wild#mushroom.#Which#of#the#following#will#be#inhibited?#
aKRNA.polymerase.I.
bKRNA.polymerase.II.
cKRNA.polymerase.III.
dKDNA.Gyrase.
Answer:.b.
..
bipolar#disorder#develop#symptoms#of#hypothyroidism.#Which#medicine#responsible?#
.Answer:.no.choices..lithium.
..
what#organism#can#cause#paralysis#??#
Answer:.botolinium.
..
scenario#about#female#sexually#active#and#came#with#symptom#i#forgot#,#in#lab#results#he#
mentioned#gram#negative#diplococci#and#asked#about#the#diagnosis#??#
Answer:.N..Gonorrhea.
..
a#drug#which#stops#sweating?.(no.option.as.hydrosamine).
aKcimtidine.
bKodensteron.
cKand.two.more.which.were.ending.with.chloride.
Answer:.?.Aluminum.chloride..
..
man#taking#diuretics#have#muscle#weakness#diarrhea#irritability?#
aKhypokalemia.
bKhyperkalemia.
cKhyponatermia.
dKhypocalcemia.
Answer:.b.
..
Mother#with#Rh#[ve#and#a#father#with#Rh#+ve,#what#the#propility#of#having#Rh#+ve#child?#
aK25%.
bK50%.
cK75%.
dK100%.
Answer:.b.
..
Phyenytoin#side#effect?#
Answer:.Gum.hyperplasia.
..
Long#thoracic#nerve#root#number?#
Answer:.no.choces..C5,C6,C7.
..
During#fights,#what#system#is#activated?#
Answer:.Sympathetic.
..
A#female#with#immunity#disorder,#where#is#the#defect?#
aKILK2.
bKILK3.
cKILK6.
Answer:.?.
..
Nitrofuratonin#side#effect?#
Answer:.no.choices.
vaginal.itching.or.discharge.
Reference:.https://www.drugs.com/nitrofurantoin.html.
..
Overdosed#drug#caused#resp#depression?#
Answer:.
..
Overdosed#lead#to#comatose?#
Answer:.no.choices..Opioids?.
..
Overdosed#lead#to#dilated#pupils?#
Answer:.
..
A#truma#case#with#facial#fractures,#his#eyes#(#something)#what#type#of#reaction?#
Answer:.
..
Muscles#responsible#for#unlocking#the#knee?#
Answer:.Poplitus.
..
Mechanism#of#warfarin#?#
Answer:.no.choices..
..
Most#common#cutaneous#manifestation#of#antimalarial#medications?#
aKPruritus.
bKPigmenation.
cKPhotosensitivity.
dKGeneralized.yellow.discoloration.of.skin.
Answer:.b.
..
What#is#a#relative#contraindication#of#Methylergometrine?#
aKAsthma.
bKDM.
cKHTN.
.Answer:.c.
Reference:.https://en.wikipedia.org/wiki/Methylergometrine.
..
Difference#between#HbA1C#and#non[modified#hemoglobin:#
Answer:.glycosalation.of.valine.
Reference:.https://books.google.com.sa/books?id=AQ0vvBFg790C&pg=PA243&lpg=PA243&dq=nonK
modified+hemoglobin+to+HbA1C&source=bl&ots=C04hA8qNFv&sig=zHSFT97YYOrLB9IuigHseYjxRqU
&hl=ar&sa=X&ved=0ahUKEwip6uSF1arXAhWJMBoKHeEBDmoQ6AEIUzAJ#v=onepage&q=nonK
modified%20hemoglobin%20to%20HbA1C&f=false.
..
hook#worm,#how#to#cause#anemia?#
aKdestructin.rbc.
bKsend.toxins.to.bone.marrow.
cKcompete.with.host.on.b.12.
Answer:..a.
Reference:.https://en.wikipedia.org/wiki/Hookworm_infection.
..
patient,#with#diarrhea#,#flaglated#protozoa,#how#it#cause#diarrhea#?#
aKincrease.screation.of.fluids.
bKkills.normal.flora.
cKcoats.the.small.bowl.
dKabsorption.is.prevented.
Answer:.a.
Reference:.https://emedicine.medscape.com/article/176718Koverview#a3.
..
Lymph#node#of#the#forehead#?#
Answer:.no.choices..preauricular.
..
Cat#bite,#which#organism:#
Answer:.Pasteurella.multocida.
..
Chronic#smoker,#presented#with#signs#of#lung#cancer,#what#is#the#origin#of#cancer?#
aK..Clara.
bK..Brush.cells.
cK..Goblet.
.Answer:.a.
Reference:.https://www.ncbi.nlm.nih.gov/pmc/articles/PMC4860431/.
..
Patient#travelled#to#Africa#where#TB#bovine#is#endemic#there,#what#prevent#him#from#
receiving#BCG#vaccine,#deficiency#in#what:#
aK.IFN.gamma.
bK.IFN.alpha.
cK.IL.4.
dK.IFN.beta.
Answer:.a.
..
Most#common#carpal#bone#involved?#
Answer:..Scaphoid.
..
Female#presented#with#hand#joints#swelling,#she#were#informed#by#physician#that#she#has#
bone#loss,#she#is#angry#about#her#permenant#bone#loss#
What#is#the#explanation#of#bone#loss:#
aK.Increase.pressure.in.joint.space.
bK.Material.secreted.from.synovial.fluid.
cK.Drug.induced.menopause.
Answer:.
..
Which#drug#will#decrase#basline#and#contact#induced#acid#secretion#from#stomach:#
aK.Ranitidine.
bK.PPI.(not.sure).
K.Other.choices.that.I.don’t.remember..
Answer:.b?.
..
#Known#case#of#sarcoidosis,#presented#with#signs#of#anemia.#From#investigation#he#has#
iron#deficiency#anemia.#What#is#the#pathophysiology:#
aK.Decreased.hepcidin.
bK.Increased.hepcidin.
Answer:.b.
Increase.hepcidin.associated.with.IDA..
..
wernicke's#area#injured,#which#type#of#aphasia?#
Answer:.no.choices.
fluent.aphasia.
..
Female#150k.g.#Weight#and#height#160#according#to#BMI#?#
aKI.Obesity.
bKII.obesity.
cKIII.obesity.
.Answer:.c.
..
Patient#complain#of#abdominal#pain#I#think#with#diarrhea,#also#his#wife#noticed#he#had#
SOB#and#tightness.#Doctor#order#5[hydroxyindoleacetic#acid#in#urine#Which#cell#
responsible?#
a..chromaffin.cell.
b..Enterocell.
c..Lympho.cell.
d..Goblet.cell.
Answer:.a.[carcinoid.syndrome.].
..
Degree#for#screening#of#abdominal#aortic#aneurysm#for#pt#age#70#y/o#who#never#smoked#?#
aKA.
bKB.
cKC.
dKD.
Answer:.c.

. .
.
What#is#biochemical#defect#in#x#linked#agammagloubinemia#!?#
.Answer:.no.choices.
the.white.blood.cell.formation.processdoes.not.generate.mature.B.cells,[2].which.manifests.as.a.
complete.or.nearKcomplete.lack.of.proteins.called.gamma.globulins,.including.antibodies,.in.their.
bloodstream..
..
Child#with#repeated#polymicrobial#chest#infection#,#skin#test#positive#for#candida#
antigen#Blood#test#all#normal#except#high#IgG#or#IgM#!?#Low#lymphocyte.#Lymph#node#
biopsy#showed#:#rudimentary#germinal#centres#.#What#is#the#pathophsyioloy#of#this#
disease!?#
Answer.:.the.diagnosis.is.Bruton.Agammaglobulinemia.
Answer:.no.choices..
..
Goblet#cells#are#mostly#found#in################################################ #
1K.stomach.
2K.duodenum.
3K.ascending.colon.
4K.rectum......... ..
Answer:.ascending.colon.
The.colon.contains.a.larger.percentage.of.goblet.cells.than.the.small.intestine.
.Reference:.https://www.ncbi.nlm.nih.gov/pmc/articles/PMC2933006/.
..
about#RAST#test#measure??# #
Answer:.no.choices..... ...
A.radioallergosorbent.test.(RAST).is.a.blood.test.using.radioimmunoassay.test.to.detect.
specific.IgE.antibodies,.to.determine.the.substances.a.subject.is.allergic.to..This.is.
different.from.a.skin.allergy.test,.which.determines.allergy.by.the.reaction.of.a.
person's.skin.to.different.substances..
..
Beta#Thalassimia?############################################### #
a)Point.mutation.
b).Deletion.
c)Insertion.
d)Fragment.shaft.
answer:.a.
..
Most#common#known#side#effect#of#beclometasone#in#children?############# #
aKExcitable.behaviour!.
bKlabail.mood.
cKgrowth.retardation.
dKintraoccular.Hypertension.
Answer:.c?.
Reference:.https://www.drugs.com/sfx/beclomethasoneKsideKeffects.html.
..
1st#responsible#for#Bp#regulation?################################## ######### #
aK.heart.
bK.aorta.
cK.arterioles.................... .. .
dK.capillaries.
.Answer:.b.
..
case#of#bell's#palsy,#which#nerve#will#be#intact############ ##### #
aK.massater.
bKbucceneter.
cK.orbicuaris.oris.
dK.orbicuaris.
Answer:.a.
..
Side#effect#of#levodopa#?################################################################### #
Htn.“.cuz.hypo.not.hyper”................................................................. .
n/v.................................. ....... .
answer:.nausea.and.vomiting.
..
what's#the#effect#of#niacin?#
aKdec.LDL.
bKDec.TGD.
cK.inc.HDL.
Answer:.c.
..
Metformin#action#is#enhanced#in#by#which#enzyme?......................................... ................
Answer:.activation.of.AMPKactivated.protein.kinase.
..
Pronator#teres#syndrome#?################################### ###### #
Answer:.Medial.nerve.compression.at.elbow.
..
about#final#destination#of#proteins#in#cell#cycle?############## #
aKAnaphase.
bKProphase.
cKCytokinesis..... .............. .
Answer:.GOLOGI.APPARTUS.OR.TELOPHASE!.
..................................................................... .
What#structure#holding#Uterus?########################## #### #
Answer:.uterosacral.ligaments.
..
pt#has#urine#culture#MRSA#he#is#on#antibiotic#after#fewa#day#develop#redness#on#face#
,neck#what#is#antibiotic?#
aKpencillin.
bKvancomycin.
.Answer:.b.
..
Which#one#of#these#vaccines#taken#by#intranasal#route?################
aKZanamavir................................ ....... .
bKOseltamivir.
Answer:.a.
..
Nasal#&#and#palate#dryness#due#to#obstruction#of#?############ ############# #
aK.submandibular.gland.
bK.Submental.
cK.Ptergoplatine.
Answer:.a.
..
Which#of#chemodrugs#will#cause#HTN#+#skin#problem#+#pulmonary#fibrosis#?#
aK.bleomycin.
bK.Methotrexate.
Answer:.a.
..
Patient#on#anti#epileptic#and#Heart#failure#medication#presented#with#signs#of#drug#
overdose#and#ECG#showing#dysarethmia#?#
aK.digoxin.
bK.SSRI...................................................... .
cK.Quinine.
Answer:.a.
..
Child#with#autoantibodies..#chances#of#getting#dm1?........ ..
aK40.%.
bK60.%.
cK80.%.
dK100.%.
Answer:?.
..
Nerve#involved#in#tarsal#tunnel#syndrome?.
Answer:.TIBIAL.NERVE.
..
Oby#surgery#clamb#the#artery#close#to#lateral#vaginal#wall#what#structure#maybe#
injured?#
aKPeudendal.nerve.
bKureter.
Answer:.b.
..
Long#scenario#about#hydrocele#typical?#################################### ############# #
Answer:.failure.of.obliteration.of.process.vaginalis.
..
pt#eye#drop#and#eye#goes#to#medial#side#what#nerve#injury#when#try#to#close#the#eye?#
aK3.
bK7.
cK4.
Answer:.a?.
..
HSP#disease#what#is#the#immunoglobulin#responsible?###### #
aKIgG.
bKIgM.
cKIgA.
dKIgE.
Answer:.
..
which#in#inguinal#canal#is#develop#from#the#external#oblique#muscle?########################## ######### ............
aKExternal.spermatic.fascia.................................. .................... .
bKInternal.spermatic.fascia.
Answer:.a.
Reference:.https://en.wikipedia.org/wiki/External_spermatic_fascia.
..
A#pt#with#hypoK#,#HypoCL#and#hypoNa#and#HyperHCO3#i#think#with#some#symptoms#,#what's#the#
1ry#defect#?#
A..NaCl.
B..H.excertion.. ................................................... .
C..H.absorption.
D..k.
Answer:.a.
bartter.syndrome,.NaCl.reabsorption.defect..
..
type#of#innate#mechanism#for#extracellular#bacteria?#
Answer:.no.choices.
........................................................................................
primary#immunodeficiency#that#expose#the#pt#to#recurrent#of#viral#and#molds#infection?#
aK.T.cell.
bK.B.cell.......................... ..... .
cK.complement.defi.
Answer:.a.
..
20#yrs#male#with#hx#of#hematuria#proteinuria#6#gm,#......#what#you#will#see#in#renal#
biopsy#(#hx#of#post#strptococcus#glomerulonephritis)?.......................................... ........... .
aK.membrane.... ................
bK.Minimal............................................................... .
cK.Post.infection.
Answer:.c?.
..
renal#vein#come#from#which#part#of#inferior#vena#cava?#################
Answer:.Subcardinal.
..
18#years#old#male#hx#of#RTA#presented#with#basal#skull#
Fracture#Reaching#to#the#jugular#foramen#,#Nerves#were#injured#.#Which#of#the#following#
muscles#will#not#be#affected#?#
AKsternocleidomastoid.muscle..
BKhyoglossus.muscle.
CKstylopharyngus.muscle.
DKtrapezius.muscle.
Answer:.B.:.Hyoglossus.muscle.
..
if#you#remove#the#pectoralis#major#muscle#,#what's#gonna#happen?#
aKloss.of.arm.adduction.
bKloss.of.arm.abduction.
cKloss.of.arm.adduction.and.internal.rotation.
dKloss.af.arm.abduction.and.external.rotation.
Answer.:.C.
..
Patient#have#bilateral#thin#walled#parenchyma#of#kidney?############
Answer:.(PCKD).
..
Child#with#DM#type#1#what's#the#Mechanism#of#action#of#the#disease#?#
aK.Triglyceride.uptake.
bK.Liver.increase.of.fatty.acid.
Answer:.
..
A#case#of#xeroderma#pigmentosum#,#whats#the#mech#of#action#?#
Answer:.defect.in.DNA.repair..
..
Which#of#the#following#genes#is#associated#with#atheroscelerosis,#but#linked#with#
polygenetic#Alzheimer's#disease#as#well?#
A).Melyiod.protien.B..
B).Apolipoptotein.E.............. ........ .
C).Presenilen.1..
D).Presenilen.2.
Answer:..b.
..
Fallopican#tube#embryology?#
.Answer:.mesoderm.intermidite.cell.>.latral.part.of.genital.ridge.
..
Patient#with#gullain#barre,#what#cell#is#affected#?#
Answer:.schwan.cell.
..
U#prescribe#roaccutane#=#retinoids#for#women#,#what#is#the#serious#side#effect?#
Answer:.birth.defect.
..
Facial#nerve#affection,#normal#lacrimation,#abnormal#tympani#reflex#and#loss#of#taste,#
site#of#injury:#
aKDistal.to.tympani.chordi.
bKProximal.to.tympani.chordi.
cKDistal.to.gbs.nerve.
dKProximal.to.gbs.nerve.
Answer:.b.
..
HDL#has#protective#mechanism#for#coronary#artery#disease#by#which#enzyme?#
Answer:.antioxidant.enzyme.paraoxonase.(PON).1.
..
Patient#had#DM#with#HTN#had#renal#transplant#after#1#month#show#rejection#biopsy#show#
hla#class#1.#Which#cell#responsible?#
aKMacrophage.
bKCd.4.
cKCd8.
dKNk.
Answer:.c.
Reference:.https://en.wikipedia.org/wiki/Human_leukocyte_antigen.
..
8[Most#specific#for#AIDS: #
A).night.sweat .
B).generalized.LAP .
C).opportunistic.infections.
Answer:.b.
..
Man#lost#smell,#which#lobe#is#affected#
A).parietal.
B).temporal.
C).frontal.
D).occipital.
Answer:.b.
..
#Man#with#stroke#and#vision#loss,#affected#lobe:#
A).parietal.
B).temporal.
C).frontal.
D).occipital.
Answer:.d.
..
Latency#period#in#HI,#cell#responsible?#
Answer:.CD4+.T.cells.with.a.memory.
..
Brown#dicoloration#of#eye#then#disappear#..#What#is#the#drug#:#
aKVanco.
bKMetacloprmide.
cKlanatoprost.
Answer:.c.
..
What#type#of#skin#reaction#in#tb#reaction?#
AK1.
BK2.
CK3.
DK4.
Answer:..d.
Reference:.https://en.wikipedia.org/wiki/Type_IV_hypersensitivity.
..
Tyrosine#kinase#inhibitors#work#against?#
a.Epidermal.growth.factor.
b.Endodermal.growth.factor.
.Answer:.a.
..
Patient#with#cardiomyopathy#and#heart#failure,#biopsy#showed:#central#necrosis#surrounded#by#
inflammatory#cells#and#large#histiocytes#(#Anitschkow#cell)#What#is#the#most#likely#diagnosis?#
aK.rheumatic.fever.
bK.myocardial.sarcoidosis.
Answer:.1.
..
Chemotherapy#medication#that#acts#on#cell#cycle#something#,skin#rash,#pulmonary#fibrosis?#
aK.methotrexate.
bK.bleomycin.
cK.cisplatin.
Answer:.b.
##
Which#beta#blocker#has#the#most#alpha#adrenergic#effect?#
aK.labetalol.
bK.nadolol.
cK.pindolol.
Answer:.a.
..
Drug#that#is#given#in#patient#with#heart#disease#that#inhibits#the#entrance#of#calcium,#which#lead#to#
decrease#in#the#myocardial#excitable#something?#
aK.diltiazim.
bK.hydralazine.
cK.nitroglycerin.
Answer:.a.
..
Mechanism#of#action#of#heparin?#
aK.reversible.bind.to.antithrombin.3.
bK.inhibit.factor.V.
cK.inhibit.factor.ii,.vii,.9,.and.10.
dK.
Answer:.1.
..
How#many#Barr#bodies#in#XXX#female?#
a).1.
b).2.
c).3.
d).4.
Answer:.2.(Barr.body.is.the.inactive.X.chromosome.).
..
Scenario#of#a#patient#with#colorectal#cancer.#Diagnosed#to#have#adenocarcinoma,#he#did#some#
tests#which#showed#high#tyrosine#kinase.#Which#of#the#following#do#u#expect#to#be#high?#
a).endo.dermal.growth.factor.
b).epidermal.growth.factor.
c).coupled.G.protein.receptor.
d).platelet.growth.factor.
Answer:.
..
Scenario#of#a#patient#who#had#jaundice#after#flu#like#symptoms.#Indirect#bilirubin#is#high,#direct#is#
normal#and#total#bilirubin#is#high.#AST,#ALT#and#GGT#normal.#What#is#the#gene#mutation#
responsible?#
a).TPEN.
b).p53.
c).APC.
d).??.(Probably.the.right.answer).
Answer:.
..
Teratogenic#effect#of#Thalidomide#
.Answer:.meromelia.
..
Inhibitor#of#uptake#of#which#of#the#following#should#be#given#to#the#patient?.(what.pt,.question.is.
missing.info).
aK.GABA.
bK.DOPA.
cK.Serotonin.
Answer:.?.
..
Trypanosoma#cruzi#works#on#or#something#like#that#on#which#receptor#(at#cellular#level):#
A..IL6.
B..IL10.
Answer:.?.
..
MOA#of#statin:#
Answer:.HMG.CoA.reductase.inhibitor.
.
.
.
.
.
.
.
.

Genetics.
..
1... Marffan#syndrome#descriptions#long#arm#and#ask#about#mood#on#inheritance?#
Answer:.AD..
..
2... Pt#with#brown#macules#on#skin#with#axillary#fleckring#dx#is#NF,#what#is#the#mode#of#
inheritance?#
.Answer:.Autosomal.Dominant..
..
3......Tall#&#thin#boy#with#joint#laxity#also#his#mother#has#same#features#what#is#mode#of#
inheritance?#
Answer:#.the.features.describes.marfan.synbdrome.and.the.mode.of.inheritance.is.Autosomal.
dominant..
..
4... pt#with#tender#flank#and#mutation#in#chromosome#16#what#mode#of#inheritance?#
Answer:#.Autosomal.Dominant.Polycystic.Kidney.disorder.[PKD.1/polycystin.1.
PKD2/polycystin.2].
..
5... SCA#mode#of#inheritance?#
Answer:#.autosomal.recessive.
References:.https://ghr.nlm.nih.gov/condition/sickleKcellKdisease#inheritance.
..
6... 48.type#of#inheritance#of#Fanconi#anaemia?#
Answer:##Autosomal#recessive#genetics;.FA.is.primarily.an.autosomal.genetic.disorder..This.means.that.
two.mutated.alleles(one.from.each.parent).are.required.to.cause.the.disease..The.risk.is.25%.risk.that.
each.subsequent.child.will.have.FA..About.2%.of.FA.cases.are.XKlinked.recessive,.which.means.that.if.the.
mother.carries.one.mutated.Fanconi.anemia.allele.on.one.X.chromosome,.a.50%.chance.exists.that.male.
offspring.will.present.with.Fanconi.anemia..
References:.https://en.wikipedia.org/wiki/Fanconi_anemia.
..
7... ehlers[danlos#what#is#the#type#of#transmission??#
K.Autosomal.dominant..
..
8... Male#present#with#multiple#maculs#5[15#mm#,#and#axilary#frickling#,#his#wife#is#
pregnant#,#what#is#the#mood#of#inheritance?#
aK.Autosoma.
bK.XKlinked.
cK.Mitochondrial.
.Answer:.a.
..
9... Mode#of#inhertance#FAP?#
Answer:..no.choices..Autosomal.dominant?.
..
10...Wiskott[Aldrich#Syndrome#mode#of#inheritance?#
Answer:.XKlinked.recessive.
..
11...type#of#inheritance#of#intrahepatic#cholestatic?######## #### #
aKAR.......... .
bK.X.linked.
cK.mitochondrial.
Answer:.a.
Reference:.https://ghr.nlm.nih.gov/condition/progressiveKfamilialKintrahepaticK
cholestasis#inheritance......... .
..
12...Burkitt#lymphoma#gene?#
Answer:#.Myc.gene.is.normally.found.in.ch.8.is.transferred.to.ch.14..in.Africa.its.associated.with.EBV..
References:.https://www.ncbi.nlm.nih.gov/books/NBK22257/.
..
13...38.A#child#from#Ghana#with#a#neck#mass,#Biopsy#showed#starry#sky#appearance,#KI#67#was#
positive,#Which#genetic#change#is#true?#
Answer.burkitts.lymphoma,.Ch.14.and.Myc.gene.translocation..
..
14...Parents#brought#their#son#with#cystic#fibrosis,#asking#about#the#chance#of#his#daughter#
to#be#carrier:#
aK.....1:2.
bK.....2:4.
cK.....1:25.
dK.....1:3.
Answer:..b.
..

. .
.
15...where#is#the#gene#of#Neurpfibromatosis?#
Answer:.17q11.2.
..
16...[#RCC#gene*#(#as#presentation#of#hematuria,#wt#loss#,#loin#pain#)?#
Answer:.VHL.
.*Gene.for.renal.cell.carcinoma..
..
17...[#Breast#ca#gene?#
Answer:.BRACA2.
..
18...Breast#cancer#gene#(no#BRCA)*?#
aK.....her2.
bK.....CA125.
cK.....P53.
Answer:.a.
..
19...Case#of#RA#asking#about#genetics?#
Answer:.HLA.B27.
..
20...Scenario#with#a#patient#who#came#with#fever#and#altered#level#of#consciousness.#Labs#
show#high#urea#and#creatinine,#low#platelets#and#anemia#(Scenario#of#TTP).#What's#the#
gene#mutation#responsible?#
a..Cardiolipins.
b..ADAMTS13.
c..Glycoprotein.
Answer:.b.
..
21...Pancreatic#cancer#which#gene#associated#with?#
Answer:.no.choices.
BRCA1,.BRCA2,.PALB2,.CDKN2A,.ATM,.STK11,.and.the.genes.linked.to.Lynch.syndrome.(MLH1,.
MSH2,.MSH6,.PMS2,.and.EPCAM)..
..
22...cardiomyopathy#mutation?#
Answer:.no.choices.
Common.Genes:.
TTN.gene.(dilated),.
MYH7,.MYBPC3,.TNNT2,.TNNI3.(hypertrophied)..
..

.
.
References:.https://ghr.nlm.nih.gov/condition/familialKdilatedKcardiomyopathy.
https://ghr.nlm.nih.gov/condition/familialKhypertrophicKcardiomyopathy#genes.
..
23...mother#had#abortion#two#times#last#child#with#45X#monosomy,#recurrence#in#next#
pregnancy?#
aK.....%30.
bK.....%50.
cK.....%70.
dK.....%90.
Answer:.a.
..
24...Familial#adenomatous#polyposis#gene?#
.Answer:.The.human.APC.gene.is.located.on.the.long.(q).arm.
of.chromosome.5.in.band.q22.2.(5q22.2).
..
25...gene#associated#with#type#1#DM#
aKDR2.
bKDR4.
cKDR6.
dKDR7.
Answer:.b..
..
26...Most#common#mutation?#
.Answer:.trisomy.21.
..
27...Marfan#syndrome#features#of#the#mother#and#her#boy:#what#is#the#probability#of#the#
children#to#have#it?#
aK25%.
bK50%.
cK75%.
dK100%.
Answer:.b.
..
28...X#linked#is?#
aKAD.
bKMultifactorial.
.Answer:?.
..
29...HCM#which#gene#has#the#worst#prognosis.(no.options)............... . .
Answer:.if.itʼsHCM.hypertrohic.cardiomyopathy.gene.assoc.MYH7,.MYBPC3,.TNNT2,.and.TNNI3.
..
30...Patient#with#tetanus#and#hypocalcemia#presented#with#pcp,#aspiragillus#pneumonia,#and#
other#infections,#what#is#the#cause?#
Answer:.Deletion.of.chromosome.22.(digoarge).
..
31...Patient#with#jaundice#,#labs#shows#high#indirect#bilirubin#and#total#bilirubin,#was#
about#the#gene#or#the#mutation?#
Answer:.no.choices.
CriglerKNajjar.syndrome.type.II.is.caused.by.homozygous.or.compound.heterozygous.
mutation.in.the.UDPKglucuronosyltransferase.gene.(UGT1A1;.191740).on.chromosome.2q37..
Mutations.in.the.same.gene.cause.Gilbert.syndrome.(143500).and.CriglerKNajjar.
syndrometype.I.(218800)..
1.).CriglerKNajjar.syndrome.is.inherited.in.an.autosomal.recessive.pattern,.which.means.
both.copies.of.the.UGT1A1.
2.).Gilbert's.syndrome.(.asymptomatic.usually.)..is.due.to.a.mutation.in.the.UGT1A1.
gene.
..
32...Gene#mutation#found#in#non[small#cell#lung#cancer?#
KAnswer:.EGFR.mutations.&.ALK.alterations.
..
33...Genotype#of#mild#thalassemia:#
A..a0.K.K./.K.K.
B...a0.K.K./.K.a.
C...a0.K.K./.a.a.
D..a+.K.a./.a.a.
Answer:.c?.
A:.one.geneK>.sever,.B:.two.gene.K>.moderate..
..
34...male#e#hemophilia#a#and#female#carrier#what#%#their#child#will#have#hemophilia?#
Answer:#.50%.
The.chance.for.a.carrier.mother.to.deliver.a.baby.with.haemophilia.is.producing.50%.carriers.and.
this.chance.is.the.same.in.all.pregnancies.even.if.she.had.a.previous.child.with.haemophilia..
References:.https://www.hog.org/handbook/section/2/howKhemophiliaKisKinherited.
..
..
Inheritance#questions:#
K.fanconi#anemia:.AR.
K.Progressive#familial#intrahepatic#cholestasis:.AR.
[#Willson:.AR,.chromosome.14.
K.Cystic#fibrosis:.AR,.chromosome.7,.long.q.arm.
K.Congenital#chloride#diarrhea:.AR.
K.Congenital#liver#fibrosis:.ass..With.AR.polycyctic.kidney.disease.
K.Von#willebrand#disease:.AR.
K.Alpha#and#beta#thalasemia,#and#SCA:.AR.
K.Otosclerosis:.AD.
K.Polycystic#kidney#adult:.AD,.chromosome.16.
K.Heridtary#spherocytosis:.AD.
K.Marfan#syndrome:.AD.
K.Hereditary#chronic#pancreatitis:.AD.
K.Apoptosis#gene:.p53.
K.Gene#of#ductal#carcinoma:.p53.
K.Enhance#apoptosis:.p53.
K.Kidney#tumor#oncogen:.MET.in.hereditary.papillary.renal.cell.cancer,.and.ps53.
K..Gene.for.copper.transport.ATPase:.
[#IBD:.chromosome.16,.genes:.NOD2,.ATG16L1.
K.DM#type#2:.chromosome.20.
K.Gilbert:.chromosome.2,.q37,.UGT1A1.gene.
K.High#cholesterol:.LDL.receptor.gene.
[#X#linked#a#gamma#globulinemia:.CD19.and.CD.20.
[#Problem#with#phagocytosis,#protein#defect#for#oxygenation:.Decrease.NADH.(chronic.
granulomatous.disease).
K.Angioedema:.SERPING1.Gene.
K.Alzheimer#disease:.&gt;60.year:.21.chromosome.,,.&lt;60:.19.chromosome.
[#Xeroderma#pigmentosa:.defect.in.DNA.break.repair.gene.
[#Alzheimer#disease:.&gt;60.year:.21.chromosome.,,.&lt;60:.19.chromosome.
K.Huntignton#disease:.chromosome.4,.CAG.trinucleotide.repeat."3.M:.mood,.memory,.
movement".
K.SCA#and#thalasemia:.point.mutation.
K.CML#and#ALL:.bcr.(on.chromosome.22).and.abl1.(on.chromosome.9).resulting.in.the.bcrK.
abl1.fusion.gene.—&gt;.philadelphia.(ph).chromosome.
[##Starry#sky#biopsy#"burkett's#lymphoma":.CKmyc.gene,.chromosome.8.
..
..
..
.
##
##
##
##
##
##
##
##
#
#

Das könnte Ihnen auch gefallen